Π£ΠΌΠ½ΠΎΠΆΠ΅Π½ΠΈΠ΅ слоТСниС Π²Ρ‹Ρ‡ΠΈΡ‚Π°Π½ΠΈΠ΅ Π΄Π΅Π»Π΅Π½ΠΈΠ΅ ΠΎΠ±Ρ‹ΠΊΠ½ΠΎΠ²Π΅Π½Π½Ρ‹Ρ… Π΄Ρ€ΠΎΠ±Π΅ΠΉ: Β«Π‘Π»ΠΎΠΆΠ΅Π½ΠΈΠ΅ ΠΈ Π²Ρ‹Ρ‡ΠΈΡ‚Π°Π½ΠΈΠ΅, Π΄Π΅Π»Π΅Π½ΠΈΠ΅ ΠΈ ΡƒΠΌΠ½ΠΎΠΆΠ΅Π½ΠΈΠ΅ ΠΎΠ±Ρ‹ΠΊΠ½ΠΎΠ²Π΅Π½Π½Ρ‹Ρ… Π΄Ρ€ΠΎΠ±Π΅ΠΉΒ» (стр. 1 ΠΈΠ· 2)

Π‘ΠΎΠ΄Π΅Ρ€ΠΆΠ°Π½ΠΈΠ΅

Β«Π‘Π»ΠΎΠΆΠ΅Π½ΠΈΠ΅ ΠΈ Π²Ρ‹Ρ‡ΠΈΡ‚Π°Π½ΠΈΠ΅, Π΄Π΅Π»Π΅Π½ΠΈΠ΅ ΠΈ ΡƒΠΌΠ½ΠΎΠΆΠ΅Π½ΠΈΠ΅ ΠΎΠ±Ρ‹ΠΊΠ½ΠΎΠ²Π΅Π½Π½Ρ‹Ρ… Π΄Ρ€ΠΎΠ±Π΅ΠΉΒ» (стр. 1 ΠΈΠ· 2)

ΠœΠ˜ΠΠ˜Π‘Π’Π•Π Π‘Π’Π’Πž ΠžΠ‘Π ΠΠ—ΠžΠ’ΠΠΠ˜Π― Π Π•Π‘ΠŸΠ£Π‘Π›Π˜ΠšΠ˜ ΠšΠΠ—ΠΠ₯БВАН

ΠšΠžΠ‘Π’ΠΠΠΠ™Π‘ΠšΠ˜Π™ Π“ΠžΠ‘Π£Π”ΠΠ Π‘Π’Π’Π•ΠΠΠ«Π™ ΠŸΠ•Π”ΠΠ“ΠžΠ“Π˜Π§Π•Π‘ΠšΠ˜Π™ ИНБВИВУВ

Π Π΅Ρ„Π΅Ρ€Π°Ρ‚

На Ρ‚Π΅ΠΌΡƒ: Β«Π‘Π»ΠΎΠΆΠ΅Π½ΠΈΠ΅ ΠΈ Π²Ρ‹Ρ‡ΠΈΡ‚Π°Π½ΠΈΠ΅, Π΄Π΅Π»Π΅Π½ΠΈΠ΅ ΠΈ ΡƒΠΌΠ½ΠΎΠΆΠ΅Π½ΠΈΠ΅ ΠΎΠ±Ρ‹ΠΊΠ½ΠΎΠ²Π΅Π½Π½Ρ‹Ρ… Π΄Ρ€ΠΎΠ±Π΅ΠΉΒ».

ΠšΠΎΡΡ‚Π°Π½Π°ΠΉ

2011 Π³ΠΎΠ΄

Π‘ΠžΠ”Π•Π Π–ΠΠΠ˜Π•

1. Из истории ΠΎΠ±Ρ‹ΠΊΠ½ΠΎΠ²Π΅Π½Π½Ρ‹Ρ… Π΄Ρ€ΠΎΠ±Π΅ΠΉ ………………………………………..3

2. ДСйствия с ΠΎΠ±Ρ‹ΠΊΠ½ΠΎΠ²Π΅Π½Π½Ρ‹ΠΌΠΈ дробями …………..…………………………..5

2.1. Π‘Π»ΠΎΠΆΠ΅Π½ΠΈΠ΅ ΠΈ Π²Ρ‹Ρ‡ΠΈΡ‚Π°Π½ΠΈΠ΅ ΠΎΠ±Ρ‹ΠΊΠ½ΠΎΠ²Π΅Π½Π½Ρ‹Ρ… Π΄Ρ€ΠΎΠ±Π΅ΠΉ …………………………..5

2.2. Π£ΠΌΠ½ΠΎΠΆΠ΅Π½ΠΈΠ΅ ΠΈ Π΄Π΅Π»Π΅Π½ΠΈΠ΅ ΠΎΠ±Ρ‹ΠΊΠ½ΠΎΠ²Π΅Π½Π½Ρ‹Ρ… Π΄Ρ€ΠΎΠ±Π΅ΠΉ ………………………….7

3. ΠŸΡ€ΠΈΠΌΠ΅Ρ€Ρ‹ Π½Π° слоТСниС, Π²Ρ‹Ρ‡ΠΈΡ‚Π°Π½ΠΈΠ΅, ΡƒΠΌΠ½ΠΎΠΆΠ΅Π½ΠΈΠ΅ ΠΈ Π΄Π΅Π»Π΅Π½ΠΈΠ΅ Π΄Ρ€ΠΎΠ±Π΅ΠΉ ……. 10

4. Бписок Π»ΠΈΡ‚Π΅Ρ€Π°Ρ‚ΡƒΡ€Ρ‹ ……………………………………………………………11

1. Из истории возникновСния ΠΎΠ±Ρ‹ΠΊΠ½ΠΎΠ²Π΅Π½Π½Ρ‹Ρ… Π΄Ρ€ΠΎΠ±Π΅ΠΉ.

Π”Ρ€ΠΎΠ±ΠΈ появились Π² Π³Π»ΡƒΠ±ΠΎΠΊΠΎΠΉ дрСвности. ΠŸΡ€ΠΈ Ρ€Π°Π·Π΄Π΅Π»Π΅ Π΄ΠΎΠ±Ρ‹Ρ‡ΠΈ, ΠΏΡ€ΠΈ измСрСниях Π²Π΅Π»ΠΈΡ‡ΠΈΠ½, Π΄Π° ΠΈ Π² Π΄Ρ€ΡƒΠ³ΠΈΡ… ΠΏΠΎΡ…ΠΎΠΆΠΈΡ… случаях люди Π²ΡΡ‚Ρ€Π΅Ρ‚ΠΈΠ»ΠΈΡΡŒ с Π½Π΅ΠΎΠ±Ρ…ΠΎΠ΄ΠΈΠΌΠΎΡΡ‚ΡŒΡŽ ввСсти Π΄Ρ€ΠΎΠ±ΠΈ.

Π”Ρ€Π΅Π²Π½ΠΈΠ΅ СгиптянС ΡƒΠΆΠ΅ Π·Π½Π°Π»ΠΈ, ΠΊΠ°ΠΊ ΠΏΠΎΠ΄Π΅Π»ΠΈΡ‚ΡŒ 2 ΠΏΡ€Π΅Π΄ΠΌΠ΅Ρ‚Π° Π½Π° Ρ‚Ρ€ΠΎΠΈΡ…, для этого числа –2/3- Ρƒ Π½ΠΈΡ… Π±Ρ‹Π» ΡΠΏΠ΅Ρ†ΠΈΠ°Π»ΡŒΠ½Ρ‹ΠΉ Π·Π½Π°Ρ‡ΠΎΠΊ. ΠœΠ΅ΠΆΠ΄Ρƒ ΠΏΡ€ΠΎΡ‡ΠΈΠΌ, это Π±Ρ‹Π»Π° СдинствСнная Π΄Ρ€ΠΎΠ±ΡŒ Π² ΠΎΠ±ΠΈΡ…ΠΎΠ΄Π΅ СгипСтских писцов, Ρƒ ΠΊΠΎΡ‚ΠΎΡ€ΠΎΠΉ Π² числитСлС Π½Π΅ стояла Π΅Π΄ΠΈΠ½ΠΈΡ†Π° – всС ΠΎΡΡ‚Π°Π»ΡŒΠ½Ρ‹Π΅ Π΄Ρ€ΠΎΠ±ΠΈ Π½Π΅ΠΏΡ€Π΅ΠΌΠ΅Π½Π½ΠΎ ΠΈΠΌΠ΅Π»ΠΈ Π² числитСлС Π΅Π΄ΠΈΠ½ΠΈΡ†Ρƒ (Ρ‚Π°ΠΊ Π½Π°Π·Ρ‹Π²Π°Π΅ΠΌΡ‹Π΅ основныС Π΄Ρ€ΠΎΠ±ΠΈ): 1/2; 1/3; 1/28; … . Если Сгиптянину Π½ΡƒΠΆΠ½ΠΎ Π±Ρ‹Π»ΠΎ ΠΈΡΠΏΠΎΠ»ΡŒΠ·ΠΎΠ²Π°Ρ‚ΡŒ Π΄Ρ€ΡƒΠ³ΠΈΠ΅ Π΄Ρ€ΠΎΠ±ΠΈ, ΠΎΠ½ прСдставлял ΠΈΡ… Π² Π²ΠΈΠ΄Π΅ суммы основных Π΄Ρ€ΠΎΠ±Π΅ΠΉ. НапримСр, вмСсто 8/15 писали 1/3+1/5. Иногда это Π±Ρ‹Π²Π°Π»ΠΎ ΡƒΠ΄ΠΎΠ±Π½ΠΎ. Π’ папирусС АхмСса Π΅ΡΡ‚ΡŒ Π·Π°Π΄Π°Ρ‡Π° :

Β«Π Π°Π·Π΄Π΅Π»ΠΈΡ‚ΡŒ 7 Ρ…Π»Π΅Π±ΠΎΠ² ΠΌΠ΅ΠΆΠ΄Ρƒ 8 людьми». Если Ρ€Π΅Π·Π°Ρ‚ΡŒ ΠΊΠ°ΠΆΠ΄Ρ‹ΠΉ Ρ…Π»Π΅Π± Π½Π° 8 частСй, придётся провСсти 49 Ρ€Π°Π·Ρ€Π΅Π·ΠΎΠ².

А ΠΏΠΎ-СгипСтски эта Π·Π°Π΄Π°Ρ‡Π° Ρ€Π΅ΡˆΠ°Π»Π°ΡΡŒ Ρ‚Π°ΠΊ: Π”Ρ€ΠΎΠ±ΡŒ 7/8 записывали Π² Π²ΠΈΠ΄Π΅ Π΄ΠΎΠ»Π΅ΠΉ: 1/2+1/4+1/8. Π—Π½Π°Ρ‡ΠΈΡ‚ ΠΊΠ°ΠΆΠ΄ΠΎΠΌΡƒ Ρ‡Π΅Π»ΠΎΠ²Π΅ΠΊΡƒ Π½Π°Π΄ΠΎ Π΄Π°Ρ‚ΡŒ ΠΏΠΎΠ»Ρ…Π»Π΅Π±Π°, Ρ‡Π΅Ρ‚Π²Π΅Ρ€Ρ‚ΡŒ Ρ…Π»Π΅Π±Π° ΠΈ Π²ΠΎΡΡŒΠΌΡƒΡˆΠΊΡƒ Ρ…Π»Π΅Π±Π°; поэтому Ρ‡Π΅Ρ‚Ρ‹Ρ€Π΅ Ρ…Π»Π΅Π±Π° Ρ€Π°Π·Ρ€Π΅Π·Π°Π»ΠΈ ΠΏΠΎΠΏΠΎΠ»Π°ΠΌ, Π΄Π²Π° Ρ…Π»Π΅Π±Π°- Π½Π° 4 части ΠΈ ΠΎΠ΄ΠΈΠ½ Ρ…Π»Π΅Π± Π½Π° 8 Π΄ΠΎΠ»Π΅ΠΉ, послС Ρ‡Π΅Π³ΠΎ ΠΊΠ°ΠΆΠ΄ΠΎΠΌΡƒ Π΄Π°Π»ΠΈ Π΅Π³ΠΎ Ρ‡Π°ΡΡ‚ΡŒ.

Но ΡΠΊΠ»Π°Π΄Ρ‹Π²Π°Ρ‚ΡŒ Ρ‚Π°ΠΊΠΈΠ΅ Π΄Ρ€ΠΎΠ±ΠΈ Π±Ρ‹Π»ΠΎ Π½Π΅ΡƒΠ΄ΠΎΠ±Π½ΠΎ. Π’Π΅Π΄ΡŒ Π² ΠΎΠ±Π° слагаСмых ΠΌΠΎΠ³ΡƒΡ‚ Π²Ρ…ΠΎΠ΄ΠΈΡ‚ΡŒ ΠΎΠ΄ΠΈΠ½Π°ΠΊΠΎΠ²Ρ‹Π΅ Π΄ΠΎΠ»ΠΈ, ΠΈ Ρ‚ΠΎΠ³Π΄Π° ΠΏΡ€ΠΈ слоТСнии появится Π΄Ρ€ΠΎΠ±ΡŒ Π²ΠΈΠ΄Π° 2/n. А Ρ‚Π°ΠΊΠΈΡ… Π΄Ρ€ΠΎΠ±Π΅ΠΉ СгиптянС Π½Π΅ допускали. ΠŸΠΎΡΡ‚ΠΎΠΌΡƒ, папирус АхмСса начинаСтся с Ρ‚Π°Π±Π»ΠΈΡ†Ρ‹, Π² ΠΊΠΎΡ‚ΠΎΡ€ΠΎΠΉ всС Π΄Ρ€ΠΎΠ±ΠΈ Ρ‚Π°ΠΊΠΎΠ³ΠΎ Π²ΠΈΠ΄Π° ΠΎΡ‚ 2/5 Π΄ΠΎ 2/99 записаны Π² Π²ΠΈΠ΄Π΅ суммы Π΄ΠΎΠ»Π΅ΠΉ. Π‘ ΠΏΠΎΠΌΠΎΡ‰ΡŒΡŽ этой Ρ‚Π°Π±Π»ΠΈΡ†Ρ‹ выполняли ΠΈ Π΄Π΅Π»Π΅Π½ΠΈΠ΅ чисСл. Π’ΠΎΡ‚, Π½Π°ΠΏΡ€ΠΈΠΌΠ΅Ρ€, ΠΊΠ°ΠΊ 5 Π΄Π΅Π»ΠΈΠ»ΠΈ Π½Π° 21: 5/21

Π£ΠΌΠ΅Π»ΠΈ СгиптянС Ρ‚Π°ΠΊΠΆΠ΅ ΡƒΠΌΠ½ΠΎΠΆΠ°Ρ‚ΡŒ ΠΈ Π΄Π΅Π»ΠΈΡ‚ΡŒ Π΄Ρ€ΠΎΠ±ΠΈ. Но для умноТСния ΠΏΡ€ΠΈΡ…ΠΎΠ΄ΠΈΠ»ΠΎΡΡŒ ΡƒΠΌΠ½ΠΎΠΆΠ°Ρ‚ΡŒ Π΄ΠΎΠ»ΠΈ Π½Π° Π΄ΠΎΠ»ΠΈ, Π° ΠΏΠΎΡ‚ΠΎΠΌ, Π±Ρ‹Ρ‚ΡŒ ΠΌΠΎΠΆΠ΅Ρ‚, снова ΠΈΡΠΏΠΎΠ»ΡŒΠ·ΠΎΠ²Π°Ρ‚ΡŒ Ρ‚Π°Π±Π»ΠΈΡ†Ρƒ. Π•Ρ‰Ρ‘ слоТнСС обстояло с Π΄Π΅Π»Π΅Π½ΠΈΠ΅ΠΌ.

Π’ Π΄Ρ€Π΅Π²Π½Π΅ΠΌ Π’Π°Π²ΠΈΠ»ΠΎΠ½Π΅ ΠΏΡ€Π΅Π΄ΠΏΠΎΡ‡ΠΈΡ‚Π°Π»ΠΈ Π½Π°ΠΎΠ±ΠΎΡ€ΠΎΡ‚, — постоянный Π·Π½Π°ΠΌΠ΅Π½Π°Ρ‚Π΅Π»ΡŒ, Ρ€Π°Π²Π½Ρ‹ΠΉ 60-Ρ‚ΠΈ. ШСстидСсятСричными дробями, унаслСдованными ΠΎΡ‚ Π’Π°Π²ΠΈΠ»ΠΎΠ½Π°, пользовались грСчСскиС ΠΈ арабскиС ΠΌΠ°Ρ‚Π΅ΠΌΠ°Ρ‚ΠΈΠΊΠΈ ΠΈ астрономы. Но Π±Ρ‹Π»ΠΎ Π½Π΅ΡƒΠ΄ΠΎΠ±Π½ΠΎ Ρ€Π°Π±ΠΎΡ‚Π°Ρ‚ΡŒ Π½Π°Π΄ Π½Π°Ρ‚ΡƒΡ€Π°Π»ΡŒΠ½Ρ‹ΠΌΠΈ числами, записанными ΠΏΠΎ дСсятичной систСмС, ΠΈ дробями, записанными ΠΏΠΎ ΡˆΠ΅ΡΡ‚ΠΈΠ΄Π΅ΡΡΡ‚Π΅Ρ€ΠΈΡ‡Π½ΠΎΠΉ. А Ρ€Π°Π±ΠΎΡ‚Π°Ρ‚ΡŒ с ΠΎΠ±Ρ‹ΠΊΠ½ΠΎΠ²Π΅Π½Π½Ρ‹ΠΌΠΈ дробями Π±Ρ‹Π»ΠΎ ΡƒΠΆΠ΅ совсСм Ρ‚Ρ€ΡƒΠ΄Π½ΠΎ. ΠŸΠΎΡΡ‚ΠΎΠΌΡƒ голландский ΠΌΠ°Ρ‚Π΅ΠΌΠ°Ρ‚ΠΈΠΊ Π‘ΠΈΠΌΠΎΠ½ Π‘Ρ‚Π΅Π²ΠΈΠ½ ΠΏΡ€Π΅Π΄Π»ΠΎΠΆΠΈΠ» ΠΏΠ΅Ρ€Π΅ΠΉΡ‚ΠΈ ΠΊ дСсятичным дробям.

Π˜Π½Ρ‚Π΅Ρ€Π΅ΡΠ½Π°Ρ систСма Π΄Ρ€ΠΎΠ±Π΅ΠΉ Π±Ρ‹Π»Π° Π² Π”Ρ€Π΅Π²Π½Π΅ΠΌ Π ΠΈΠΌΠ΅. Она ΠΎΡΠ½ΠΎΠ²Ρ‹Π²Π°Π»Π°ΡΡŒ Π½Π° Π΄Π΅Π»Π΅Π½ΠΈΠΈ Π½Π° 12 Π΄ΠΎΠ»Π΅ΠΉ Π΅Π΄ΠΈΠ½ΠΈΡ†Ρ‹ вСса, которая Π½Π°Π·Ρ‹Π²Π°Π»Π°ΡΡŒ асс. Π”Π²Π΅Π½Π°Π΄Ρ†Π°Ρ‚ΡƒΡŽ долю асса Π½Π°Π·Ρ‹Π²Π°Π»ΠΈ ΡƒΠ½Ρ†ΠΈΠ΅ΠΉ. А ΠΏΡƒΡ‚ΡŒ, врСмя ΠΈ Π΄Ρ€ΡƒΠ³ΠΈΠ΅ Π²Π΅Π»ΠΈΡ‡ΠΈΠ½Ρ‹ сравнивали с наглядной Π²Π΅Ρ‰ΡŒΡŽ- вСсом. НапримСр, римлянин ΠΌΠΎΠ³ ΡΠΊΠ°Π·Π°Ρ‚ΡŒ, Ρ‡Ρ‚ΠΎ ΠΎΠ½ ΠΏΡ€ΠΎΡˆΠ΅Π» сСмь ΡƒΠ½Ρ†ΠΈΠΉ ΠΏΡƒΡ‚ΠΈ ΠΈΠ»ΠΈ ΠΏΡ€ΠΎΡ‡Π΅Π» ΠΏΡΡ‚ΡŒ ΡƒΠ½Ρ†ΠΈΠΉ ΠΊΠ½ΠΈΠ³ΠΈ. ΠŸΡ€ΠΈ этом, ΠΊΠΎΠ½Π΅Ρ‡Π½ΠΎ, Ρ€Π΅Ρ‡ΡŒ шла Π½Π΅ ΠΎ взвСшивании ΠΏΡƒΡ‚ΠΈ ΠΈΠ»ΠΈ ΠΊΠ½ΠΈΠ³ΠΈ. ИмСлось Π² Π²ΠΈΠ΄Ρƒ, Ρ‡Ρ‚ΠΎ ΠΏΡ€ΠΎΠΉΠ΄Π΅Π½ΠΎ 7/12 ΠΏΡƒΡ‚ΠΈ ΠΈΠ»ΠΈ ΠΏΡ€ΠΎΡ‡Ρ‚Π΅Π½ΠΎ 5/12 ΠΊΠ½ΠΈΠ³ΠΈ. А для Π΄Ρ€ΠΎΠ±Π΅ΠΉ, ΠΏΠΎΠ»ΡƒΡ‡Π°ΡŽΡ‰ΠΈΡ…ΡΡ сокращСниСм Π΄Ρ€ΠΎΠ±Π΅ΠΉ со Π·Π½Π°ΠΌΠ΅Π½Π°Ρ‚Π΅Π»Π΅ΠΌ 12 ΠΈΠ»ΠΈ Ρ€Π°Π·Π΄Ρ€ΠΎΠ±Π»Π΅Π½ΠΈΠ΅ΠΌ Π΄Π²Π΅Π½Π°Π΄Ρ†Π°Ρ‚Ρ‹Ρ… Π΄ΠΎΠ»Π΅ΠΉ Π½Π° Π±ΠΎΠ»Π΅Π΅ ΠΌΠ΅Π»ΠΊΠΈΠ΅, Π±Ρ‹Π»ΠΈ особыС названия.

Π”Π°ΠΆΠ΅ сСйчас ΠΈΠ½ΠΎΠ³Π΄Π° говорят:β€ΠžΠ½ скрупулёзно ΠΈΠ·ΡƒΡ‡ΠΈΠ» этот вопрос.” Π­Ρ‚ΠΎ Π·Π½Π°Ρ‡ΠΈΡ‚, Ρ‡Ρ‚ΠΎ вопрос ΠΈΠ·ΡƒΡ‡Π΅Π½Π΄ΠΎ ΠΊΠΎΠ½Ρ†Π°, Ρ‡Ρ‚ΠΎ Π½Π΅ ΠΎΠ΄Π½ΠΎΠΉ самой ΠΌΠ°Π»ΠΎΠΉ нСясности Π½Π΅ ΠΎΡΡ‚Π°Π»ΠΎΡΡŒ. А происходит странноС слово β€œΡΠΊΡ€ΡƒΠΏΡƒΠ»Ρ‘Π·Π½ΠΎβ€ ΠΎΡ‚ римского названия 1/288 асса — β€œΡΠΊΡ€ΡƒΠΏΡƒΠ»ΡƒΡβ€. Π’ Ρ…ΠΎΠ΄Ρƒ Π±Ρ‹Π»ΠΈ ΠΈ Ρ‚Π°ΠΊΠΈΠ΅ названия: ”сСмис”- ΠΏΠΎΠ»ΠΎΠ²ΠΈΠ½Π° асса, β€œΡΠ΅ΠΊΡΡ‚Π°Π½Ρβ€- ΡˆΠ΅ΡΡ‚Π°Ρ Π΅Π³ΠΎ доля, β€œΡΠ΅ΠΌΠΈΡƒΠ½Ρ†ΠΈΡβ€- ΠΏΠΎΠ»ΠΎΠ²ΠΈΠ½Π° ΡƒΠ½Ρ†ΠΈΠΈ, Ρ‚.Π΅. 1/24 асса ΠΈ Ρ‚.Π΄. ВсСго ΠΏΡ€ΠΈΠΌΠ΅Π½ΡΠ»ΠΎΡΡŒ 18 Ρ€Π°Π·Π»ΠΈΡ‡Π½Ρ‹Ρ… Π½Π°Π·Π²Π°Π½ΠΈΠΉ Π΄Ρ€ΠΎΠ±Π΅ΠΉ. Π§Ρ‚ΠΎΠ±Ρ‹ Ρ€Π°Π±ΠΎΡ‚Π°Ρ‚ΡŒ с дробями, Π½Π°Π΄ΠΎ Π±Ρ‹Π»ΠΎ ΠΏΠΎΠΌΠ½ΠΈΡ‚ΡŒ для этих Π΄Ρ€ΠΎΠ±Π΅ΠΉ Ρ‚Π°Π±Π»ΠΈΡ†Ρƒ слоТСния ΠΈ Ρ‚Π°Π±Π»ΠΈΡ†Ρƒ умноТСния. ΠŸΠΎΡΡ‚ΠΎΠΌΡƒ римскиС ΠΊΡƒΠΏΡ†Ρ‹ Ρ‚Π²Ρ‘Ρ€Π΄ΠΎ Π·Π½Π°Π»ΠΈ, Ρ‡Ρ‚ΠΎ ΠΏΡ€ΠΈ слоТСнии триСнса (1/3 асса) ΠΈ сСкстанса получаСтся сСмис, Π° ΠΏΡ€ΠΈ ΡƒΠΌΠ½ΠΎΠΆΠ΅Π½ΠΈΠΈ бСса (2/3 асса) Π½Π° ΡΠ΅ΡΠΊΡƒΠ½Ρ†ΠΈΡŽ( 2/3 ΡƒΠ½Ρ†ΠΈΠΈ, Ρ‚. Π΅.1/8 асса) получаСтся унция. Для облСгчСния Ρ€Π°Π±ΠΎΡ‚Ρ‹ ΡΠΎΡΡ‚Π°Π²Π»ΡΠ»ΠΈΡΡŒ ΡΠΏΠ΅Ρ†ΠΈΠ°Π»ΡŒΠ½Ρ‹Π΅ Ρ‚Π°Π±Π»ΠΈΡ†Ρ‹, Π½Π΅ΠΊΠΎΡ‚ΠΎΡ€Ρ‹Π΅ ΠΈΠ· ΠΊΠΎΡ‚ΠΎΡ€Ρ‹Ρ… дошли Π΄ΠΎ нас.

Π‘ΠΎΠ²Ρ€Π΅ΠΌΠ΅Π½Π½ΡƒΡŽ систСму записи Π΄Ρ€ΠΎΠ±Π΅ΠΉ с числитСлСм ΠΈ Π·Π½Π°ΠΌΠ΅Π½Π°Ρ‚Π΅Π»Π΅ΠΌ создали Π² Индии. Волько Ρ‚Π°ΠΌ писали Π·Π½Π°ΠΌΠ΅Π½Π°Ρ‚Π΅Π»ΡŒ свСрху, Π° Ρ‡ΠΈΡΠ»ΠΈΡ‚Π΅Π»ΡŒ — снизу, ΠΈ Π½Π΅ писали Π΄Ρ€ΠΎΠ±Π½ΠΎΠΉ Ρ‡Π΅Ρ€Ρ‚Ρ‹. А Π·Π°ΠΏΠΈΡΡ‹Π²Π°Ρ‚ΡŒ Π΄Ρ€ΠΎΠ±ΠΈ Π² точности, ΠΊΠ°ΠΊ сСйчас, стали Π°Ρ€Π°Π±Ρ‹.

ΠžΠ±Ρ‹ΠΊΠ½ΠΎΠ²Π΅Π½Π½Π°Ρ Π΄Ρ€ΠΎΠ±ΡŒ – это число Π²ΠΈΠ΄Π°

, Π³Π΄Π΅ m ΠΈ n – Π½Π°Ρ‚ΡƒΡ€Π°Π»ΡŒΠ½Ρ‹Π΅ числа, Π½Π°ΠΏΡ€ΠΈΠΌΠ΅Ρ€

. Число m называСтся числитСлСм Π΄Ρ€ΠΎΠ±ΠΈ,n – Π·Π½Π°ΠΌΠ΅Π½Π°Ρ‚Π΅Π»Π΅ΠΌ. Π‘Ρ€Π΅Π΄ΠΈ ΠΎΠ±Ρ‹ΠΊΠ½ΠΎΠ²Π΅Π½Π½Ρ‹Ρ… Π΄Ρ€ΠΎΠ±Π΅ΠΉ Ρ€Π°Π·Π»ΠΈΡ‡Π°ΡŽΡ‚ ΠΏΡ€Π°Π²ΠΈΠ»ΡŒΠ½Ρ‹Π΅ ΠΈ Π½Π΅ΠΏΡ€Π°Π²ΠΈΠ»ΡŒΠ½Ρ‹Π΅ Π΄Ρ€ΠΎΠ±ΠΈ. Π”Ρ€ΠΎΠ±ΡŒ

называСтся ΠΏΡ€Π°Π²ΠΈΠ»ΡŒΠ½ΠΎΠΉ, Ссли Π΅Π΅ Ρ‡ΠΈΡΠ»ΠΈΡ‚Π΅Π»ΡŒ мСньшС знамСнатСля, ΠΈ Π½Π΅ΠΏΡ€Π°Π²ΠΈΠ»ΡŒΠ½ΠΎΠΉ, Ссли Π΅Π΅ Ρ‡ΠΈΡΠ»ΠΈΡ‚Π΅Π»ΡŒ большС знамСнатСля ΠΈΠ»ΠΈ Ρ€Π°Π²Π΅Π½ Π΅ΠΌΡƒ.

2. ДСйствия с ΠΎΠ±Ρ‹ΠΊΠ½ΠΎΠ²Π΅Π½Π½Ρ‹ΠΌΠΈ дробями.

2.1. Π‘Π»ΠΎΠΆΠ΅Π½ΠΈΠ΅ ΠΈ Π²Ρ‹Ρ‡ΠΈΡ‚Π°Π½ΠΈΠ΅ ΠΎΠ±Ρ‹ΠΊΠ½ΠΎΠ²Π΅Π½Π½Ρ‹Ρ… Π΄Ρ€ΠΎΠ±Π΅ΠΉ.

Π‘Π»ΠΎΠΆΠ΅Π½ΠΈΠ΅ ΠΎΠ±Ρ‹ΠΊΠ½ΠΎΠ²Π΅Π½Π½Ρ‹Ρ… Π΄Ρ€ΠΎΠ±Π΅ΠΉ выполняСтся Ρ‚Π°ΠΊ:

Π°) Ссли Π·Π½Π°ΠΌΠ΅Π½Π°Ρ‚Π΅Π»ΠΈ Π΄Ρ€ΠΎΠ±Π΅ΠΉ ΠΎΠ΄ΠΈΠ½Π°ΠΊΠΎΠ²Ρ‹, Ρ‚ΠΎ ΠΊ Ρ‡ΠΈΡΠ»ΠΈΡ‚Π΅Π»ΡŽ ΠΏΠ΅Ρ€Π²ΠΎΠΉ Π΄Ρ€ΠΎΠ±ΠΈ ΠΏΡ€ΠΈΠ±Π°Π²Π»ΡΡŽΡ‚ Ρ‡ΠΈΡΠ»ΠΈΡ‚Π΅Π»ΡŒ Π²Ρ‚ΠΎΡ€ΠΎΠΉ Π΄Ρ€ΠΎΠ±ΠΈ ΠΈ ΠΎΡΡ‚Π°Π²Π»ΡΡŽΡ‚ Ρ‚ΠΎΡ‚ ΠΆΠ΅ Π·Π½Π°ΠΌΠ΅Π½Π°Ρ‚Π΅Π»ΡŒ, Ρ‚. Π΅.

;

Π±) Ссли Π·Π½Π°ΠΌΠ΅Π½Π°Ρ‚Π΅Π»ΠΈ Π΄Ρ€ΠΎΠ±Π΅ΠΉ Ρ€Π°Π·Π»ΠΈΡ‡Π½Ρ‹, Ρ‚ΠΎ Π΄Ρ€ΠΎΠ±ΠΈ сначала приводят ΠΊ ΠΎΠ±Ρ‰Π΅ΠΌΡƒ Π·Π½Π°ΠΌΠ΅Π½Π°Ρ‚Π΅Π»ΡŽ, ΠΏΡ€Π΅Π΄ΠΏΠΎΡ‡Ρ‚ΠΈΡ‚Π΅Π»ΡŒΠ½Π΅Π΅ ΠΊ Π½Π°ΠΈΠΌΠ΅Π½ΡŒΡˆΠ΅ΠΌΡƒ, Π° Π·Π°Ρ‚Π΅ΠΌ ΠΊ Ρ‡ΠΈΡΠ»ΠΈΡ‚Π΅Π»ΡŽ ΠΏΠ΅Ρ€Π²ΠΎΠΉ Π΄Ρ€ΠΎΠ±ΠΈ ΠΏΡ€ΠΈΠ±Π°Π²Π»ΡΡŽΡ‚ Ρ‡ΠΈΡΠ»ΠΈΡ‚Π΅Π»ΡŒ Π²Ρ‚ΠΎΡ€ΠΎΠΉ Π΄Ρ€ΠΎΠ±ΠΈ, Ρ‚.Π΅.

.

Π’Ρ‹Ρ‡ΠΈΡ‚Π°Π½ΠΈΠ΅ ΠΎΠ±Ρ‹ΠΊΠ½ΠΎΠ²Π΅Π½Π½Ρ‹Ρ… Π΄Ρ€ΠΎΠ±Π΅ΠΉ Π²Ρ‹ΠΏΠΎΠ»Π½ΡΡŽΡ‚ ΡΠ»Π΅Π΄ΡƒΡŽΡ‰ΠΈΠΌ ΠΎΠ±Ρ€Π°Π·ΠΎΠΌ:

Π°) Ссли Π·Π½Π°ΠΌΠ΅Π½Π°Ρ‚Π΅Π»ΠΈ Π΄Ρ€ΠΎΠ±Π΅ΠΉ ΠΎΠ΄ΠΈΠ½Π°ΠΊΠΎΠ²Ρ‹, Ρ‚ΠΎ ΠΎΡ‚ числитСля ΠΏΠ΅Ρ€Π²ΠΎΠΉ Π΄Ρ€ΠΎΠ±ΠΈ Π²Ρ‹Ρ‡ΠΈΡ‚Π°ΡŽΡ‚ Ρ‡ΠΈΡΠ»ΠΈΡ‚Π΅Π»ΡŒ Π²Ρ‚ΠΎΡ€ΠΎΠΉ Π΄Ρ€ΠΎΠ±ΠΈ ΠΈ ΠΎΡΡ‚Π°Π²Π»ΡΡŽΡ‚ Ρ‚ΠΎΡ‚ ΠΆΠ΅ Π·Π½Π°ΠΌΠ΅Π½Π°Ρ‚Π΅Π»ΡŒ, Ρ‚.Π΅.

.

Π±) Ссли Π·Π½Π°ΠΌΠ΅Π½Π°Ρ‚Π΅Π»ΠΈ Ρ€Π°Π·Π»ΠΈΡ‡Π½Ρ‹, Ρ‚ΠΎ сначала Π΄Ρ€ΠΎΠ±ΠΈ приводят ΠΊ ΠΎΠ±Ρ‰Π΅ΠΌΡƒ Π·Π½Π°ΠΌΠ΅Π½Π°Ρ‚Π΅Π»ΡŽ, Π° Π·Π°Ρ‚Π΅ΠΌ ΠΎΡ‚ числитСля ΠΏΠ΅Ρ€Π²ΠΎΠΉ Π΄Ρ€ΠΎΠ±ΠΈ Π²Ρ‹Ρ‡ΠΈΡ‚Π°ΡŽΡ‚ Ρ‡ΠΈΡΠ»ΠΈΡ‚Π΅Π»ΡŒ Π²Ρ‚ΠΎΡ€ΠΎΠΉ Π΄Ρ€ΠΎΠ±ΠΈ, Ρ‚.Π΅.

.

Π‘Π»ΠΎΠΆΠ΅Π½ΠΈΠ΅ ΠΈ Π²Ρ‹Ρ‡ΠΈΡ‚Π°Π½ΠΈΠ΅ Π΄Ρ€ΠΎΠ±Π΅ΠΉ. Если Π·Π½Π°ΠΌΠ΅Π½Π°Ρ‚Π΅Π»ΠΈ Π΄Ρ€ΠΎΠ±Π΅ΠΉ ΠΎΠ΄ΠΈΠ½Π°ΠΊΠΎΠ²Ρ‹, Ρ‚ΠΎ для Ρ‚ΠΎΠ³ΠΎ, Ρ‡Ρ‚ΠΎΠ±Ρ‹ ΡΠ»ΠΎΠΆΠΈΡ‚ΡŒ Π΄Ρ€ΠΎΠ±ΠΈ, Π½Π°Π΄ΠΎ ΡΠ»ΠΎΠΆΠΈΡ‚ΡŒ ΠΈΡ… числитСли, Π° для Ρ‚ΠΎΠ³ΠΎ, Ρ‡Ρ‚ΠΎΠ±Ρ‹ Π²Ρ‹Ρ‡Π΅ΡΡ‚ΡŒ Π΄Ρ€ΠΎΠ±ΠΈ, Π½Π°Π΄ΠΎ Π²Ρ‹Ρ‡Π΅ΡΡ‚ΡŒ ΠΈΡ… числитСли (Π² Ρ‚ΠΎΠΌ ΠΆΠ΅ порядкС). ΠŸΠΎΠ»ΡƒΡ‡Π΅Π½Π½Π°Ρ сумма ΠΈΠ»ΠΈ Ρ€Π°Π·Π½ΠΎΡΡ‚ΡŒ Π±ΡƒΠ΄Π΅Ρ‚ числитСлСм Ρ€Π΅Π·ΡƒΠ»ΡŒΡ‚Π°Ρ‚Π°; Π·Π½Π°ΠΌΠ΅Π½Π°Ρ‚Π΅Π»ΡŒ останСтся Ρ‚Π΅ΠΌ ΠΆΠ΅.

НапримСр:

1.

2.

3.

4.

5.

6.

Если Π·Π½Π°ΠΌΠ΅Π½Π°Ρ‚Π΅Π»ΠΈ Π΄Ρ€ΠΎΠ±Π΅ΠΉ Ρ€Π°Π·Π»ΠΈΡ‡Π½Ρ‹, Π½Π΅ΠΎΠ±Ρ…ΠΎΠ΄ΠΈΠΌΠΎ сначала привСсти Π΄Ρ€ΠΎΠ±ΠΈ ΠΊ ΠΎΠ±Ρ‰Π΅ΠΌΡƒ Π·Π½Π°ΠΌΠ΅Π½Π°Ρ‚Π΅Π»ΡŽ. ΠŸΡ€ΠΈ слоТСнии ΡΠΌΠ΅ΡˆΠ°Π½Π½Ρ‹Ρ… чисСл ΠΈΡ… Ρ†Π΅Π»Ρ‹Π΅ ΠΈ Π΄Ρ€ΠΎΠ±Π½Ρ‹Π΅ части ΡΠΊΠ»Π°Π΄Ρ‹Π²Π°ΡŽΡ‚ΡΡ ΠΎΡ‚Π΄Π΅Π»ΡŒΠ½ΠΎ. ΠŸΡ€ΠΈ Π²Ρ‹Ρ‡ΠΈΡ‚Π°Π½ΠΈΠΈ ΡΠΌΠ΅ΡˆΠ°Π½Π½Ρ‹Ρ… чисСл ΠΌΡ‹ Ρ€Π΅ΠΊΠΎΠΌΠ΅Π½Π΄ΡƒΠ΅ΠΌ сначала ΠΏΡ€Π΅ΠΎΠ±Ρ€Π°Π·ΠΎΠ²Π°Ρ‚ΡŒ ΠΈΡ… ΠΊ Π²ΠΈΠ΄Ρƒ Π½Π΅ΠΏΡ€Π°Π²ΠΈΠ»ΡŒΠ½Ρ‹Ρ… Π΄Ρ€ΠΎΠ±Π΅ΠΉ, Π·Π°Ρ‚Π΅ΠΌ Π²Ρ‹Ρ‡Π΅ΡΡ‚ΡŒ ΠΈΠ· ΠΎΠ΄Π½ΠΎΠΉ Π΄Ρ€ΡƒΠ³ΡƒΡŽ, Π° послС этого вновь привСсти Ρ€Π΅Π·ΡƒΠ»ΡŒΡ‚Π°Ρ‚, Ссли трСбуСтся, ΠΊ Π²ΠΈΠ΄Ρƒ смСшанного числа.

НапримСр:

1.

2.

3.

4.

5.

6.

7.

8.

2.2. Π£ΠΌΠ½ΠΎΠΆΠ΅Π½ΠΈΠ΅ ΠΈ Π΄Π΅Π»Π΅Π½ΠΈΠ΅ ΠΎΠ±Ρ‹ΠΊΠ½ΠΎΠ²Π΅Π½Π½Ρ‹Ρ… Π΄Ρ€ΠΎΠ±Π΅ΠΉ.

Π£ΠΌΠ½ΠΎΠΆΠ΅Π½ΠΈΠ΅ ΠΎΠ±Ρ‹ΠΊΠ½ΠΎΠ²Π΅Π½Π½Ρ‹Ρ… Π΄Ρ€ΠΎΠ±Π΅ΠΉ выполняСтся ΡΠ»Π΅Π΄ΡƒΡŽΡ‰ΠΈΠΌ ΠΎΠ±Ρ€Π°Π·ΠΎΠΌ:

,

Ρ‚.Π΅. ΠΏΠ΅Ρ€Π΅ΠΌΠ½ΠΎΠΆΠ°ΡŽΡ‚ΡΡ ΠΎΡ‚Π΄Π΅Π»ΡŒΠ½ΠΎ числитСли, ΠΎΡ‚Π΄Π΅Π»ΡŒΠ½ΠΎ Π·Π½Π°ΠΌΠ΅Π½Π°Ρ‚Π΅Π»ΠΈ, ΠΏΠ΅Ρ€Π²ΠΎΠ΅ ΠΏΡ€ΠΎΠΈΠ·Π²Π΅Π΄Π΅Π½ΠΈΠ΅ Π΄Π΅Π»Π°ΡŽΡ‚ числитСлСм, Π²Ρ‚ΠΎΡ€ΠΎΠ΅ – Π·Π½Π°ΠΌΠ΅Π½Π°Ρ‚Π΅Π»Π΅ΠΌ.

ΠŸΡ€ΠΈ ΡƒΠΌΠ½ΠΎΠΆΠ΅Π½ΠΈΠΈ Π΄Ρ€ΠΎΠ±ΠΈ Π½Π° Π½Π°Ρ‚ΡƒΡ€Π°Π»ΡŒΠ½ΠΎΠ΅ число, Ρ‡ΠΈΡΠ»ΠΈΡ‚Π΅Π»ΡŒ Π΄Ρ€ΠΎΠ±ΠΈ ΡƒΠΌΠ½ΠΎΠΆΠ°ΡŽΡ‚ Π½Π° это число, Π° Π·Π½Π°ΠΌΠ΅Π½Π°Ρ‚Π΅Π»ΡŒ ΠΎΡΡ‚Π°Π²Π»ΡΡŽΡ‚ Π±Π΅Π· измСнСния.

Если ΠΌΠ½ΠΎΠΆΠΈΡ‚Π΅Π»ΠΈ ΡΠ²Π»ΡΡŽΡ‚ΡΡ ΡΠΌΠ΅ΡˆΠ°Π½Π½Ρ‹ΠΌΠΈ числами, Ρ‚ΠΎ сначала ΠΈΡ… Π½ΡƒΠΆΠ½ΠΎ Π·Π°ΠΏΠΈΡΠ°Ρ‚ΡŒ Π² Π²ΠΈΠ΄Π΅ Π½Π΅ΠΏΡ€Π°Π²ΠΈΠ»ΡŒΠ½Ρ‹Ρ… Π΄Ρ€ΠΎΠ±Π΅ΠΉ, Π·Π°Ρ‚Π΅ΠΌ Π²ΠΎΡΠΏΠΎΠ»ΡŒΠ·ΠΎΠ²Π°Ρ‚ΡŒΡΡ ΠΏΡ€Π°Π²ΠΈΠ»ΠΎΠΌ умноТСния Π΄Ρ€ΠΎΠ±Π΅ΠΉ.

Π”Π΅Π»Π΅Π½ΠΈΠ΅ ΠΎΠ±Ρ‹ΠΊΠ½ΠΎΠ²Π΅Π½Π½Ρ‹Ρ… Π΄Ρ€ΠΎΠ±Π΅ΠΉ Π²Ρ‹ΠΏΠΎΠ»Π½ΡΡŽΡ‚ ΡΠ»Π΅Π΄ΡƒΡŽΡ‰ΠΈΠΌ ΠΎΠ±Ρ€Π°Π·ΠΎΠΌ:

,

Ρ‚.Π΅. Π΄Π΅Π»ΠΈΠΌΠΎΠ΅

ΡƒΠΌΠ½ΠΎΠΆΠ°ΡŽΡ‚ Π½Π° Π΄Ρ€ΠΎΠ±ΡŒ

, ΠΎΠ±Ρ€Π°Ρ‚Π½ΡƒΡŽ Π΄Π΅Π»ΠΈΡ‚Π΅Π»ΡŽ

.

Π£ΠΌΠ½ΠΎΠΆΠ΅Π½ΠΈΠ΅ ΠΎΠ±Ρ‹ΠΊΠ½ΠΎΠ²Π΅Π½Π½ΠΎΠΉ Π΄Ρ€ΠΎΠ±ΠΈ Π½Π° Ρ†Π΅Π»ΠΎΠ΅ число.

Π§Ρ‚ΠΎΠ±Ρ‹ ΡƒΠΌΠ½ΠΎΠΆΠΈΡ‚ΡŒ Π΄Ρ€ΠΎΠ±ΡŒ Π½Π° Ρ†Π΅Π»ΠΎΠ΅ число, достаточно Ρ‡ΠΈΡΠ»ΠΈΡ‚Π΅Π»ΡŒ Π΄Ρ€ΠΎΠ±ΠΈ ΡƒΠΌΠ½ΠΎΠΆΠΈΡ‚ΡŒ Π½Π° это число, оставив ΠΏΡ€Π΅ΠΆΠ½ΠΈΠΉ Π·Π½Π°ΠΌΠ΅Π½Π°Ρ‚Π΅Π»ΡŒ.

ΠŸΡ€Π°Π²ΠΈΠ»Π° дСлСния ΠΎΠ±Ρ‹ΠΊΠ½ΠΎΠ²Π΅Π½Π½Ρ‹Ρ… Π΄Ρ€ΠΎΠ±Π΅ΠΉ с Ρ€Π°Π·Π½Ρ‹ΠΌΠΈ знамСнатСлями. Π”Ρ€ΠΎΠ±ΠΈ. Π£ΠΌΠ½ΠΎΠΆΠ΅Π½ΠΈΠ΅ ΠΈ Π΄Π΅Π»Π΅Π½ΠΈΠ΅ Π΄Ρ€ΠΎΠ±Π΅ΠΉ

Π”Ρ€ΠΎΠ±ΡŒ – это ΠΎΠ΄Π½Π° ΠΈΠ»ΠΈ Π±ΠΎΠ»Π΅Π΅ Π΄ΠΎΠ»Π΅ΠΉ Ρ†Π΅Π»ΠΎΠ³ΠΎ, Π·Π° ΠΊΠΎΡ‚ΠΎΡ€ΠΎΠ΅ ΠΎΠ±Ρ‹Ρ‡Π½ΠΎ принимаСтся Π΅Π΄ΠΈΠ½ΠΈΡ†Π° (1). Как ΠΈ с Π½Π°Ρ‚ΡƒΡ€Π°Π»ΡŒΠ½Ρ‹ΠΌΠΈ числами, с дробями ΠΌΠΎΠΆΠ½ΠΎ Π²Ρ‹ΠΏΠΎΠ»Π½ΡΡ‚ΡŒ всС основныС арифмСтичСскиС дСйствия (слоТСниС, Π²Ρ‹Ρ‡ΠΈΡ‚Π°Π½ΠΈΠ΅, Π΄Π΅Π»Π΅Π½ΠΈΠ΅, умноТСния), для этого Π½ΡƒΠΆΠ½ΠΎ Π·Π½Π°Ρ‚ΡŒ особСнности Ρ€Π°Π±ΠΎΡ‚Ρ‹ с дробями ΠΈ Ρ€Π°Π·Π»ΠΈΡ‡Π°Ρ‚ΡŒ ΠΈΡ… Π²ΠΈΠ΄Ρ‹. БущСствуСт нСсколько Π²ΠΈΠ΄ΠΎΠ² Π΄Ρ€ΠΎΠ±Π΅ΠΉ: дСсятичныС ΠΈ ΠΎΠ±Ρ‹ΠΊΠ½ΠΎΠ²Π΅Π½Π½Ρ‹Π΅, ΠΈΠ»ΠΈ простыС. Бвоя спСцифика Π΅ΡΡ‚ΡŒ Ρƒ ΠΊΠ°ΠΆΠ΄ΠΎΠ³ΠΎ Π²ΠΈΠ΄Π° Π΄Ρ€ΠΎΠ±Π΅ΠΉ, Π½ΠΎ, ΠΎΠ±ΡΡ‚ΠΎΡΡ‚Π΅Π»ΡŒΠ½ΠΎ Ρ€Π°Π·ΠΎΠ±Ρ€Π°Π²ΡˆΠΈΡΡŒ ΠΎΠ΄ΠΈΠ½ Ρ€Π°Π·, ΠΊΠ°ΠΊ с Π½ΠΈΠΌΠΈ ΠΎΠ±Ρ€Π°Ρ‰Π°Ρ‚ΡŒΡΡ, Π²Ρ‹ смоТСтС Ρ€Π΅ΡˆΠ°Ρ‚ΡŒ Π»ΡŽΠ±Ρ‹Π΅ ΠΏΡ€ΠΈΠΌΠ΅Ρ€Ρ‹ с дробями, ΠΏΠΎΡΠΊΠΎΠ»ΡŒΠΊΡƒ Π±ΡƒΠ΄Π΅Ρ‚Π΅ Π·Π½Π°Ρ‚ΡŒ основныС ΠΏΡ€ΠΈΠ½Ρ†ΠΈΠΏΡ‹ выполнСния арифмСтичСских вычислСний с дробями. Рассмотрим Π½Π° ΠΏΡ€ΠΈΠΌΠ΅Ρ€Π°Ρ… ΠΊΠ°ΠΊ Ρ€Π°Π·Π΄Π΅Π»ΠΈΡ‚ΡŒ Π΄Ρ€ΠΎΠ±ΡŒ Π½Π° Ρ†Π΅Π»ΠΎΠ΅ число, ΠΈΡΠΏΠΎΠ»ΡŒΠ·ΡƒΡ Ρ€Π°Π·Π½Ρ‹Π΅ Π²ΠΈΠ΄Ρ‹ Π΄Ρ€ΠΎΠ±Π΅ΠΉ.

Как Ρ€Π°Π·Π΄Π΅Π»ΠΈΡ‚ΡŒ ΠΏΡ€ΠΎΡΡ‚ΡƒΡŽ Π΄Ρ€ΠΎΠ±ΡŒ Π½Π° Π½Π°Ρ‚ΡƒΡ€Π°Π»ΡŒΠ½ΠΎΠ΅ число?
ΠžΠ±Ρ‹ΠΊΠ½ΠΎΠ²Π΅Π½Π½Ρ‹ΠΌΠΈ ΠΈΠ»ΠΈ простыми Π½Π°Π·Ρ‹Π²Π°ΡŽΡ‚ Π΄Ρ€ΠΎΠ±ΠΈ, Π·Π°ΠΏΠΈΡΡ‹Π²Π°ΡŽΡ‰ΠΈΠ΅ΡΡ Π² Π²ΠΈΠ΄Π΅ Ρ‚Π°ΠΊΠΎΠ³ΠΎ ΠΎΡ‚Π½ΠΎΡˆΠ΅Π½ΠΈΡ чисСл, ΠΏΡ€ΠΈ ΠΊΠΎΡ‚ΠΎΡ€ΠΎΠΌ Π²Π²Π΅Ρ€Ρ…Ρƒ Π΄Ρ€ΠΎΠ±ΠΈ указываСтся Π΄Π΅Π»ΠΈΠΌΠΎΠ΅ (Ρ‡ΠΈΡΠ»ΠΈΡ‚Π΅Π»ΡŒ), Π° Π²Π½ΠΈΠ·Ρƒ – Π΄Π΅Π»ΠΈΡ‚Π΅Π»ΡŒ (Π·Π½Π°ΠΌΠ΅Π½Π°Ρ‚Π΅Π»ΡŒ) Π΄Ρ€ΠΎΠ±ΠΈ. Как Ρ€Π°Π·Π΄Π΅Π»ΠΈΡ‚ΡŒ Ρ‚Π°ΠΊΡƒΡŽ Π΄Ρ€ΠΎΠ±ΡŒ Π½Π° Ρ†Π΅Π»ΠΎΠ΅ число? Рассмотрим Π½Π° ΠΏΡ€ΠΈΠΌΠ΅Ρ€Π΅! Допустим, Π½Π°ΠΌ Π½ΡƒΠΆΠ½ΠΎ Ρ€Π°Π·Π΄Π΅Π»ΠΈΡ‚ΡŒ 8/12 Π½Π° 2.

Для этого ΠΌΡ‹ Π΄ΠΎΠ»ΠΆΠ½Ρ‹ Π²Ρ‹ΠΏΠΎΠ»Π½ΠΈΡ‚ΡŒ ряд дСйствий:
Π’Π°ΠΊΠΈΠΌ ΠΎΠ±Ρ€Π°Π·ΠΎΠΌ, Ссли ΠΏΠ΅Ρ€Π΅Π΄ Π½Π°ΠΌΠΈ стоит Π·Π°Π΄Π°Ρ‡Π° Ρ€Π°Π·Π΄Π΅Π»ΠΈΡ‚ΡŒ Π΄Ρ€ΠΎΠ±ΡŒ Π½Π° Ρ†Π΅Π»ΠΎΠ΅ число, схСма Ρ€Π΅ΡˆΠ΅Π½ΠΈΡ Π±ΡƒΠ΄Π΅Ρ‚ Π²Ρ‹Π³Π»ΡΠ΄Π΅Ρ‚ΡŒ ΠΏΡ€ΠΈΠΌΠ΅Ρ€Π½ΠΎ Ρ‚Π°ΠΊ:

ΠŸΠΎΠ΄ΠΎΠ±Π½Ρ‹ΠΌ ΠΎΠ±Ρ€Π°Π·ΠΎΠΌ ΠΌΠΎΠΆΠ½ΠΎ Ρ€Π°Π·Π΄Π΅Π»ΠΈΡ‚ΡŒ Π»ΡŽΠ±ΡƒΡŽ ΠΎΠ±Ρ‹ΠΊΠ½ΠΎΠ²Π΅Π½Π½ΡƒΡŽ (ΠΏΡ€ΠΎΡΡ‚ΡƒΡŽ) Π΄Ρ€ΠΎΠ±ΡŒ Π½Π° Ρ†Π΅Π»ΠΎΠ΅ число.

Как Ρ€Π°Π·Π΄Π΅Π»ΠΈΡ‚ΡŒ Π΄Π΅ΡΡΡ‚ΠΈΡ‡Π½ΡƒΡŽ Π΄Ρ€ΠΎΠ±ΡŒ Π½Π° Ρ†Π΅Π»ΠΎΠ΅ число?
ДСсятичная Π΄Ρ€ΠΎΠ±ΡŒ — это такая Π΄Ρ€ΠΎΠ±ΡŒ, которая получаСтся вслСдствиС дСлСния Π΅Π΄ΠΈΠ½ΠΈΡ†Ρ‹ Π½Π° Π΄Π΅ΡΡΡ‚ΡŒ, тысячу ΠΈ Ρ‚Π°ΠΊ Π΄Π°Π»Π΅Π΅ частСй. АрифмСтичСскиС дСйствия с дСсятичными дробями Π²Ρ‹ΠΏΠΎΠ»Π½ΡΡŽΡ‚ΡΡ довольно просто.

Рассмотрим Π½Π° ΠΏΡ€ΠΈΠΌΠ΅Ρ€Π΅ ΠΊΠ°ΠΊ Ρ€Π°Π·Π΄Π΅Π»ΠΈΡ‚ΡŒ Π΄Ρ€ΠΎΠ±ΡŒ Π½Π° Ρ†Π΅Π»ΠΎΠ΅ число. Допустим, Π½Π°ΠΌ Π½ΡƒΠΆΠ½ΠΎ ΠΏΠΎΠ΄Π΅Π»ΠΈΡ‚ΡŒ Π΄Π΅ΡΡΡ‚ΠΈΡ‡Π½ΡƒΡŽ Π΄Ρ€ΠΎΠ±ΡŒ 0,925 Π½Π° Π½Π°Ρ‚ΡƒΡ€Π°Π»ΡŒΠ½ΠΎΠ΅ число 5.


Подводя ΠΈΡ‚ΠΎΠ³ΠΈ, остановимся Π½Π° Π΄Π²ΡƒΡ… основных ΠΌΠΎΠΌΠ΅Π½Ρ‚Π°Ρ…, ΠΊΠΎΡ‚ΠΎΡ€Ρ‹Π΅ Π²Π°ΠΆΠ½Ρ‹ ΠΏΡ€ΠΈ Π²Ρ‹ΠΏΠΎΠ»Π½Π΅Π½ΠΈΠΈ ΠΎΠΏΠ΅Ρ€Π°Ρ†ΠΈΠΈ дСлСния дСсятичных Π΄Ρ€ΠΎΠ±Π΅ΠΉ Π½Π° Ρ†Π΅Π»ΠΎΠ΅ число:

  • для раздСлСния дСсятичной Π΄Ρ€ΠΎΠ±ΠΈ Π½Π° Π½Π°Ρ‚ΡƒΡ€Π°Π»ΡŒΠ½ΠΎΠ΅ число ΠΏΡ€ΠΈΠΌΠ΅Π½ΡΡŽΡ‚ Π΄Π΅Π»Π΅Π½ΠΈΠ΅ Π² столбик;
  • запятая ставится Π² частном Ρ‚ΠΎΠ³Π΄Π°, ΠΊΠΎΠ³Π΄Π° Π·Π°ΠΊΠΎΠ½Ρ‡Π΅Π½ΠΎ Π΄Π΅Π»Π΅Π½ΠΈΠ΅ Ρ†Π΅Π»ΠΎΠΉ части Π΄Π΅Π»ΠΈΠΌΠΎΠ³ΠΎ.

ΠŸΡ€ΠΈΠΌΠ΅Π½ΡΡ эти простыС ΠΏΡ€Π°Π²ΠΈΠ»Π°, всСгда ΠΌΠΎΠΆΠ½ΠΎ Π±Π΅Π· особого Ρ‚Ρ€ΡƒΠ΄Π° Ρ€Π°Π·Π΄Π΅Π»ΠΈΡ‚ΡŒ Π»ΡŽΠ±ΡƒΡŽ Π΄Π΅ΡΡΡ‚ΠΈΡ‡Π½ΡƒΡŽ ΠΈΠ»ΠΈ ΠΏΡ€ΠΎΡΡ‚ΡƒΡŽ Π΄Ρ€ΠΎΠ±ΠΈ Π½Π° Ρ†Π΅Π»ΠΎΠ΅ число. Π‘ΠΎΠ΄Π΅Ρ€ΠΆΠ°Π½ΠΈΠ΅ ΡƒΡ€ΠΎΠΊΠ°

Π‘Π»ΠΎΠΆΠ΅Π½ΠΈΠ΅ Π΄Ρ€ΠΎΠ±Π΅ΠΉ с ΠΎΠ΄ΠΈΠ½Π°ΠΊΠΎΠ²Ρ‹ΠΌΠΈ знамСнатСлями

Π‘Π»ΠΎΠΆΠ΅Π½ΠΈΠ΅ Π΄Ρ€ΠΎΠ±Π΅ΠΉ Π±Ρ‹Π²Π°Π΅Ρ‚ Π΄Π²ΡƒΡ… Π²ΠΈΠ΄ΠΎΠ²:

  1. Π‘Π»ΠΎΠΆΠ΅Π½ΠΈΠ΅ Π΄Ρ€ΠΎΠ±Π΅ΠΉ с ΠΎΠ΄ΠΈΠ½Π°ΠΊΠΎΠ²Ρ‹ΠΌΠΈ знамСнатСлями
  2. Π‘Π»ΠΎΠΆΠ΅Π½ΠΈΠ΅ Π΄Ρ€ΠΎΠ±Π΅ΠΉ с Ρ€Π°Π·Π½Ρ‹ΠΌΠΈ знамСнатСлями

Π‘Π½Π°Ρ‡Π°Π»Π° ΠΈΠ·ΡƒΡ‡ΠΈΠΌ слоТСниС Π΄Ρ€ΠΎΠ±Π΅ΠΉ с ΠΎΠ΄ΠΈΠ½Π°ΠΊΠΎΠ²Ρ‹ΠΌΠΈ знамСнатСлями.

Π’ΡƒΡ‚ всё просто. Π§Ρ‚ΠΎΠ±Ρ‹ ΡΠ»ΠΎΠΆΠΈΡ‚ΡŒ Π΄Ρ€ΠΎΠ±ΠΈ с ΠΎΠ΄ΠΈΠ½Π°ΠΊΠΎΠ²Ρ‹ΠΌΠΈ знамСнатСлями, Π½ΡƒΠΆΠ½ΠΎ ΡΠ»ΠΎΠΆΠΈΡ‚ΡŒ ΠΈΡ… числитСли, Π° Π·Π½Π°ΠΌΠ΅Π½Π°Ρ‚Π΅Π»ΡŒ ΠΎΡΡ‚Π°Π²ΠΈΡ‚ΡŒ Π±Π΅Π· измСнСния. НапримСр, слоТим Π΄Ρ€ΠΎΠ±ΠΈ ΠΈ . Π‘ΠΊΠ»Π°Π΄Ρ‹Π²Π°Π΅ΠΌ числитСли, Π° Π·Π½Π°ΠΌΠ΅Π½Π°Ρ‚Π΅Π»ΡŒ оставляСм Π±Π΅Π· измСнСния:

Π­Ρ‚ΠΎΡ‚ ΠΏΡ€ΠΈΠΌΠ΅Ρ€ ΠΌΠΎΠΆΠ½ΠΎ Π»Π΅Π³ΠΊΠΎ ΠΏΠΎΠ½ΡΡ‚ΡŒ, Ссли Π²ΡΠΏΠΎΠΌΠ½ΠΈΡ‚ΡŒ ΠΏΡ€ΠΎ ΠΏΠΈΡ†Ρ†Ρƒ, которая Ρ€Π°Π·Π΄Π΅Π»Π΅Π½Π° Π½Π° Ρ‡Π΅Ρ‚Ρ‹Ρ€Π΅ части. Если ΠΊ ΠΏΠΈΡ†Ρ†Ρ‹ ΠΏΡ€ΠΈΠ±Π°Π²ΠΈΡ‚ΡŒ ΠΏΠΈΡ†Ρ†Ρ‹, Ρ‚ΠΎ получится ΠΏΠΈΡ†Ρ†Ρ‹:

ΠŸΡ€ΠΈΠΌΠ΅Ρ€ 2. Π‘Π»ΠΎΠΆΠΈΡ‚ΡŒ Π΄Ρ€ΠΎΠ±ΠΈ ΠΈ .

Π’ ΠΎΡ‚Π²Π΅Ρ‚Π΅ ΠΏΠΎΠ»ΡƒΡ‡ΠΈΠ»Π°ΡΡŒ Π½Π΅ΠΏΡ€Π°Π²ΠΈΠ»ΡŒΠ½Π°Ρ Π΄Ρ€ΠΎΠ±ΡŒ . Если наступаСт ΠΊΠΎΠ½Π΅Ρ† Π·Π°Π΄Π°Ρ‡ΠΈ, Ρ‚ΠΎ ΠΎΡ‚ Π½Π΅ΠΏΡ€Π°Π²ΠΈΠ»ΡŒΠ½Ρ‹Ρ… Π΄Ρ€ΠΎΠ±Π΅ΠΉ принято ΠΈΠ·Π±Π°Π²Π»ΡΡ‚ΡŒΡΡ. Π§Ρ‚ΠΎΠ±Ρ‹ избавится ΠΎΡ‚ Π½Π΅ΠΏΡ€Π°Π²ΠΈΠ»ΡŒΠ½ΠΎΠΉ Π΄Ρ€ΠΎΠ±ΠΈ, Π½ΡƒΠΆΠ½ΠΎ Π²Ρ‹Π΄Π΅Π»ΠΈΡ‚ΡŒ Π² Π½Π΅ΠΉ Ρ†Π΅Π»ΡƒΡŽ Ρ‡Π°ΡΡ‚ΡŒ. Π’ нашСм случаС цСлая Ρ‡Π°ΡΡ‚ΡŒ выдСляСтся Π»Π΅Π³ΠΊΠΎ β€” Π΄Π²Π° Ρ€Π°Π·Π΄Π΅Π»ΠΈΡ‚ΡŒ Π½Π° Π΄Π²Π° Ρ€Π°Π²Π½ΠΎ Π΅Π΄ΠΈΠ½ΠΈΡ†Π΅:

Π­Ρ‚ΠΎΡ‚ ΠΏΡ€ΠΈΠΌΠ΅Ρ€ ΠΌΠΎΠΆΠ½ΠΎ Π»Π΅Π³ΠΊΠΎ ΠΏΠΎΠ½ΡΡ‚ΡŒ, Ссли Π²ΡΠΏΠΎΠΌΠ½ΠΈΡ‚ΡŒ ΠΏΡ€ΠΎ ΠΏΠΈΡ†Ρ†Ρƒ, которая Ρ€Π°Π·Π΄Π΅Π»Π΅Π½Π° Π½Π° Π΄Π²Π΅ части. Если ΠΊ ΠΏΠΈΡ†Ρ†Ρ‹ ΠΏΡ€ΠΈΠ±Π°Π²ΠΈΡ‚ΡŒ Π΅Ρ‰Π΅ ΠΏΠΈΡ†Ρ†Ρ‹, Ρ‚ΠΎ получится ΠΎΠ΄Π½Π° цСлая ΠΏΠΈΡ†Ρ†Π°:

ΠŸΡ€ΠΈΠΌΠ΅Ρ€ 3 . Π‘Π»ΠΎΠΆΠΈΡ‚ΡŒ Π΄Ρ€ΠΎΠ±ΠΈ ΠΈ .

ΠžΠΏΡΡ‚ΡŒ ΠΆΠ΅ складываСм числитСли, Π° Π·Π½Π°ΠΌΠ΅Π½Π°Ρ‚Π΅Π»ΡŒ оставляСм Π±Π΅Π· измСнСния:

Π­Ρ‚ΠΎΡ‚ ΠΏΡ€ΠΈΠΌΠ΅Ρ€ ΠΌΠΎΠΆΠ½ΠΎ Π»Π΅Π³ΠΊΠΎ ΠΏΠΎΠ½ΡΡ‚ΡŒ, Ссли Π²ΡΠΏΠΎΠΌΠ½ΠΈΡ‚ΡŒ ΠΏΡ€ΠΎ ΠΏΠΈΡ†Ρ†Ρƒ, которая Ρ€Π°Π·Π΄Π΅Π»Π΅Π½Π° Π½Π° Ρ‚Ρ€ΠΈ части. Если ΠΊ ΠΏΠΈΡ†Ρ†Ρ‹ ΠΏΡ€ΠΈΠ±Π°Π²ΠΈΡ‚ΡŒ Π΅Ρ‰Ρ‘ ΠΏΠΈΡ†Ρ†Ρ‹, Ρ‚ΠΎ получится ΠΏΠΈΡ†Ρ†Ρ‹:

ΠŸΡ€ΠΈΠΌΠ΅Ρ€ 4. Найти Π·Π½Π°Ρ‡Π΅Π½ΠΈΠ΅ выраТСния

Π­Ρ‚ΠΎΡ‚ ΠΏΡ€ΠΈΠΌΠ΅Ρ€ Ρ€Π΅ΡˆΠ°Π΅Ρ‚ΡΡ Ρ‚ΠΎΡ‡Π½ΠΎ Ρ‚Π°ΠΊΠΆΠ΅, ΠΊΠ°ΠΊ ΠΈ ΠΏΡ€Π΅Π΄Ρ‹Π΄ΡƒΡ‰ΠΈΠ΅. ЧислитСли Π½Π΅ΠΎΠ±Ρ…ΠΎΠ΄ΠΈΠΌΠΎ ΡΠ»ΠΎΠΆΠΈΡ‚ΡŒ, Π° Π·Π½Π°ΠΌΠ΅Π½Π°Ρ‚Π΅Π»ΡŒ ΠΎΡΡ‚Π°Π²ΠΈΡ‚ΡŒ Π±Π΅Π· измСнСния:

ΠŸΠΎΠΏΡ€ΠΎΠ±ΡƒΠ΅ΠΌ ΠΈΠ·ΠΎΠ±Ρ€Π°Π·ΠΈΡ‚ΡŒ нашС Ρ€Π΅ΡˆΠ΅Π½ΠΈΠ΅ с ΠΏΠΎΠΌΠΎΡ‰ΡŒΡŽ рисунка. Если ΠΊ ΠΏΠΈΡ†Ρ†Ρ‹ ΠΏΡ€ΠΈΠ±Π°Π²ΠΈΡ‚ΡŒ ΠΏΠΈΡ†Ρ†Ρ‹ ΠΈ Π΅Ρ‰Ρ‘ ΠΏΡ€ΠΈΠ±Π°Π²ΠΈΡ‚ΡŒ ΠΏΠΈΡ†Ρ†Ρ‹, Ρ‚ΠΎ получится 1 цСлая ΠΈ Π΅Ρ‰Ρ‘ ΠΏΠΈΡ†Ρ†Ρ‹.

Как Π²ΠΈΠ΄ΠΈΡ‚Π΅ Π² слоТСнии Π΄Ρ€ΠΎΠ±Π΅ΠΉ с ΠΎΠ΄ΠΈΠ½Π°ΠΊΠΎΠ²Ρ‹ΠΌΠΈ знамСнатСлями Π½ΠΈΡ‡Π΅Π³ΠΎ слоТного Π½Π΅Ρ‚. Достаточно ΠΏΠΎΠ½ΠΈΠΌΠ°Ρ‚ΡŒ ΡΠ»Π΅Π΄ΡƒΡŽΡ‰ΠΈΠ΅ ΠΏΡ€Π°Π²ΠΈΠ»Π°:

  1. Π§Ρ‚ΠΎΠ±Ρ‹ ΡΠ»ΠΎΠΆΠΈΡ‚ΡŒ Π΄Ρ€ΠΎΠ±ΠΈ с ΠΎΠ΄ΠΈΠ½Π°ΠΊΠΎΠ²Ρ‹ΠΌΠΈ знамСнатСля, Π½ΡƒΠΆΠ½ΠΎ ΡΠ»ΠΎΠΆΠΈΡ‚ΡŒ ΠΈΡ… числитСли, Π° Π·Π½Π°ΠΌΠ΅Π½Π°Ρ‚Π΅Π»ΡŒ ΠΎΡΡ‚Π°Π²ΠΈΡ‚ΡŒ Π±Π΅Π· измСнСния;

Π‘Π»ΠΎΠΆΠ΅Π½ΠΈΠ΅ Π΄Ρ€ΠΎΠ±Π΅ΠΉ с Ρ€Π°Π·Π½Ρ‹ΠΌΠΈ знамСнатСлями

Π’Π΅ΠΏΠ΅Ρ€ΡŒ научимся ΡΠΊΠ»Π°Π΄Ρ‹Π²Π°Ρ‚ΡŒ Π΄Ρ€ΠΎΠ±ΠΈ с Ρ€Π°Π·Π½Ρ‹ΠΌΠΈ знамСнатСлями. Когда ΡΠΊΠ»Π°Π΄Ρ‹Π²Π°ΡŽΡ‚ Π΄Ρ€ΠΎΠ±ΠΈ, Π·Π½Π°ΠΌΠ΅Π½Π°Ρ‚Π΅Π»ΠΈ этих Π΄Ρ€ΠΎΠ±Π΅ΠΉ Π΄ΠΎΠ»ΠΆΠ½Ρ‹ Π±Ρ‹Ρ‚ΡŒ ΠΎΠ΄ΠΈΠ½Π°ΠΊΠΎΠ²Ρ‹ΠΌΠΈ. Но ΠΎΠ΄ΠΈΠ½Π°ΠΊΠΎΠ²Ρ‹ΠΌΠΈ ΠΎΠ½ΠΈ Π±Ρ‹Π²Π°ΡŽΡ‚ Π½Π΅ всСгда.

НапримСр, Π΄Ρ€ΠΎΠ±ΠΈ ΠΈ ΡΠ»ΠΎΠΆΠΈΡ‚ΡŒ ΠΌΠΎΠΆΠ½ΠΎ, ΠΏΠΎΡΠΊΠΎΠ»ΡŒΠΊΡƒ Ρƒ Π½ΠΈΡ… ΠΎΠ΄ΠΈΠ½Π°ΠΊΠΎΠ²Ρ‹Π΅ Π·Π½Π°ΠΌΠ΅Π½Π°Ρ‚Π΅Π»ΠΈ.

А Π²ΠΎΡ‚ Π΄Ρ€ΠΎΠ±ΠΈ ΠΈ сразу ΡΠ»ΠΎΠΆΠΈΡ‚ΡŒ нСльзя, ΠΏΠΎΡΠΊΠΎΠ»ΡŒΠΊΡƒ Ρƒ этих Π΄Ρ€ΠΎΠ±Π΅ΠΉ Ρ€Π°Π·Π½Ρ‹Π΅ Π·Π½Π°ΠΌΠ΅Π½Π°Ρ‚Π΅Π»ΠΈ. Π’ Ρ‚Π°ΠΊΠΈΡ… случаях Π΄Ρ€ΠΎΠ±ΠΈ Π½ΡƒΠΆΠ½ΠΎ ΠΏΡ€ΠΈΠ²ΠΎΠ΄ΠΈΡ‚ΡŒ ΠΊ ΠΎΠ΄ΠΈΠ½Π°ΠΊΠΎΠ²ΠΎΠΌΡƒ (ΠΎΠ±Ρ‰Π΅ΠΌΡƒ) Π·Π½Π°ΠΌΠ΅Π½Π°Ρ‚Π΅Π»ΡŽ.

БущСствуСт нСсколько способов привСдСния Π΄Ρ€ΠΎΠ±Π΅ΠΉ ΠΊ ΠΎΠ΄ΠΈΠ½Π°ΠΊΠΎΠ²ΠΎΠΌΡƒ Π·Π½Π°ΠΌΠ΅Π½Π°Ρ‚Π΅Π»ΡŽ. БСгодня ΠΌΡ‹ рассмотрим Ρ‚ΠΎΠ»ΡŒΠΊΠΎ ΠΎΠ΄ΠΈΠ½ ΠΈΠ· Π½ΠΈΡ…, ΠΏΠΎΡΠΊΠΎΠ»ΡŒΠΊΡƒ ΠΎΡΡ‚Π°Π»ΡŒΠ½Ρ‹Π΅ способы ΠΌΠΎΠ³ΡƒΡ‚ ΠΏΠΎΠΊΠ°Π·Π°Ρ‚ΡŒΡΡ слоТными для Π½Π°Ρ‡ΠΈΠ½Π°ΡŽΡ‰Π΅Π³ΠΎ.

Π‘ΡƒΡ‚ΡŒ этого способа Π·Π°ΠΊΠ»ΡŽΡ‡Π°Π΅Ρ‚ΡΡ Π² Ρ‚ΠΎΠΌ, Ρ‡Ρ‚ΠΎ сначала ищСтся (НОК) Π·Π½Π°ΠΌΠ΅Π½Π°Ρ‚Π΅Π»Π΅ΠΉ ΠΎΠ±Π΅ΠΈΡ… Π΄Ρ€ΠΎΠ±Π΅ΠΉ. Π—Π°Ρ‚Π΅ΠΌ НОК дСлят Π½Π° Π·Π½Π°ΠΌΠ΅Π½Π°Ρ‚Π΅Π»ΡŒ ΠΏΠ΅Ρ€Π²ΠΎΠΉ Π΄Ρ€ΠΎΠ±ΠΈ ΠΈ ΠΏΠΎΠ»ΡƒΡ‡Π°ΡŽΡ‚ ΠΏΠ΅Ρ€Π²Ρ‹ΠΉ Π΄ΠΎΠΏΠΎΠ»Π½ΠΈΡ‚Π΅Π»ΡŒΠ½Ρ‹ΠΉ ΠΌΠ½ΠΎΠΆΠΈΡ‚Π΅Π»ΡŒ. Аналогично ΠΏΠΎΡΡ‚ΡƒΠΏΠ°ΡŽΡ‚ ΠΈ со Π²Ρ‚ΠΎΡ€ΠΎΠΉ Π΄Ρ€ΠΎΠ±ΡŒΡŽ β€” НОК дСлят Π½Π° Π·Π½Π°ΠΌΠ΅Π½Π°Ρ‚Π΅Π»ΡŒ Π²Ρ‚ΠΎΡ€ΠΎΠΉ Π΄Ρ€ΠΎΠ±ΠΈ ΠΈ ΠΏΠΎΠ»ΡƒΡ‡Π°ΡŽΡ‚ Π²Ρ‚ΠΎΡ€ΠΎΠΉ Π΄ΠΎΠΏΠΎΠ»Π½ΠΈΡ‚Π΅Π»ΡŒΠ½Ρ‹ΠΉ ΠΌΠ½ΠΎΠΆΠΈΡ‚Π΅Π»ΡŒ.

Π—Π°Ρ‚Π΅ΠΌ числитСли ΠΈ Π·Π½Π°ΠΌΠ΅Π½Π°Ρ‚Π΅Π»ΠΈ Π΄Ρ€ΠΎΠ±Π΅ΠΉ ΡƒΠΌΠ½ΠΎΠΆΠ°ΡŽΡ‚ΡΡ Π½Π° свои Π΄ΠΎΠΏΠΎΠ»Π½ΠΈΡ‚Π΅Π»ΡŒΠ½Ρ‹Π΅ ΠΌΠ½ΠΎΠΆΠΈΡ‚Π΅Π»ΠΈ. Π’ Ρ€Π΅Π·ΡƒΠ»ΡŒΡ‚Π°Ρ‚Π΅ этих дСйствий, Π΄Ρ€ΠΎΠ±ΠΈ Ρƒ ΠΊΠΎΡ‚ΠΎΡ€Ρ‹Ρ… Π±Ρ‹Π»ΠΈ Ρ€Π°Π·Π½Ρ‹Π΅ Π·Π½Π°ΠΌΠ΅Π½Π°Ρ‚Π΅Π»ΠΈ, ΠΎΠ±Ρ€Π°Ρ‰Π°ΡŽΡ‚ΡΡ Π² Π΄Ρ€ΠΎΠ±ΠΈ, Ρƒ ΠΊΠΎΡ‚ΠΎΡ€Ρ‹Ρ… ΠΎΠ΄ΠΈΠ½Π°ΠΊΠΎΠ²Ρ‹Π΅ Π·Π½Π°ΠΌΠ΅Π½Π°Ρ‚Π΅Π»ΠΈ. А ΠΊΠ°ΠΊ ΡΠΊΠ»Π°Π΄Ρ‹Π²Π°Ρ‚ΡŒ Ρ‚Π°ΠΊΠΈΠ΅ Π΄Ρ€ΠΎΠ±ΠΈ ΠΌΡ‹ ΡƒΠΆΠ΅ Π·Π½Π°Π΅ΠΌ.

ΠŸΡ€ΠΈΠΌΠ΅Ρ€ 1 . Π‘Π»ΠΎΠΆΠΈΠΌ Π΄Ρ€ΠΎΠ±ΠΈ ΠΈ

Π’ ΠΏΠ΅Ρ€Π²ΡƒΡŽ ΠΎΡ‡Π΅Ρ€Π΅Π΄ΡŒ Π½Π°Ρ…ΠΎΠ΄ΠΈΠΌ наимСньшСС ΠΎΠ±Ρ‰Π΅Π΅ ΠΊΡ€Π°Ρ‚Π½ΠΎΠ΅ Π·Π½Π°ΠΌΠ΅Π½Π°Ρ‚Π΅Π»Π΅ΠΉ ΠΎΠ±Π΅ΠΈΡ… Π΄Ρ€ΠΎΠ±Π΅ΠΉ. Π—Π½Π°ΠΌΠ΅Π½Π°Ρ‚Π΅Π»ΡŒ ΠΏΠ΅Ρ€Π²ΠΎΠΉ Π΄Ρ€ΠΎΠ±ΠΈ это число 3, Π° Π·Π½Π°ΠΌΠ΅Π½Π°Ρ‚Π΅Π»ΡŒ Π²Ρ‚ΠΎΡ€ΠΎΠΉ Π΄Ρ€ΠΎΠ±ΠΈ β€” число 2. НаимСньшСС ΠΎΠ±Ρ‰Π΅Π΅ ΠΊΡ€Π°Ρ‚Π½ΠΎΠ΅ этих чисСл Ρ€Π°Π²Π½ΠΎ 6

НОК (2 и 3) = 6

Π’Π΅ΠΏΠ΅Ρ€ΡŒ возвращаСмся ΠΊ дробям ΠΈ . Π‘Π½Π°Ρ‡Π°Π»Π° Ρ€Π°Π·Π΄Π΅Π»ΠΈΠΌ НОК Π½Π° Π·Π½Π°ΠΌΠ΅Π½Π°Ρ‚Π΅Π»ΡŒ ΠΏΠ΅Ρ€Π²ΠΎΠΉ Π΄Ρ€ΠΎΠ±ΠΈ ΠΈ ΠΏΠΎΠ»ΡƒΡ‡ΠΈΠΌ ΠΏΠ΅Ρ€Π²Ρ‹ΠΉ Π΄ΠΎΠΏΠΎΠ»Π½ΠΈΡ‚Π΅Π»ΡŒΠ½Ρ‹ΠΉ ΠΌΠ½ΠΎΠΆΠΈΡ‚Π΅Π»ΡŒ. НОК это число 6, Π° Π·Π½Π°ΠΌΠ΅Π½Π°Ρ‚Π΅Π»ΡŒ ΠΏΠ΅Ρ€Π²ΠΎΠΉ Π΄Ρ€ΠΎΠ±ΠΈ это число 3. Π”Π΅Π»ΠΈΠΌ 6 Π½Π° 3, ΠΏΠΎΠ»ΡƒΡ‡Π°Π΅ΠΌ 2.

ΠŸΠΎΠ»ΡƒΡ‡Π΅Π½Π½ΠΎΠ΅ число 2 это ΠΏΠ΅Ρ€Π²Ρ‹ΠΉ Π΄ΠΎΠΏΠΎΠ»Π½ΠΈΡ‚Π΅Π»ΡŒΠ½Ρ‹ΠΉ ΠΌΠ½ΠΎΠΆΠΈΡ‚Π΅Π»ΡŒ. ЗаписываСм Π΅Π³ΠΎ ΠΊ ΠΏΠ΅Ρ€Π²ΠΎΠΉ Π΄Ρ€ΠΎΠ±ΠΈ. Для этого Π΄Π΅Π»Π°Π΅ΠΌ Π½Π΅Π±ΠΎΠ»ΡŒΡˆΡƒΡŽ ΠΊΠΎΡΡƒΡŽ линию Π½Π°Π΄ Π΄Ρ€ΠΎΠ±ΡŒΡŽ ΠΈ записываСм Π½Π°Π΄ Π½Π΅ΠΉ Π½Π°ΠΉΠ΄Π΅Π½Π½Ρ‹ΠΉ Π΄ΠΎΠΏΠΎΠ»Π½ΠΈΡ‚Π΅Π»ΡŒΠ½Ρ‹ΠΉ ΠΌΠ½ΠΎΠΆΠΈΡ‚Π΅Π»ΡŒ:

Аналогично поступаСм ΠΈ со Π²Ρ‚ΠΎΡ€ΠΎΠΉ Π΄Ρ€ΠΎΠ±ΡŒΡŽ. Π”Π΅Π»ΠΈΠΌ НОК Π½Π° Π·Π½Π°ΠΌΠ΅Π½Π°Ρ‚Π΅Π»ΡŒ Π²Ρ‚ΠΎΡ€ΠΎΠΉ Π΄Ρ€ΠΎΠ±ΠΈ ΠΈ ΠΏΠΎΠ»ΡƒΡ‡Π°Π΅ΠΌ Π²Ρ‚ΠΎΡ€ΠΎΠΉ Π΄ΠΎΠΏΠΎΠ»Π½ΠΈΡ‚Π΅Π»ΡŒΠ½Ρ‹ΠΉ ΠΌΠ½ΠΎΠΆΠΈΡ‚Π΅Π»ΡŒ. НОК это число 6, Π° Π·Π½Π°ΠΌΠ΅Π½Π°Ρ‚Π΅Π»ΡŒ Π²Ρ‚ΠΎΡ€ΠΎΠΉ Π΄Ρ€ΠΎΠ±ΠΈ β€” число 2. Π”Π΅Π»ΠΈΠΌ 6 Π½Π° 2, ΠΏΠΎΠ»ΡƒΡ‡Π°Π΅ΠΌ 3.

ΠŸΠΎΠ»ΡƒΡ‡Π΅Π½Π½ΠΎΠ΅ число 3 это Π²Ρ‚ΠΎΡ€ΠΎΠΉ Π΄ΠΎΠΏΠΎΠ»Π½ΠΈΡ‚Π΅Π»ΡŒΠ½Ρ‹ΠΉ ΠΌΠ½ΠΎΠΆΠΈΡ‚Π΅Π»ΡŒ. ЗаписываСм Π΅Π³ΠΎ ΠΊΠΎ Π²Ρ‚ΠΎΡ€ΠΎΠΉ Π΄Ρ€ΠΎΠ±ΠΈ. ΠžΠΏΡΡ‚ΡŒ ΠΆΠ΅ Π΄Π΅Π»Π°Π΅ΠΌ Π½Π΅Π±ΠΎΠ»ΡŒΡˆΡƒΡŽ ΠΊΠΎΡΡƒΡŽ линию Π½Π°Π΄ Π²Ρ‚ΠΎΡ€ΠΎΠΉ Π΄Ρ€ΠΎΠ±ΡŒΡŽ ΠΈ записываСм Π½Π°Π΄ Π½Π΅ΠΉ Π½Π°ΠΉΠ΄Π΅Π½Π½Ρ‹ΠΉ Π΄ΠΎΠΏΠΎΠ»Π½ΠΈΡ‚Π΅Π»ΡŒΠ½Ρ‹ΠΉ ΠΌΠ½ΠΎΠΆΠΈΡ‚Π΅Π»ΡŒ:

Π’Π΅ΠΏΠ΅Ρ€ΡŒ Ρƒ нас всё Π³ΠΎΡ‚ΠΎΠ²ΠΎ для слоТСния. ΠžΡΡ‚Π°Π»ΠΎΡΡŒ ΡƒΠΌΠ½ΠΎΠΆΠΈΡ‚ΡŒ числитСли ΠΈ Π·Π½Π°ΠΌΠ΅Π½Π°Ρ‚Π΅Π»ΠΈ Π΄Ρ€ΠΎΠ±Π΅ΠΉ Π½Π° свои Π΄ΠΎΠΏΠΎΠ»Π½ΠΈΡ‚Π΅Π»ΡŒΠ½Ρ‹Π΅ ΠΌΠ½ΠΎΠΆΠΈΡ‚Π΅Π»ΠΈ:

ΠŸΠΎΡΠΌΠΎΡ‚Ρ€ΠΈΡ‚Π΅ Π²Π½ΠΈΠΌΠ°Ρ‚Π΅Π»ΡŒΠ½ΠΎ ΠΊ Ρ‡Π΅ΠΌΡƒ ΠΌΡ‹ ΠΏΡ€ΠΈΡˆΠ»ΠΈ. ΠœΡ‹ ΠΏΡ€ΠΈΡˆΠ»ΠΈ ΠΊ Ρ‚ΠΎΠΌΡƒ, Ρ‡Ρ‚ΠΎ Π΄Ρ€ΠΎΠ±ΠΈ Ρƒ ΠΊΠΎΡ‚ΠΎΡ€Ρ‹Ρ… Π±Ρ‹Π»ΠΈ Ρ€Π°Π·Π½Ρ‹Π΅ Π·Π½Π°ΠΌΠ΅Π½Π°Ρ‚Π΅Π»ΠΈ, ΠΏΡ€Π΅Π²Ρ€Π°Ρ‚ΠΈΠ»ΠΈΡΡŒ Π² Π΄Ρ€ΠΎΠ±ΠΈ Ρƒ ΠΊΠΎΡ‚ΠΎΡ€Ρ‹Ρ… ΠΎΠ΄ΠΈΠ½Π°ΠΊΠΎΠ²Ρ‹Π΅ Π·Π½Π°ΠΌΠ΅Π½Π°Ρ‚Π΅Π»ΠΈ. А ΠΊΠ°ΠΊ ΡΠΊΠ»Π°Π΄Ρ‹Π²Π°Ρ‚ΡŒ Ρ‚Π°ΠΊΠΈΠ΅ Π΄Ρ€ΠΎΠ±ΠΈ ΠΌΡ‹ ΡƒΠΆΠ΅ Π·Π½Π°Π΅ΠΌ. Π”Π°Π²Π°ΠΉΡ‚Π΅ Π΄ΠΎΡ€Π΅ΡˆΠ°Π΅ΠΌ этот ΠΏΡ€ΠΈΠΌΠ΅Ρ€ Π΄ΠΎ ΠΊΠΎΠ½Ρ†Π°:

Π’Π°ΠΊΠΈΠΌ ΠΎΠ±Ρ€Π°Π·ΠΎΠΌ, ΠΏΡ€ΠΈΠΌΠ΅Ρ€ Π·Π°Π²Π΅Ρ€ΡˆΠ°Π΅Ρ‚ΡΡ. К ΠΏΡ€ΠΈΠ±Π°Π²ΠΈΡ‚ΡŒ получаСтся .

ΠŸΠΎΠΏΡ€ΠΎΠ±ΡƒΠ΅ΠΌ ΠΈΠ·ΠΎΠ±Ρ€Π°Π·ΠΈΡ‚ΡŒ нашС Ρ€Π΅ΡˆΠ΅Π½ΠΈΠ΅ с ΠΏΠΎΠΌΠΎΡ‰ΡŒΡŽ рисунка. Если ΠΊ ΠΏΠΈΡ†Ρ†Ρ‹ ΠΏΡ€ΠΈΠ±Π°Π²ΠΈΡ‚ΡŒ ΠΏΠΈΡ†Ρ†Ρ‹, Ρ‚ΠΎ получится ΠΎΠ΄Π½Π° цСлая ΠΏΠΈΡ†Ρ†Π° ΠΈ Π΅Ρ‰Π΅ ΠΎΠ΄Π½Π° ΡˆΠ΅ΡΡ‚Π°Ρ ΠΏΠΈΡ†Ρ†Ρ‹:

ΠŸΡ€ΠΈΠ²Π΅Π΄Π΅Π½ΠΈΠ΅ Π΄Ρ€ΠΎΠ±Π΅ΠΉ ΠΊ ΠΎΠ΄ΠΈΠ½Π°ΠΊΠΎΠ²ΠΎΠΌΡƒ (ΠΎΠ±Ρ‰Π΅ΠΌΡƒ) Π·Π½Π°ΠΌΠ΅Π½Π°Ρ‚Π΅Π»ΡŽ Ρ‚Π°ΠΊΠΆΠ΅ ΠΌΠΎΠΆΠ½ΠΎ ΠΈΠ·ΠΎΠ±Ρ€Π°Π·ΠΈΡ‚ΡŒ с ΠΏΠΎΠΌΠΎΡ‰ΡŒΡŽ рисунка. ΠŸΡ€ΠΈΠ²Π΅Π΄Ρ Π΄Ρ€ΠΎΠ±ΠΈ ΠΈ ΠΊ ΠΎΠ±Ρ‰Π΅ΠΌΡƒ Π·Π½Π°ΠΌΠ΅Π½Π°Ρ‚Π΅Π»ΡŽ, ΠΌΡ‹ ΠΏΠΎΠ»ΡƒΡ‡ΠΈΠ»ΠΈ Π΄Ρ€ΠΎΠ±ΠΈ ΠΈ . Π­Ρ‚ΠΈ Π΄Π²Π΅ Π΄Ρ€ΠΎΠ±ΠΈ Π±ΡƒΠ΄ΡƒΡ‚ ΠΈΠ·ΠΎΠ±Ρ€Π°ΠΆΠ°Ρ‚ΡŒΡΡ Ρ‚Π΅ΠΌΠΈ ΠΆΠ΅ кусками ΠΏΠΈΡ†Ρ†. Π Π°Π·Π»ΠΈΡ‡ΠΈΠ΅ Π±ΡƒΠ΄Π΅Ρ‚ лишь Π² Ρ‚ΠΎΠΌ, Ρ‡Ρ‚ΠΎ Π² этот Ρ€Π°Π· ΠΎΠ½ΠΈ Π±ΡƒΠ΄ΡƒΡ‚ Ρ€Π°Π·Π΄Π΅Π»Π΅Π½Ρ‹ Π½Π° ΠΎΠ΄ΠΈΠ½Π°ΠΊΠΎΠ²Ρ‹Π΅ Π΄ΠΎΠ»ΠΈ (ΠΏΡ€ΠΈΠ²Π΅Π΄Π΅Π½Ρ‹ ΠΊ ΠΎΠ΄ΠΈΠ½Π°ΠΊΠΎΠ²ΠΎΠΌΡƒ Π·Π½Π°ΠΌΠ΅Π½Π°Ρ‚Π΅Π»ΡŽ).

ΠŸΠ΅Ρ€Π²Ρ‹ΠΉ рисунок ΠΈΠ·ΠΎΠ±Ρ€Π°ΠΆΠ°Π΅Ρ‚ Π΄Ρ€ΠΎΠ±ΡŒ (Ρ‡Π΅Ρ‚Ρ‹Ρ€Π΅ кусочка ΠΈΠ· ΡˆΠ΅ΡΡ‚ΠΈ), Π° Π²Ρ‚ΠΎΡ€ΠΎΠΉ рисунок ΠΈΠ·ΠΎΠ±Ρ€Π°ΠΆΠ°Π΅Ρ‚ Π΄Ρ€ΠΎΠ±ΡŒ (Ρ‚Ρ€ΠΈ кусочка ΠΈΠ· ΡˆΠ΅ΡΡ‚ΠΈ). Π‘Π»ΠΎΠΆΠΈΠ² эти кусочки ΠΌΡ‹ ΠΏΠΎΠ»ΡƒΡ‡Π°Π΅ΠΌ (сСмь кусочков ΠΈΠ· ΡˆΠ΅ΡΡ‚ΠΈ). Π­Ρ‚Π° Π΄Ρ€ΠΎΠ±ΡŒ Π½Π΅ΠΏΡ€Π°Π²ΠΈΠ»ΡŒΠ½Π°Ρ, поэтому ΠΌΡ‹ Π²Ρ‹Π΄Π΅Π»ΠΈΠ»ΠΈ Π² Π½Π΅ΠΉ Ρ†Π΅Π»ΡƒΡŽ Ρ‡Π°ΡΡ‚ΡŒ. Π’ Ρ€Π΅Π·ΡƒΠ»ΡŒΡ‚Π°Ρ‚Π΅ ΠΏΠΎΠ»ΡƒΡ‡ΠΈΠ»ΠΈ (ΠΎΠ΄Π½Ρƒ Ρ†Π΅Π»ΡƒΡŽ ΠΏΠΈΡ†Ρ†Ρƒ ΠΈ Π΅Ρ‰Π΅ ΠΎΠ΄Π½Ρƒ ΡˆΠ΅ΡΡ‚ΡƒΡŽ ΠΏΠΈΡ†Ρ†Ρ‹).

ΠžΡ‚ΠΌΠ΅Ρ‚ΠΈΠΌ, Ρ‡Ρ‚ΠΎ ΠΌΡ‹ с Π²Π°ΠΌΠΈ расписали Π΄Π°Π½Π½Ρ‹ΠΉ ΠΏΡ€ΠΈΠΌΠ΅Ρ€ слишком ΠΏΠΎΠ΄Ρ€ΠΎΠ±Π½ΠΎ. Π’ ΡƒΡ‡Π΅Π±Π½Ρ‹Ρ… завСдСниях Π½Π΅ принято ΠΏΠΈΡΠ°Ρ‚ΡŒ Ρ‚Π°ΠΊ Ρ€Π°Π·Π²Ρ‘Ρ€Π½ΡƒΡ‚ΠΎ. НуТно ΡƒΠΌΠ΅Ρ‚ΡŒ быстро Π½Π°Ρ…ΠΎΠ΄ΠΈΡ‚ΡŒ НОК ΠΎΠ±ΠΎΠΈΡ… Π·Π½Π°ΠΌΠ΅Π½Π°Ρ‚Π΅Π»Π΅ΠΉ ΠΈ Π΄ΠΎΠΏΠΎΠ»Π½ΠΈΡ‚Π΅Π»ΡŒΠ½Ρ‹Π΅ ΠΌΠ½ΠΎΠΆΠΈΡ‚Π΅Π»ΠΈ ΠΊ Π½ΠΈΠΌ, Π° Ρ‚Π°ΠΊΠΆΠ΅ быстро ΡƒΠΌΠ½ΠΎΠΆΠ°Ρ‚ΡŒ Π½Π°ΠΉΠ΄Π΅Π½Π½Ρ‹Π΅ Π΄ΠΎΠΏΠΎΠ»Π½ΠΈΡ‚Π΅Π»ΡŒΠ½Ρ‹Π΅ ΠΌΠ½ΠΎΠΆΠΈΡ‚Π΅Π»ΠΈ Π½Π° свои числитСли ΠΈ Π·Π½Π°ΠΌΠ΅Π½Π°Ρ‚Π΅Π»ΠΈ. ΠΠ°Ρ…ΠΎΠ΄ΡΡΡŒ Π² школС, Π΄Π°Π½Π½Ρ‹ΠΉ ΠΏΡ€ΠΈΠΌΠ΅Ρ€ Π½Π°ΠΌ ΠΏΡ€ΠΈΡˆΠ»ΠΎΡΡŒ Π±Ρ‹ Π·Π°ΠΏΠΈΡΠ°Ρ‚ΡŒ ΡΠ»Π΅Π΄ΡƒΡŽΡ‰ΠΈΠΌ ΠΎΠ±Ρ€Π°Π·ΠΎΠΌ:

Но Π΅ΡΡ‚ΡŒ ΠΈ обратная сторона ΠΌΠ΅Π΄Π°Π»ΠΈ. Если Π½Π° ΠΏΠ΅Ρ€Π²Ρ‹Ρ… этапах изучСния ΠΌΠ°Ρ‚Π΅ΠΌΠ°Ρ‚ΠΈΠΊΠΈ Π½Π΅ Π΄Π΅Π»Π°Ρ‚ΡŒ ΠΏΠΎΠ΄Ρ€ΠΎΠ±Π½Ρ‹Ρ… записСй, Ρ‚ΠΎ Π½Π°Ρ‡ΠΈΠ½Π°ΡŽΡ‚ ΠΏΠΎΡΠ²Π»ΡΡ‚ΡŒΡΡ вопросы Ρ€ΠΎΠ΄Π° Β«Π° ΠΎΡ‚ΠΊΡƒΠ΄Π° Π²ΠΎΠ½ Ρ‚Π° Ρ†ΠΈΡ„Ρ€Π°?Β», Β«ΠΏΠΎΡ‡Π΅ΠΌΡƒ Π΄Ρ€ΠΎΠ±ΠΈ Π²Π΄Ρ€ΡƒΠ³ ΠΏΡ€Π΅Π²Ρ€Π°Ρ‰Π°ΡŽΡ‚ΡΡ совсСм Π² Π΄Ρ€ΡƒΠ³ΠΈΠ΅ Π΄Ρ€ΠΎΠ±ΠΈ? Β«.

Π§Ρ‚ΠΎΠ±Ρ‹ Π»Π΅Π³Ρ‡Π΅ Π±Ρ‹Π»ΠΎ ΡΠΊΠ»Π°Π΄Ρ‹Π²Π°Ρ‚ΡŒ Π΄Ρ€ΠΎΠ±ΠΈ с Ρ€Π°Π·Π½Ρ‹ΠΌΠΈ знамСнатСлями, ΠΌΠΎΠΆΠ½ΠΎ Π²ΠΎΡΠΏΠΎΠ»ΡŒΠ·ΠΎΠ²Π°Ρ‚ΡŒΡΡ ΡΠ»Π΅Π΄ΡƒΡŽΡ‰Π΅ΠΉ пошаговой инструкциСй:

  1. Найти НОК Π·Π½Π°ΠΌΠ΅Π½Π°Ρ‚Π΅Π»Π΅ΠΉ Π΄Ρ€ΠΎΠ±Π΅ΠΉ;
  2. Π Π°Π·Π΄Π΅Π»ΠΈΡ‚ΡŒ НОК Π½Π° Π·Π½Π°ΠΌΠ΅Π½Π°Ρ‚Π΅Π»ΡŒ ΠΊΠ°ΠΆΠ΄ΠΎΠΉ Π΄Ρ€ΠΎΠ±ΠΈ ΠΈ ΠΏΠΎΠ»ΡƒΡ‡ΠΈΡ‚ΡŒ Π΄ΠΎΠΏΠΎΠ»Π½ΠΈΡ‚Π΅Π»ΡŒΠ½Ρ‹ΠΉ ΠΌΠ½ΠΎΠΆΠΈΡ‚Π΅Π»ΡŒ для ΠΊΠ°ΠΆΠ΄ΠΎΠΉ Π΄Ρ€ΠΎΠ±ΠΈ;
  3. Π£ΠΌΠ½ΠΎΠΆΠΈΡ‚ΡŒ числитСли ΠΈ Π·Π½Π°ΠΌΠ΅Π½Π°Ρ‚Π΅Π»ΠΈ Π΄Ρ€ΠΎΠ±Π΅ΠΉ Π½Π° свои Π΄ΠΎΠΏΠΎΠ»Π½ΠΈΡ‚Π΅Π»ΡŒΠ½Ρ‹Π΅ ΠΌΠ½ΠΎΠΆΠΈΡ‚Π΅Π»ΠΈ;
  4. Π‘Π»ΠΎΠΆΠΈΡ‚ΡŒ Π΄Ρ€ΠΎΠ±ΠΈ, Ρƒ ΠΊΠΎΡ‚ΠΎΡ€Ρ‹Ρ… ΠΎΠ΄ΠΈΠ½Π°ΠΊΠΎΠ²Ρ‹Π΅ Π·Π½Π°ΠΌΠ΅Π½Π°Ρ‚Π΅Π»ΠΈ;
  5. Если Π² ΠΎΡ‚Π²Π΅Ρ‚Π΅ ΠΏΠΎΠ»ΡƒΡ‡ΠΈΠ»Π°ΡΡŒ Π½Π΅ΠΏΡ€Π°Π²ΠΈΠ»ΡŒΠ½Π°Ρ Π΄Ρ€ΠΎΠ±ΡŒ, Ρ‚ΠΎ Π²Ρ‹Π΄Π΅Π»ΠΈΡ‚ΡŒ Π΅Ρ‘ Ρ†Π΅Π»ΡƒΡŽ Ρ‡Π°ΡΡ‚ΡŒ;

ΠŸΡ€ΠΈΠΌΠ΅Ρ€ 2. Найти Π·Π½Π°Ρ‡Π΅Π½ΠΈΠ΅ выраТСния .

Π’ΠΎΡΠΏΠΎΠ»ΡŒΠ·ΡƒΠ΅ΠΌΡΡ инструкциСй, которая ΠΏΡ€ΠΈΠ²Π΅Π΄Π΅Π½Π° Π²Ρ‹ΡˆΠ΅.

Π¨Π°Π³ 1. Найти НОК Π·Π½Π°ΠΌΠ΅Π½Π°Ρ‚Π΅Π»Π΅ΠΉ Π΄Ρ€ΠΎΠ±Π΅ΠΉ

Находим НОК Π·Π½Π°ΠΌΠ΅Π½Π°Ρ‚Π΅Π»Π΅ΠΉ ΠΎΠ±Π΅ΠΈΡ… Π΄Ρ€ΠΎΠ±Π΅ΠΉ. Π—Π½Π°ΠΌΠ΅Π½Π°Ρ‚Π΅Π»ΠΈ Π΄Ρ€ΠΎΠ±Π΅ΠΉ это числа 2, 3 ΠΈ 4

Π¨Π°Π³ 2. Π Π°Π·Π΄Π΅Π»ΠΈΡ‚ΡŒ НОК Π½Π° Π·Π½Π°ΠΌΠ΅Π½Π°Ρ‚Π΅Π»ΡŒ ΠΊΠ°ΠΆΠ΄ΠΎΠΉ Π΄Ρ€ΠΎΠ±ΠΈ ΠΈ ΠΏΠΎΠ»ΡƒΡ‡ΠΈΡ‚ΡŒ Π΄ΠΎΠΏΠΎΠ»Π½ΠΈΡ‚Π΅Π»ΡŒΠ½Ρ‹ΠΉ ΠΌΠ½ΠΎΠΆΠΈΡ‚Π΅Π»ΡŒ для ΠΊΠ°ΠΆΠ΄ΠΎΠΉ Π΄Ρ€ΠΎΠ±ΠΈ

Π”Π΅Π»ΠΈΠΌ НОК Π½Π° Π·Π½Π°ΠΌΠ΅Π½Π°Ρ‚Π΅Π»ΡŒ ΠΏΠ΅Ρ€Π²ΠΎΠΉ Π΄Ρ€ΠΎΠ±ΠΈ. НОК это число 12, Π° Π·Π½Π°ΠΌΠ΅Π½Π°Ρ‚Π΅Π»ΡŒ ΠΏΠ΅Ρ€Π²ΠΎΠΉ Π΄Ρ€ΠΎΠ±ΠΈ это число 2. Π”Π΅Π»ΠΈΠΌ 12 Π½Π° 2, ΠΏΠΎΠ»ΡƒΡ‡Π°Π΅ΠΌ 6. ΠŸΠΎΠ»ΡƒΡ‡ΠΈΠ»ΠΈ ΠΏΠ΅Ρ€Π²Ρ‹ΠΉ Π΄ΠΎΠΏΠΎΠ»Π½ΠΈΡ‚Π΅Π»ΡŒΠ½Ρ‹ΠΉ ΠΌΠ½ΠΎΠΆΠΈΡ‚Π΅Π»ΡŒ 6. ЗаписываСм Π΅Π³ΠΎ Π½Π°Π΄ ΠΏΠ΅Ρ€Π²ΠΎΠΉ Π΄Ρ€ΠΎΠ±ΡŒΡŽ:

Π’Π΅ΠΏΠ΅Ρ€ΡŒ Π΄Π΅Π»ΠΈΠΌ НОК Π½Π° Π·Π½Π°ΠΌΠ΅Π½Π°Ρ‚Π΅Π»ΡŒ Π²Ρ‚ΠΎΡ€ΠΎΠΉ Π΄Ρ€ΠΎΠ±ΠΈ. НОК это число 12, Π° Π·Π½Π°ΠΌΠ΅Π½Π°Ρ‚Π΅Π»ΡŒ Π²Ρ‚ΠΎΡ€ΠΎΠΉ Π΄Ρ€ΠΎΠ±ΠΈ это число 3. Π”Π΅Π»ΠΈΠΌ 12 Π½Π° 3, ΠΏΠΎΠ»ΡƒΡ‡Π°Π΅ΠΌ 4. ΠŸΠΎΠ»ΡƒΡ‡ΠΈΠ»ΠΈ Π²Ρ‚ΠΎΡ€ΠΎΠΉ Π΄ΠΎΠΏΠΎΠ»Π½ΠΈΡ‚Π΅Π»ΡŒΠ½Ρ‹ΠΉ ΠΌΠ½ΠΎΠΆΠΈΡ‚Π΅Π»ΡŒ 4. ЗаписываСм Π΅Π³ΠΎ Π½Π°Π΄ Π²Ρ‚ΠΎΡ€ΠΎΠΉ Π΄Ρ€ΠΎΠ±ΡŒΡŽ:

Π’Π΅ΠΏΠ΅Ρ€ΡŒ Π΄Π΅Π»ΠΈΠΌ НОК Π½Π° Π·Π½Π°ΠΌΠ΅Π½Π°Ρ‚Π΅Π»ΡŒ Ρ‚Ρ€Π΅Ρ‚ΡŒΠ΅ΠΉ Π΄Ρ€ΠΎΠ±ΠΈ. НОК это число 12, Π° Π·Π½Π°ΠΌΠ΅Π½Π°Ρ‚Π΅Π»ΡŒ Ρ‚Ρ€Π΅Ρ‚ΡŒΠ΅ΠΉ Π΄Ρ€ΠΎΠ±ΠΈ это число 4. Π”Π΅Π»ΠΈΠΌ 12 Π½Π° 4, ΠΏΠΎΠ»ΡƒΡ‡Π°Π΅ΠΌ 3. ΠŸΠΎΠ»ΡƒΡ‡ΠΈΠ»ΠΈ Ρ‚Ρ€Π΅Ρ‚ΠΈΠΉ Π΄ΠΎΠΏΠΎΠ»Π½ΠΈΡ‚Π΅Π»ΡŒΠ½Ρ‹ΠΉ ΠΌΠ½ΠΎΠΆΠΈΡ‚Π΅Π»ΡŒ 3. ЗаписываСм Π΅Π³ΠΎ Π½Π°Π΄ Ρ‚Ρ€Π΅Ρ‚ΡŒΠ΅ΠΉ Π΄Ρ€ΠΎΠ±ΡŒΡŽ:

Π¨Π°Π³ 3. Π£ΠΌΠ½ΠΎΠΆΠΈΡ‚ΡŒ числитСли ΠΈ Π·Π½Π°ΠΌΠ΅Π½Π°Ρ‚Π΅Π»ΠΈ Π΄Ρ€ΠΎΠ±Π΅ΠΉ Π½Π° свои Π΄ΠΎΠΏΠΎΠ»Π½ΠΈΡ‚Π΅Π»ΡŒΠ½Ρ‹Π΅ ΠΌΠ½ΠΎΠΆΠΈΡ‚Π΅Π»ΠΈ

Π£ΠΌΠ½ΠΎΠΆΠ°Π΅ΠΌ числитСли ΠΈ Π·Π½Π°ΠΌΠ΅Π½Π°Ρ‚Π΅Π»ΠΈ Π½Π° свои Π΄ΠΎΠΏΠΎΠ»Π½ΠΈΡ‚Π΅Π»ΡŒΠ½Ρ‹Π΅ ΠΌΠ½ΠΎΠΆΠΈΡ‚Π΅Π»ΠΈ:

Π¨Π°Π³ 4. Π‘Π»ΠΎΠΆΠΈΡ‚ΡŒ Π΄Ρ€ΠΎΠ±ΠΈ Ρƒ ΠΊΠΎΡ‚ΠΎΡ€Ρ‹Ρ… ΠΎΠ΄ΠΈΠ½Π°ΠΊΠΎΠ²Ρ‹Π΅ Π·Π½Π°ΠΌΠ΅Π½Π°Ρ‚Π΅Π»ΠΈ

ΠœΡ‹ ΠΏΡ€ΠΈΡˆΠ»ΠΈ ΠΊ Ρ‚ΠΎΠΌΡƒ, Ρ‡Ρ‚ΠΎ Π΄Ρ€ΠΎΠ±ΠΈ Ρƒ ΠΊΠΎΡ‚ΠΎΡ€Ρ‹Ρ… Π±Ρ‹Π»ΠΈ Ρ€Π°Π·Π½Ρ‹Π΅ Π·Π½Π°ΠΌΠ΅Π½Π°Ρ‚Π΅Π»ΠΈ, ΠΏΡ€Π΅Π²Ρ€Π°Ρ‚ΠΈΠ»ΠΈΡΡŒ Π² Π΄Ρ€ΠΎΠ±ΠΈ, Ρƒ ΠΊΠΎΡ‚ΠΎΡ€Ρ‹Ρ… ΠΎΠ΄ΠΈΠ½Π°ΠΊΠΎΠ²Ρ‹Π΅ (ΠΎΠ±Ρ‰ΠΈΠ΅) Π·Π½Π°ΠΌΠ΅Π½Π°Ρ‚Π΅Π»ΠΈ. ΠžΡΡ‚Π°Π»ΠΎΡΡŒ ΡΠ»ΠΎΠΆΠΈΡ‚ΡŒ эти Π΄Ρ€ΠΎΠ±ΠΈ. Π‘ΠΊΠ»Π°Π΄Ρ‹Π²Π°Π΅ΠΌ:

Π‘Π»ΠΎΠΆΠ΅Π½ΠΈΠ΅ Π½Π΅ ΠΏΠΎΠΌΠ΅ΡΡ‚ΠΈΠ»ΠΎΡΡŒ Π½Π° ΠΎΠ΄Π½ΠΎΠΉ строкС, поэтому ΠΌΡ‹ пСрСнСсли ΠΎΡΡ‚Π°Π²ΡˆΠ΅Π΅ΡΡ Π²Ρ‹Ρ€Π°ΠΆΠ΅Π½ΠΈΠ΅ Π½Π° ΡΠ»Π΅Π΄ΡƒΡŽΡ‰ΡƒΡŽ строку. Π­Ρ‚ΠΎ допускаСтся Π² ΠΌΠ°Ρ‚Π΅ΠΌΠ°Ρ‚ΠΈΠΊΠ΅. Когда Π²Ρ‹Ρ€Π°ΠΆΠ΅Π½ΠΈΠ΅ Π½Π΅ помСщаСтся Π½Π° ΠΎΠ΄Π½Ρƒ строку, Π΅Π³ΠΎ пСрСносят Π½Π° ΡΠ»Π΅Π΄ΡƒΡŽΡ‰ΡƒΡŽ строку, ΠΏΡ€ΠΈ этом Π½Π°Π΄ΠΎ ΠΎΠ±ΡΠ·Π°Ρ‚Π΅Π»ΡŒΠ½ΠΎ ΠΏΠΎΡΡ‚Π°Π²ΠΈΡ‚ΡŒ Π·Π½Π°ΠΊ равСнства (=) Π½Π° ΠΊΠΎΠ½Ρ†Π΅ ΠΏΠ΅Ρ€Π²ΠΎΠΉ строки ΠΈ Π² Π½Π°Ρ‡Π°Π»Π΅ Π½ΠΎΠ²ΠΎΠΉ строки. Π—Π½Π°ΠΊ равСнства Π½Π° Π²Ρ‚ΠΎΡ€ΠΎΠΉ строкС Π³ΠΎΠ²ΠΎΡ€ΠΈΡ‚ ΠΎ Ρ‚ΠΎΠΌ, Ρ‡Ρ‚ΠΎ это ΠΏΡ€ΠΎΠ΄ΠΎΠ»ΠΆΠ΅Π½ΠΈΠ΅ выраТСния, ΠΊΠΎΡ‚ΠΎΡ€ΠΎΠ΅ Π±Ρ‹Π»ΠΎ Π½Π° ΠΏΠ΅Ρ€Π²ΠΎΠΉ строкС.

Π¨Π°Π³ 5. Если Π² ΠΎΡ‚Π²Π΅Ρ‚Π΅ ΠΏΠΎΠ»ΡƒΡ‡ΠΈΠ»Π°ΡΡŒ Π½Π΅ΠΏΡ€Π°Π²ΠΈΠ»ΡŒΠ½Π°Ρ Π΄Ρ€ΠΎΠ±ΡŒ, Ρ‚ΠΎ Π²Ρ‹Π΄Π΅Π»ΠΈΡ‚ΡŒ Π² Π½Π΅ΠΉ Ρ†Π΅Π»ΡƒΡŽ Ρ‡Π°ΡΡ‚ΡŒ

Π£ нас Π² ΠΎΡ‚Π²Π΅Ρ‚Π΅ ΠΏΠΎΠ»ΡƒΡ‡ΠΈΠ»Π°ΡΡŒ Π½Π΅ΠΏΡ€Π°Π²ΠΈΠ»ΡŒΠ½Π°Ρ Π΄Ρ€ΠΎΠ±ΡŒ. ΠœΡ‹ Π΄ΠΎΠ»ΠΆΠ½Ρ‹ Π²Ρ‹Π΄Π΅Π»ΠΈΡ‚ΡŒ Ρƒ Π½Π΅Ρ‘ Ρ†Π΅Π»ΡƒΡŽ Ρ‡Π°ΡΡ‚ΡŒ. ВыдСляСм:

ΠŸΠΎΠ»ΡƒΡ‡ΠΈΠ»ΠΈ ΠΎΡ‚Π²Π΅Ρ‚

Π’Ρ‹Ρ‡ΠΈΡ‚Π°Π½ΠΈΠ΅ Π΄Ρ€ΠΎΠ±Π΅ΠΉ с ΠΎΠ΄ΠΈΠ½Π°ΠΊΠΎΠ²Ρ‹ΠΌΠΈ знамСнатСлями

Π’Ρ‹Ρ‡ΠΈΡ‚Π°Π½ΠΈΠ΅ Π΄Ρ€ΠΎΠ±Π΅ΠΉ Π±Ρ‹Π²Π°Π΅Ρ‚ Π΄Π²ΡƒΡ… Π²ΠΈΠ΄ΠΎΠ²:

  1. Π’Ρ‹Ρ‡ΠΈΡ‚Π°Π½ΠΈΠ΅ Π΄Ρ€ΠΎΠ±Π΅ΠΉ с ΠΎΠ΄ΠΈΠ½Π°ΠΊΠΎΠ²Ρ‹ΠΌΠΈ знамСнатСлями
  2. Π’Ρ‹Ρ‡ΠΈΡ‚Π°Π½ΠΈΠ΅ Π΄Ρ€ΠΎΠ±Π΅ΠΉ с Ρ€Π°Π·Π½Ρ‹ΠΌΠΈ знамСнатСлями

Π‘Π½Π°Ρ‡Π°Π»Π° ΠΈΠ·ΡƒΡ‡ΠΈΠΌ Π²Ρ‹Ρ‡ΠΈΡ‚Π°Π½ΠΈΠ΅ Π΄Ρ€ΠΎΠ±Π΅ΠΉ с ΠΎΠ΄ΠΈΠ½Π°ΠΊΠΎΠ²Ρ‹ΠΌΠΈ знамСнатСлями. Π’ΡƒΡ‚ всё просто. Π§Ρ‚ΠΎΠ±Ρ‹ Π²Ρ‹Ρ‡Π΅ΡΡ‚ΡŒ ΠΈΠ· ΠΎΠ΄Π½ΠΎΠΉ Π΄Ρ€ΠΎΠ±ΠΈ Π΄Ρ€ΡƒΠ³ΡƒΡŽ, Π½ΡƒΠΆΠ½ΠΎ ΠΈΠ· числитСля ΠΏΠ΅Ρ€Π²ΠΎΠΉ Π΄Ρ€ΠΎΠ±ΠΈ Π²Ρ‹Ρ‡Π΅ΡΡ‚ΡŒ Ρ‡ΠΈΡΠ»ΠΈΡ‚Π΅Π»ΡŒ Π²Ρ‚ΠΎΡ€ΠΎΠΉ Π΄Ρ€ΠΎΠ±ΠΈ, Π° Π·Π½Π°ΠΌΠ΅Π½Π°Ρ‚Π΅Π»ΡŒ ΠΎΡΡ‚Π°Π²ΠΈΡ‚ΡŒ ΠΏΡ€Π΅ΠΆΠ½ΠΈΠΌ.

НапримСр, Π½Π°ΠΉΠ΄Ρ‘ΠΌ Π·Π½Π°Ρ‡Π΅Π½ΠΈΠ΅ выраТСния . Π§Ρ‚ΠΎΠ±Ρ‹ Ρ€Π΅ΡˆΠΈΡ‚ΡŒ этот ΠΏΡ€ΠΈΠΌΠ΅Ρ€, Π½Π°Π΄ΠΎ ΠΈΠ· числитСля ΠΏΠ΅Ρ€Π²ΠΎΠΉ Π΄Ρ€ΠΎΠ±ΠΈ Π²Ρ‹Ρ‡Π΅ΡΡ‚ΡŒ Ρ‡ΠΈΡΠ»ΠΈΡ‚Π΅Π»ΡŒ Π²Ρ‚ΠΎΡ€ΠΎΠΉ Π΄Ρ€ΠΎΠ±ΠΈ, Π° Π·Π½Π°ΠΌΠ΅Π½Π°Ρ‚Π΅Π»ΡŒ ΠΎΡΡ‚Π°Π²ΠΈΡ‚ΡŒ Π±Π΅Π· измСнСния. Π’Π°ΠΊ ΠΈ сдСлаСм:

Π­Ρ‚ΠΎΡ‚ ΠΏΡ€ΠΈΠΌΠ΅Ρ€ ΠΌΠΎΠΆΠ½ΠΎ Π»Π΅Π³ΠΊΠΎ ΠΏΠΎΠ½ΡΡ‚ΡŒ, Ссли Π²ΡΠΏΠΎΠΌΠ½ΠΈΡ‚ΡŒ ΠΏΡ€ΠΎ ΠΏΠΈΡ†Ρ†Ρƒ, которая Ρ€Π°Π·Π΄Π΅Π»Π΅Π½Π° Π½Π° Ρ‡Π΅Ρ‚Ρ‹Ρ€Π΅ части. Если ΠΎΡ‚ ΠΏΠΈΡ†Ρ†Ρ‹ ΠΎΡ‚Ρ€Π΅Π·Π°Ρ‚ΡŒ ΠΏΠΈΡ†Ρ†Ρ‹, Ρ‚ΠΎ получится ΠΏΠΈΡ†Ρ†Ρ‹:

ΠŸΡ€ΠΈΠΌΠ΅Ρ€ 2. Найти Π·Π½Π°Ρ‡Π΅Π½ΠΈΠ΅ выраТСния .

ΠžΠΏΡΡ‚ΡŒ ΠΆΠ΅ ΠΈΠ· числитСля ΠΏΠ΅Ρ€Π²ΠΎΠΉ Π΄Ρ€ΠΎΠ±ΠΈ Π²Ρ‹Ρ‡ΠΈΡ‚Π°Π΅ΠΌ Ρ‡ΠΈΡΠ»ΠΈΡ‚Π΅Π»ΡŒ Π²Ρ‚ΠΎΡ€ΠΎΠΉ Π΄Ρ€ΠΎΠ±ΠΈ, Π° Π·Π½Π°ΠΌΠ΅Π½Π°Ρ‚Π΅Π»ΡŒ оставляСм Π±Π΅Π· измСнСния:

Π­Ρ‚ΠΎΡ‚ ΠΏΡ€ΠΈΠΌΠ΅Ρ€ ΠΌΠΎΠΆΠ½ΠΎ Π»Π΅Π³ΠΊΠΎ ΠΏΠΎΠ½ΡΡ‚ΡŒ, Ссли Π²ΡΠΏΠΎΠΌΠ½ΠΈΡ‚ΡŒ ΠΏΡ€ΠΎ ΠΏΠΈΡ†Ρ†Ρƒ, которая Ρ€Π°Π·Π΄Π΅Π»Π΅Π½Π° Π½Π° Ρ‚Ρ€ΠΈ части. Если ΠΎΡ‚ ΠΏΠΈΡ†Ρ†Ρ‹ ΠΎΡ‚Ρ€Π΅Π·Π°Ρ‚ΡŒ ΠΏΠΈΡ†Ρ†Ρ‹, Ρ‚ΠΎ получится ΠΏΠΈΡ†Ρ†Ρ‹:

ΠŸΡ€ΠΈΠΌΠ΅Ρ€ 3. Найти Π·Π½Π°Ρ‡Π΅Π½ΠΈΠ΅ выраТСния

Π­Ρ‚ΠΎΡ‚ ΠΏΡ€ΠΈΠΌΠ΅Ρ€ Ρ€Π΅ΡˆΠ°Π΅Ρ‚ΡΡ Ρ‚ΠΎΡ‡Π½ΠΎ Ρ‚Π°ΠΊΠΆΠ΅, ΠΊΠ°ΠΊ ΠΈ ΠΏΡ€Π΅Π΄Ρ‹Π΄ΡƒΡ‰ΠΈΠ΅. Из числитСля ΠΏΠ΅Ρ€Π²ΠΎΠΉ Π΄Ρ€ΠΎΠ±ΠΈ Π½ΡƒΠΆΠ½ΠΎ Π²Ρ‹Ρ‡Π΅ΡΡ‚ΡŒ числитСли ΠΎΡΡ‚Π°Π»ΡŒΠ½Ρ‹Ρ… Π΄Ρ€ΠΎΠ±Π΅ΠΉ:

Как Π²ΠΈΠ΄ΠΈΡ‚Π΅ Π² Π²Ρ‹Ρ‡ΠΈΡ‚Π°Π½ΠΈΠΈ Π΄Ρ€ΠΎΠ±Π΅ΠΉ с ΠΎΠ΄ΠΈΠ½Π°ΠΊΠΎΠ²Ρ‹ΠΌΠΈ знамСнатСлями Π½ΠΈΡ‡Π΅Π³ΠΎ слоТного Π½Π΅Ρ‚. Достаточно ΠΏΠΎΠ½ΠΈΠΌΠ°Ρ‚ΡŒ ΡΠ»Π΅Π΄ΡƒΡŽΡ‰ΠΈΠ΅ ΠΏΡ€Π°Π²ΠΈΠ»Π°:

  1. Π§Ρ‚ΠΎΠ±Ρ‹ Π²Ρ‹Ρ‡Π΅ΡΡ‚ΡŒ ΠΈΠ· ΠΎΠ΄Π½ΠΎΠΉ Π΄Ρ€ΠΎΠ±ΠΈ Π΄Ρ€ΡƒΠ³ΡƒΡŽ, Π½ΡƒΠΆΠ½ΠΎ ΠΈΠ· числитСля ΠΏΠ΅Ρ€Π²ΠΎΠΉ Π΄Ρ€ΠΎΠ±ΠΈ Π²Ρ‹Ρ‡Π΅ΡΡ‚ΡŒ Ρ‡ΠΈΡΠ»ΠΈΡ‚Π΅Π»ΡŒ Π²Ρ‚ΠΎΡ€ΠΎΠΉ Π΄Ρ€ΠΎΠ±ΠΈ, Π° Π·Π½Π°ΠΌΠ΅Π½Π°Ρ‚Π΅Π»ΡŒ ΠΎΡΡ‚Π°Π²ΠΈΡ‚ΡŒ Π±Π΅Π· измСнСния;
  2. Если Π² ΠΎΡ‚Π²Π΅Ρ‚Π΅ ΠΏΠΎΠ»ΡƒΡ‡ΠΈΠ»Π°ΡΡŒ Π½Π΅ΠΏΡ€Π°Π²ΠΈΠ»ΡŒΠ½Π°Ρ Π΄Ρ€ΠΎΠ±ΡŒ, Ρ‚ΠΎ Π½ΡƒΠΆΠ½ΠΎ Π²Ρ‹Π΄Π΅Π»ΠΈΡ‚ΡŒ Π² Π½Π΅ΠΉ Ρ†Π΅Π»ΡƒΡŽ Ρ‡Π°ΡΡ‚ΡŒ.

Π’Ρ‹Ρ‡ΠΈΡ‚Π°Π½ΠΈΠ΅ Π΄Ρ€ΠΎΠ±Π΅ΠΉ с Ρ€Π°Π·Π½Ρ‹ΠΌΠΈ знамСнатСлями

НапримСр, ΠΎΡ‚ Π΄Ρ€ΠΎΠ±ΠΈ ΠΌΠΎΠΆΠ½ΠΎ Π²Ρ‹Ρ‡Π΅ΡΡ‚ΡŒ Π΄Ρ€ΠΎΠ±ΡŒ , ΠΏΠΎΡΠΊΠΎΠ»ΡŒΠΊΡƒ Ρƒ этих Π΄Ρ€ΠΎΠ±Π΅ΠΉ ΠΎΠ΄ΠΈΠ½Π°ΠΊΠΎΠ²Ρ‹Π΅ Π·Π½Π°ΠΌΠ΅Π½Π°Ρ‚Π΅Π»ΠΈ. А Π²ΠΎΡ‚ ΠΎΡ‚ Π΄Ρ€ΠΎΠ±ΠΈ нСльзя Π²Ρ‹Ρ‡Π΅ΡΡ‚ΡŒ Π΄Ρ€ΠΎΠ±ΡŒ , ΠΏΠΎΡΠΊΠΎΠ»ΡŒΠΊΡƒ Ρƒ этих Π΄Ρ€ΠΎΠ±Π΅ΠΉ Ρ€Π°Π·Π½Ρ‹Π΅ Π·Π½Π°ΠΌΠ΅Π½Π°Ρ‚Π΅Π»ΠΈ. Π’ Ρ‚Π°ΠΊΠΈΡ… случаях Π΄Ρ€ΠΎΠ±ΠΈ Π½ΡƒΠΆΠ½ΠΎ ΠΏΡ€ΠΈΠ²ΠΎΠ΄ΠΈΡ‚ΡŒ ΠΊ ΠΎΠ΄ΠΈΠ½Π°ΠΊΠΎΠ²ΠΎΠΌΡƒ (ΠΎΠ±Ρ‰Π΅ΠΌΡƒ) Π·Π½Π°ΠΌΠ΅Π½Π°Ρ‚Π΅Π»ΡŽ.

ΠžΠ±Ρ‰ΠΈΠΉ Π·Π½Π°ΠΌΠ΅Π½Π°Ρ‚Π΅Π»ΡŒ находят ΠΏΠΎ Ρ‚ΠΎΠΌΡƒ ΠΆΠ΅ ΠΏΡ€ΠΈΠ½Ρ†ΠΈΠΏΡƒ, ΠΊΠΎΡ‚ΠΎΡ€Ρ‹ΠΌ ΠΌΡ‹ пользовались ΠΏΡ€ΠΈ слоТСнии Π΄Ρ€ΠΎΠ±Π΅ΠΉ с Ρ€Π°Π·Π½Ρ‹ΠΌΠΈ знамСнатСлями. Π’ ΠΏΠ΅Ρ€Π²ΡƒΡŽ ΠΎΡ‡Π΅Ρ€Π΅Π΄ΡŒ находят НОК Π·Π½Π°ΠΌΠ΅Π½Π°Ρ‚Π΅Π»Π΅ΠΉ ΠΎΠ±Π΅ΠΈΡ… Π΄Ρ€ΠΎΠ±Π΅ΠΉ. Π—Π°Ρ‚Π΅ΠΌ НОК дСлят Π½Π° Π·Π½Π°ΠΌΠ΅Π½Π°Ρ‚Π΅Π»ΡŒ ΠΏΠ΅Ρ€Π²ΠΎΠΉ Π΄Ρ€ΠΎΠ±ΠΈ ΠΈ ΠΏΠΎΠ»ΡƒΡ‡Π°ΡŽΡ‚ ΠΏΠ΅Ρ€Π²Ρ‹ΠΉ Π΄ΠΎΠΏΠΎΠ»Π½ΠΈΡ‚Π΅Π»ΡŒΠ½Ρ‹ΠΉ ΠΌΠ½ΠΎΠΆΠΈΡ‚Π΅Π»ΡŒ, ΠΊΠΎΡ‚ΠΎΡ€Ρ‹ΠΉ записываСтся Π½Π°Π΄ ΠΏΠ΅Ρ€Π²ΠΎΠΉ Π΄Ρ€ΠΎΠ±ΡŒΡŽ. Аналогично НОК дСлят Π½Π° Π·Π½Π°ΠΌΠ΅Π½Π°Ρ‚Π΅Π»ΡŒ Π²Ρ‚ΠΎΡ€ΠΎΠΉ Π΄Ρ€ΠΎΠ±ΠΈ ΠΈ ΠΏΠΎΠ»ΡƒΡ‡Π°ΡŽΡ‚ Π²Ρ‚ΠΎΡ€ΠΎΠΉ Π΄ΠΎΠΏΠΎΠ»Π½ΠΈΡ‚Π΅Π»ΡŒΠ½Ρ‹ΠΉ ΠΌΠ½ΠΎΠΆΠΈΡ‚Π΅Π»ΡŒ, ΠΊΠΎΡ‚ΠΎΡ€Ρ‹ΠΉ записываСтся Π½Π°Π΄ Π²Ρ‚ΠΎΡ€ΠΎΠΉ Π΄Ρ€ΠΎΠ±ΡŒΡŽ.

Π—Π°Ρ‚Π΅ΠΌ Π΄Ρ€ΠΎΠ±ΠΈ ΡƒΠΌΠ½ΠΎΠΆΠ°ΡŽΡ‚ΡΡ Π½Π° свои Π΄ΠΎΠΏΠΎΠ»Π½ΠΈΡ‚Π΅Π»ΡŒΠ½Ρ‹Π΅ ΠΌΠ½ΠΎΠΆΠΈΡ‚Π΅Π»ΠΈ. Π’ Ρ€Π΅Π·ΡƒΠ»ΡŒΡ‚Π°Ρ‚Π΅ этих ΠΎΠΏΠ΅Ρ€Π°Ρ†ΠΈΠΉ, Π΄Ρ€ΠΎΠ±ΠΈ Ρƒ ΠΊΠΎΡ‚ΠΎΡ€Ρ‹Ρ… Π±Ρ‹Π»ΠΈ Ρ€Π°Π·Π½Ρ‹Π΅ Π·Π½Π°ΠΌΠ΅Π½Π°Ρ‚Π΅Π»ΠΈ, ΠΎΠ±Ρ€Π°Ρ‰Π°ΡŽΡ‚ΡΡ Π² Π΄Ρ€ΠΎΠ±ΠΈ, Ρƒ ΠΊΠΎΡ‚ΠΎΡ€Ρ‹Ρ… ΠΎΠ΄ΠΈΠ½Π°ΠΊΠΎΠ²Ρ‹Π΅ Π·Π½Π°ΠΌΠ΅Π½Π°Ρ‚Π΅Π»ΠΈ. А ΠΊΠ°ΠΊ Π²Ρ‹Ρ‡ΠΈΡ‚Π°Ρ‚ΡŒ Ρ‚Π°ΠΊΠΈΠ΅ Π΄Ρ€ΠΎΠ±ΠΈ ΠΌΡ‹ ΡƒΠΆΠ΅ Π·Π½Π°Π΅ΠΌ.

ΠŸΡ€ΠΈΠΌΠ΅Ρ€ 1. Найти Π·Π½Π°Ρ‡Π΅Π½ΠΈΠ΅ выраТСния:

Π£ этих Π΄Ρ€ΠΎΠ±Π΅ΠΉ Ρ€Π°Π·Π½Ρ‹Π΅ Π·Π½Π°ΠΌΠ΅Π½Π°Ρ‚Π΅Π»ΠΈ, поэтому Π½ΡƒΠΆΠ½ΠΎ привСсти ΠΈΡ… ΠΊ ΠΎΠ΄ΠΈΠ½Π°ΠΊΠΎΠ²ΠΎΠΌΡƒ (ΠΎΠ±Ρ‰Π΅ΠΌΡƒ) Π·Π½Π°ΠΌΠ΅Π½Π°Ρ‚Π΅Π»ΡŽ.

Π‘Π½Π°Ρ‡Π°Π»Π° Π½Π°Ρ…ΠΎΠ΄ΠΈΠΌ НОК Π·Π½Π°ΠΌΠ΅Π½Π°Ρ‚Π΅Π»Π΅ΠΉ ΠΎΠ±Π΅ΠΈΡ… Π΄Ρ€ΠΎΠ±Π΅ΠΉ. Π—Π½Π°ΠΌΠ΅Π½Π°Ρ‚Π΅Π»ΡŒ ΠΏΠ΅Ρ€Π²ΠΎΠΉ Π΄Ρ€ΠΎΠ±ΠΈ это число 3, Π° Π·Π½Π°ΠΌΠ΅Π½Π°Ρ‚Π΅Π»ΡŒ Π²Ρ‚ΠΎΡ€ΠΎΠΉ Π΄Ρ€ΠΎΠ±ΠΈ β€” число 4. НаимСньшСС ΠΎΠ±Ρ‰Π΅Π΅ ΠΊΡ€Π°Ρ‚Π½ΠΎΠ΅ этих чисСл Ρ€Π°Π²Π½ΠΎ 12

НОК (3 и 4) = 12

Π’Π΅ΠΏΠ΅Ρ€ΡŒ возвращаСмся ΠΊ дробям ΠΈ

Найдём Π΄ΠΎΠΏΠΎΠ»Π½ΠΈΡ‚Π΅Π»ΡŒΠ½Ρ‹ΠΉ ΠΌΠ½ΠΎΠΆΠΈΡ‚Π΅Π»ΡŒ для ΠΏΠ΅Ρ€Π²ΠΎΠΉ Π΄Ρ€ΠΎΠ±ΠΈ. Для этого Ρ€Π°Π·Π΄Π΅Π»ΠΈΠΌ НОК Π½Π° Π·Π½Π°ΠΌΠ΅Π½Π°Ρ‚Π΅Π»ΡŒ ΠΏΠ΅Ρ€Π²ΠΎΠΉ Π΄Ρ€ΠΎΠ±ΠΈ. НОК это число 12, Π° Π·Π½Π°ΠΌΠ΅Π½Π°Ρ‚Π΅Π»ΡŒ ΠΏΠ΅Ρ€Π²ΠΎΠΉ Π΄Ρ€ΠΎΠ±ΠΈ β€” число 3. Π”Π΅Π»ΠΈΠΌ 12 Π½Π° 3, ΠΏΠΎΠ»ΡƒΡ‡Π°Π΅ΠΌ 4. ЗаписываСм Ρ‡Π΅Ρ‚Π²Ρ‘Ρ€ΠΊΡƒ Π½Π°Π΄ ΠΏΠ΅Ρ€Π²ΠΎΠΉ Π΄Ρ€ΠΎΠ±ΡŒΡŽ:

Аналогично поступаСм ΠΈ со Π²Ρ‚ΠΎΡ€ΠΎΠΉ Π΄Ρ€ΠΎΠ±ΡŒΡŽ. Π”Π΅Π»ΠΈΠΌ НОК Π½Π° Π·Π½Π°ΠΌΠ΅Π½Π°Ρ‚Π΅Π»ΡŒ Π²Ρ‚ΠΎΡ€ΠΎΠΉ Π΄Ρ€ΠΎΠ±ΠΈ. НОК это число 12, Π° Π·Π½Π°ΠΌΠ΅Π½Π°Ρ‚Π΅Π»ΡŒ Π²Ρ‚ΠΎΡ€ΠΎΠΉ Π΄Ρ€ΠΎΠ±ΠΈ β€” число 4. Π”Π΅Π»ΠΈΠΌ 12 Π½Π° 4, ΠΏΠΎΠ»ΡƒΡ‡Π°Π΅ΠΌ 3. ЗаписываСм Ρ‚Ρ€ΠΎΠΉΠΊΡƒ Π½Π°Π΄ Π²Ρ‚ΠΎΡ€ΠΎΠΉ Π΄Ρ€ΠΎΠ±ΡŒΡŽ:

Π’Π΅ΠΏΠ΅Ρ€ΡŒ Ρƒ нас всё Π³ΠΎΡ‚ΠΎΠ²ΠΎ для вычитания. ΠžΡΡ‚Π°Π»ΠΎΡΡŒ ΡƒΠΌΠ½ΠΎΠΆΠΈΡ‚ΡŒ Π΄Ρ€ΠΎΠ±ΠΈ Π½Π° свои Π΄ΠΎΠΏΠΎΠ»Π½ΠΈΡ‚Π΅Π»ΡŒΠ½Ρ‹Π΅ ΠΌΠ½ΠΎΠΆΠΈΡ‚Π΅Π»ΠΈ:

ΠœΡ‹ ΠΏΡ€ΠΈΡˆΠ»ΠΈ ΠΊ Ρ‚ΠΎΠΌΡƒ, Ρ‡Ρ‚ΠΎ Π΄Ρ€ΠΎΠ±ΠΈ Ρƒ ΠΊΠΎΡ‚ΠΎΡ€Ρ‹Ρ… Π±Ρ‹Π»ΠΈ Ρ€Π°Π·Π½Ρ‹Π΅ Π·Π½Π°ΠΌΠ΅Π½Π°Ρ‚Π΅Π»ΠΈ, ΠΏΡ€Π΅Π²Ρ€Π°Ρ‚ΠΈΠ»ΠΈΡΡŒ Π² Π΄Ρ€ΠΎΠ±ΠΈ Ρƒ ΠΊΠΎΡ‚ΠΎΡ€Ρ‹Ρ… ΠΎΠ΄ΠΈΠ½Π°ΠΊΠΎΠ²Ρ‹Π΅ Π·Π½Π°ΠΌΠ΅Π½Π°Ρ‚Π΅Π»ΠΈ. А ΠΊΠ°ΠΊ Π²Ρ‹Ρ‡ΠΈΡ‚Π°Ρ‚ΡŒ Ρ‚Π°ΠΊΠΈΠ΅ Π΄Ρ€ΠΎΠ±ΠΈ ΠΌΡ‹ ΡƒΠΆΠ΅ Π·Π½Π°Π΅ΠΌ. Π”Π°Π²Π°ΠΉΡ‚Π΅ Π΄ΠΎΡ€Π΅ΡˆΠ°Π΅ΠΌ этот ΠΏΡ€ΠΈΠΌΠ΅Ρ€ Π΄ΠΎ ΠΊΠΎΠ½Ρ†Π°:

ΠŸΠΎΠ»ΡƒΡ‡ΠΈΠ»ΠΈ ΠΎΡ‚Π²Π΅Ρ‚

ΠŸΠΎΠΏΡ€ΠΎΠ±ΡƒΠ΅ΠΌ ΠΈΠ·ΠΎΠ±Ρ€Π°Π·ΠΈΡ‚ΡŒ нашС Ρ€Π΅ΡˆΠ΅Π½ΠΈΠ΅ с ΠΏΠΎΠΌΠΎΡ‰ΡŒΡŽ рисунка. Если ΠΎΡ‚ ΠΏΠΈΡ†Ρ†Ρ‹ ΠΎΡ‚Ρ€Π΅Π·Π°Ρ‚ΡŒ ΠΏΠΈΡ†Ρ†Ρ‹, Ρ‚ΠΎ получится ΠΏΠΈΡ†Ρ†Ρ‹

Π­Ρ‚ΠΎ подробная вСрсия Ρ€Π΅ΡˆΠ΅Π½ΠΈΡ. ΠΠ°Ρ…ΠΎΠ΄ΡΡΡŒ Π² школС, Π½Π°ΠΌ ΠΏΡ€ΠΈΡˆΠ»ΠΎΡΡŒ Π±Ρ‹ Ρ€Π΅ΡˆΠΈΡ‚ΡŒ этот ΠΏΡ€ΠΈΠΌΠ΅Ρ€ ΠΏΠΎΠΊΠΎΡ€ΠΎΡ‡Π΅. ВыглядСло Π±Ρ‹ Ρ‚Π°ΠΊΠΎΠ΅ Ρ€Π΅ΡˆΠ΅Π½ΠΈΠ΅ ΡΠ»Π΅Π΄ΡƒΡŽΡ‰ΠΈΠΌ ΠΎΠ±Ρ€Π°Π·ΠΎΠΌ:

ΠŸΡ€ΠΈΠ²Π΅Π΄Π΅Π½ΠΈΠ΅ Π΄Ρ€ΠΎΠ±Π΅ΠΉ ΠΈ ΠΊ ΠΎΠ±Ρ‰Π΅ΠΌΡƒ Π·Π½Π°ΠΌΠ΅Π½Π°Ρ‚Π΅Π»ΡŽ Ρ‚Π°ΠΊΠΆΠ΅ ΠΌΠΎΠΆΠ΅Ρ‚ Π±Ρ‹Ρ‚ΡŒ ΠΈΠ·ΠΎΠ±Ρ€Π°ΠΆΠ΅Π½ΠΎ с ΠΏΠΎΠΌΠΎΡ‰ΡŒΡŽ рисунка. ΠŸΡ€ΠΈΠ²Π΅Π΄Ρ эти Π΄Ρ€ΠΎΠ±ΠΈ ΠΊ ΠΎΠ±Ρ‰Π΅ΠΌΡƒ Π·Π½Π°ΠΌΠ΅Π½Π°Ρ‚Π΅Π»ΡŽ, ΠΌΡ‹ ΠΏΠΎΠ»ΡƒΡ‡ΠΈΠ»ΠΈ Π΄Ρ€ΠΎΠ±ΠΈ ΠΈ . Π­Ρ‚ΠΈ Π΄Ρ€ΠΎΠ±ΠΈ Π±ΡƒΠ΄ΡƒΡ‚ ΠΈΠ·ΠΎΠ±Ρ€Π°ΠΆΠ°Ρ‚ΡŒΡΡ Ρ‚Π΅ΠΌΠΈ ΠΆΠ΅ кусочками ΠΏΠΈΡ†Ρ†, Π½ΠΎ Π² этот Ρ€Π°Π· ΠΎΠ½ΠΈ Π±ΡƒΠ΄ΡƒΡ‚ Ρ€Π°Π·Π΄Π΅Π»Π΅Π½Ρ‹ Π½Π° ΠΎΠ΄ΠΈΠ½Π°ΠΊΠΎΠ²Ρ‹Π΅ Π΄ΠΎΠ»ΠΈ (ΠΏΡ€ΠΈΠ²Π΅Π΄Π΅Π½Ρ‹ ΠΊ ΠΎΠ΄ΠΈΠ½Π°ΠΊΠΎΠ²ΠΎΠΌΡƒ Π·Π½Π°ΠΌΠ΅Π½Π°Ρ‚Π΅Π»ΡŽ):

ΠŸΠ΅Ρ€Π²Ρ‹ΠΉ рисунок ΠΈΠ·ΠΎΠ±Ρ€Π°ΠΆΠ°Π΅Ρ‚ Π΄Ρ€ΠΎΠ±ΡŒ (восСмь кусочков ΠΈΠ· Π΄Π²Π΅Π½Π°Π΄Ρ†Π°Ρ‚ΠΈ), Π° Π²Ρ‚ΠΎΡ€ΠΎΠΉ рисунок β€” Π΄Ρ€ΠΎΠ±ΡŒ (Ρ‚Ρ€ΠΈ кусочка ΠΈΠ· Π΄Π²Π΅Π½Π°Π΄Ρ†Π°Ρ‚ΠΈ). ΠžΡ‚Ρ€Π΅Π·Π°Π² ΠΎΡ‚ восьми кусочков Ρ‚Ρ€ΠΈ кусочка ΠΌΡ‹ ΠΏΠΎΠ»ΡƒΡ‡Π°Π΅ΠΌ ΠΏΡΡ‚ΡŒ кусочков ΠΈΠ· Π΄Π²Π΅Π½Π°Π΄Ρ†Π°Ρ‚ΠΈ. Π”Ρ€ΠΎΠ±ΡŒ ΠΈ описываСт эти ΠΏΡΡ‚ΡŒ кусочков.

ΠŸΡ€ΠΈΠΌΠ΅Ρ€ 2. Найти Π·Π½Π°Ρ‡Π΅Π½ΠΈΠ΅ выраТСния

Π£ этих Π΄Ρ€ΠΎΠ±Π΅ΠΉ Ρ€Π°Π·Π½Ρ‹Π΅ Π·Π½Π°ΠΌΠ΅Π½Π°Ρ‚Π΅Π»ΠΈ, поэтому сначала Π½ΡƒΠΆΠ½ΠΎ привСсти ΠΈΡ… ΠΊ ΠΎΠ΄ΠΈΠ½Π°ΠΊΠΎΠ²ΠΎΠΌΡƒ (ΠΎΠ±Ρ‰Π΅ΠΌΡƒ) Π·Π½Π°ΠΌΠ΅Π½Π°Ρ‚Π΅Π»ΡŽ.

Найдём НОК Π·Π½Π°ΠΌΠ΅Π½Π°Ρ‚Π΅Π»Π΅ΠΉ этих Π΄Ρ€ΠΎΠ±Π΅ΠΉ.

Π—Π½Π°ΠΌΠ΅Π½Π°Ρ‚Π΅Π»ΠΈ Π΄Ρ€ΠΎΠ±Π΅ΠΉ это числа 10, 3 ΠΈ 5. НаимСньшСС ΠΎΠ±Ρ‰Π΅Π΅ ΠΊΡ€Π°Ρ‚Π½ΠΎΠ΅ этих чисСл Ρ€Π°Π²Π½ΠΎ 30

НОК (10, 3, 5) = 30

Π’Π΅ΠΏΠ΅Ρ€ΡŒ Π½Π°Ρ…ΠΎΠ΄ΠΈΠΌ Π΄ΠΎΠΏΠΎΠ»Π½ΠΈΡ‚Π΅Π»ΡŒΠ½Ρ‹Π΅ ΠΌΠ½ΠΎΠΆΠΈΡ‚Π΅Π»ΠΈ для ΠΊΠ°ΠΆΠ΄ΠΎΠΉ Π΄Ρ€ΠΎΠ±ΠΈ. Для этого Ρ€Π°Π·Π΄Π΅Π»ΠΈΠΌ НОК Π½Π° Π·Π½Π°ΠΌΠ΅Π½Π°Ρ‚Π΅Π»ΡŒ ΠΊΠ°ΠΆΠ΄ΠΎΠΉ Π΄Ρ€ΠΎΠ±ΠΈ.

Найдём Π΄ΠΎΠΏΠΎΠ»Π½ΠΈΡ‚Π΅Π»ΡŒΠ½Ρ‹ΠΉ ΠΌΠ½ΠΎΠΆΠΈΡ‚Π΅Π»ΡŒ для ΠΏΠ΅Ρ€Π²ΠΎΠΉ Π΄Ρ€ΠΎΠ±ΠΈ. НОК это число 30, Π° Π·Π½Π°ΠΌΠ΅Π½Π°Ρ‚Π΅Π»ΡŒ ΠΏΠ΅Ρ€Π²ΠΎΠΉ Π΄Ρ€ΠΎΠ±ΠΈ β€” число 10. Π”Π΅Π»ΠΈΠΌ 30 Π½Π° 10, ΠΏΠΎΠ»ΡƒΡ‡Π°Π΅ΠΌ ΠΏΠ΅Ρ€Π²Ρ‹ΠΉ Π΄ΠΎΠΏΠΎΠ»Π½ΠΈΡ‚Π΅Π»ΡŒΠ½Ρ‹ΠΉ ΠΌΠ½ΠΎΠΆΠΈΡ‚Π΅Π»ΡŒ 3. ЗаписываСм Π΅Π³ΠΎ Π½Π°Π΄ ΠΏΠ΅Ρ€Π²ΠΎΠΉ Π΄Ρ€ΠΎΠ±ΡŒΡŽ:

Π’Π΅ΠΏΠ΅Ρ€ΡŒ Π½Π°Ρ…ΠΎΠ΄ΠΈΠΌ Π΄ΠΎΠΏΠΎΠ»Π½ΠΈΡ‚Π΅Π»ΡŒΠ½Ρ‹ΠΉ ΠΌΠ½ΠΎΠΆΠΈΡ‚Π΅Π»ΡŒ для Π²Ρ‚ΠΎΡ€ΠΎΠΉ Π΄Ρ€ΠΎΠ±ΠΈ. Π Π°Π·Π΄Π΅Π»ΠΈΠΌ НОК Π½Π° Π·Π½Π°ΠΌΠ΅Π½Π°Ρ‚Π΅Π»ΡŒ Π²Ρ‚ΠΎΡ€ΠΎΠΉ Π΄Ρ€ΠΎΠ±ΠΈ. НОК это число 30, Π° Π·Π½Π°ΠΌΠ΅Π½Π°Ρ‚Π΅Π»ΡŒ Π²Ρ‚ΠΎΡ€ΠΎΠΉ Π΄Ρ€ΠΎΠ±ΠΈ β€” число 3. Π”Π΅Π»ΠΈΠΌ 30 Π½Π° 3, ΠΏΠΎΠ»ΡƒΡ‡Π°Π΅ΠΌ Π²Ρ‚ΠΎΡ€ΠΎΠΉ Π΄ΠΎΠΏΠΎΠ»Π½ΠΈΡ‚Π΅Π»ΡŒΠ½Ρ‹ΠΉ ΠΌΠ½ΠΎΠΆΠΈΡ‚Π΅Π»ΡŒ 10. ЗаписываСм Π΅Π³ΠΎ Π½Π°Π΄ Π²Ρ‚ΠΎΡ€ΠΎΠΉ Π΄Ρ€ΠΎΠ±ΡŒΡŽ:

Π’Π΅ΠΏΠ΅Ρ€ΡŒ Π½Π°Ρ…ΠΎΠ΄ΠΈΠΌ Π΄ΠΎΠΏΠΎΠ»Π½ΠΈΡ‚Π΅Π»ΡŒΠ½Ρ‹ΠΉ ΠΌΠ½ΠΎΠΆΠΈΡ‚Π΅Π»ΡŒ для Ρ‚Ρ€Π΅Ρ‚ΡŒΠ΅ΠΉ Π΄Ρ€ΠΎΠ±ΠΈ. Π Π°Π·Π΄Π΅Π»ΠΈΠΌ НОК Π½Π° Π·Π½Π°ΠΌΠ΅Π½Π°Ρ‚Π΅Π»ΡŒ Ρ‚Ρ€Π΅Ρ‚ΡŒΠ΅ΠΉ Π΄Ρ€ΠΎΠ±ΠΈ. НОК это число 30, Π° Π·Π½Π°ΠΌΠ΅Π½Π°Ρ‚Π΅Π»ΡŒ Ρ‚Ρ€Π΅Ρ‚ΡŒΠ΅ΠΉ Π΄Ρ€ΠΎΠ±ΠΈ β€” число 5. Π”Π΅Π»ΠΈΠΌ 30 Π½Π° 5, ΠΏΠΎΠ»ΡƒΡ‡Π°Π΅ΠΌ Ρ‚Ρ€Π΅Ρ‚ΠΈΠΉ Π΄ΠΎΠΏΠΎΠ»Π½ΠΈΡ‚Π΅Π»ΡŒΠ½Ρ‹ΠΉ ΠΌΠ½ΠΎΠΆΠΈΡ‚Π΅Π»ΡŒ 6. ЗаписываСм Π΅Π³ΠΎ Π½Π°Π΄ Ρ‚Ρ€Π΅Ρ‚ΡŒΠ΅ΠΉ Π΄Ρ€ΠΎΠ±ΡŒΡŽ:

Π’Π΅ΠΏΠ΅Ρ€ΡŒ всё Π³ΠΎΡ‚ΠΎΠ²ΠΎ для вычитания. ΠžΡΡ‚Π°Π»ΠΎΡΡŒ ΡƒΠΌΠ½ΠΎΠΆΠΈΡ‚ΡŒ Π΄Ρ€ΠΎΠ±ΠΈ Π½Π° свои Π΄ΠΎΠΏΠΎΠ»Π½ΠΈΡ‚Π΅Π»ΡŒΠ½Ρ‹Π΅ ΠΌΠ½ΠΎΠΆΠΈΡ‚Π΅Π»ΠΈ:

ΠœΡ‹ ΠΏΡ€ΠΈΡˆΠ»ΠΈ ΠΊ Ρ‚ΠΎΠΌΡƒ, Ρ‡Ρ‚ΠΎ Π΄Ρ€ΠΎΠ±ΠΈ Ρƒ ΠΊΠΎΡ‚ΠΎΡ€Ρ‹Ρ… Π±Ρ‹Π»ΠΈ Ρ€Π°Π·Π½Ρ‹Π΅ Π·Π½Π°ΠΌΠ΅Π½Π°Ρ‚Π΅Π»ΠΈ, ΠΏΡ€Π΅Π²Ρ€Π°Ρ‚ΠΈΠ»ΠΈΡΡŒ Π² Π΄Ρ€ΠΎΠ±ΠΈ Ρƒ ΠΊΠΎΡ‚ΠΎΡ€Ρ‹Ρ… ΠΎΠ΄ΠΈΠ½Π°ΠΊΠΎΠ²Ρ‹Π΅ (ΠΎΠ±Ρ‰ΠΈΠ΅) Π·Π½Π°ΠΌΠ΅Π½Π°Ρ‚Π΅Π»ΠΈ. А ΠΊΠ°ΠΊ Π²Ρ‹Ρ‡ΠΈΡ‚Π°Ρ‚ΡŒ Ρ‚Π°ΠΊΠΈΠ΅ Π΄Ρ€ΠΎΠ±ΠΈ ΠΌΡ‹ ΡƒΠΆΠ΅ Π·Π½Π°Π΅ΠΌ. Π”Π°Π²Π°ΠΉΡ‚Π΅ Π΄ΠΎΡ€Π΅ΡˆΠ°Π΅ΠΌ этот ΠΏΡ€ΠΈΠΌΠ΅Ρ€.

ΠŸΡ€ΠΎΠ΄ΠΎΠ»ΠΆΠ΅Π½ΠΈΠ΅ ΠΏΡ€ΠΈΠΌΠ΅Ρ€Π° Π½Π΅ помСстится Π½Π° ΠΎΠ΄Π½ΠΎΠΉ строкС, поэтому пСрСносим ΠΏΡ€ΠΎΠ΄ΠΎΠ»ΠΆΠ΅Π½ΠΈΠ΅ Π½Π° ΡΠ»Π΅Π΄ΡƒΡŽΡ‰ΡƒΡŽ строку. НС Π·Π°Π±Ρ‹Π²Π°Π΅ΠΌ ΠΏΡ€ΠΎ Π·Π½Π°ΠΊ равСнства (=) Π½Π° Π½ΠΎΠ²ΠΎΠΉ строкС:

Π’ ΠΎΡ‚Π²Π΅Ρ‚Π΅ ΠΏΠΎΠ»ΡƒΡ‡ΠΈΠ»Π°ΡΡŒ ΠΏΡ€Π°Π²ΠΈΠ»ΡŒΠ½Π°Ρ Π΄Ρ€ΠΎΠ±ΡŒ, ΠΈ Π²Ρ€ΠΎΠ΄Π΅ Π±Ρ‹ нас всё устраиваСт, Π½ΠΎ ΠΎΠ½Π° слишком Π³Ρ€ΠΎΠΌΠΎΠ·Π΄ΠΊΠ° ΠΈ нСкрасива. Надо Π±Ρ‹ ΡΠ΄Π΅Π»Π°Ρ‚ΡŒ Π΅Ρ‘ ΠΏΡ€ΠΎΡ‰Π΅. А Ρ‡Ρ‚ΠΎ ΠΌΠΎΠΆΠ½ΠΎ ΡΠ΄Π΅Π»Π°Ρ‚ΡŒ? МоТно ΡΠΎΠΊΡ€Π°Ρ‚ΠΈΡ‚ΡŒ эту Π΄Ρ€ΠΎΠ±ΡŒ.

Π§Ρ‚ΠΎΠ±Ρ‹ ΡΠΎΠΊΡ€Π°Ρ‚ΠΈΡ‚ΡŒ Π΄Ρ€ΠΎΠ±ΡŒ , Π½ΡƒΠΆΠ½ΠΎ Ρ€Π°Π·Π΄Π΅Π»ΠΈΡ‚ΡŒ Π΅Ρ‘ Ρ‡ΠΈΡΠ»ΠΈΡ‚Π΅Π»ΡŒ ΠΈ Π·Π½Π°ΠΌΠ΅Π½Π°Ρ‚Π΅Π»ΡŒ Π½Π° (ΠΠžΠ”) чисСл 20 ΠΈ 30.

Π˜Ρ‚Π°ΠΊ, Π½Π°Ρ…ΠΎΠ΄ΠΈΠΌ ΠΠžΠ” чисСл 20 ΠΈ 30:

Π’Π΅ΠΏΠ΅Ρ€ΡŒ возвращаСмся ΠΊ Π½Π°ΡˆΠ΅ΠΌΡƒ ΠΏΡ€ΠΈΠΌΠ΅Ρ€Ρƒ ΠΈ Π΄Π΅Π»ΠΈΠΌ Ρ‡ΠΈΡΠ»ΠΈΡ‚Π΅Π»ΡŒ ΠΈ Π·Π½Π°ΠΌΠ΅Π½Π°Ρ‚Π΅Π»ΡŒ Π΄Ρ€ΠΎΠ±ΠΈ Π½Π° Π½Π°ΠΉΠ΄Π΅Π½Π½Ρ‹ΠΉ ΠΠžΠ”, Ρ‚ΠΎ Π΅ΡΡ‚ΡŒ Π½Π° 10

ΠŸΠΎΠ»ΡƒΡ‡ΠΈΠ»ΠΈ ΠΎΡ‚Π²Π΅Ρ‚

Π£ΠΌΠ½ΠΎΠΆΠ΅Π½ΠΈΠ΅ Π΄Ρ€ΠΎΠ±ΠΈ Π½Π° число

Π§Ρ‚ΠΎΠ±Ρ‹ ΡƒΠΌΠ½ΠΎΠΆΠΈΡ‚ΡŒ Π΄Ρ€ΠΎΠ±ΡŒ Π½Π° число, Π½ΡƒΠΆΠ½ΠΎ Ρ‡ΠΈΡΠ»ΠΈΡ‚Π΅Π»ΡŒ Π΄Π°Π½Π½ΠΎΠΉ Π΄Ρ€ΠΎΠ±ΠΈ ΡƒΠΌΠ½ΠΎΠΆΠΈΡ‚ΡŒ Π½Π° это число, Π° Π·Π½Π°ΠΌΠ΅Π½Π°Ρ‚Π΅Π»ΡŒ ΠΎΡΡ‚Π°Π²ΠΈΡ‚ΡŒ ΠΏΡ€Π΅ΠΆΠ½ΠΈΠΌ.

ΠŸΡ€ΠΈΠΌΠ΅Ρ€ 1 . Π£ΠΌΠ½ΠΎΠΆΠΈΡ‚ΡŒ Π΄Ρ€ΠΎΠ±ΡŒ Π½Π° число 1 .

Π£ΠΌΠ½ΠΎΠΆΠΈΠΌ Ρ‡ΠΈΡΠ»ΠΈΡ‚Π΅Π»ΡŒ Π΄Ρ€ΠΎΠ±ΠΈ Π½Π° число 1

Π—Π°ΠΏΠΈΡΡŒ ΠΌΠΎΠΆΠ½ΠΎ ΠΏΠΎΠ½ΠΈΠΌΠ°Ρ‚ΡŒ, ΠΊΠ°ΠΊ Π²Π·ΡΡ‚ΡŒ ΠΏΠΎΠ»ΠΎΠ²ΠΈΠ½Ρƒ 1 Ρ€Π°Π·. К ΠΏΡ€ΠΈΠΌΠ΅Ρ€Ρƒ, Ссли ΠΏΠΈΡ†Ρ†Ρ‹ Π²Π·ΡΡ‚ΡŒ 1 Ρ€Π°Π·, Ρ‚ΠΎ получится ΠΏΠΈΡ†Ρ†Ρ‹

Из Π·Π°ΠΊΠΎΠ½ΠΎΠ² умноТСния ΠΌΡ‹ Π·Π½Π°Π΅ΠΌ, Ρ‡Ρ‚ΠΎ Ссли ΠΌΠ½ΠΎΠΆΠΈΠΌΠΎΠ΅ ΠΈ ΠΌΠ½ΠΎΠΆΠΈΡ‚Π΅Π»ΡŒ ΠΏΠΎΠΌΠ΅Π½ΡΡ‚ΡŒ мСстами, Ρ‚ΠΎ ΠΏΡ€ΠΎΠΈΠ·Π²Π΅Π΄Π΅Π½ΠΈΠ΅ Π½Π΅ измСнится. Если Π²Ρ‹Ρ€Π°ΠΆΠ΅Π½ΠΈΠ΅ , Π·Π°ΠΏΠΈΡΠ°Ρ‚ΡŒ ΠΊΠ°ΠΊ , Ρ‚ΠΎ ΠΏΡ€ΠΎΠΈΠ·Π²Π΅Π΄Π΅Π½ΠΈΠ΅ ΠΏΠΎ ΠΏΡ€Π΅ΠΆΠ½Π΅ΠΌΡƒ Π±ΡƒΠ΄Π΅Ρ‚ Ρ€Π°Π²Π½ΠΎ . ΠžΠΏΡΡ‚ΡŒ ΠΆΠ΅ срабатываСт ΠΏΡ€Π°Π²ΠΈΠ»ΠΎ пСрСмноТСния Ρ†Π΅Π»ΠΎΠ³ΠΎ числа ΠΈ Π΄Ρ€ΠΎΠ±ΠΈ:

Π­Ρ‚Ρƒ запись ΠΌΠΎΠΆΠ½ΠΎ ΠΏΠΎΠ½ΠΈΠΌΠ°Ρ‚ΡŒ, ΠΊΠ°ΠΊ взятиС ΠΏΠΎΠ»ΠΎΠ²ΠΈΠ½Ρ‹ ΠΎΡ‚ Π΅Π΄ΠΈΠ½ΠΈΡ†Ρ‹. К ΠΏΡ€ΠΈΠΌΠ΅Ρ€Ρƒ, Ссли имССтся 1 цСлая ΠΏΠΈΡ†Ρ†Π° ΠΈ ΠΌΡ‹ возьмСм ΠΎΡ‚ Π½Π΅Ρ‘ ΠΏΠΎΠ»ΠΎΠ²ΠΈΠ½Ρƒ, Ρ‚ΠΎ Ρƒ нас окаТСтся ΠΏΠΈΡ†Ρ†Ρ‹:

ΠŸΡ€ΠΈΠΌΠ΅Ρ€ 2 . Найти Π·Π½Π°Ρ‡Π΅Π½ΠΈΠ΅ выраТСния

Π£ΠΌΠ½ΠΎΠΆΠΈΠΌ Ρ‡ΠΈΡΠ»ΠΈΡ‚Π΅Π»ΡŒ Π΄Ρ€ΠΎΠ±ΠΈ Π½Π° 4

Π’ ΠΎΡ‚Π²Π΅Ρ‚Π΅ ΠΏΠΎΠ»ΡƒΡ‡ΠΈΠ»Π°ΡΡŒ Π½Π΅ΠΏΡ€Π°Π²ΠΈΠ»ΡŒΠ½Π°Ρ Π΄Ρ€ΠΎΠ±ΡŒ. Π’Ρ‹Π΄Π΅Π»ΠΈΠΌ Π² Π½Π΅ΠΉ Ρ†Π΅Π»ΡƒΡŽ Ρ‡Π°ΡΡ‚ΡŒ:

Π’Ρ‹Ρ€Π°ΠΆΠ΅Π½ΠΈΠ΅ ΠΌΠΎΠΆΠ½ΠΎ ΠΏΠΎΠ½ΠΈΠΌΠ°Ρ‚ΡŒ, ΠΊΠ°ΠΊ взятиС Π΄Π²ΡƒΡ… Ρ‡Π΅Ρ‚Π²Π΅Ρ€Ρ‚Π΅ΠΉ 4 Ρ€Π°Π·Π°. К ΠΏΡ€ΠΈΠΌΠ΅Ρ€Ρƒ, Ссли ΠΏΠΈΡ†Ρ†Ρ‹ Π²Π·ΡΡ‚ΡŒ 4 Ρ€Π°Π·Π°, Ρ‚ΠΎ получится Π΄Π²Π΅ Ρ†Π΅Π»Ρ‹Π΅ ΠΏΠΈΡ†Ρ†Ρ‹

А Ссли ΠΏΠΎΠΌΠ΅Π½ΡΡ‚ΡŒ ΠΌΠ½ΠΎΠΆΠΈΠΌΠΎΠ΅ ΠΈ ΠΌΠ½ΠΎΠΆΠΈΡ‚Π΅Π»ΡŒ мСстами, Ρ‚ΠΎ ΠΏΠΎΠ»ΡƒΡ‡ΠΈΠΌ Π²Ρ‹Ρ€Π°ΠΆΠ΅Π½ΠΈΠ΅ . Оно Ρ‚ΠΎΠΆΠ΅ Π±ΡƒΠ΄Π΅Ρ‚ Ρ€Π°Π²Π½ΠΎ 2. Π­Ρ‚ΠΎ Π²Ρ‹Ρ€Π°ΠΆΠ΅Π½ΠΈΠ΅ ΠΌΠΎΠΆΠ½ΠΎ ΠΏΠΎΠ½ΠΈΠΌΠ°Ρ‚ΡŒ, ΠΊΠ°ΠΊ взятиС Π΄Π²ΡƒΡ… ΠΏΠΈΡ†Ρ† ΠΎΡ‚ Ρ‡Π΅Ρ‚Ρ‹Ρ€Π΅Ρ… Ρ†Π΅Π»Ρ‹Ρ… ΠΏΠΈΡ†Ρ†:

Π£ΠΌΠ½ΠΎΠΆΠ΅Π½ΠΈΠ΅ Π΄Ρ€ΠΎΠ±Π΅ΠΉ

Π§Ρ‚ΠΎΠ±Ρ‹ ΠΏΠ΅Ρ€Π΅ΠΌΠ½ΠΎΠΆΠΈΡ‚ΡŒ Π΄Ρ€ΠΎΠ±ΠΈ, Π½ΡƒΠΆΠ½ΠΎ ΠΏΠ΅Ρ€Π΅ΠΌΠ½ΠΎΠΆΠΈΡ‚ΡŒ ΠΈΡ… числитСли ΠΈ Π·Π½Π°ΠΌΠ΅Π½Π°Ρ‚Π΅Π»ΠΈ. Если Π² ΠΎΡ‚Π²Π΅Ρ‚Π΅ получится Π½Π΅ΠΏΡ€Π°Π²ΠΈΠ»ΡŒΠ½Π°Ρ Π΄Ρ€ΠΎΠ±ΡŒ, Π½ΡƒΠΆΠ½ΠΎ Π²Ρ‹Π΄Π΅Π»ΠΈΡ‚ΡŒ Π² Π½Π΅ΠΉ Ρ†Π΅Π»ΡƒΡŽ Ρ‡Π°ΡΡ‚ΡŒ.

ΠŸΡ€ΠΈΠΌΠ΅Ρ€ 1. Найти Π·Π½Π°Ρ‡Π΅Π½ΠΈΠ΅ выраТСния .

ΠŸΠΎΠ»ΡƒΡ‡ΠΈΠ»ΠΈ ΠΎΡ‚Π²Π΅Ρ‚ . Π–Π΅Π»Π°Ρ‚Π΅Π»ΡŒΠ½ΠΎ ΡΠΎΠΊΡ€Π°Ρ‚ΠΈΡ‚ΡŒ Π΄Π°Π½Π½ΡƒΡŽ Π΄Ρ€ΠΎΠ±ΡŒ. Π”Ρ€ΠΎΠ±ΡŒ ΠΌΠΎΠΆΠ½ΠΎ ΡΠΎΠΊΡ€Π°Ρ‚ΠΈΡ‚ΡŒ Π½Π° 2. Π’ΠΎΠ³Π΄Π° ΠΎΠΊΠΎΠ½Ρ‡Π°Ρ‚Π΅Π»ΡŒΠ½ΠΎΠ΅ Ρ€Π΅ΡˆΠ΅Π½ΠΈΠ΅ ΠΏΡ€ΠΈΠΌΠ΅Ρ‚ ΡΠ»Π΅Π΄ΡƒΡŽΡ‰ΠΈΠΉ Π²ΠΈΠ΄:

Π’Ρ‹Ρ€Π°ΠΆΠ΅Π½ΠΈΠ΅ ΠΌΠΎΠΆΠ½ΠΎ ΠΏΠΎΠ½ΠΈΠΌΠ°Ρ‚ΡŒ, ΠΊΠ°ΠΊ взятиС ΠΏΠΈΡ†Ρ†Ρ‹ ΠΎΡ‚ ΠΏΠΎΠ»ΠΎΠ²ΠΈΠ½Ρ‹ ΠΏΠΈΡ†Ρ†Ρ‹. Допустим, Ρƒ нас Π΅ΡΡ‚ΡŒ ΠΏΠΎΠ»ΠΎΠ²ΠΈΠ½Π° ΠΏΠΈΡ†Ρ†Ρ‹:

Как Π²Π·ΡΡ‚ΡŒ ΠΎΡ‚ этой ΠΏΠΎΠ»ΠΎΠ²ΠΈΠ½Ρ‹ Π΄Π²Π΅ Ρ‚Ρ€Π΅Ρ‚ΡŒΠΈΡ…? Π‘Π½Π°Ρ‡Π°Π»Π° Π½ΡƒΠΆΠ½ΠΎ ΠΏΠΎΠ΄Π΅Π»ΠΈΡ‚ΡŒ эту ΠΏΠΎΠ»ΠΎΠ²ΠΈΠ½Ρƒ Π½Π° Ρ‚Ρ€ΠΈ Ρ€Π°Π²Π½Ρ‹Π΅ части:

И Π²Π·ΡΡ‚ΡŒ ΠΎΡ‚ этих Ρ‚Ρ€Π΅Ρ… кусочков Π΄Π²Π°:

Π£ нас получится ΠΏΠΈΡ†Ρ†Ρ‹. ВспомнитС, ΠΊΠ°ΠΊ выглядит ΠΏΠΈΡ†Ρ†Π°, раздСлСнная Π½Π° Ρ‚Ρ€ΠΈ части:

Один кусок ΠΎΡ‚ этой ΠΏΠΈΡ†Ρ†Ρ‹ ΠΈ взятыС Π½Π°ΠΌΠΈ Π΄Π²Π° кусочка Π±ΡƒΠ΄ΡƒΡ‚ ΠΈΠΌΠ΅Ρ‚ΡŒ ΠΎΠ΄ΠΈΠ½Π°ΠΊΠΎΠ²Ρ‹Π΅ Ρ€Π°Π·ΠΌΠ΅Ρ€Ρ‹:

Π”Ρ€ΡƒΠ³ΠΈΠΌΠΈ словами, Ρ€Π΅Ρ‡ΡŒ ΠΈΠ΄Π΅Ρ‚ ΠΎΠ± ΠΎΠ΄Π½ΠΎΠΌ ΠΈ Ρ‚ΠΎΠΌ ΠΆΠ΅ Ρ€Π°Π·ΠΌΠ΅Ρ€Π΅ ΠΏΠΈΡ†Ρ†Ρ‹. ΠŸΠΎΡΡ‚ΠΎΠΌΡƒ Π·Π½Π°Ρ‡Π΅Π½ΠΈΠ΅ выраТСния Ρ€Π°Π²Π½ΠΎ

ΠŸΡ€ΠΈΠΌΠ΅Ρ€ 2 . Найти Π·Π½Π°Ρ‡Π΅Π½ΠΈΠ΅ выраТСния

Π£ΠΌΠ½ΠΎΠΆΠ°Π΅ΠΌ Ρ‡ΠΈΡΠ»ΠΈΡ‚Π΅Π»ΡŒ ΠΏΠ΅Ρ€Π²ΠΎΠΉ Π΄Ρ€ΠΎΠ±ΠΈ Π½Π° Ρ‡ΠΈΡΠ»ΠΈΡ‚Π΅Π»ΡŒ Π²Ρ‚ΠΎΡ€ΠΎΠΉ Π΄Ρ€ΠΎΠ±ΠΈ, Π° Π·Π½Π°ΠΌΠ΅Π½Π°Ρ‚Π΅Π»ΡŒ ΠΏΠ΅Ρ€Π²ΠΎΠΉ Π΄Ρ€ΠΎΠ±ΠΈ Π½Π° Π·Π½Π°ΠΌΠ΅Π½Π°Ρ‚Π΅Π»ΡŒ Π²Ρ‚ΠΎΡ€ΠΎΠΉ Π΄Ρ€ΠΎΠ±ΠΈ:

Π’ ΠΎΡ‚Π²Π΅Ρ‚Π΅ ΠΏΠΎΠ»ΡƒΡ‡ΠΈΠ»Π°ΡΡŒ Π½Π΅ΠΏΡ€Π°Π²ΠΈΠ»ΡŒΠ½Π°Ρ Π΄Ρ€ΠΎΠ±ΡŒ. Π’Ρ‹Π΄Π΅Π»ΠΈΠΌ Π² Π½Π΅ΠΉ Ρ†Π΅Π»ΡƒΡŽ Ρ‡Π°ΡΡ‚ΡŒ:

ΠŸΡ€ΠΈΠΌΠ΅Ρ€ 3. Найти Π·Π½Π°Ρ‡Π΅Π½ΠΈΠ΅ выраТСния

Π£ΠΌΠ½ΠΎΠΆΠ°Π΅ΠΌ Ρ‡ΠΈΡΠ»ΠΈΡ‚Π΅Π»ΡŒ ΠΏΠ΅Ρ€Π²ΠΎΠΉ Π΄Ρ€ΠΎΠ±ΠΈ Π½Π° Ρ‡ΠΈΡΠ»ΠΈΡ‚Π΅Π»ΡŒ Π²Ρ‚ΠΎΡ€ΠΎΠΉ Π΄Ρ€ΠΎΠ±ΠΈ, Π° Π·Π½Π°ΠΌΠ΅Π½Π°Ρ‚Π΅Π»ΡŒ ΠΏΠ΅Ρ€Π²ΠΎΠΉ Π΄Ρ€ΠΎΠ±ΠΈ Π½Π° Π·Π½Π°ΠΌΠ΅Π½Π°Ρ‚Π΅Π»ΡŒ Π²Ρ‚ΠΎΡ€ΠΎΠΉ Π΄Ρ€ΠΎΠ±ΠΈ:

Π’ ΠΎΡ‚Π²Π΅Ρ‚Π΅ ΠΏΠΎΠ»ΡƒΡ‡ΠΈΠ»Π°ΡΡŒ ΠΏΡ€Π°Π²ΠΈΠ»ΡŒΠ½Π°Ρ Π΄Ρ€ΠΎΠ±ΡŒ, Π½ΠΎ Π±ΡƒΠ΄Π΅Ρ‚ Ρ…ΠΎΡ€ΠΎΡˆΠΎ, Ссли Π΅Ρ‘ ΡΠΎΠΊΡ€Π°Ρ‚ΠΈΡ‚ΡŒ. Π§Ρ‚ΠΎΠ±Ρ‹ ΡΠΎΠΊΡ€Π°Ρ‚ΠΈΡ‚ΡŒ эту Π΄Ρ€ΠΎΠ±ΡŒ, Π½ΡƒΠΆΠ½ΠΎ Ρ‡ΠΈΡΠ»ΠΈΡ‚Π΅Π»ΡŒ ΠΈ Π·Π½Π°ΠΌΠ΅Π½Π°Ρ‚Π΅Π»ΡŒ Π΄Π°Π½Π½ΠΎΠΉ Π΄Ρ€ΠΎΠ±ΠΈ Ρ€Π°Π·Π΄Π΅Π»ΠΈΡ‚ΡŒ Π½Π° наибольший ΠΎΠ±Ρ‰ΠΈΠΉ Π΄Π΅Π»ΠΈΡ‚Π΅Π»ΡŒ (ΠΠžΠ”) чисСл 105 ΠΈ 450.

Π˜Ρ‚Π°ΠΊ, Π½Π°ΠΉΠ΄Ρ‘ΠΌ ΠΠžΠ” чисСл 105 ΠΈ 450:

Π’Π΅ΠΏΠ΅Ρ€ΡŒ Π΄Π΅Π»ΠΈΠΌ Ρ‡ΠΈΡΠ»ΠΈΡ‚Π΅Π»ΡŒ ΠΈ Π·Π½Π°ΠΌΠ΅Π½Π°Ρ‚Π΅Π»ΡŒ нашСго ΠΎΡ‚Π²Π΅Ρ‚Π° Π½Π° ΠΠžΠ”, ΠΊΠΎΡ‚ΠΎΡ€Ρ‹ΠΉ ΠΌΡ‹ сСйчас нашли, Ρ‚ΠΎ Π΅ΡΡ‚ΡŒ Π½Π° 15

ΠŸΡ€Π΅Π΄ΡΡ‚Π°Π²Π»Π΅Π½ΠΈΠ΅ Ρ†Π΅Π»ΠΎΠ³ΠΎ числа Π² Π²ΠΈΠ΄Π΅ Π΄Ρ€ΠΎΠ±ΠΈ

Π›ΡŽΠ±ΠΎΠ΅ Ρ†Π΅Π»ΠΎΠ΅ число ΠΌΠΎΠΆΠ½ΠΎ ΠΏΡ€Π΅Π΄ΡΡ‚Π°Π²ΠΈΡ‚ΡŒ Π² Π²ΠΈΠ΄Π΅ Π΄Ρ€ΠΎΠ±ΠΈ. НапримСр, число 5 ΠΌΠΎΠΆΠ½ΠΎ ΠΏΡ€Π΅Π΄ΡΡ‚Π°Π²ΠΈΡ‚ΡŒ ΠΊΠ°ΠΊ . ΠžΡ‚ этого пятёрка своСго значСния Π½Π΅ помСняСт, ΠΏΠΎΡΠΊΠΎΠ»ΡŒΠΊΡƒ Π²Ρ‹Ρ€Π°ΠΆΠ΅Π½ΠΈΠ΅ ΠΎΠ·Π½Π°Ρ‡Π°Π΅Ρ‚ «число ΠΏΡΡ‚ΡŒ Ρ€Π°Π·Π΄Π΅Π»ΠΈΡ‚ΡŒ Π½Π° Π΅Π΄ΠΈΠ½ΠΈΡ†ΡƒΒ», Π° это, ΠΊΠ°ΠΊ извСстно Ρ€Π°Π²Π½ΠΎ пятёркС:

ΠžΠ±Ρ€Π°Ρ‚Π½Ρ‹Π΅ числа

БСйчас ΠΌΡ‹ познакомимся с ΠΎΡ‡Π΅Π½ΡŒ интСрСсной Ρ‚Π΅ΠΌΠΎΠΉ Π² ΠΌΠ°Ρ‚Π΅ΠΌΠ°Ρ‚ΠΈΠΊΠ΅. Она называСтся Β«ΠΎΠ±Ρ€Π°Ρ‚Π½Ρ‹Π΅ числа».

ΠžΠΏΡ€Π΅Π΄Π΅Π»Π΅Π½ΠΈΠ΅. ΠžΠ±Ρ€Π°Ρ‚Π½Ρ‹ΠΌ ΠΊ числу a называСтся число, ΠΊΠΎΡ‚ΠΎΡ€ΠΎΠ΅ ΠΏΡ€ΠΈ ΡƒΠΌΠ½ΠΎΠΆΠ΅Π½ΠΈΠΈ Π½Π° a Π΄Π°Ρ‘Ρ‚ Π΅Π΄ΠΈΠ½ΠΈΡ†Ρƒ.

Π”Π°Π²Π°ΠΉΡ‚Π΅ подставим Π² это ΠΎΠΏΡ€Π΅Π΄Π΅Π»Π΅Π½ΠΈΠ΅ вмСсто ΠΏΠ΅Ρ€Π΅ΠΌΠ΅Π½Π½ΠΎΠΉ a число 5 ΠΈ ΠΏΠΎΠΏΡ€ΠΎΠ±ΡƒΠ΅ΠΌ ΠΏΡ€ΠΎΡ‡ΠΈΡ‚Π°Ρ‚ΡŒ ΠΎΠΏΡ€Π΅Π΄Π΅Π»Π΅Π½ΠΈΠ΅:

ΠžΠ±Ρ€Π°Ρ‚Π½Ρ‹ΠΌ ΠΊ числу 5 называСтся число, ΠΊΠΎΡ‚ΠΎΡ€ΠΎΠ΅ ΠΏΡ€ΠΈ ΡƒΠΌΠ½ΠΎΠΆΠ΅Π½ΠΈΠΈ Π½Π° 5 Π΄Π°Ρ‘Ρ‚ Π΅Π΄ΠΈΠ½ΠΈΡ†Ρƒ.

МоТно Π»ΠΈ Π½Π°ΠΉΡ‚ΠΈ Ρ‚Π°ΠΊΠΎΠ΅ число, ΠΊΠΎΡ‚ΠΎΡ€ΠΎΠ΅ ΠΏΡ€ΠΈ ΡƒΠΌΠ½ΠΎΠΆΠ΅Π½ΠΈΠΈ Π½Π° 5, Π΄Π°Ρ‘Ρ‚ Π΅Π΄ΠΈΠ½ΠΈΡ†Ρƒ? ΠžΠΊΠ°Π·Ρ‹Π²Π°Π΅Ρ‚ΡΡ ΠΌΠΎΠΆΠ½ΠΎ. ΠŸΡ€Π΅Π΄ΡΡ‚Π°Π²ΠΈΠΌ пятёрку Π² Π²ΠΈΠ΄Π΅ Π΄Ρ€ΠΎΠ±ΠΈ:

Π—Π°Ρ‚Π΅ΠΌ ΡƒΠΌΠ½ΠΎΠΆΠΈΡ‚ΡŒ эту Π΄Ρ€ΠΎΠ±ΡŒ Π½Π° саму сСбя, Ρ‚ΠΎΠ»ΡŒΠΊΠΎ помСняСм мСстами Ρ‡ΠΈΡΠ»ΠΈΡ‚Π΅Π»ΡŒ ΠΈ Π·Π½Π°ΠΌΠ΅Π½Π°Ρ‚Π΅Π»ΡŒ. Π”Ρ€ΡƒΠ³ΠΈΠΌΠΈ словами, ΡƒΠΌΠ½ΠΎΠΆΠΈΠΌ Π΄Ρ€ΠΎΠ±ΡŒ Π½Π° саму сСбя, Ρ‚ΠΎΠ»ΡŒΠΊΠΎ ΠΏΠ΅Ρ€Π΅Π²Ρ‘Ρ€Π½ΡƒΡ‚ΡƒΡŽ:

Π§Ρ‚ΠΎ получится Π² Ρ€Π΅Π·ΡƒΠ»ΡŒΡ‚Π°Ρ‚Π΅ этого? Если ΠΌΡ‹ ΠΏΡ€ΠΎΠ΄ΠΎΠ»ΠΆΠΈΠΌ Ρ€Π΅ΡˆΠ°Ρ‚ΡŒ этот ΠΏΡ€ΠΈΠΌΠ΅Ρ€, Ρ‚ΠΎ ΠΏΠΎΠ»ΡƒΡ‡ΠΈΠΌ Π΅Π΄ΠΈΠ½ΠΈΡ†Ρƒ:

Π—Π½Π°Ρ‡ΠΈΡ‚ ΠΎΠ±Ρ€Π°Ρ‚Π½Ρ‹ΠΌ ΠΊ числу 5, являСтся число , ΠΏΠΎΡΠΊΠΎΠ»ΡŒΠΊΡƒ ΠΏΡ€ΠΈ ΡƒΠΌΠ½ΠΎΠΆΠ΅Π½ΠΈΠΈ 5 Π½Π° получаСтся Π΅Π΄ΠΈΠ½ΠΈΡ†Π°.

ΠžΠ±Ρ€Π°Ρ‚Π½ΠΎΠ΅ число ΠΌΠΎΠΆΠ½ΠΎ Π½Π°ΠΉΡ‚ΠΈ Ρ‚Π°ΠΊΠΆΠ΅ для любого Π΄Ρ€ΡƒΠ³ΠΎΠ³ΠΎ Ρ†Π΅Π»ΠΎΠ³ΠΎ числа.

Найти ΠΎΠ±Ρ€Π°Ρ‚Π½ΠΎΠ΅ число ΠΌΠΎΠΆΠ½ΠΎ Ρ‚Π°ΠΊΠΆΠ΅ для любой Π΄Ρ€ΡƒΠ³ΠΎΠΉ Π΄Ρ€ΠΎΠ±ΠΈ. Для этого достаточно ΠΏΠ΅Ρ€Π΅Π²Π΅Ρ€Π½ΡƒΡ‚ΡŒ Π΅Ρ‘.

Π”Π΅Π»Π΅Π½ΠΈΠ΅ Π΄Ρ€ΠΎΠ±ΠΈ Π½Π° число

Допустим, Ρƒ нас имССтся ΠΏΠΎΠ»ΠΎΠ²ΠΈΠ½Π° ΠΏΠΈΡ†Ρ†Ρ‹:

Π Π°Π·Π΄Π΅Π»ΠΈΠΌ Π΅Ρ‘ ΠΏΠΎΡ€ΠΎΠ²Π½Ρƒ Π½Π° Π΄Π²ΠΎΠΈΡ…. Бколько ΠΏΠΈΡ†Ρ†Ρ‹ достанСтся ΠΊΠ°ΠΆΠ΄ΠΎΠΌΡƒ?

Π’ΠΈΠ΄Π½ΠΎ, Ρ‡Ρ‚ΠΎ послС раздСлСния ΠΏΠΎΠ»ΠΎΠ²ΠΈΠ½Ρ‹ ΠΏΠΈΡ†Ρ†Ρ‹ ΠΏΠΎΠ»ΡƒΡ‡ΠΈΠ»ΠΎΡΡŒ Π΄Π²Π° Ρ€Π°Π²Π½Ρ‹Ρ… кусочка, ΠΊΠ°ΠΆΠ΄Ρ‹ΠΉ ΠΈΠ· ΠΊΠΎΡ‚ΠΎΡ€Ρ‹Ρ… составляСт ΠΏΠΈΡ†Ρ†Ρ‹. Π—Π½Π°Ρ‡ΠΈΡ‚ ΠΊΠ°ΠΆΠ΄ΠΎΠΌΡƒ достанСтся ΠΏΠΎ ΠΏΠΈΡ†Ρ†Ρ‹.

Π”Π΅Π»Π΅Π½ΠΈΠ΅ Π΄Ρ€ΠΎΠ±Π΅ΠΉ выполняСтся с ΠΏΠΎΠΌΠΎΡ‰ΡŒΡŽ ΠΎΠ±Ρ€Π°Ρ‚Π½Ρ‹Ρ… чисСл. ΠžΠ±Ρ€Π°Ρ‚Π½Ρ‹Π΅ числа ΠΏΠΎΠ·Π²ΠΎΠ»ΡΡŽΡ‚ Π·Π°ΠΌΠ΅Π½ΠΈΡ‚ΡŒ Π΄Π΅Π»Π΅Π½ΠΈΠ΅ ΡƒΠΌΠ½ΠΎΠΆΠ΅Π½ΠΈΠ΅ΠΌ.

Π§Ρ‚ΠΎΠ±Ρ‹ Ρ€Π°Π·Π΄Π΅Π»ΠΈΡ‚ΡŒ Π΄Ρ€ΠΎΠ±ΡŒ Π½Π° число, Π½ΡƒΠΆΠ½ΠΎ эту Π΄Ρ€ΠΎΠ±ΡŒ ΡƒΠΌΠ½ΠΎΠΆΠΈΡ‚ΡŒ Π½Π° число, ΠΎΠ±Ρ€Π°Ρ‚Π½ΠΎΠ΅ Π΄Π΅Π»ΠΈΡ‚Π΅Π»ΡŽ.

ΠŸΠΎΠ»ΡŒΠ·ΡƒΡΡΡŒ этим ΠΏΡ€Π°Π²ΠΈΠ»ΠΎΠΌ, запишСм Π΄Π΅Π»Π΅Π½ΠΈΠ΅ нашСй ΠΏΠΎΠ»ΠΎΠ²ΠΈΠ½Ρ‹ ΠΏΠΈΡ†Ρ†Ρ‹ Π½Π° Π΄Π²Π΅ части.

Π˜Ρ‚Π°ΠΊ, трСбуСтся Ρ€Π°Π·Π΄Π΅Π»ΠΈΡ‚ΡŒ Π΄Ρ€ΠΎΠ±ΡŒ Π½Π° число 2 . Π—Π΄Π΅ΡΡŒ Π΄Π΅Π»ΠΈΠΌΡ‹ΠΌ являСтся Π΄Ρ€ΠΎΠ±ΡŒ , Π° Π΄Π΅Π»ΠΈΡ‚Π΅Π»Π΅ΠΌ число 2.

Π§Ρ‚ΠΎΠ±Ρ‹ Ρ€Π°Π·Π΄Π΅Π»ΠΈΡ‚ΡŒ Π΄Ρ€ΠΎΠ±ΡŒ Π½Π° число 2, Π½ΡƒΠΆΠ½ΠΎ эту Π΄Ρ€ΠΎΠ±ΡŒ ΡƒΠΌΠ½ΠΎΠΆΠΈΡ‚ΡŒ Π½Π° число, ΠΎΠ±Ρ€Π°Ρ‚Π½ΠΎΠ΅ Π΄Π΅Π»ΠΈΡ‚Π΅Π»ΡŽ 2. ΠžΠ±Ρ€Π°Ρ‚Π½ΠΎΠ΅ Π΄Π΅Π»ΠΈΡ‚Π΅Π»ΡŽ 2 это Π΄Ρ€ΠΎΠ±ΡŒ . Π—Π½Π°Ρ‡ΠΈΡ‚ Π½ΡƒΠΆΠ½ΠΎ ΡƒΠΌΠ½ΠΎΠΆΠΈΡ‚ΡŒ Π½Π°

Для Ρ€Π΅ΡˆΠ΅Π½ΠΈΡ Ρ€Π°Π·Π»ΠΈΡ‡Π½Ρ‹Ρ… Π·Π°Π΄Π°Π½ΠΈΠΉ ΠΈΠ· курса ΠΌΠ°Ρ‚Π΅ΠΌΠ°Ρ‚ΠΈΠΊΠΈ, Ρ„ΠΈΠ·ΠΈΠΊΠΈ приходится ΠΏΡ€ΠΎΠΈΠ·Π²ΠΎΠ΄ΠΈΡ‚ΡŒ Π΄Π΅Π»Π΅Π½ΠΈΠ΅ Π΄Ρ€ΠΎΠ±Π΅ΠΉ. Π­Ρ‚ΠΎ ΡΠ΄Π΅Π»Π°Ρ‚ΡŒ ΠΎΡ‡Π΅Π½ΡŒ Π»Π΅Π³ΠΊΠΎ, Ссли Π·Π½Π°Ρ‚ΡŒ ΠΎΠΏΡ€Π΅Π΄Π΅Π»Π΅Π½Π½Ρ‹Π΅ ΠΏΡ€Π°Π²ΠΈΠ»Π° выполнСния этого матСматичСского дСйствия.

ΠŸΡ€Π΅ΠΆΠ΄Π΅ Ρ‡Π΅ΠΌ ΠΏΠ΅Ρ€Π΅ΠΉΡ‚ΠΈ ΠΊ Ρ„ΠΎΡ€ΠΌΡƒΠ»ΠΈΡ€ΠΎΠ²Π°Π½ΠΈΡŽ ΠΏΡ€Π°Π²ΠΈΠ»ΠΎ Ρ‚ΠΎΠΌ, ΠΊΠ°ΠΊ Π΄Π΅Π»ΠΈΡ‚ΡŒ Π΄Ρ€ΠΎΠ±ΠΈ, Π΄Π°Π²Π°ΠΉΡ‚Π΅ вспомним Π½Π΅ΠΊΠΎΡ‚ΠΎΡ€Ρ‹Π΅ матСматичСскиС Ρ‚Π΅Ρ€ΠΌΠΈΠ½Ρ‹:

  1. ВСрхняя Ρ‡Π°ΡΡ‚ΡŒ Π΄Ρ€ΠΎΠ±ΠΈ называСтся числитСлСм, Π° ниТняя – Π·Π½Π°ΠΌΠ΅Π½Π°Ρ‚Π΅Π»Π΅ΠΌ.
  2. ΠŸΡ€ΠΈ Π΄Π΅Π»Π΅Π½ΠΈΠΈ числа Π½Π°Π·Ρ‹Π²Π°ΡŽΡ‚ΡΡ Ρ‚Π°ΠΊ: Π΄Π΅Π»ΠΈΠΌΠΎΠ΅: Π΄Π΅Π»ΠΈΡ‚Π΅Π»ΡŒ = частноС

Как Π΄Π΅Π»ΠΈΡ‚ΡŒ Π΄Ρ€ΠΎΠ±ΠΈ: простыС Π΄Ρ€ΠΎΠ±ΠΈ

Для выполнСния дСлСния Π΄Π²ΡƒΡ… простых Π΄Ρ€ΠΎΠ±Π΅ΠΉ слСдуСт ΡƒΠΌΠ½ΠΎΠΆΠΈΡ‚ΡŒ Π΄Π΅Π»ΠΈΠΌΠΎΠ΅ Π½Π° Π΄Ρ€ΠΎΠ±ΡŒ, ΠΎΠ±Ρ€Π°Ρ‚Π½ΡƒΡŽ Π΄Π΅Π»ΠΈΡ‚Π΅Π»ΡŽ. Π­Ρ‚Ρƒ Π΄Ρ€ΠΎΠ±ΡŒ ΠΏΠΎ-Π΄Ρ€ΡƒΠ³ΠΎΠΌΡƒ Π½Π°Π·Ρ‹Π²Π°ΡŽΡ‚ Π΅Ρ‰Π΅ ΠΏΠ΅Ρ€Π΅Π²Π΅Ρ€Π½ΡƒΡ‚ΠΎΠΉ, ΠΏΠΎΡ‚ΠΎΠΌΡƒ Ρ‡Ρ‚ΠΎ ΠΎΠ½Π° получаСтся Π² Ρ€Π΅Π·ΡƒΠ»ΡŒΡ‚Π°Ρ‚Π΅ Π·Π°ΠΌΠ΅Π½Ρ‹ мСстами числитСля ΠΈ знамСнатСля. НапримСр:

3/77: 1/11 = 3 /77 * 11 /1 = 3/7

Как Π΄Π΅Π»ΠΈΡ‚ΡŒ Π΄Ρ€ΠΎΠ±ΠΈ: ΡΠΌΠ΅ΡˆΠ°Π½Π½Ρ‹Π΅ Π΄Ρ€ΠΎΠ±ΠΈ

Если Π½Π°ΠΌ прСдстоит Ρ€Π°Π·Π΄Π΅Π»ΠΈΡ‚ΡŒ ΡΠΌΠ΅ΡˆΠ°Π½Π½Ρ‹Π΅ Π΄Ρ€ΠΎΠ±ΠΈ, Ρ‚ΠΎ здСсь Ρ‚ΠΎΠΆΠ΅ всС достаточно просто ΠΈ понятно. Π‘Π½Π°Ρ‡Π°Π»Π° ΠΏΠ΅Ρ€Π΅Π²ΠΎΠ΄ΠΈΠΌ ΡΠΌΠ΅ΡˆΠ°Π½Π½ΡƒΡŽ Π΄Ρ€ΠΎΠ±ΡŒ Π² ΠΎΠ±Ρ‹Ρ‡Π½ΡƒΡŽ Π½Π΅ΠΏΡ€Π°Π²ΠΈΠ»ΡŒΠ½ΡƒΡŽ Π΄Ρ€ΠΎΠ±ΡŒ. Для этого ΡƒΠΌΠ½ΠΎΠΆΠ°Π΅ΠΌ Π·Π½Π°ΠΌΠ΅Π½Π°Ρ‚Π΅Π»ΡŒ Ρ‚Π°ΠΊΠΎΠΉ Π΄Ρ€ΠΎΠ±ΠΈ Π½Π° Ρ†Π΅Π»ΠΎΠ΅ число ΠΈ Ρ‡ΠΈΡΠ»ΠΈΡ‚Π΅Π»ΡŒ прибавляСм ΠΊ ΠΏΠΎΠ»ΡƒΡ‡Π΅Π½Π½ΠΎΠΌΡƒ ΠΏΡ€ΠΎΠΈΠ·Π²Π΅Π΄Π΅Π½ΠΈΡŽ. Π’ ΠΈΡ‚ΠΎΠ³Π΅ ΠΌΡ‹ ΠΏΠΎΠ»ΡƒΡ‡ΠΈΠ»ΠΈ Π½ΠΎΠ²Ρ‹ΠΉ Ρ‡ΠΈΡΠ»ΠΈΡ‚Π΅Π»ΡŒ смСшанной Π΄Ρ€ΠΎΠ±ΠΈ, Π° Π·Π½Π°ΠΌΠ΅Π½Π°Ρ‚Π΅Π»ΡŒ Π΅Π΅ останСтся Π±Π΅Π· измСнСния. Π”Π°Π»ΡŒΡˆΠ΅ Π΄Π΅Π»Π΅Π½ΠΈΠ΅ Π΄Ρ€ΠΎΠ±Π΅ΠΉ Π±ΡƒΠ΄Π΅Ρ‚ ΠΎΡΡƒΡ‰Π΅ΡΡ‚Π²Π»ΡΡ‚ΡŒΡΡ Ρ‚ΠΎΡ‡Π½ΠΎ Ρ‚Π°ΠΊ ΠΆΠ΅, ΠΊΠ°ΠΊ ΠΈ Π΄Π΅Π»Π΅Π½ΠΈΠ΅ простых Π΄Ρ€ΠΎΠ±Π΅ΠΉ. НапримСр:

10 2/3: 4/15 = 32/3: 4/15 = 32/3 * 15 /4 = 40/1 = 40

Как Π΄Π΅Π»ΠΈΡ‚ΡŒ Π΄Ρ€ΠΎΠ±ΡŒ Π½Π° число

Для Ρ‚ΠΎΠ³ΠΎ Ρ‡Ρ‚ΠΎΠ±Ρ‹ Ρ€Π°Π·Π΄Π΅Π»ΠΈΡ‚ΡŒ ΠΏΡ€ΠΎΡΡ‚ΡƒΡŽ Π΄Ρ€ΠΎΠ±ΡŒ Π½Π° число, послСднСС слСдуСт Π½Π°ΠΏΠΈΡΠ°Ρ‚ΡŒ Π² Π²ΠΈΠ΄Π΅ Π΄Ρ€ΠΎΠ±ΠΈ (Π½Π΅ΠΏΡ€Π°Π²ΠΈΠ»ΡŒΠ½ΠΎΠΉ). Π­Ρ‚ΠΎ ΡΠ΄Π΅Π»Π°Ρ‚ΡŒ ΠΎΡ‡Π΅Π½ΡŒ Π»Π΅Π³ΠΊΠΎ: Π½Π° мСстС числитСля ΠΏΠΈΡˆΠ΅Ρ‚ΡΡ это число, Π° Π·Π½Π°ΠΌΠ΅Π½Π°Ρ‚Π΅Π»ΡŒ Ρ‚Π°ΠΊΠΎΠΉ Π΄Ρ€ΠΎΠ±ΠΈ Ρ€Π°Π²Π΅Π½ Π΅Π΄ΠΈΠ½ΠΈΡ†Π΅. Π”Π°Π»ΡŒΡˆΠ΅ Π΄Π΅Π»Π΅Π½ΠΈΠ΅ выполняСтся ΠΎΠ±Ρ‹Ρ‡Π½Ρ‹ΠΌ способом. Рассмотрим это Π½Π° ΠΏΡ€ΠΈΠΌΠ΅Ρ€Π΅:

5/11: 7 = 5/11: 7/1 = 5/11 * 1/7 = 5/77

Как Π΄Π΅Π»ΠΈΡ‚ΡŒ дСсятичныС Π΄Ρ€ΠΎΠ±ΠΈ

НСрСдко взрослый Ρ‡Π΅Π»ΠΎΠ²Π΅ΠΊ испытываСт затруднСния ΠΏΡ€ΠΈ нСобходимости Π±Π΅Π· ΠΏΠΎΠΌΠΎΡ‰ΠΈ ΠΊΠ°Π»ΡŒΠΊΡƒΠ»ΡΡ‚ΠΎΡ€Π° Ρ€Π°Π·Π΄Π΅Π»ΠΈΡ‚ΡŒ Ρ†Π΅Π»ΠΎΠ΅ число ΠΈΠ»ΠΈ Π΄Π΅ΡΡΡ‚ΠΈΡ‡Π½ΡƒΡŽ Π΄Ρ€ΠΎΠ±ΡŒ Π½Π° Π΄Π΅ΡΡΡ‚ΠΈΡ‡Π½ΡƒΡŽ Π΄Ρ€ΠΎΠ±ΡŒ.

Π˜Ρ‚Π°ΠΊ, Ρ‡Ρ‚ΠΎΠ±Ρ‹ Π²Ρ‹ΠΏΠΎΠ»Π½ΠΈΡ‚ΡŒ Π΄Π΅Π»Π΅Π½ΠΈΠ΅ дСсятичных Π΄Ρ€ΠΎΠ±Π΅ΠΉ, Π½ΡƒΠΆΠ½ΠΎ Π² Π΄Π΅Π»ΠΈΡ‚Π΅Π»Π΅ просто Π·Π°Ρ‡Π΅Ρ€ΠΊΠ½ΡƒΡ‚ΡŒ Π·Π°ΠΏΡΡ‚ΡƒΡŽ ΠΈ ΠΏΠ΅Ρ€Π΅ΡΡ‚Π°Ρ‚ΡŒ ΠΎΠ±Ρ€Π°Ρ‰Π°Ρ‚ΡŒ Π½Π° Π½Π΅Π΅ Π²Π½ΠΈΠΌΠ°Π½ΠΈΠ΅. Π’ Π΄Π΅Π»ΠΈΠΌΠΎΠΌ Π·Π°ΠΏΡΡ‚ΡƒΡŽ Π½ΡƒΠΆΠ½ΠΎ ΠΏΠ΅Ρ€Π΅Π΄Π²ΠΈΠ½ΡƒΡ‚ΡŒ Π²ΠΏΡ€Π°Π²ΠΎ Ρ€ΠΎΠ²Π½ΠΎ Π½Π° ΡΡ‚ΠΎΠ»ΡŒΠΊΠΎ Π·Π½Π°ΠΊΠΎΠ², сколько Π±Ρ‹Π»ΠΎ Π² Π΄Ρ€ΠΎΠ±Π½ΠΎΠΉ части дСлитСля, ΠΏΡ€ΠΈ нСобходимости дописывая Π½ΡƒΠ»ΠΈ. И дальшС производят ΠΎΠ±Ρ‹Ρ‡Π½ΠΎΠ΅ Π΄Π΅Π»Π΅Π½ΠΈΠ΅ Π½Π° Ρ†Π΅Π»ΠΎΠ΅ число. Π§Ρ‚ΠΎΠ±Ρ‹ это стало Π±ΠΎΠ»Π΅Π΅ понятно, ΠΏΡ€ΠΈΠ²Π΅Π΄Π΅ΠΌ ΡΠ»Π΅Π΄ΡƒΡŽΡ‰ΠΈΠΉ ΠΏΡ€ΠΈΠΌΠ΅Ρ€.

Π£ΠΌΠ½ΠΎΠΆΠ΅Π½ΠΈΠ΅ ΠΈ Π΄Π΅Π»Π΅Π½ΠΈΠ΅ Π΄Ρ€ΠΎΠ±Π΅ΠΉ.

Π’Π½ΠΈΠΌΠ°Π½ΠΈΠ΅!
К этой Ρ‚Π΅ΠΌΠ΅ ΠΈΠΌΠ΅ΡŽΡ‚ΡΡ Π΄ΠΎΠΏΠΎΠ»Π½ΠΈΡ‚Π΅Π»ΡŒΠ½Ρ‹Π΅
ΠΌΠ°Ρ‚Π΅Ρ€ΠΈΠ°Π»Ρ‹ Π² Особом Ρ€Π°Π·Π΄Π΅Π»Π΅ 555.
Для Ρ‚Π΅Ρ…, ΠΊΡ‚ΠΎ сильно «Π½Π΅ ΠΎΡ‡Π΅Π½ΡŒ…»
И для Ρ‚Π΅Ρ…, ΠΊΡ‚ΠΎ «ΠΎΡ‡Π΅Π½ΡŒ Π΄Π°ΠΆΠ΅. ..»)

Π­Ρ‚Π° опСрация Π³ΠΎΡ€Π°Π·Π΄ΠΎ приятнСС слоТСния-вычитания ! ΠŸΠΎΡ‚ΠΎΠΌΡƒ Ρ‡Ρ‚ΠΎ ΠΏΡ€ΠΎΡ‰Π΅. Напоминаю: Ρ‡Ρ‚ΠΎΠ±Ρ‹ ΡƒΠΌΠ½ΠΎΠΆΠΈΡ‚ΡŒ Π΄Ρ€ΠΎΠ±ΡŒ Π½Π° Π΄Ρ€ΠΎΠ±ΡŒ, Π½ΡƒΠΆΠ½ΠΎ ΠΏΠ΅Ρ€Π΅ΠΌΠ½ΠΎΠΆΠΈΡ‚ΡŒ числитСли (это Π±ΡƒΠ΄Π΅Ρ‚ Ρ‡ΠΈΡΠ»ΠΈΡ‚Π΅Π»ΡŒ Ρ€Π΅Π·ΡƒΠ»ΡŒΡ‚Π°Ρ‚Π°) ΠΈ Π·Π½Π°ΠΌΠ΅Π½Π°Ρ‚Π΅Π»ΠΈ (это Π±ΡƒΠ΄Π΅Ρ‚ Π·Π½Π°ΠΌΠ΅Π½Π°Ρ‚Π΅Π»ΡŒ). Π’ΠΎ Π΅ΡΡ‚ΡŒ:

НапримСр:

Всё ΠΏΡ€Π΅Π΄Π΅Π»ΡŒΠ½ΠΎ просто . И, поТалуйста, Π½Π΅ ΠΈΡ‰ΠΈΡ‚Π΅ ΠΎΠ±Ρ‰ΠΈΠΉ Π·Π½Π°ΠΌΠ΅Π½Π°Ρ‚Π΅Π»ΡŒ! НС Π½Π°Π΄ΠΎ Π΅Π³ΠΎ Π·Π΄Π΅ΡΡŒβ€¦

Π§Ρ‚ΠΎΠ±Ρ‹ Ρ€Π°Π·Π΄Π΅Π»ΠΈΡ‚ΡŒ Π΄Ρ€ΠΎΠ±ΡŒ Π½Π° Π΄Ρ€ΠΎΠ±ΡŒ, Π½ΡƒΠΆΠ½ΠΎ ΠΏΠ΅Ρ€Π΅Π²Π΅Ρ€Π½ΡƒΡ‚ΡŒ Π²Ρ‚ΠΎΡ€ΡƒΡŽ (это Π²Π°ΠΆΠ½ΠΎ!) Π΄Ρ€ΠΎΠ±ΡŒ ΠΈ ΠΈΡ… ΠΏΠ΅Ρ€Π΅ΠΌΠ½ΠΎΠΆΠΈΡ‚ΡŒ, Ρ‚.Π΅.:

НапримСр:

Если попалось ΡƒΠΌΠ½ΠΎΠΆΠ΅Π½ΠΈΠ΅ ΠΈΠ»ΠΈ Π΄Π΅Π»Π΅Π½ΠΈΠ΅ с Ρ†Π΅Π»Ρ‹ΠΌΠΈ числами ΠΈ дробями — Π½ΠΈΡ‡Π΅Π³ΠΎ ΡΡ‚Ρ€Π°ΡˆΠ½ΠΎΠ³ΠΎ. Как ΠΈ ΠΏΡ€ΠΈ слоТСнии, Π΄Π΅Π»Π°Π΅ΠΌ ΠΈΠ· Ρ†Π΅Π»ΠΎΠ³ΠΎ числа Π΄Ρ€ΠΎΠ±ΡŒ с Π΅Π΄ΠΈΠ½ΠΈΡ†Π΅ΠΉ Π² Π·Π½Π°ΠΌΠ΅Π½Π°Ρ‚Π΅Π»Π΅ — ΠΈ Π²ΠΏΠ΅Ρ€Ρ‘Π΄! НапримСр:

Π’ ΡΡ‚Π°Ρ€ΡˆΠΈΡ… классах часто приходится ΠΈΠΌΠ΅Ρ‚ΡŒ Π΄Π΅Π»ΠΎ с трСхэтаТными (Π° Ρ‚ΠΎ ΠΈ чСтырСхэтаТными!) дробями. НапримСр:

Как эту Π΄Ρ€ΠΎΠ±ΡŒ привСсти ΠΊ ΠΏΡ€ΠΈΠ»ΠΈΡ‡Π½ΠΎΠΌΡƒ Π²ΠΈΠ΄Ρƒ? Π”Π° ΠΎΡ‡Π΅Π½ΡŒ просто! Π˜ΡΠΏΠΎΠ»ΡŒΠ·ΠΎΠ²Π°Ρ‚ΡŒ Π΄Π΅Π»Π΅Π½ΠΈΠ΅ Ρ‡Π΅Ρ€Π΅Π· Π΄Π²Π΅ Ρ‚ΠΎΡ‡ΠΊΠΈ:

Но Π½Π΅ Π·Π°Π±Ρ‹Π²Π°ΠΉΡ‚Π΅ ΠΎ порядкС дСлСния! Π’ ΠΎΡ‚Π»ΠΈΡ‡ΠΈΠ΅ ΠΎΡ‚ умноТСния, здСсь это ΠΎΡ‡Π΅Π½ΡŒ Π²Π°ΠΆΠ½ΠΎ! ΠšΠΎΠ½Π΅Ρ‡Π½ΠΎ, 4:2, ΠΈΠ»ΠΈ 2:4 ΠΌΡ‹ Π½Π΅ спутаСм. А Π²ΠΎΡ‚ Π² трёхэтаТной Π΄Ρ€ΠΎΠ±ΠΈ Π»Π΅Π³ΠΊΠΎ ΠΎΡˆΠΈΠ±ΠΈΡ‚ΡŒΡΡ. ΠžΠ±Ρ€Π°Ρ‚ΠΈΡ‚Π΅ Π²Π½ΠΈΠΌΠ°Π½ΠΈΠ΅, Π½Π°ΠΏΡ€ΠΈΠΌΠ΅Ρ€:

Π’ ΠΏΠ΅Ρ€Π²ΠΎΠΌ случаС (Π²Ρ‹Ρ€Π°ΠΆΠ΅Π½ΠΈΠ΅ слСва):

Π’ΠΎ Π²Ρ‚ΠΎΡ€ΠΎΠΌ (Π²Ρ‹Ρ€Π°ΠΆΠ΅Π½ΠΈΠ΅ справа):

ЧувствуСтС Ρ€Π°Π·Π½ΠΈΡ†Ρƒ? 4 ΠΈ 1/9!

А Ρ‡Π΅ΠΌ задаСтся порядок дСлСния? Или скобками, ΠΈΠ»ΠΈ (ΠΊΠ°ΠΊ здСсь) Π΄Π»ΠΈΠ½ΠΎΠΉ Π³ΠΎΡ€ΠΈΠ·ΠΎΠ½Ρ‚Π°Π»ΡŒΠ½Ρ‹Ρ… Ρ‡Π΅Ρ€Ρ‚ΠΎΡ‡Π΅ΠΊ. Π Π°Π·Π²ΠΈΠ²Π°ΠΉΡ‚Π΅ Π³Π»Π°Π·ΠΎΠΌΠ΅Ρ€. А Ссли Π½Π΅Ρ‚ Π½ΠΈ скобок, Π½ΠΈ Ρ‡Π΅Ρ€Ρ‚ΠΎΡ‡Π΅ΠΊ, Ρ‚ΠΈΠΏΠ°:

Ρ‚ΠΎ Π΄Π΅Π»ΠΈΠΌ-ΡƒΠΌΠ½ΠΎΠΆΠ°Π΅ΠΌ ΠΏΠΎ порядочку, слСва Π½Π°ΠΏΡ€Π°Π²ΠΎ !

И Π΅Ρ‰Π΅ ΠΎΡ‡Π΅Π½ΡŒ простой ΠΈ Π²Π°ΠΆΠ½Ρ‹ΠΉ ΠΏΡ€ΠΈΡ‘ΠΌ. Π’ дСйствиях со стСпСнями ΠΎΠ½ Π²Π°ΠΌ ΠΎΡ… ΠΊΠ°ΠΊ пригодится! ПодСлим Π΅Π΄ΠΈΠ½ΠΈΡ†Ρƒ Π½Π° Π»ΡŽΠ±ΡƒΡŽ Π΄Ρ€ΠΎΠ±ΡŒ, Π½Π°ΠΏΡ€ΠΈΠΌΠ΅Ρ€, Π½Π° 13/15:

Π”Ρ€ΠΎΠ±ΡŒ ΠΏΠ΅Ρ€Π΅Π²Π΅Ρ€Π½ΡƒΠ»Π°ΡΡŒ! И Ρ‚Π°ΠΊ Π±Ρ‹Π²Π°Π΅Ρ‚ всСгда. ΠŸΡ€ΠΈ Π΄Π΅Π»Π΅Π½ΠΈΠΈ 1 Π½Π° Π»ΡŽΠ±ΡƒΡŽ Π΄Ρ€ΠΎΠ±ΡŒ, Π² Ρ€Π΅Π·ΡƒΠ»ΡŒΡ‚Π°Ρ‚Π΅ ΠΏΠΎΠ»ΡƒΡ‡Π°Π΅ΠΌ Ρ‚Ρƒ ΠΆΠ΅ Π΄Ρ€ΠΎΠ±ΡŒ, Ρ‚ΠΎΠ»ΡŒΠΊΠΎ ΠΏΠ΅Ρ€Π΅Π²Π΅Ρ€Π½ΡƒΡ‚ΡƒΡŽ.

Π’ΠΎΡ‚ ΠΈ всС дСйствия с дробями. Π’Π΅Ρ‰ΡŒ достаточно простая, Π½ΠΎ ошибок Π΄Π°Ρ‘Ρ‚ Π±ΠΎΠ»Π΅Π΅, Ρ‡Π΅ΠΌ достаточно. ΠŸΡ€ΠΈΠΌΠΈΡ‚Π΅ ΠΊ свСдСнию практичСскиС совСты, ΠΈ ΠΈΡ… (ошибок) Π±ΡƒΠ΄Π΅Ρ‚ мСньшС!

ΠŸΡ€Π°ΠΊΡ‚ΠΈΡ‡Π΅ΡΠΊΠΈΠ΅ совСты:

1. Π‘Π°ΠΌΠΎΠ΅ Π³Π»Π°Π²Π½ΠΎΠ΅ ΠΏΡ€ΠΈ Ρ€Π°Π±ΠΎΡ‚Π΅ с Π΄Ρ€ΠΎΠ±Π½Ρ‹ΠΌΠΈ выраТСниями — Π°ΠΊΠΊΡƒΡ€Π°Ρ‚Π½ΠΎΡΡ‚ΡŒ ΠΈ Π²Π½ΠΈΠΌΠ°Ρ‚Π΅Π»ΡŒΠ½ΠΎΡΡ‚ΡŒ! Π­Ρ‚ΠΎ Π½Π΅ ΠΎΠ±Ρ‰ΠΈΠ΅ слова, Π½Π΅ Π±Π»Π°Π³ΠΈΠ΅ поТСлания! Π­Ρ‚ΠΎ суровая Π½Π΅ΠΎΠ±Ρ…ΠΎΠ΄ΠΈΠΌΠΎΡΡ‚ΡŒ! ВсС вычислСния Π½Π° Π•Π“Π­ Π΄Π΅Π»Π°ΠΉΡ‚Π΅ ΠΊΠ°ΠΊ ΠΏΠΎΠ»Π½ΠΎΡ†Π΅Π½Π½ΠΎΠ΅ Π·Π°Π΄Π°Π½ΠΈΠ΅, сосрСдоточСнно ΠΈ Ρ‡Ρ‘Ρ‚ΠΊΠΎ. Π›ΡƒΡ‡ΡˆΠ΅ Π½Π°ΠΏΠΈΡΠ°Ρ‚ΡŒ Π΄Π²Π΅ лишниС строчки Π² Ρ‡Π΅Ρ€Π½ΠΎΠ²ΠΈΠΊΠ΅, Ρ‡Π΅ΠΌ Π½Π°ΠΊΠΎΡΡΡ‡ΠΈΡ‚ΡŒ ΠΏΡ€ΠΈ расчётС Π² ΡƒΠΌΠ΅.

2. Π’ ΠΏΡ€ΠΈΠΌΠ΅Ρ€Π°Ρ… с Ρ€Π°Π·Π½Ρ‹ΠΌΠΈ Π²ΠΈΠ΄Π°ΠΌΠΈ Π΄Ρ€ΠΎΠ±Π΅ΠΉ — ΠΏΠ΅Ρ€Π΅Ρ…ΠΎΠ΄ΠΈΠΌ ΠΊ ΠΎΠ±Ρ‹ΠΊΠ½ΠΎΠ²Π΅Π½Π½Ρ‹ΠΌ дробям.

3. ВсС Π΄Ρ€ΠΎΠ±ΠΈ сокращаСм Π΄ΠΎ ΡƒΠΏΠΎΡ€Π°.

4. ΠœΠ½ΠΎΠ³ΠΎΡΡ‚Π°ΠΆΠ½Ρ‹Π΅ Π΄Ρ€ΠΎΠ±Π½Ρ‹Π΅ выраТСния сводим ΠΊ ΠΎΠ±Ρ‹ΠΊΠ½ΠΎΠ²Π΅Π½Π½Ρ‹ΠΌ, ΠΈΡΠΏΠΎΠ»ΡŒΠ·ΡƒΡ Π΄Π΅Π»Π΅Π½ΠΈΠ΅ Ρ‡Π΅Ρ€Π΅Π· Π΄Π²Π΅ Ρ‚ΠΎΡ‡ΠΊΠΈ (слСдим Π·Π° порядком дСлСния!).

5. Π•Π΄ΠΈΠ½ΠΈΡ†Ρƒ Π½Π° Π΄Ρ€ΠΎΠ±ΡŒ Π΄Π΅Π»ΠΈΠΌ Π² ΡƒΠΌΠ΅, просто пСрСворачивая Π΄Ρ€ΠΎΠ±ΡŒ.

Π’ΠΎΡ‚ Π²Π°ΠΌ задания, ΠΊΠΎΡ‚ΠΎΡ€Ρ‹Π΅ Π½ΡƒΠΆΠ½ΠΎ ΠΎΠ±ΡΠ·Π°Ρ‚Π΅Π»ΡŒΠ½ΠΎ ΠΏΡ€ΠΎΡ€Π΅ΡˆΠ°Ρ‚ΡŒ. ΠžΡ‚Π²Π΅Ρ‚Ρ‹ Π΄Π°Π½Ρ‹ послС всСх Π·Π°Π΄Π°Π½ΠΈΠΉ. Π˜ΡΠΏΠΎΠ»ΡŒΠ·ΡƒΠΉΡ‚Π΅ ΠΌΠ°Ρ‚Π΅Ρ€ΠΈΠ°Π»Ρ‹ этой Ρ‚Π΅ΠΌΡ‹ ΠΈ практичСскиС совСты. ΠŸΡ€ΠΈΠΊΠΈΠ½ΡŒΡ‚Π΅, сколько ΠΏΡ€ΠΈΠΌΠ΅Ρ€ΠΎΠ² Π²Ρ‹ смогли Ρ€Π΅ΡˆΠΈΡ‚ΡŒ ΠΏΡ€Π°Π²ΠΈΠ»ΡŒΠ½ΠΎ. Π‘ ΠΏΠ΅Ρ€Π²ΠΎΠ³ΠΎ Ρ€Π°Π·Π°! Π‘Π΅Π· ΠΊΠ°Π»ΡŒΠΊΡƒΠ»ΡΡ‚ΠΎΡ€Π°! И сдСлайтС Π²Π΅Ρ€Π½Ρ‹Π΅ Π²Ρ‹Π²ΠΎΠ΄Ρ‹…

ΠŸΠΎΠΌΠ½ΠΈΡ‚Π΅ – ΠΏΡ€Π°Π²ΠΈΠ»ΡŒΠ½Ρ‹ΠΉ ΠΎΡ‚Π²Π΅Ρ‚, ΠΏΠΎΠ»ΡƒΡ‡Π΅Π½Π½Ρ‹ΠΉ со Π²Ρ‚ΠΎΡ€ΠΎΠ³ΠΎ (Ρ‚Π΅ΠΌ Π±ΠΎΠ»Π΅Π΅ – Ρ‚Ρ€Π΅Ρ‚ΡŒΠ΅Π³ΠΎ) Ρ€Π°Π·Π° – Π½Π΅ считаСтся! Π’Π°ΠΊΠΎΠ²Π° суровая Тизнь.

Π˜Ρ‚Π°ΠΊ, Ρ€Π΅ΡˆΠ°Π΅ΠΌ Π² Ρ€Π΅ΠΆΠΈΠΌΠ΅ экзамСна ! Π­Ρ‚ΠΎ ΡƒΠΆΠ΅ ΠΏΠΎΠ΄Π³ΠΎΡ‚ΠΎΠ²ΠΊΠ° ΠΊ Π•Π“Π­, ΠΌΠ΅ΠΆΠ΄Ρƒ ΠΏΡ€ΠΎΡ‡ΠΈΠΌ. РСшаСм ΠΏΡ€ΠΈΠΌΠ΅Ρ€, провСряСм, Ρ€Π΅ΡˆΠ°Π΅ΠΌ ΡΠ»Π΅Π΄ΡƒΡŽΡ‰ΠΈΠΉ. РСшили всС — ΠΏΡ€ΠΎΠ²Π΅Ρ€ΠΈΠ»ΠΈ снова с ΠΏΠ΅Ρ€Π²ΠΎΠ³ΠΎ ΠΏΠΎ послСдний. И Ρ‚ΠΎΠ»ΡŒΠΊΠΎ ΠΏΠΎΡ‚ΠΎΠΌ смотрим ΠΎΡ‚Π²Π΅Ρ‚Ρ‹.

Π’Ρ‹Ρ‡ΠΈΡΠ»ΠΈΡ‚ΡŒ:

ΠŸΠΎΡ€Π΅ΡˆΠ°Π»ΠΈ?

Π˜Ρ‰Π΅ΠΌ ΠΎΡ‚Π²Π΅Ρ‚Ρ‹, ΠΊΠΎΡ‚ΠΎΡ€Ρ‹Π΅ ΡΠΎΠ²ΠΏΠ°Π΄Π°ΡŽΡ‚ с вашими. Π― ΡΠΏΠ΅Ρ†ΠΈΠ°Π»ΡŒΠ½ΠΎ ΠΈΡ… Π² бСспорядкС записал, подальшС ΠΎΡ‚ соблазна, Ρ‚Π°ΠΊ ΡΠΊΠ°Π·Π°Ρ‚ΡŒ… Π’ΠΎΡ‚ ΠΎΠ½ΠΈ, ΠΎΡ‚Π²Π΅Ρ‚Ρ‹, Ρ‡Π΅Ρ€Π΅Π· Ρ‚ΠΎΡ‡ΠΊΡƒ с запятой записаны.

0; 17/22; 3/4; 2/5; 1; 25.

А Ρ‚Π΅ΠΏΠ΅Ρ€ΡŒ Π΄Π΅Π»Π°Π΅ΠΌ Π²Ρ‹Π²ΠΎΠ΄Ρ‹. Если всё ΠΏΠΎΠ»ΡƒΡ‡ΠΈΠ»ΠΎΡΡŒ — Ρ€Π°Π΄ Π·Π° вас! Π­Π»Π΅ΠΌΠ΅Π½Ρ‚Π°Ρ€Π½Ρ‹Π΅ вычислСния с дробями — Π½Π΅ ваша ΠΏΡ€ΠΎΠ±Π»Π΅ΠΌΠ°! МоТно Π·Π°Π½ΡΡ‚ΡŒΡΡ Π±ΠΎΠ»Π΅Π΅ ΡΠ΅Ρ€ΡŒΡ‘Π·Π½Ρ‹ΠΌΠΈ Π²Π΅Ρ‰Π°ΠΌΠΈ. Если Π½Π΅Ρ‚…

Π—Π½Π°Ρ‡ΠΈΡ‚, Ρƒ вас ΠΎΠ΄Π½Π° ΠΈΠ· Π΄Π²ΡƒΡ… ΠΏΡ€ΠΎΠ±Π»Π΅ΠΌ. Или ΠΎΠ±Π΅ сразу.) НСхватка Π·Π½Π°Π½ΠΈΠΉ ΠΈ (ΠΈΠ»ΠΈ) Π½Π΅Π²Π½ΠΈΠΌΠ°Ρ‚Π΅Π»ΡŒΠ½ΠΎΡΡ‚ΡŒ. Но… Π­Ρ‚ΠΎ Ρ€Π΅ΡˆΠ°Π΅ΠΌΡ‹Π΅ ΠΏΡ€ΠΎΠ±Π»Π΅ΠΌΡ‹.

Если Π’Π°ΠΌ нравится этот сайт…

ΠšΡΡ‚Π°Ρ‚ΠΈ, Ρƒ мСня Π΅ΡΡ‚ΡŒ Π΅Ρ‰Ρ‘ ΠΏΠ°Ρ€ΠΎΡ‡ΠΊΠ° интСрСсных сайтов для Вас.)

МоТно ΠΏΠΎΡ‚Ρ€Π΅Π½ΠΈΡ€ΠΎΠ²Π°Ρ‚ΡŒΡΡ Π² Ρ€Π΅ΡˆΠ΅Π½ΠΈΠΈ ΠΏΡ€ΠΈΠΌΠ΅Ρ€ΠΎΠ² ΠΈ ΡƒΠ·Π½Π°Ρ‚ΡŒ свой ΡƒΡ€ΠΎΠ²Π΅Π½ΡŒ. ВСстированиС с ΠΌΠ³Π½ΠΎΠ²Π΅Π½Π½ΠΎΠΉ ΠΏΡ€ΠΎΠ²Π΅Ρ€ΠΊΠΎΠΉ. Учимся — с интСрСсом!)

ΠΌΠΎΠΆΠ½ΠΎ ΠΏΠΎΠ·Π½Π°ΠΊΠΎΠΌΠΈΡ‚ΡŒΡΡ с функциями ΠΈ ΠΏΡ€ΠΎΠΈΠ·Π²ΠΎΠ΄Π½Ρ‹ΠΌΠΈ.

Π’ ΠΏΡ€ΠΎΡˆΠ»Ρ‹ΠΉ Ρ€Π°Π· ΠΌΡ‹ Π½Π°ΡƒΡ‡ΠΈΠ»ΠΈΡΡŒ ΡΠΊΠ»Π°Π΄Ρ‹Π²Π°Ρ‚ΡŒ ΠΈ Π²Ρ‹Ρ‡ΠΈΡ‚Π°Ρ‚ΡŒ Π΄Ρ€ΠΎΠ±ΠΈ (см. ΡƒΡ€ΠΎΠΊ Β«Π‘Π»ΠΎΠΆΠ΅Π½ΠΈΠ΅ ΠΈ Π²Ρ‹Ρ‡ΠΈΡ‚Π°Π½ΠΈΠ΅ Π΄Ρ€ΠΎΠ±Π΅ΠΉ Β»). НаиболСС слоТным ΠΌΠΎΠΌΠ΅Π½Ρ‚ΠΎΠΌ Π² Ρ‚Π΅Ρ… дСйствиях Π±Ρ‹Π»ΠΎ ΠΏΡ€ΠΈΠ²Π΅Π΄Π΅Π½ΠΈΠ΅ Π΄Ρ€ΠΎΠ±Π΅ΠΉ ΠΊ ΠΎΠ±Ρ‰Π΅ΠΌΡƒ Π·Π½Π°ΠΌΠ΅Π½Π°Ρ‚Π΅Π»ΡŽ.

Π’Π΅ΠΏΠ΅Ρ€ΡŒ настала ΠΏΠΎΡ€Π° Ρ€Π°Π·ΠΎΠ±Ρ€Π°Ρ‚ΡŒΡΡ с ΡƒΠΌΠ½ΠΎΠΆΠ΅Π½ΠΈΠ΅ΠΌ ΠΈ Π΄Π΅Π»Π΅Π½ΠΈΠ΅ΠΌ. Π₯ΠΎΡ€ΠΎΡˆΠ°Ρ Π½ΠΎΠ²ΠΎΡΡ‚ΡŒ состоит Π² Ρ‚ΠΎΠΌ, Ρ‡Ρ‚ΠΎ эти ΠΎΠΏΠ΅Ρ€Π°Ρ†ΠΈΠΈ Π²Ρ‹ΠΏΠΎΠ»Π½ΡΡŽΡ‚ΡΡ Π΄Π°ΠΆΠ΅ ΠΏΡ€ΠΎΡ‰Π΅, Ρ‡Π΅ΠΌ слоТСниС ΠΈ Π²Ρ‹Ρ‡ΠΈΡ‚Π°Π½ΠΈΠ΅. Для Π½Π°Ρ‡Π°Π»Π° рассмотрим ΠΏΡ€ΠΎΡΡ‚Π΅ΠΉΡˆΠΈΠΉ случай, ΠΊΠΎΠ³Π΄Π° Π΅ΡΡ‚ΡŒ Π΄Π²Π΅ ΠΏΠΎΠ»ΠΎΠΆΠΈΡ‚Π΅Π»ΡŒΠ½Ρ‹Π΅ Π΄Ρ€ΠΎΠ±ΠΈ Π±Π΅Π· Π²Ρ‹Π΄Π΅Π»Π΅Π½Π½ΠΎΠΉ Ρ†Π΅Π»ΠΎΠΉ части.

Π§Ρ‚ΠΎΠ±Ρ‹ ΡƒΠΌΠ½ΠΎΠΆΠΈΡ‚ΡŒ Π΄Π²Π΅ Π΄Ρ€ΠΎΠ±ΠΈ, Π½Π°Π΄ΠΎ ΠΎΡ‚Π΄Π΅Π»ΡŒΠ½ΠΎ ΡƒΠΌΠ½ΠΎΠΆΠΈΡ‚ΡŒ ΠΈΡ… числитСли ΠΈ Π·Π½Π°ΠΌΠ΅Π½Π°Ρ‚Π΅Π»ΠΈ. ΠŸΠ΅Ρ€Π²ΠΎΠ΅ число Π±ΡƒΠ΄Π΅Ρ‚ числитСлСм Π½ΠΎΠ²ΠΎΠΉ Π΄Ρ€ΠΎΠ±ΠΈ, Π° Π²Ρ‚ΠΎΡ€ΠΎΠ΅ — Π·Π½Π°ΠΌΠ΅Π½Π°Ρ‚Π΅Π»Π΅ΠΌ.

Π§Ρ‚ΠΎΠ±Ρ‹ Ρ€Π°Π·Π΄Π΅Π»ΠΈΡ‚ΡŒ Π΄Π²Π΅ Π΄Ρ€ΠΎΠ±ΠΈ, Π½Π°Π΄ΠΎ ΠΏΠ΅Ρ€Π²ΡƒΡŽ Π΄Ρ€ΠΎΠ±ΡŒ ΡƒΠΌΠ½ΠΎΠΆΠΈΡ‚ΡŒ Π½Π° Β«ΠΏΠ΅Ρ€Π΅Π²Π΅Ρ€Π½ΡƒΡ‚ΡƒΡŽΒ» Π²Ρ‚ΠΎΡ€ΡƒΡŽ.

ΠžΠ±ΠΎΠ·Π½Π°Ρ‡Π΅Π½ΠΈΠ΅:

Из опрСдСлСния слСдуСт, Ρ‡Ρ‚ΠΎ Π΄Π΅Π»Π΅Π½ΠΈΠ΅ Π΄Ρ€ΠΎΠ±Π΅ΠΉ сводится ΠΊ ΡƒΠΌΠ½ΠΎΠΆΠ΅Π½ΠΈΡŽ. Π§Ρ‚ΠΎΠ±Ρ‹ Β«ΠΏΠ΅Ρ€Π΅Π²Π΅Ρ€Π½ΡƒΡ‚ΡŒΒ» Π΄Ρ€ΠΎΠ±ΡŒ, достаточно ΠΏΠΎΠΌΠ΅Π½ΡΡ‚ΡŒ мСстами Ρ‡ΠΈΡΠ»ΠΈΡ‚Π΅Π»ΡŒ ΠΈ Π·Π½Π°ΠΌΠ΅Π½Π°Ρ‚Π΅Π»ΡŒ. ΠŸΠΎΡΡ‚ΠΎΠΌΡƒ вСсь ΡƒΡ€ΠΎΠΊ ΠΌΡ‹ Π±ΡƒΠ΄Π΅ΠΌ Ρ€Π°ΡΡΠΌΠ°Ρ‚Ρ€ΠΈΠ²Π°Ρ‚ΡŒ Π² основном ΡƒΠΌΠ½ΠΎΠΆΠ΅Π½ΠΈΠ΅.

Π’ Ρ€Π΅Π·ΡƒΠ»ΡŒΡ‚Π°Ρ‚Π΅ умноТСния ΠΌΠΎΠΆΠ΅Ρ‚ Π²ΠΎΠ·Π½ΠΈΠΊΠ½ΡƒΡ‚ΡŒ (ΠΈ Π·Π°Ρ‡Π°ΡΡ‚ΡƒΡŽ Π΄Π΅ΠΉΡΡ‚Π²ΠΈΡ‚Π΅Π»ΡŒΠ½ΠΎ Π²ΠΎΠ·Π½ΠΈΠΊΠ°Π΅Ρ‚) сократимая Π΄Ρ€ΠΎΠ±ΡŒ — Π΅Π΅, разумССтся, Π½Π°Π΄ΠΎ ΡΠΎΠΊΡ€Π°Ρ‚ΠΈΡ‚ΡŒ. Если послС всСх сокращСний Π΄Ρ€ΠΎΠ±ΡŒ оказалась Π½Π΅ΠΏΡ€Π°Π²ΠΈΠ»ΡŒΠ½ΠΎΠΉ, Π² Π½Π΅ΠΉ слСдуСт Π²Ρ‹Π΄Π΅Π»ΠΈΡ‚ΡŒ Ρ†Π΅Π»ΡƒΡŽ Ρ‡Π°ΡΡ‚ΡŒ. Но Ρ‡Π΅Π³ΠΎ Ρ‚ΠΎΡ‡Π½ΠΎ Π½Π΅ Π±ΡƒΠ΄Π΅Ρ‚ ΠΏΡ€ΠΈ ΡƒΠΌΠ½ΠΎΠΆΠ΅Π½ΠΈΠΈ, Ρ‚Π°ΠΊ это привСдСния ΠΊ ΠΎΠ±Ρ‰Π΅ΠΌΡƒ Π·Π½Π°ΠΌΠ΅Π½Π°Ρ‚Π΅Π»ΡŽ: Π½ΠΈΠΊΠ°ΠΊΠΈΡ… ΠΌΠ΅Ρ‚ΠΎΠ΄ΠΎΠ² «крСст-накрСст», Π½Π°ΠΈΠ±ΠΎΠ»ΡŒΡˆΠΈΡ… ΠΌΠ½ΠΎΠΆΠΈΡ‚Π΅Π»Π΅ΠΉ ΠΈ Π½Π°ΠΈΠΌΠ΅Π½ΡŒΡˆΠΈΡ… ΠΎΠ±Ρ‰ΠΈΡ… ΠΊΡ€Π°Ρ‚Π½Ρ‹Ρ….

По ΠΎΠΏΡ€Π΅Π΄Π΅Π»Π΅Π½ΠΈΡŽ ΠΈΠΌΠ΅Π΅ΠΌ:

Π£ΠΌΠ½ΠΎΠΆΠ΅Π½ΠΈΠ΅ Π΄Ρ€ΠΎΠ±Π΅ΠΉ с Ρ†Π΅Π»ΠΎΠΉ Ρ‡Π°ΡΡ‚ΡŒΡŽ ΠΈ ΠΎΡ‚Ρ€ΠΈΡ†Π°Ρ‚Π΅Π»ΡŒΠ½Ρ‹Ρ… Π΄Ρ€ΠΎΠ±Π΅ΠΉ

Если Π² дробях присутствуСт цСлая Ρ‡Π°ΡΡ‚ΡŒ, ΠΈΡ… Π½Π°Π΄ΠΎ пСрСвСсти Π² Π½Π΅ΠΏΡ€Π°Π²ΠΈΠ»ΡŒΠ½Ρ‹Π΅ — ΠΈ Ρ‚ΠΎΠ»ΡŒΠΊΠΎ Π·Π°Ρ‚Π΅ΠΌ ΡƒΠΌΠ½ΠΎΠΆΠ°Ρ‚ΡŒ ΠΏΠΎ схСмам, ΠΈΠ·Π»ΠΎΠΆΠ΅Π½Π½Ρ‹ΠΌ Π²Ρ‹ΡˆΠ΅.

Если Π² числитСлС Π΄Ρ€ΠΎΠ±ΠΈ, Π² Π·Π½Π°ΠΌΠ΅Π½Π°Ρ‚Π΅Π»Π΅ ΠΈΠ»ΠΈ ΠΏΠ΅Ρ€Π΅Π΄ Π½Π΅ΠΉ стоит минус, Π΅Π³ΠΎ ΠΌΠΎΠΆΠ½ΠΎ вынСсти Π·Π° ΠΏΡ€Π΅Π΄Π΅Π»Ρ‹ умноТСния ΠΈΠ»ΠΈ Π²ΠΎΠΎΠ±Ρ‰Π΅ ΡƒΠ±Ρ€Π°Ρ‚ΡŒ ΠΏΠΎ ΡΠ»Π΅Π΄ΡƒΡŽΡ‰ΠΈΠΌ ΠΏΡ€Π°Π²ΠΈΠ»Π°ΠΌ:

  1. Плюс Π½Π° минус Π΄Π°Π΅Ρ‚ минус;
  2. ΠœΠΈΠ½ΡƒΡ Π½Π° минус Π΄Π°Π΅Ρ‚ плюс.

Π”ΠΎ сих ΠΏΠΎΡ€ эти ΠΏΡ€Π°Π²ΠΈΠ»Π° Π²ΡΡ‚Ρ€Π΅Ρ‡Π°Π»ΠΈΡΡŒ Ρ‚ΠΎΠ»ΡŒΠΊΠΎ ΠΏΡ€ΠΈ слоТСнии ΠΈ Π²Ρ‹Ρ‡ΠΈΡ‚Π°Π½ΠΈΠΈ ΠΎΡ‚Ρ€ΠΈΡ†Π°Ρ‚Π΅Π»ΡŒΠ½Ρ‹Ρ… Π΄Ρ€ΠΎΠ±Π΅ΠΉ, ΠΊΠΎΠ³Π΄Π° Ρ‚Ρ€Π΅Π±ΠΎΠ²Π°Π»ΠΎΡΡŒ ΠΈΠ·Π±Π°Π²ΠΈΡ‚ΡŒΡΡ ΠΎΡ‚ Ρ†Π΅Π»ΠΎΠΉ части. Для произвСдСния ΠΈΡ… ΠΌΠΎΠΆΠ½ΠΎ ΠΎΠ±ΠΎΠ±Ρ‰ΠΈΡ‚ΡŒ, Ρ‡Ρ‚ΠΎΠ±Ρ‹ Β«ΡΠΆΠΈΠ³Π°Ρ‚ΡŒΒ» сразу нСсколько минусов:

  1. Π’Ρ‹Ρ‡Π΅Ρ€ΠΊΠΈΠ²Π°Π΅ΠΌ минусы ΠΏΠ°Ρ€Π°ΠΌΠΈ Π΄ΠΎ Ρ‚Π΅Ρ… ΠΏΠΎΡ€, ΠΏΠΎΠΊΠ° ΠΎΠ½ΠΈ ΠΏΠΎΠ»Π½ΠΎΡΡ‚ΡŒΡŽ Π½Π΅ исчСзнут. Π’ ΠΊΡ€Π°ΠΉΠ½Π΅ΠΌ случаС, ΠΎΠ΄ΠΈΠ½ минус ΠΌΠΎΠΆΠ΅Ρ‚ Π²Ρ‹ΠΆΠΈΡ‚ΡŒ — Ρ‚ΠΎΡ‚, ΠΊΠΎΡ‚ΠΎΡ€ΠΎΠΌΡƒ Π½Π΅ нашлось ΠΏΠ°Ρ€Ρ‹;
  2. Если минусов Π½Π΅ ΠΎΡΡ‚Π°Π»ΠΎΡΡŒ, опСрация Π²Ρ‹ΠΏΠΎΠ»Π½Π΅Π½Π° — ΠΌΠΎΠΆΠ½ΠΎ ΠΏΡ€ΠΈΡΡ‚ΡƒΠΏΠ°Ρ‚ΡŒ ΠΊ ΡƒΠΌΠ½ΠΎΠΆΠ΅Π½ΠΈΡŽ. Если ΠΆΠ΅ послСдний минус Π½Π΅ Π·Π°Ρ‡Π΅Ρ€ΠΊΠ½ΡƒΡ‚, ΠΏΠΎΡΠΊΠΎΠ»ΡŒΠΊΡƒ Π΅ΠΌΡƒ Π½Π΅ нашлось ΠΏΠ°Ρ€Ρ‹, выносим Π΅Π³ΠΎ Π·Π° ΠΏΡ€Π΅Π΄Π΅Π»Ρ‹ умноТСния. ΠŸΠΎΠ»ΡƒΡ‡ΠΈΡ‚ΡΡ ΠΎΡ‚Ρ€ΠΈΡ†Π°Ρ‚Π΅Π»ΡŒΠ½Π°Ρ Π΄Ρ€ΠΎΠ±ΡŒ.

Π—Π°Π΄Π°Ρ‡Π°. НайдитС Π·Π½Π°Ρ‡Π΅Π½ΠΈΠ΅ выраТСния:

ВсС Π΄Ρ€ΠΎΠ±ΠΈ ΠΏΠ΅Ρ€Π΅Π²ΠΎΠ΄ΠΈΠΌ Π² Π½Π΅ΠΏΡ€Π°Π²ΠΈΠ»ΡŒΠ½Ρ‹Π΅, Π° Π·Π°Ρ‚Π΅ΠΌ выносим минусы Π·Π° ΠΏΡ€Π΅Π΄Π΅Π»Ρ‹ умноТСния. Π’ΠΎ, Ρ‡Ρ‚ΠΎ ΠΎΡΡ‚Π°Π»ΠΎΡΡŒ, ΡƒΠΌΠ½ΠΎΠΆΠ°Π΅ΠΌ ΠΏΠΎ ΠΎΠ±Ρ‹Ρ‡Π½Ρ‹ΠΌ ΠΏΡ€Π°Π²ΠΈΠ»Π°ΠΌ. ΠŸΠΎΠ»ΡƒΡ‡Π°Π΅ΠΌ:

Π•Ρ‰Π΅ Ρ€Π°Π· напомню, Ρ‡Ρ‚ΠΎ минус, ΠΊΠΎΡ‚ΠΎΡ€Ρ‹ΠΉ стоит ΠΏΠ΅Ρ€Π΅Π΄ Π΄Ρ€ΠΎΠ±ΡŒΡŽ с Π²Ρ‹Π΄Π΅Π»Π΅Π½Π½ΠΎΠΉ Ρ†Π΅Π»ΠΎΠΉ Ρ‡Π°ΡΡ‚ΡŒΡŽ, относится ΠΈΠΌΠ΅Π½Π½ΠΎ ΠΊΠΎ всСй Π΄Ρ€ΠΎΠ±ΠΈ, Π° Π½Π΅ Ρ‚ΠΎΠ»ΡŒΠΊΠΎ ΠΊ Π΅Π΅ Ρ†Π΅Π»ΠΎΠΉ части (это касаСтся Π΄Π²ΡƒΡ… послСдних ΠΏΡ€ΠΈΠΌΠ΅Ρ€ΠΎΠ²).

Π’Π°ΠΊΠΆΠ΅ ΠΎΠ±Ρ€Π°Ρ‚ΠΈΡ‚Π΅ Π²Π½ΠΈΠΌΠ°Π½ΠΈΠ΅ Π½Π° ΠΎΡ‚Ρ€ΠΈΡ†Π°Ρ‚Π΅Π»ΡŒΠ½Ρ‹Π΅ числа: ΠΏΡ€ΠΈ ΡƒΠΌΠ½ΠΎΠΆΠ΅Π½ΠΈΠΈ ΠΎΠ½ΠΈ Π·Π°ΠΊΠ»ΡŽΡ‡Π°ΡŽΡ‚ΡΡ Π² скобки. Π­Ρ‚ΠΎ сдСлано для Ρ‚ΠΎΠ³ΠΎ, Ρ‡Ρ‚ΠΎΠ±Ρ‹ ΠΎΡ‚Π΄Π΅Π»ΠΈΡ‚ΡŒ минусы ΠΎΡ‚ Π·Π½Π°ΠΊΠΎΠ² умноТСния ΠΈ ΡΠ΄Π΅Π»Π°Ρ‚ΡŒ всю запись Π±ΠΎΠ»Π΅Π΅ Π°ΠΊΠΊΡƒΡ€Π°Ρ‚Π½ΠΎΠΉ.

Π‘ΠΎΠΊΡ€Π°Ρ‰Π΅Π½ΠΈΠ΅ Π΄Ρ€ΠΎΠ±Π΅ΠΉ Β«Π½Π° Π»Π΅Ρ‚ΡƒΒ»

Π£ΠΌΠ½ΠΎΠΆΠ΅Π½ΠΈΠ΅ — вСсьма трудоСмкая опСрация. Числа здСсь ΠΏΠΎΠ»ΡƒΡ‡Π°ΡŽΡ‚ΡΡ довольно большиС, ΠΈ Ρ‡Ρ‚ΠΎΠ±Ρ‹ ΡƒΠΏΡ€ΠΎΡΡ‚ΠΈΡ‚ΡŒ Π·Π°Π΄Π°Ρ‡Ρƒ, ΠΌΠΎΠΆΠ½ΠΎ ΠΏΠΎΠΏΡ€ΠΎΠ±ΠΎΠ²Π°Ρ‚ΡŒ ΡΠΎΠΊΡ€Π°Ρ‚ΠΈΡ‚ΡŒ Π΄Ρ€ΠΎΠ±ΡŒ Π΅Ρ‰Π΅ Π΄ΠΎ умноТСния . Π’Π΅Π΄ΡŒ ΠΏΠΎ сущСству, числитСли ΠΈ Π·Π½Π°ΠΌΠ΅Π½Π°Ρ‚Π΅Π»ΠΈ Π΄Ρ€ΠΎΠ±Π΅ΠΉ — это ΠΎΠ±Ρ‹Ρ‡Π½Ρ‹Π΅ ΠΌΠ½ΠΎΠΆΠΈΡ‚Π΅Π»ΠΈ, ΠΈ, ΡΠ»Π΅Π΄ΠΎΠ²Π°Ρ‚Π΅Π»ΡŒΠ½ΠΎ, ΠΈΡ… ΠΌΠΎΠΆΠ½ΠΎ ΡΠΎΠΊΡ€Π°Ρ‰Π°Ρ‚ΡŒ, ΠΈΡΠΏΠΎΠ»ΡŒΠ·ΡƒΡ основноС свойство Π΄Ρ€ΠΎΠ±ΠΈ. ВзглянитС Π½Π° ΠΏΡ€ΠΈΠΌΠ΅Ρ€Ρ‹:

Π—Π°Π΄Π°Ρ‡Π°. НайдитС Π·Π½Π°Ρ‡Π΅Π½ΠΈΠ΅ выраТСния:

По ΠΎΠΏΡ€Π΅Π΄Π΅Π»Π΅Π½ΠΈΡŽ ΠΈΠΌΠ΅Π΅ΠΌ:

Π’ΠΎ всСх ΠΏΡ€ΠΈΠΌΠ΅Ρ€Π°Ρ… красным Ρ†Π²Π΅Ρ‚ΠΎΠΌ ΠΎΡ‚ΠΌΠ΅Ρ‡Π΅Π½Ρ‹ числа, ΠΊΠΎΡ‚ΠΎΡ€Ρ‹Π΅ ΠΏΠΎΠ΄Π²Π΅Ρ€Π³Π»ΠΈΡΡŒ ΡΠΎΠΊΡ€Π°Ρ‰Π΅Π½ΠΈΡŽ, ΠΈ Ρ‚ΠΎ, Ρ‡Ρ‚ΠΎ ΠΎΡ‚ Π½ΠΈΡ… ΠΎΡΡ‚Π°Π»ΠΎΡΡŒ.

ΠžΠ±Ρ€Π°Ρ‚ΠΈΡ‚Π΅ Π²Π½ΠΈΠΌΠ°Π½ΠΈΠ΅: Π² ΠΏΠ΅Ρ€Π²ΠΎΠΌ случаС ΠΌΠ½ΠΎΠΆΠΈΡ‚Π΅Π»ΠΈ ΡΠΎΠΊΡ€Π°Ρ‚ΠΈΠ»ΠΈΡΡŒ ΠΏΠΎΠ»Π½ΠΎΡΡ‚ΡŒΡŽ. На ΠΈΡ… мСстС ΠΎΡΡ‚Π°Π»ΠΈΡΡŒ Π΅Π΄ΠΈΠ½ΠΈΡ†Ρ‹, ΠΊΠΎΡ‚ΠΎΡ€Ρ‹Π΅, Π²ΠΎΠΎΠ±Ρ‰Π΅ говоря, ΠΌΠΎΠΆΠ½ΠΎ Π½Π΅ ΠΏΠΈΡΠ°Ρ‚ΡŒ. Π’ΠΎ Π²Ρ‚ΠΎΡ€ΠΎΠΌ ΠΏΡ€ΠΈΠΌΠ΅Ρ€Π΅ ΠΏΠΎΠ»Π½ΠΎΠ³ΠΎ сокращСния Π΄ΠΎΠ±ΠΈΡ‚ΡŒΡΡ Π½Π΅ ΡƒΠ΄Π°Π»ΠΎΡΡŒ, Π½ΠΎ суммарный объСм вычислСний всС Ρ€Π°Π²Π½ΠΎ ΡƒΠΌΠ΅Π½ΡŒΡˆΠΈΠ»ΡΡ.

Однако Π½ΠΈ Π² ΠΊΠΎΠ΅ΠΌ случаС Π½Π΅ ΠΈΡΠΏΠΎΠ»ΡŒΠ·ΡƒΠΉΡ‚Π΅ этот ΠΏΡ€ΠΈΠ΅ΠΌ ΠΏΡ€ΠΈ слоТСнии ΠΈ Π²Ρ‹Ρ‡ΠΈΡ‚Π°Π½ΠΈΠΈ Π΄Ρ€ΠΎΠ±Π΅ΠΉ! Π”Π°, ΠΈΠ½ΠΎΠ³Π΄Π° Ρ‚Π°ΠΌ Π²ΡΡ‚Ρ€Π΅Ρ‡Π°ΡŽΡ‚ΡΡ ΠΏΠΎΡ…ΠΎΠΆΠΈΠ΅ числа, ΠΊΠΎΡ‚ΠΎΡ€Ρ‹Π΅ Ρ‚Π°ΠΊ ΠΈ хочСтся ΡΠΎΠΊΡ€Π°Ρ‚ΠΈΡ‚ΡŒ. Π’ΠΎΡ‚, посмотритС:

Π’Π°ΠΊ Π΄Π΅Π»Π°Ρ‚ΡŒ нСльзя!

Ошибка Π²ΠΎΠ·Π½ΠΈΠΊΠ°Π΅Ρ‚ ΠΈΠ·-Π·Π° Ρ‚ΠΎΠ³ΠΎ, Ρ‡Ρ‚ΠΎ ΠΏΡ€ΠΈ слоТСнии Π² числитСлС Π΄Ρ€ΠΎΠ±ΠΈ появляСтся сумма, Π° Π½Π΅ ΠΏΡ€ΠΎΠΈΠ·Π²Π΅Π΄Π΅Π½ΠΈΠ΅ чисСл. Π‘Π»Π΅Π΄ΠΎΠ²Π°Ρ‚Π΅Π»ΡŒΠ½ΠΎ, ΠΏΡ€ΠΈΠΌΠ΅Π½ΡΡ‚ΡŒ основноС свойство Π΄Ρ€ΠΎΠ±ΠΈ нСльзя, ΠΏΠΎΡΠΊΠΎΠ»ΡŒΠΊΡƒ Π² этом свойствС Ρ€Π΅Ρ‡ΡŒ ΠΈΠ΄Π΅Ρ‚ ΠΈΠΌΠ΅Π½Π½ΠΎ ΠΎΠ± ΡƒΠΌΠ½ΠΎΠΆΠ΅Π½ΠΈΠΈ чисСл.

Π”Ρ€ΡƒΠ³ΠΈΡ… оснований для сокращСния Π΄Ρ€ΠΎΠ±Π΅ΠΉ просто Π½Π΅ сущСствуСт, поэтому ΠΏΡ€Π°Π²ΠΈΠ»ΡŒΠ½ΠΎΠ΅ Ρ€Π΅ΡˆΠ΅Π½ΠΈΠ΅ ΠΏΡ€Π΅Π΄Ρ‹Π΄ΡƒΡ‰Π΅ΠΉ Π·Π°Π΄Π°Ρ‡ΠΈ выглядит Ρ‚Π°ΠΊ:

ΠŸΡ€Π°Π²ΠΈΠ»ΡŒΠ½ΠΎΠ΅ Ρ€Π΅ΡˆΠ΅Π½ΠΈΠ΅:

Как Π²ΠΈΠ΄ΠΈΡ‚Π΅, ΠΏΡ€Π°Π²ΠΈΠ»ΡŒΠ½Ρ‹ΠΉ ΠΎΡ‚Π²Π΅Ρ‚ оказался Π½Π΅ Ρ‚Π°ΠΊΠΈΠΌ красивым. Π’ ΠΎΠ±Ρ‰Π΅ΠΌ, Π±ΡƒΠ΄ΡŒΡ‚Π΅ Π²Π½ΠΈΠΌΠ°Ρ‚Π΅Π»ΡŒΠ½Ρ‹.

Как Π΄Π΅Π»ΠΈΡ‚ΡŒ ΠΎΠ±Ρ‹ΠΊΠ½ΠΎΠ²Π΅Π½Π½Ρ‹Π΅. Π”Ρ€ΠΎΠ±ΠΈ. Π”Π΅Π»Π΅Π½ΠΈΠ΅ Π΄Ρ€ΠΎΠ±Π΅ΠΉ. Π€ΠΎΡ€ΠΌΡƒΠ»Π° умноТСния Π΄Ρ€ΠΎΠ±Π΅ΠΉ

Π‘ΠΎΠ΄Π΅Ρ€ΠΆΠ°Π½ΠΈΠ΅ ΡƒΡ€ΠΎΠΊΠ°

Π‘Π»ΠΎΠΆΠ΅Π½ΠΈΠ΅ Π΄Ρ€ΠΎΠ±Π΅ΠΉ с ΠΎΠ΄ΠΈΠ½Π°ΠΊΠΎΠ²Ρ‹ΠΌΠΈ знамСнатСлями

Π‘Π»ΠΎΠΆΠ΅Π½ΠΈΠ΅ Π΄Ρ€ΠΎΠ±Π΅ΠΉ Π±Ρ‹Π²Π°Π΅Ρ‚ Π΄Π²ΡƒΡ… Π²ΠΈΠ΄ΠΎΠ²:

  1. Π‘Π»ΠΎΠΆΠ΅Π½ΠΈΠ΅ Π΄Ρ€ΠΎΠ±Π΅ΠΉ с ΠΎΠ΄ΠΈΠ½Π°ΠΊΠΎΠ²Ρ‹ΠΌΠΈ знамСнатСлями;
  2. Π‘Π»ΠΎΠΆΠ΅Π½ΠΈΠ΅ Π΄Ρ€ΠΎΠ±Π΅ΠΉ с Ρ€Π°Π·Π½Ρ‹ΠΌΠΈ знамСнатСлями.

Π‘Π½Π°Ρ‡Π°Π»Π° ΠΈΠ·Γ½Ρ‡ΠΈΠΌ слоТСниС Π΄Ρ€ΠΎΠ±Π΅ΠΉ с ΠΎΠ΄ΠΈΠ½Π°ΠΊΠΎΠ²Ρ‹ΠΌΠΈ знамСнатСлями. Π’ΡƒΡ‚ всё просто. Π§Ρ‚ΠΎΠ±Ρ‹ ΡΠ»ΠΎΠΆΠΈΡ‚ΡŒ Π΄Ρ€ΠΎΠ±ΠΈ с ΠΎΠ΄ΠΈΠ½Π°ΠΊΠΎΠ²Ρ‹ΠΌΠΈ знамСнатСлями, Π½ΡƒΠΆΠ½ΠΎ ΡΠ»ΠΎΠΆΠΈΡ‚ΡŒ ΠΈΡ… числитСли, Π° Π·Π½Π°ΠΌΠ΅Π½Π°Ρ‚Π΅Π»ΡŒ ΠΎΡΡ‚Π°Π²ΠΈΡ‚ΡŒ Π±Π΅Π· измСнСния.

НапримСр, слóТим Π΄Ρ€ΠΎΠ±ΠΈ ΠΈ . Π‘ΠΊΠ»Π°Π΄Ρ‹Π²Π°Π΅ΠΌ числитСли, Π° Π·Π½Π°ΠΌΠ΅Π½Π°Ρ‚Π΅Π»ΡŒ оставляСм Π±Π΅Π· измСнСния:

Π­Ρ‚ΠΎΡ‚ ΠΏΡ€ΠΈΠΌΠ΅Ρ€ ΠΌΠΎΠΆΠ½ΠΎ Π»Π΅Π³ΠΊΠΎ ΠΏΠΎΠ½ΡΡ‚ΡŒ, Ссли Π²ΡΠΏΠΎΠΌΠ½ΠΈΡ‚ΡŒ ΠΏΡ€ΠΎ ΠΏΠΈΡ†Ρ†Ρƒ, которая Ρ€Π°Π·Π΄Π΅Π»Π΅Π½Π° Π½Π° Ρ‡Π΅Ρ‚Ρ‹Ρ€Π΅ части. Если ΠΊ ΠΏΠΈΡ†Ρ†Ρ‹ ΠΏΡ€ΠΈΠ±Π°Π²ΠΈΡ‚ΡŒ ΠΏΠΈΡ†Ρ†Ρ‹, Ρ‚ΠΎ получится ΠΏΠΈΡ†Ρ†Ρ‹:

ΠŸΡ€ΠΈΠΌΠ΅Ρ€ 2. Π‘Π»ΠΎΠΆΠΈΡ‚ΡŒ Π΄Ρ€ΠΎΠ±ΠΈ ΠΈ .

Π’ ΠΎΡ‚Π²Π΅Ρ‚Π΅ ΠΏΠΎΠ»ΡƒΡ‡ΠΈΠ»Π°ΡΡŒ Π½Π΅ΠΏΡ€Π°Π²ΠΈΠ»ΡŒΠ½Π°Ρ Π΄Ρ€ΠΎΠ±ΡŒ . Если наступаСт ΠΊΠΎΠ½Π΅Ρ† Π·Π°Π΄Π°Ρ‡ΠΈ, Ρ‚ΠΎ ΠΎΡ‚ Π½Π΅ΠΏΡ€Π°Π²ΠΈΠ»ΡŒΠ½Ρ‹Ρ… Π΄Ρ€ΠΎΠ±Π΅ΠΉ принято ΠΈΠ·Π±Π°Π²Π»ΡΡ‚ΡŒΡΡ. Π§Ρ‚ΠΎΠ±Ρ‹ избавится ΠΎΡ‚ Π½Π΅ΠΏΡ€Π°Π²ΠΈΠ»ΡŒΠ½ΠΎΠΉ Π΄Ρ€ΠΎΠ±ΠΈ, Π½ΡƒΠΆΠ½ΠΎ Π²Ρ‹Π΄Π΅Π»ΠΈΡ‚ΡŒ Π² Π½Π΅ΠΉ Ρ†Π΅Π»ΡƒΡŽ Ρ‡Π°ΡΡ‚ΡŒ. Π’ нашСм случаС цСлая Ρ‡Π°ΡΡ‚ΡŒ выдСляСтся Π»Π΅Π³ΠΊΠΎ β€” Π΄Π²Π° Ρ€Π°Π·Π΄Π΅Π»ΠΈΡ‚ΡŒ Π½Π° Π΄Π²Π° Π±ΡƒΠ΄Π΅Ρ‚ ΠΎΠ΄ΠΈΠ½:

Π­Ρ‚ΠΎΡ‚ ΠΏΡ€ΠΈΠΌΠ΅Ρ€ ΠΌΠΎΠΆΠ½ΠΎ Π»Π΅Π³ΠΊΠΎ ΠΏΠΎΠ½ΡΡ‚ΡŒ, Ссли Π²ΡΠΏΠΎΠΌΠ½ΠΈΡ‚ΡŒ ΠΏΡ€ΠΎ ΠΏΠΈΡ†Ρ†Ρƒ, которая Ρ€Π°Π·Π΄Π΅Π»Π΅Π½Π° Π½Π° Π΄Π²Π΅ части. Если ΠΊ ΠΏΠΈΡ†Ρ†Ρ‹ ΠΏΡ€ΠΈΠ±Π°Π²ΠΈΡ‚ΡŒ Π΅Ρ‰Π΅ ΠΏΠΈΡ†Ρ†Ρ‹, Ρ‚ΠΎ получится ΠΎΠ΄Π½Π° цСлая ΠΏΠΈΡ†Ρ†Π°:

ΠŸΡ€ΠΈΠΌΠ΅Ρ€ 3 . Π‘Π»ΠΎΠΆΠΈΡ‚ΡŒ Π΄Ρ€ΠΎΠ±ΠΈ ΠΈ .

ΠžΠΏΡΡ‚ΡŒ ΠΆΠ΅ складываСм числитСли, Π° Π·Π½Π°ΠΌΠ΅Π½Π°Ρ‚Π΅Π»ΡŒ оставляСм Π±Π΅Π· измСнСния:

Π­Ρ‚ΠΎΡ‚ ΠΏΡ€ΠΈΠΌΠ΅Ρ€ ΠΌΠΎΠΆΠ½ΠΎ Π»Π΅Π³ΠΊΠΎ ΠΏΠΎΠ½ΡΡ‚ΡŒ, Ссли Π²ΡΠΏΠΎΠΌΠ½ΠΈΡ‚ΡŒ ΠΏΡ€ΠΎ ΠΏΠΈΡ†Ρ†Ρƒ, которая Ρ€Π°Π·Π΄Π΅Π»Π΅Π½Π° Π½Π° Ρ‚Ρ€ΠΈ части. Если ΠΊ ΠΏΠΈΡ†Ρ†Ρ‹ ΠΏΡ€ΠΈΠ±Π°Π²ΠΈΡ‚ΡŒ Π΅Ρ‰Ρ‘ ΠΏΠΈΡ†Ρ†Ρ‹, Ρ‚ΠΎ получится ΠΏΠΈΡ†Ρ†Ρ‹:

ΠŸΡ€ΠΈΠΌΠ΅Ρ€ 4. Найти Π·Π½Π°Ρ‡Π΅Π½ΠΈΠ΅ выраТСния

Π­Ρ‚ΠΎΡ‚ ΠΏΡ€ΠΈΠΌΠ΅Ρ€ Ρ€Π΅ΡˆΠ°Π΅Ρ‚ΡΡ Ρ‚ΠΎΡ‡Π½ΠΎ Ρ‚Π°ΠΊΠΆΠ΅, ΠΊΠ°ΠΊ ΠΈ ΠΏΡ€Π΅Π΄Ρ‹Π΄ΡƒΡ‰ΠΈΠ΅. ЧислитСли Π½Π΅ΠΎΠ±Ρ…ΠΎΠ΄ΠΈΠΌΠΎ ΡΠ»ΠΎΠΆΠΈΡ‚ΡŒ, Π° Π·Π½Π°ΠΌΠ΅Π½Π°Ρ‚Π΅Π»ΡŒ ΠΎΡΡ‚Π°Π²ΠΈΡ‚ΡŒ Π±Π΅Π· измСнСния:

ΠŸΠΎΠΏΡ€ΠΎΠ±ΡƒΠ΅ΠΌ ΠΈΠ·ΠΎΠ±Ρ€Π°Π·ΠΈΡ‚ΡŒ нашС Ρ€Π΅ΡˆΠ΅Π½ΠΈΠ΅ с ΠΏΠΎΠΌΠΎΡ‰ΡŒΡŽ рисунка. Если ΠΊ ΠΏΠΈΡ†Ρ†Ρ‹ ΠΏΡ€ΠΈΠ±Π°Π²ΠΈΡ‚ΡŒ ΠΏΠΈΡ†Ρ†Ρ‹ ΠΈ Π΅Ρ‰Ρ‘ ΠΏΡ€ΠΈΠ±Π°Π²ΠΈΡ‚ΡŒ ΠΏΠΈΡ†Ρ†Ρ‹, Ρ‚ΠΎ получится 1 цСлая ΠΈ Π΅Ρ‰Ρ‘ ΠΏΠΈΡ†Ρ†Ρ‹.

Как Π²ΠΈΠ΄ΠΈΡ‚Π΅ Π² слоТСнии Π΄Ρ€ΠΎΠ±Π΅ΠΉ с ΠΎΠ΄ΠΈΠ½Π°ΠΊΠΎΠ²Ρ‹ΠΌΠΈ знамСнатСлями Π½Π΅Ρ‚ Π½ΠΈΡ‡Π΅Π³ΠΎ слоТного. Достаточно ΠΏΠΎΠ½ΠΈΠΌΠ°Ρ‚ΡŒ ΡΠ»Π΅Π΄ΡƒΡŽΡ‰ΠΈΠ΅ ΠΏΡ€Π°Π²ΠΈΠ»Π°:

  1. Π§Ρ‚ΠΎΠ±Ρ‹ ΡΠ»ΠΎΠΆΠΈΡ‚ΡŒ Π΄Ρ€ΠΎΠ±ΠΈ с ΠΎΠ΄ΠΈΠ½Π°ΠΊΠΎΠ²Ρ‹ΠΌΠΈ знамСнатСлями, Π½ΡƒΠΆΠ½ΠΎ ΡΠ»ΠΎΠΆΠΈΡ‚ΡŒ ΠΈΡ… числитСли, Π° Π·Π½Π°ΠΌΠ΅Π½Π°Ρ‚Π΅Π»ΡŒ ΠΎΡΡ‚Π°Π²ΠΈΡ‚ΡŒ Π±Π΅Π· измСнСния;

Π‘Π»ΠΎΠΆΠ΅Π½ΠΈΠ΅ Π΄Ρ€ΠΎΠ±Π΅ΠΉ с Ρ€Π°Π·Π½Ρ‹ΠΌΠΈ знамСнатСлями

Π’Π΅ΠΏΠ΅Ρ€ΡŒ научимся ΡΠΊΠ»Π°Π΄Ρ‹Π²Π°Ρ‚ΡŒ Π΄Ρ€ΠΎΠ±ΠΈ с Ρ€Π°Π·Π½Ρ‹ΠΌΠΈ знамСнатСлями. Когда ΡΠΊΠ»Π°Π΄Ρ‹Π²Π°ΡŽΡ‚ Π΄Ρ€ΠΎΠ±ΠΈ, Π·Π½Π°ΠΌΠ΅Π½Π°Ρ‚Π΅Π»ΠΈ этих Π΄Ρ€ΠΎΠ±Π΅ΠΉ Π΄ΠΎΠ»ΠΆΠ½Ρ‹ Π±Ρ‹Ρ‚ΡŒ ΠΎΠ΄ΠΈΠ½Π°ΠΊΠΎΠ²Ρ‹ΠΌΠΈ. Но ΠΎΠ΄ΠΈΠ½Π°ΠΊΠΎΠ²Ρ‹ΠΌΠΈ ΠΎΠ½ΠΈ Π±Ρ‹Π²Π°ΡŽΡ‚ Π½Π΅ всСгда.

НапримСр, Π΄Ρ€ΠΎΠ±ΠΈ ΠΈ ΡΠ»ΠΎΠΆΠΈΡ‚ΡŒ ΠΌΠΎΠΆΠ½ΠΎ, ΠΏΠΎΡΠΊΠΎΠ»ΡŒΠΊΡƒ Ρƒ Π½ΠΈΡ… ΠΎΠ΄ΠΈΠ½Π°ΠΊΠΎΠ²Ρ‹Π΅ Π·Π½Π°ΠΌΠ΅Π½Π°Ρ‚Π΅Π»ΠΈ.

А Π²ΠΎΡ‚ Π΄Ρ€ΠΎΠ±ΠΈ ΠΈ сразу ΡΠ»ΠΎΠΆΠΈΡ‚ΡŒ нСльзя, ΠΏΠΎΡΠΊΠΎΠ»ΡŒΠΊΡƒ Ρƒ этих Π΄Ρ€ΠΎΠ±Π΅ΠΉ Ρ€Π°Π·Π½Ρ‹Π΅ Π·Π½Π°ΠΌΠ΅Π½Π°Ρ‚Π΅Π»ΠΈ. Π’ Ρ‚Π°ΠΊΠΈΡ… случаях Π΄Ρ€ΠΎΠ±ΠΈ Π½ΡƒΠΆΠ½ΠΎ ΠΏΡ€ΠΈΠ²ΠΎΠ΄ΠΈΡ‚ΡŒ ΠΊ ΠΎΠ΄ΠΈΠ½Π°ΠΊΠΎΠ²ΠΎΠΌΡƒ (ΠΎΠ±Ρ‰Π΅ΠΌΡƒ) Π·Π½Π°ΠΌΠ΅Π½Π°Ρ‚Π΅Π»ΡŽ.

БущСствуСт нСсколько способов привСдСния Π΄Ρ€ΠΎΠ±Π΅ΠΉ ΠΊ ΠΎΠ΄ΠΈΠ½Π°ΠΊΠΎΠ²ΠΎΠΌΡƒ Π·Π½Π°ΠΌΠ΅Π½Π°Ρ‚Π΅Π»ΡŽ. БСгодня ΠΌΡ‹ рассмотрим Ρ‚ΠΎΠ»ΡŒΠΊΠΎ ΠΎΠ΄ΠΈΠ½ ΠΈΠ· Π½ΠΈΡ…, ΠΏΠΎΡΠΊΠΎΠ»ΡŒΠΊΡƒ ΠΎΡΡ‚Π°Π»ΡŒΠ½Ρ‹Π΅ способы ΠΌΠΎΠ³ΡƒΡ‚ ΠΏΠΎΠΊΠ°Π·Π°Ρ‚ΡŒΡΡ слоТными для Π½Π°Ρ‡ΠΈΠ½Π°ΡŽΡ‰Π΅Π³ΠΎ.

Π‘ΡƒΡ‚ΡŒ этого способа Π·Π°ΠΊΠ»ΡŽΡ‡Π°Π΅Ρ‚ΡΡ Π² Ρ‚ΠΎΠΌ, Ρ‡Ρ‚ΠΎ сначала ищСтся (НОК) Π·Π½Π°ΠΌΠ΅Π½Π°Ρ‚Π΅Π»Π΅ΠΉ ΠΎΠ±Π΅ΠΈΡ… Π΄Ρ€ΠΎΠ±Π΅ΠΉ. Π—Π°Ρ‚Π΅ΠΌ НОК дСлят Π½Π° Π·Π½Π°ΠΌΠ΅Π½Π°Ρ‚Π΅Π»ΡŒ ΠΏΠ΅Ρ€Π²ΠΎΠΉ Π΄Ρ€ΠΎΠ±ΠΈ ΠΈ ΠΏΠΎΠ»ΡƒΡ‡Π°ΡŽΡ‚ ΠΏΠ΅Ρ€Π²Ρ‹ΠΉ Π΄ΠΎΠΏΠΎΠ»Π½ΠΈΡ‚Π΅Π»ΡŒΠ½Ρ‹ΠΉ ΠΌΠ½ΠΎΠΆΠΈΡ‚Π΅Π»ΡŒ. Аналогично ΠΏΠΎΡΡ‚ΡƒΠΏΠ°ΡŽΡ‚ ΠΈ со Π²Ρ‚ΠΎΡ€ΠΎΠΉ Π΄Ρ€ΠΎΠ±ΡŒΡŽ β€” НОК дСлят Π½Π° Π·Π½Π°ΠΌΠ΅Π½Π°Ρ‚Π΅Π»ΡŒ Π²Ρ‚ΠΎΡ€ΠΎΠΉ Π΄Ρ€ΠΎΠ±ΠΈ ΠΈ ΠΏΠΎΠ»ΡƒΡ‡Π°ΡŽΡ‚ Π²Ρ‚ΠΎΡ€ΠΎΠΉ Π΄ΠΎΠΏΠΎΠ»Π½ΠΈΡ‚Π΅Π»ΡŒΠ½Ρ‹ΠΉ ΠΌΠ½ΠΎΠΆΠΈΡ‚Π΅Π»ΡŒ.

Π—Π°Ρ‚Π΅ΠΌ числитСли ΠΈ Π·Π½Π°ΠΌΠ΅Π½Π°Ρ‚Π΅Π»ΠΈ Π΄Ρ€ΠΎΠ±Π΅ΠΉ ΡƒΠΌΠ½ΠΎΠΆΠ°ΡŽΡ‚ΡΡ Π½Π° свои Π΄ΠΎΠΏΠΎΠ»Π½ΠΈΡ‚Π΅Π»ΡŒΠ½Ρ‹Π΅ ΠΌΠ½ΠΎΠΆΠΈΡ‚Π΅Π»ΠΈ. Π’ Ρ€Π΅Π·ΡƒΠ»ΡŒΡ‚Π°Ρ‚Π΅ этих дСйствий, Π΄Ρ€ΠΎΠ±ΠΈ Ρƒ ΠΊΠΎΡ‚ΠΎΡ€Ρ‹Ρ… Π±Ρ‹Π»ΠΈ Ρ€Π°Π·Π½Ρ‹Π΅ Π·Π½Π°ΠΌΠ΅Π½Π°Ρ‚Π΅Π»ΠΈ, ΠΎΠ±Ρ€Π°Ρ‰Π°ΡŽΡ‚ΡΡ Π² Π΄Ρ€ΠΎΠ±ΠΈ, Ρƒ ΠΊΠΎΡ‚ΠΎΡ€Ρ‹Ρ… ΠΎΠ΄ΠΈΠ½Π°ΠΊΠΎΠ²Ρ‹Π΅ Π·Π½Π°ΠΌΠ΅Π½Π°Ρ‚Π΅Π»ΠΈ. А ΠΊΠ°ΠΊ ΡΠΊΠ»Π°Π΄Ρ‹Π²Π°Ρ‚ΡŒ Ρ‚Π°ΠΊΠΈΠ΅ Π΄Ρ€ΠΎΠ±ΠΈ ΠΌΡ‹ ΡƒΠΆΠ΅ Π·Π½Π°Π΅ΠΌ.

ΠŸΡ€ΠΈΠΌΠ΅Ρ€ 1 . Π‘Π»ΠΎΠΆΠΈΠΌ Π΄Ρ€ΠΎΠ±ΠΈ ΠΈ

Π’ ΠΏΠ΅Ρ€Π²ΡƒΡŽ ΠΎΡ‡Π΅Ρ€Π΅Π΄ΡŒ Π½Π°Ρ…ΠΎΠ΄ΠΈΠΌ наимСньшСС ΠΎΠ±Ρ‰Π΅Π΅ ΠΊΡ€Π°Ρ‚Π½ΠΎΠ΅ Π·Π½Π°ΠΌΠ΅Π½Π°Ρ‚Π΅Π»Π΅ΠΉ ΠΎΠ±Π΅ΠΈΡ… Π΄Ρ€ΠΎΠ±Π΅ΠΉ. Π—Π½Π°ΠΌΠ΅Π½Π°Ρ‚Π΅Π»ΡŒ ΠΏΠ΅Ρ€Π²ΠΎΠΉ Π΄Ρ€ΠΎΠ±ΠΈ это число 3, Π° Π·Π½Π°ΠΌΠ΅Π½Π°Ρ‚Π΅Π»ΡŒ Π²Ρ‚ΠΎΡ€ΠΎΠΉ Π΄Ρ€ΠΎΠ±ΠΈ β€” число 2. НаимСньшСС ΠΎΠ±Ρ‰Π΅Π΅ ΠΊΡ€Π°Ρ‚Π½ΠΎΠ΅ этих чисСл Ρ€Π°Π²Π½ΠΎ 6

НОК (2 и 3) = 6

Π’Π΅ΠΏΠ΅Ρ€ΡŒ возвращаСмся ΠΊ дробям ΠΈ . Π‘Π½Π°Ρ‡Π°Π»Π° Ρ€Π°Π·Π΄Π΅Π»ΠΈΠΌ НОК Π½Π° Π·Π½Π°ΠΌΠ΅Π½Π°Ρ‚Π΅Π»ΡŒ ΠΏΠ΅Ρ€Π²ΠΎΠΉ Π΄Ρ€ΠΎΠ±ΠΈ ΠΈ ΠΏΠΎΠ»ΡƒΡ‡ΠΈΠΌ ΠΏΠ΅Ρ€Π²Ρ‹ΠΉ Π΄ΠΎΠΏΠΎΠ»Π½ΠΈΡ‚Π΅Π»ΡŒΠ½Ρ‹ΠΉ ΠΌΠ½ΠΎΠΆΠΈΡ‚Π΅Π»ΡŒ. НОК это число 6, Π° Π·Π½Π°ΠΌΠ΅Π½Π°Ρ‚Π΅Π»ΡŒ ΠΏΠ΅Ρ€Π²ΠΎΠΉ Π΄Ρ€ΠΎΠ±ΠΈ это число 3. Π”Π΅Π»ΠΈΠΌ 6 Π½Π° 3, ΠΏΠΎΠ»ΡƒΡ‡Π°Π΅ΠΌ 2.

ΠŸΠΎΠ»ΡƒΡ‡Π΅Π½Π½ΠΎΠ΅ число 2 это ΠΏΠ΅Ρ€Π²Ρ‹ΠΉ Π΄ΠΎΠΏΠΎΠ»Π½ΠΈΡ‚Π΅Π»ΡŒΠ½Ρ‹ΠΉ ΠΌΠ½ΠΎΠΆΠΈΡ‚Π΅Π»ΡŒ. ЗаписываСм Π΅Π³ΠΎ ΠΊ ΠΏΠ΅Ρ€Π²ΠΎΠΉ Π΄Ρ€ΠΎΠ±ΠΈ. Для этого Π΄Π΅Π»Π°Π΅ΠΌ Π½Π΅Π±ΠΎΠ»ΡŒΡˆΡƒΡŽ ΠΊΠΎΡΡƒΡŽ линию Π½Π°Π΄ Π΄Ρ€ΠΎΠ±ΡŒΡŽ ΠΈ записываСм Π½Π°Π΄ Π½Π΅ΠΉ Π½Π°ΠΉΠ΄Π΅Π½Π½Ρ‹ΠΉ Π΄ΠΎΠΏΠΎΠ»Π½ΠΈΡ‚Π΅Π»ΡŒΠ½Ρ‹ΠΉ ΠΌΠ½ΠΎΠΆΠΈΡ‚Π΅Π»ΡŒ:

Аналогично поступаСм ΠΈ со Π²Ρ‚ΠΎΡ€ΠΎΠΉ Π΄Ρ€ΠΎΠ±ΡŒΡŽ. Π”Π΅Π»ΠΈΠΌ НОК Π½Π° Π·Π½Π°ΠΌΠ΅Π½Π°Ρ‚Π΅Π»ΡŒ Π²Ρ‚ΠΎΡ€ΠΎΠΉ Π΄Ρ€ΠΎΠ±ΠΈ ΠΈ ΠΏΠΎΠ»ΡƒΡ‡Π°Π΅ΠΌ Π²Ρ‚ΠΎΡ€ΠΎΠΉ Π΄ΠΎΠΏΠΎΠ»Π½ΠΈΡ‚Π΅Π»ΡŒΠ½Ρ‹ΠΉ ΠΌΠ½ΠΎΠΆΠΈΡ‚Π΅Π»ΡŒ. НОК это число 6, Π° Π·Π½Π°ΠΌΠ΅Π½Π°Ρ‚Π΅Π»ΡŒ Π²Ρ‚ΠΎΡ€ΠΎΠΉ Π΄Ρ€ΠΎΠ±ΠΈ β€” число 2. Π”Π΅Π»ΠΈΠΌ 6 Π½Π° 2, ΠΏΠΎΠ»ΡƒΡ‡Π°Π΅ΠΌ 3.

ΠŸΠΎΠ»ΡƒΡ‡Π΅Π½Π½ΠΎΠ΅ число 3 это Π²Ρ‚ΠΎΡ€ΠΎΠΉ Π΄ΠΎΠΏΠΎΠ»Π½ΠΈΡ‚Π΅Π»ΡŒΠ½Ρ‹ΠΉ ΠΌΠ½ΠΎΠΆΠΈΡ‚Π΅Π»ΡŒ. ЗаписываСм Π΅Π³ΠΎ ΠΊΠΎ Π²Ρ‚ΠΎΡ€ΠΎΠΉ Π΄Ρ€ΠΎΠ±ΠΈ. ΠžΠΏΡΡ‚ΡŒ ΠΆΠ΅ Π΄Π΅Π»Π°Π΅ΠΌ Π½Π΅Π±ΠΎΠ»ΡŒΡˆΡƒΡŽ ΠΊΠΎΡΡƒΡŽ линию Π½Π°Π΄ Π²Ρ‚ΠΎΡ€ΠΎΠΉ Π΄Ρ€ΠΎΠ±ΡŒΡŽ ΠΈ записываСм Π½Π°Π΄ Π½Π΅ΠΉ Π½Π°ΠΉΠ΄Π΅Π½Π½Ρ‹ΠΉ Π΄ΠΎΠΏΠΎΠ»Π½ΠΈΡ‚Π΅Π»ΡŒΠ½Ρ‹ΠΉ ΠΌΠ½ΠΎΠΆΠΈΡ‚Π΅Π»ΡŒ:

Π’Π΅ΠΏΠ΅Ρ€ΡŒ Ρƒ нас всё Π³ΠΎΡ‚ΠΎΠ²ΠΎ для слоТСния. ΠžΡΡ‚Π°Π»ΠΎΡΡŒ ΡƒΠΌΠ½ΠΎΠΆΠΈΡ‚ΡŒ числитСли ΠΈ Π·Π½Π°ΠΌΠ΅Π½Π°Ρ‚Π΅Π»ΠΈ Π΄Ρ€ΠΎΠ±Π΅ΠΉ Π½Π° свои Π΄ΠΎΠΏΠΎΠ»Π½ΠΈΡ‚Π΅Π»ΡŒΠ½Ρ‹Π΅ ΠΌΠ½ΠΎΠΆΠΈΡ‚Π΅Π»ΠΈ:

ΠŸΠΎΡΠΌΠΎΡ‚Ρ€ΠΈΡ‚Π΅ Π²Π½ΠΈΠΌΠ°Ρ‚Π΅Π»ΡŒΠ½ΠΎ ΠΊ Ρ‡Π΅ΠΌΡƒ ΠΌΡ‹ ΠΏΡ€ΠΈΡˆΠ»ΠΈ. ΠœΡ‹ ΠΏΡ€ΠΈΡˆΠ»ΠΈ ΠΊ Ρ‚ΠΎΠΌΡƒ, Ρ‡Ρ‚ΠΎ Π΄Ρ€ΠΎΠ±ΠΈ Ρƒ ΠΊΠΎΡ‚ΠΎΡ€Ρ‹Ρ… Π±Ρ‹Π»ΠΈ Ρ€Π°Π·Π½Ρ‹Π΅ Π·Π½Π°ΠΌΠ΅Π½Π°Ρ‚Π΅Π»ΠΈ, ΠΏΡ€Π΅Π²Ρ€Π°Ρ‚ΠΈΠ»ΠΈΡΡŒ Π² Π΄Ρ€ΠΎΠ±ΠΈ Ρƒ ΠΊΠΎΡ‚ΠΎΡ€Ρ‹Ρ… ΠΎΠ΄ΠΈΠ½Π°ΠΊΠΎΠ²Ρ‹Π΅ Π·Π½Π°ΠΌΠ΅Π½Π°Ρ‚Π΅Π»ΠΈ. А ΠΊΠ°ΠΊ ΡΠΊΠ»Π°Π΄Ρ‹Π²Π°Ρ‚ΡŒ Ρ‚Π°ΠΊΠΈΠ΅ Π΄Ρ€ΠΎΠ±ΠΈ ΠΌΡ‹ ΡƒΠΆΠ΅ Π·Π½Π°Π΅ΠΌ. Π”Π°Π²Π°ΠΉΡ‚Π΅ Π΄ΠΎΡ€Π΅ΡˆΠ°Π΅ΠΌ этот ΠΏΡ€ΠΈΠΌΠ΅Ρ€ Π΄ΠΎ ΠΊΠΎΠ½Ρ†Π°:

Π’Π°ΠΊΠΈΠΌ ΠΎΠ±Ρ€Π°Π·ΠΎΠΌ, ΠΏΡ€ΠΈΠΌΠ΅Ρ€ Π·Π°Π²Π΅Ρ€ΡˆΠ°Π΅Ρ‚ΡΡ. К ΠΏΡ€ΠΈΠ±Π°Π²ΠΈΡ‚ΡŒ получаСтся .

ΠŸΠΎΠΏΡ€ΠΎΠ±ΡƒΠ΅ΠΌ ΠΈΠ·ΠΎΠ±Ρ€Π°Π·ΠΈΡ‚ΡŒ нашС Ρ€Π΅ΡˆΠ΅Π½ΠΈΠ΅ с ΠΏΠΎΠΌΠΎΡ‰ΡŒΡŽ рисунка. Если ΠΊ ΠΏΠΈΡ†Ρ†Ρ‹ ΠΏΡ€ΠΈΠ±Π°Π²ΠΈΡ‚ΡŒ ΠΏΠΈΡ†Ρ†Ρ‹, Ρ‚ΠΎ получится ΠΎΠ΄Π½Π° цСлая ΠΏΠΈΡ†Ρ†Π° ΠΈ Π΅Ρ‰Π΅ ΠΎΠ΄Π½Π° ΡˆΠ΅ΡΡ‚Π°Ρ ΠΏΠΈΡ†Ρ†Ρ‹:

ΠŸΡ€ΠΈΠ²Π΅Π΄Π΅Π½ΠΈΠ΅ Π΄Ρ€ΠΎΠ±Π΅ΠΉ ΠΊ ΠΎΠ΄ΠΈΠ½Π°ΠΊΠΎΠ²ΠΎΠΌΡƒ (ΠΎΠ±Ρ‰Π΅ΠΌΡƒ) Π·Π½Π°ΠΌΠ΅Π½Π°Ρ‚Π΅Π»ΡŽ Ρ‚Π°ΠΊΠΆΠ΅ ΠΌΠΎΠΆΠ½ΠΎ ΠΈΠ·ΠΎΠ±Ρ€Π°Π·ΠΈΡ‚ΡŒ с ΠΏΠΎΠΌΠΎΡ‰ΡŒΡŽ рисунка. ΠŸΡ€ΠΈΠ²Π΅Π΄Ρ Π΄Ρ€ΠΎΠ±ΠΈ ΠΈ ΠΊ ΠΎΠ±Ρ‰Π΅ΠΌΡƒ Π·Π½Π°ΠΌΠ΅Π½Π°Ρ‚Π΅Π»ΡŽ, ΠΌΡ‹ ΠΏΠΎΠ»ΡƒΡ‡ΠΈΠ»ΠΈ Π΄Ρ€ΠΎΠ±ΠΈ ΠΈ . Π­Ρ‚ΠΈ Π΄Π²Π΅ Π΄Ρ€ΠΎΠ±ΠΈ Π±ΡƒΠ΄ΡƒΡ‚ ΠΈΠ·ΠΎΠ±Ρ€Π°ΠΆΠ°Ρ‚ΡŒΡΡ Ρ‚Π΅ΠΌΠΈ ΠΆΠ΅ кусками ΠΏΠΈΡ†Ρ†. Π Π°Π·Π»ΠΈΡ‡ΠΈΠ΅ Π±ΡƒΠ΄Π΅Ρ‚ лишь Π² Ρ‚ΠΎΠΌ, Ρ‡Ρ‚ΠΎ Π² этот Ρ€Π°Π· ΠΎΠ½ΠΈ Π±ΡƒΠ΄ΡƒΡ‚ Ρ€Π°Π·Π΄Π΅Π»Π΅Π½Ρ‹ Π½Π° ΠΎΠ΄ΠΈΠ½Π°ΠΊΠΎΠ²Ρ‹Π΅ Π΄ΠΎΠ»ΠΈ (ΠΏΡ€ΠΈΠ²Π΅Π΄Π΅Π½Ρ‹ ΠΊ ΠΎΠ΄ΠΈΠ½Π°ΠΊΠΎΠ²ΠΎΠΌΡƒ Π·Π½Π°ΠΌΠ΅Π½Π°Ρ‚Π΅Π»ΡŽ).

ΠŸΠ΅Ρ€Π²Ρ‹ΠΉ рисунок ΠΈΠ·ΠΎΠ±Ρ€Π°ΠΆΠ°Π΅Ρ‚ Π΄Ρ€ΠΎΠ±ΡŒ (Ρ‡Π΅Ρ‚Ρ‹Ρ€Π΅ кусочка ΠΈΠ· ΡˆΠ΅ΡΡ‚ΠΈ), Π° Π²Ρ‚ΠΎΡ€ΠΎΠΉ рисунок ΠΈΠ·ΠΎΠ±Ρ€Π°ΠΆΠ°Π΅Ρ‚ Π΄Ρ€ΠΎΠ±ΡŒ (Ρ‚Ρ€ΠΈ кусочка ΠΈΠ· ΡˆΠ΅ΡΡ‚ΠΈ). Π‘Π»ΠΎΠΆΠΈΠ² эти кусочки ΠΌΡ‹ ΠΏΠΎΠ»ΡƒΡ‡Π°Π΅ΠΌ (сСмь кусочков ΠΈΠ· ΡˆΠ΅ΡΡ‚ΠΈ). Π­Ρ‚Π° Π΄Ρ€ΠΎΠ±ΡŒ Π½Π΅ΠΏΡ€Π°Π²ΠΈΠ»ΡŒΠ½Π°Ρ, поэтому ΠΌΡ‹ Π²Ρ‹Π΄Π΅Π»ΠΈΠ»ΠΈ Π² Π½Π΅ΠΉ Ρ†Π΅Π»ΡƒΡŽ Ρ‡Π°ΡΡ‚ΡŒ. Π’ Ρ€Π΅Π·ΡƒΠ»ΡŒΡ‚Π°Ρ‚Π΅ ΠΏΠΎΠ»ΡƒΡ‡ΠΈΠ»ΠΈ (ΠΎΠ΄Π½Ρƒ Ρ†Π΅Π»ΡƒΡŽ ΠΏΠΈΡ†Ρ†Ρƒ ΠΈ Π΅Ρ‰Π΅ ΠΎΠ΄Π½Ρƒ ΡˆΠ΅ΡΡ‚ΡƒΡŽ ΠΏΠΈΡ†Ρ†Ρ‹).

ΠžΡ‚ΠΌΠ΅Ρ‚ΠΈΠΌ, Ρ‡Ρ‚ΠΎ ΠΌΡ‹ с Π²Π°ΠΌΠΈ расписали Π΄Π°Π½Π½Ρ‹ΠΉ ΠΏΡ€ΠΈΠΌΠ΅Ρ€ слишком ΠΏΠΎΠ΄Ρ€ΠΎΠ±Π½ΠΎ. Π’ ΡƒΡ‡Π΅Π±Π½Ρ‹Ρ… завСдСниях Π½Π΅ принято ΠΏΠΈΡΠ°Ρ‚ΡŒ Ρ‚Π°ΠΊ Ρ€Π°Π·Π²Ρ‘Ρ€Π½ΡƒΡ‚ΠΎ. НуТно ΡƒΠΌΠ΅Ρ‚ΡŒ быстро Π½Π°Ρ…ΠΎΠ΄ΠΈΡ‚ΡŒ НОК ΠΎΠ±ΠΎΠΈΡ… Π·Π½Π°ΠΌΠ΅Π½Π°Ρ‚Π΅Π»Π΅ΠΉ ΠΈ Π΄ΠΎΠΏΠΎΠ»Π½ΠΈΡ‚Π΅Π»ΡŒΠ½Ρ‹Π΅ ΠΌΠ½ΠΎΠΆΠΈΡ‚Π΅Π»ΠΈ ΠΊ Π½ΠΈΠΌ, Π° Ρ‚Π°ΠΊΠΆΠ΅ быстро ΡƒΠΌΠ½ΠΎΠΆΠ°Ρ‚ΡŒ Π½Π°ΠΉΠ΄Π΅Π½Π½Ρ‹Π΅ Π΄ΠΎΠΏΠΎΠ»Π½ΠΈΡ‚Π΅Π»ΡŒΠ½Ρ‹Π΅ ΠΌΠ½ΠΎΠΆΠΈΡ‚Π΅Π»ΠΈ Π½Π° свои числитСли ΠΈ Π·Π½Π°ΠΌΠ΅Π½Π°Ρ‚Π΅Π»ΠΈ. ΠΠ°Ρ…ΠΎΠ΄ΡΡΡŒ Π² школС, Π΄Π°Π½Π½Ρ‹ΠΉ ΠΏΡ€ΠΈΠΌΠ΅Ρ€ Π½Π°ΠΌ ΠΏΡ€ΠΈΡˆΠ»ΠΎΡΡŒ Π±Ρ‹ Π·Π°ΠΏΠΈΡΠ°Ρ‚ΡŒ ΡΠ»Π΅Π΄ΡƒΡŽΡ‰ΠΈΠΌ ΠΎΠ±Ρ€Π°Π·ΠΎΠΌ:

Но Π΅ΡΡ‚ΡŒ ΠΈ обратная сторона ΠΌΠ΅Π΄Π°Π»ΠΈ. Если Π½Π° ΠΏΠ΅Ρ€Π²Ρ‹Ρ… этапах изучСния ΠΌΠ°Ρ‚Π΅ΠΌΠ°Ρ‚ΠΈΠΊΠΈ Π½Π΅ Π΄Π΅Π»Π°Ρ‚ΡŒ ΠΏΠΎΠ΄Ρ€ΠΎΠ±Π½Ρ‹Ρ… записСй, Ρ‚ΠΎ Π½Π°Ρ‡ΠΈΠ½Π°ΡŽΡ‚ ΠΏΠΎΡΠ²Π»ΡΡ‚ΡŒΡΡ вопросы Ρ€ΠΎΠ΄Π° Β«Π° ΠΎΡ‚ΠΊΡƒΠ΄Π° Π²ΠΎΠ½ Ρ‚Π° Ρ†ΠΈΡ„Ρ€Π°?Β», Β«ΠΏΠΎΡ‡Π΅ΠΌΡƒ Π΄Ρ€ΠΎΠ±ΠΈ Π²Π΄Ρ€ΡƒΠ³ ΠΏΡ€Π΅Π²Ρ€Π°Ρ‰Π°ΡŽΡ‚ΡΡ совсСм Π² Π΄Ρ€ΡƒΠ³ΠΈΠ΅ Π΄Ρ€ΠΎΠ±ΠΈ? Β«.

Π§Ρ‚ΠΎΠ±Ρ‹ Π»Π΅Π³Ρ‡Π΅ Π±Ρ‹Π»ΠΎ ΡΠΊΠ»Π°Π΄Ρ‹Π²Π°Ρ‚ΡŒ Π΄Ρ€ΠΎΠ±ΠΈ с Ρ€Π°Π·Π½Ρ‹ΠΌΠΈ знамСнатСлями, ΠΌΠΎΠΆΠ½ΠΎ Π²ΠΎΡΠΏΠΎΠ»ΡŒΠ·ΠΎΠ²Π°Ρ‚ΡŒΡΡ ΡΠ»Π΅Π΄ΡƒΡŽΡ‰Π΅ΠΉ пошаговой инструкциСй:

  1. Найти НОК Π·Π½Π°ΠΌΠ΅Π½Π°Ρ‚Π΅Π»Π΅ΠΉ Π΄Ρ€ΠΎΠ±Π΅ΠΉ;
  2. Π Π°Π·Π΄Π΅Π»ΠΈΡ‚ΡŒ НОК Π½Π° Π·Π½Π°ΠΌΠ΅Π½Π°Ρ‚Π΅Π»ΡŒ ΠΊΠ°ΠΆΠ΄ΠΎΠΉ Π΄Ρ€ΠΎΠ±ΠΈ ΠΈ ΠΏΠΎΠ»ΡƒΡ‡ΠΈΡ‚ΡŒ Π΄ΠΎΠΏΠΎΠ»Π½ΠΈΡ‚Π΅Π»ΡŒΠ½Ρ‹ΠΉ ΠΌΠ½ΠΎΠΆΠΈΡ‚Π΅Π»ΡŒ для ΠΊΠ°ΠΆΠ΄ΠΎΠΉ Π΄Ρ€ΠΎΠ±ΠΈ;
  3. Π£ΠΌΠ½ΠΎΠΆΠΈΡ‚ΡŒ числитСли ΠΈ Π·Π½Π°ΠΌΠ΅Π½Π°Ρ‚Π΅Π»ΠΈ Π΄Ρ€ΠΎΠ±Π΅ΠΉ Π½Π° свои Π΄ΠΎΠΏΠΎΠ»Π½ΠΈΡ‚Π΅Π»ΡŒΠ½Ρ‹Π΅ ΠΌΠ½ΠΎΠΆΠΈΡ‚Π΅Π»ΠΈ;
  4. Π‘Π»ΠΎΠΆΠΈΡ‚ΡŒ Π΄Ρ€ΠΎΠ±ΠΈ, Ρƒ ΠΊΠΎΡ‚ΠΎΡ€Ρ‹Ρ… ΠΎΠ΄ΠΈΠ½Π°ΠΊΠΎΠ²Ρ‹Π΅ Π·Π½Π°ΠΌΠ΅Π½Π°Ρ‚Π΅Π»ΠΈ;
  5. Если Π² ΠΎΡ‚Π²Π΅Ρ‚Π΅ ΠΏΠΎΠ»ΡƒΡ‡ΠΈΠ»Π°ΡΡŒ Π½Π΅ΠΏΡ€Π°Π²ΠΈΠ»ΡŒΠ½Π°Ρ Π΄Ρ€ΠΎΠ±ΡŒ, Ρ‚ΠΎ Π²Ρ‹Π΄Π΅Π»ΠΈΡ‚ΡŒ Π΅Ρ‘ Ρ†Π΅Π»ΡƒΡŽ Ρ‡Π°ΡΡ‚ΡŒ;

ΠŸΡ€ΠΈΠΌΠ΅Ρ€ 2. Найти Π·Π½Π°Ρ‡Π΅Π½ΠΈΠ΅ выраТСния .

Π’ΠΎΡΠΏΠΎΠ»ΡŒΠ·ΡƒΠ΅ΠΌΡΡ инструкциСй, которая ΠΏΡ€ΠΈΠ²Π΅Π΄Π΅Π½Π° Π²Ρ‹ΡˆΠ΅.

Π¨Π°Π³ 1. Найти НОК Π·Π½Π°ΠΌΠ΅Π½Π°Ρ‚Π΅Π»Π΅ΠΉ Π΄Ρ€ΠΎΠ±Π΅ΠΉ

Находим НОК Π·Π½Π°ΠΌΠ΅Π½Π°Ρ‚Π΅Π»Π΅ΠΉ ΠΎΠ±Π΅ΠΈΡ… Π΄Ρ€ΠΎΠ±Π΅ΠΉ. Π—Π½Π°ΠΌΠ΅Π½Π°Ρ‚Π΅Π»ΠΈ Π΄Ρ€ΠΎΠ±Π΅ΠΉ это числа 2, 3 ΠΈ 4

Π¨Π°Π³ 2. Π Π°Π·Π΄Π΅Π»ΠΈΡ‚ΡŒ НОК Π½Π° Π·Π½Π°ΠΌΠ΅Π½Π°Ρ‚Π΅Π»ΡŒ ΠΊΠ°ΠΆΠ΄ΠΎΠΉ Π΄Ρ€ΠΎΠ±ΠΈ ΠΈ ΠΏΠΎΠ»ΡƒΡ‡ΠΈΡ‚ΡŒ Π΄ΠΎΠΏΠΎΠ»Π½ΠΈΡ‚Π΅Π»ΡŒΠ½Ρ‹ΠΉ ΠΌΠ½ΠΎΠΆΠΈΡ‚Π΅Π»ΡŒ для ΠΊΠ°ΠΆΠ΄ΠΎΠΉ Π΄Ρ€ΠΎΠ±ΠΈ

Π”Π΅Π»ΠΈΠΌ НОК Π½Π° Π·Π½Π°ΠΌΠ΅Π½Π°Ρ‚Π΅Π»ΡŒ ΠΏΠ΅Ρ€Π²ΠΎΠΉ Π΄Ρ€ΠΎΠ±ΠΈ. НОК это число 12, Π° Π·Π½Π°ΠΌΠ΅Π½Π°Ρ‚Π΅Π»ΡŒ ΠΏΠ΅Ρ€Π²ΠΎΠΉ Π΄Ρ€ΠΎΠ±ΠΈ это число 2. Π”Π΅Π»ΠΈΠΌ 12 Π½Π° 2, ΠΏΠΎΠ»ΡƒΡ‡Π°Π΅ΠΌ 6. ΠŸΠΎΠ»ΡƒΡ‡ΠΈΠ»ΠΈ ΠΏΠ΅Ρ€Π²Ρ‹ΠΉ Π΄ΠΎΠΏΠΎΠ»Π½ΠΈΡ‚Π΅Π»ΡŒΠ½Ρ‹ΠΉ ΠΌΠ½ΠΎΠΆΠΈΡ‚Π΅Π»ΡŒ 6. ЗаписываСм Π΅Π³ΠΎ Π½Π°Π΄ ΠΏΠ΅Ρ€Π²ΠΎΠΉ Π΄Ρ€ΠΎΠ±ΡŒΡŽ:

Π’Π΅ΠΏΠ΅Ρ€ΡŒ Π΄Π΅Π»ΠΈΠΌ НОК Π½Π° Π·Π½Π°ΠΌΠ΅Π½Π°Ρ‚Π΅Π»ΡŒ Π²Ρ‚ΠΎΡ€ΠΎΠΉ Π΄Ρ€ΠΎΠ±ΠΈ. НОК это число 12, Π° Π·Π½Π°ΠΌΠ΅Π½Π°Ρ‚Π΅Π»ΡŒ Π²Ρ‚ΠΎΡ€ΠΎΠΉ Π΄Ρ€ΠΎΠ±ΠΈ это число 3. Π”Π΅Π»ΠΈΠΌ 12 Π½Π° 3, ΠΏΠΎΠ»ΡƒΡ‡Π°Π΅ΠΌ 4. ΠŸΠΎΠ»ΡƒΡ‡ΠΈΠ»ΠΈ Π²Ρ‚ΠΎΡ€ΠΎΠΉ Π΄ΠΎΠΏΠΎΠ»Π½ΠΈΡ‚Π΅Π»ΡŒΠ½Ρ‹ΠΉ ΠΌΠ½ΠΎΠΆΠΈΡ‚Π΅Π»ΡŒ 4. ЗаписываСм Π΅Π³ΠΎ Π½Π°Π΄ Π²Ρ‚ΠΎΡ€ΠΎΠΉ Π΄Ρ€ΠΎΠ±ΡŒΡŽ:

Π’Π΅ΠΏΠ΅Ρ€ΡŒ Π΄Π΅Π»ΠΈΠΌ НОК Π½Π° Π·Π½Π°ΠΌΠ΅Π½Π°Ρ‚Π΅Π»ΡŒ Ρ‚Ρ€Π΅Ρ‚ΡŒΠ΅ΠΉ Π΄Ρ€ΠΎΠ±ΠΈ. НОК это число 12, Π° Π·Π½Π°ΠΌΠ΅Π½Π°Ρ‚Π΅Π»ΡŒ Ρ‚Ρ€Π΅Ρ‚ΡŒΠ΅ΠΉ Π΄Ρ€ΠΎΠ±ΠΈ это число 4. Π”Π΅Π»ΠΈΠΌ 12 Π½Π° 4, ΠΏΠΎΠ»ΡƒΡ‡Π°Π΅ΠΌ 3. ΠŸΠΎΠ»ΡƒΡ‡ΠΈΠ»ΠΈ Ρ‚Ρ€Π΅Ρ‚ΠΈΠΉ Π΄ΠΎΠΏΠΎΠ»Π½ΠΈΡ‚Π΅Π»ΡŒΠ½Ρ‹ΠΉ ΠΌΠ½ΠΎΠΆΠΈΡ‚Π΅Π»ΡŒ 3. ЗаписываСм Π΅Π³ΠΎ Π½Π°Π΄ Ρ‚Ρ€Π΅Ρ‚ΡŒΠ΅ΠΉ Π΄Ρ€ΠΎΠ±ΡŒΡŽ:

Π¨Π°Π³ 3. Π£ΠΌΠ½ΠΎΠΆΠΈΡ‚ΡŒ числитСли ΠΈ Π·Π½Π°ΠΌΠ΅Π½Π°Ρ‚Π΅Π»ΠΈ Π΄Ρ€ΠΎΠ±Π΅ΠΉ Π½Π° свои Π΄ΠΎΠΏΠΎΠ»Π½ΠΈΡ‚Π΅Π»ΡŒΠ½Ρ‹Π΅ ΠΌΠ½ΠΎΠΆΠΈΡ‚Π΅Π»ΠΈ

Π£ΠΌΠ½ΠΎΠΆΠ°Π΅ΠΌ числитСли ΠΈ Π·Π½Π°ΠΌΠ΅Π½Π°Ρ‚Π΅Π»ΠΈ Π½Π° свои Π΄ΠΎΠΏΠΎΠ»Π½ΠΈΡ‚Π΅Π»ΡŒΠ½Ρ‹Π΅ ΠΌΠ½ΠΎΠΆΠΈΡ‚Π΅Π»ΠΈ:

Π¨Π°Π³ 4. Π‘Π»ΠΎΠΆΠΈΡ‚ΡŒ Π΄Ρ€ΠΎΠ±ΠΈ Ρƒ ΠΊΠΎΡ‚ΠΎΡ€Ρ‹Ρ… ΠΎΠ΄ΠΈΠ½Π°ΠΊΠΎΠ²Ρ‹Π΅ Π·Π½Π°ΠΌΠ΅Π½Π°Ρ‚Π΅Π»ΠΈ

ΠœΡ‹ ΠΏΡ€ΠΈΡˆΠ»ΠΈ ΠΊ Ρ‚ΠΎΠΌΡƒ, Ρ‡Ρ‚ΠΎ Π΄Ρ€ΠΎΠ±ΠΈ Ρƒ ΠΊΠΎΡ‚ΠΎΡ€Ρ‹Ρ… Π±Ρ‹Π»ΠΈ Ρ€Π°Π·Π½Ρ‹Π΅ Π·Π½Π°ΠΌΠ΅Π½Π°Ρ‚Π΅Π»ΠΈ, ΠΏΡ€Π΅Π²Ρ€Π°Ρ‚ΠΈΠ»ΠΈΡΡŒ Π² Π΄Ρ€ΠΎΠ±ΠΈ, Ρƒ ΠΊΠΎΡ‚ΠΎΡ€Ρ‹Ρ… ΠΎΠ΄ΠΈΠ½Π°ΠΊΠΎΠ²Ρ‹Π΅ (ΠΎΠ±Ρ‰ΠΈΠ΅) Π·Π½Π°ΠΌΠ΅Π½Π°Ρ‚Π΅Π»ΠΈ. ΠžΡΡ‚Π°Π»ΠΎΡΡŒ ΡΠ»ΠΎΠΆΠΈΡ‚ΡŒ эти Π΄Ρ€ΠΎΠ±ΠΈ. Π‘ΠΊΠ»Π°Π΄Ρ‹Π²Π°Π΅ΠΌ:

Π‘Π»ΠΎΠΆΠ΅Π½ΠΈΠ΅ Π½Π΅ ΠΏΠΎΠΌΠ΅ΡΡ‚ΠΈΠ»ΠΎΡΡŒ Π½Π° ΠΎΠ΄Π½ΠΎΠΉ строкС, поэтому ΠΌΡ‹ пСрСнСсли ΠΎΡΡ‚Π°Π²ΡˆΠ΅Π΅ΡΡ Π²Ρ‹Ρ€Π°ΠΆΠ΅Π½ΠΈΠ΅ Π½Π° ΡΠ»Π΅Π΄ΡƒΡŽΡ‰ΡƒΡŽ строку. Π­Ρ‚ΠΎ допускаСтся Π² ΠΌΠ°Ρ‚Π΅ΠΌΠ°Ρ‚ΠΈΠΊΠ΅. Когда Π²Ρ‹Ρ€Π°ΠΆΠ΅Π½ΠΈΠ΅ Π½Π΅ помСщаСтся Π½Π° ΠΎΠ΄Π½Ρƒ строку, Π΅Π³ΠΎ пСрСносят Π½Π° ΡΠ»Π΅Π΄ΡƒΡŽΡ‰ΡƒΡŽ строку, ΠΏΡ€ΠΈ этом Π½Π°Π΄ΠΎ ΠΎΠ±ΡΠ·Π°Ρ‚Π΅Π»ΡŒΠ½ΠΎ ΠΏΠΎΡΡ‚Π°Π²ΠΈΡ‚ΡŒ Π·Π½Π°ΠΊ равСнства (=) Π½Π° ΠΊΠΎΠ½Ρ†Π΅ ΠΏΠ΅Ρ€Π²ΠΎΠΉ строки ΠΈ Π² Π½Π°Ρ‡Π°Π»Π΅ Π½ΠΎΠ²ΠΎΠΉ строки. Π—Π½Π°ΠΊ равСнства Π½Π° Π²Ρ‚ΠΎΡ€ΠΎΠΉ строкС Π³ΠΎΠ²ΠΎΡ€ΠΈΡ‚ ΠΎ Ρ‚ΠΎΠΌ, Ρ‡Ρ‚ΠΎ это ΠΏΡ€ΠΎΠ΄ΠΎΠ»ΠΆΠ΅Π½ΠΈΠ΅ выраТСния, ΠΊΠΎΡ‚ΠΎΡ€ΠΎΠ΅ Π±Ρ‹Π»ΠΎ Π½Π° ΠΏΠ΅Ρ€Π²ΠΎΠΉ строкС.

Π¨Π°Π³ 5. Если Π² ΠΎΡ‚Π²Π΅Ρ‚Π΅ ΠΏΠΎΠ»ΡƒΡ‡ΠΈΠ»Π°ΡΡŒ Π½Π΅ΠΏΡ€Π°Π²ΠΈΠ»ΡŒΠ½Π°Ρ Π΄Ρ€ΠΎΠ±ΡŒ, Ρ‚ΠΎ Π²Ρ‹Π΄Π΅Π»ΠΈΡ‚ΡŒ Π² Π½Π΅ΠΉ Ρ†Π΅Π»ΡƒΡŽ Ρ‡Π°ΡΡ‚ΡŒ

Π£ нас Π² ΠΎΡ‚Π²Π΅Ρ‚Π΅ ΠΏΠΎΠ»ΡƒΡ‡ΠΈΠ»Π°ΡΡŒ Π½Π΅ΠΏΡ€Π°Π²ΠΈΠ»ΡŒΠ½Π°Ρ Π΄Ρ€ΠΎΠ±ΡŒ. ΠœΡ‹ Π΄ΠΎΠ»ΠΆΠ½Ρ‹ Π²Ρ‹Π΄Π΅Π»ΠΈΡ‚ΡŒ Ρƒ Π½Π΅Ρ‘ Ρ†Π΅Π»ΡƒΡŽ Ρ‡Π°ΡΡ‚ΡŒ. ВыдСляСм:

ΠŸΠΎΠ»ΡƒΡ‡ΠΈΠ»ΠΈ ΠΎΡ‚Π²Π΅Ρ‚

Π’Ρ‹Ρ‡ΠΈΡ‚Π°Π½ΠΈΠ΅ Π΄Ρ€ΠΎΠ±Π΅ΠΉ с ΠΎΠ΄ΠΈΠ½Π°ΠΊΠΎΠ²Ρ‹ΠΌΠΈ знамСнатСлями

Π’Ρ‹Ρ‡ΠΈΡ‚Π°Π½ΠΈΠ΅ Π΄Ρ€ΠΎΠ±Π΅ΠΉ Π±Ρ‹Π²Π°Π΅Ρ‚ Π΄Π²ΡƒΡ… Π²ΠΈΠ΄ΠΎΠ²:

  1. Π’Ρ‹Ρ‡ΠΈΡ‚Π°Π½ΠΈΠ΅ Π΄Ρ€ΠΎΠ±Π΅ΠΉ с ΠΎΠ΄ΠΈΠ½Π°ΠΊΠΎΠ²Ρ‹ΠΌΠΈ знамСнатСлями
  2. Π’Ρ‹Ρ‡ΠΈΡ‚Π°Π½ΠΈΠ΅ Π΄Ρ€ΠΎΠ±Π΅ΠΉ с Ρ€Π°Π·Π½Ρ‹ΠΌΠΈ знамСнатСлями

Π‘Π½Π°Ρ‡Π°Π»Π° ΠΈΠ·ΡƒΡ‡ΠΈΠΌ Π²Ρ‹Ρ‡ΠΈΡ‚Π°Π½ΠΈΠ΅ Π΄Ρ€ΠΎΠ±Π΅ΠΉ с ΠΎΠ΄ΠΈΠ½Π°ΠΊΠΎΠ²Ρ‹ΠΌΠΈ знамСнатСлями.

Π§Ρ‚ΠΎΠ±Ρ‹ Π²Ρ‹Ρ‡Π΅ΡΡ‚ΡŒ ΠΈΠ· ΠΎΠ΄Π½ΠΎΠΉ Π΄Ρ€ΠΎΠ±ΠΈ Π΄Ρ€ΡƒΠ³ΡƒΡŽ, Π½ΡƒΠΆΠ½ΠΎ ΠΈΠ· числитСля ΠΏΠ΅Ρ€Π²ΠΎΠΉ Π΄Ρ€ΠΎΠ±ΠΈ Π²Ρ‹Ρ‡Π΅ΡΡ‚ΡŒ Ρ‡ΠΈΡΠ»ΠΈΡ‚Π΅Π»ΡŒ Π²Ρ‚ΠΎΡ€ΠΎΠΉ Π΄Ρ€ΠΎΠ±ΠΈ, Π° Π·Π½Π°ΠΌΠ΅Π½Π°Ρ‚Π΅Π»ΡŒ ΠΎΡΡ‚Π°Π²ΠΈΡ‚ΡŒ Π±Π΅Π· измСнСния.

НапримСр, Π½Π°ΠΉΠ΄Ρ‘ΠΌ Π·Π½Π°Ρ‡Π΅Π½ΠΈΠ΅ выраТСния . Π§Ρ‚ΠΎΠ±Ρ‹ Ρ€Π΅ΡˆΠΈΡ‚ΡŒ этот ΠΏΡ€ΠΈΠΌΠ΅Ρ€, Π½Π°Π΄ΠΎ ΠΈΠ· числитСля ΠΏΠ΅Ρ€Π²ΠΎΠΉ Π΄Ρ€ΠΎΠ±ΠΈ Π²Ρ‹Ρ‡Π΅ΡΡ‚ΡŒ Ρ‡ΠΈΡΠ»ΠΈΡ‚Π΅Π»ΡŒ Π²Ρ‚ΠΎΡ€ΠΎΠΉ Π΄Ρ€ΠΎΠ±ΠΈ, Π° Π·Π½Π°ΠΌΠ΅Π½Π°Ρ‚Π΅Π»ΡŒ ΠΎΡΡ‚Π°Π²ΠΈΡ‚ΡŒ Π±Π΅Π· измСнСния. Π’Π°ΠΊ ΠΈ сдСлаСм:

Π­Ρ‚ΠΎΡ‚ ΠΏΡ€ΠΈΠΌΠ΅Ρ€ ΠΌΠΎΠΆΠ½ΠΎ Π»Π΅Π³ΠΊΠΎ ΠΏΠΎΠ½ΡΡ‚ΡŒ, Ссли Π²ΡΠΏΠΎΠΌΠ½ΠΈΡ‚ΡŒ ΠΏΡ€ΠΎ ΠΏΠΈΡ†Ρ†Ρƒ, которая Ρ€Π°Π·Π΄Π΅Π»Π΅Π½Π° Π½Π° Ρ‡Π΅Ρ‚Ρ‹Ρ€Π΅ части. Если ΠΎΡ‚ ΠΏΠΈΡ†Ρ†Ρ‹ ΠΎΡ‚Ρ€Π΅Π·Π°Ρ‚ΡŒ ΠΏΠΈΡ†Ρ†Ρ‹, Ρ‚ΠΎ получится ΠΏΠΈΡ†Ρ†Ρ‹:

ΠŸΡ€ΠΈΠΌΠ΅Ρ€ 2. Найти Π·Π½Π°Ρ‡Π΅Π½ΠΈΠ΅ выраТСния .

ΠžΠΏΡΡ‚ΡŒ ΠΆΠ΅ ΠΈΠ· числитСля ΠΏΠ΅Ρ€Π²ΠΎΠΉ Π΄Ρ€ΠΎΠ±ΠΈ Π²Ρ‹Ρ‡ΠΈΡ‚Π°Π΅ΠΌ Ρ‡ΠΈΡΠ»ΠΈΡ‚Π΅Π»ΡŒ Π²Ρ‚ΠΎΡ€ΠΎΠΉ Π΄Ρ€ΠΎΠ±ΠΈ, Π° Π·Π½Π°ΠΌΠ΅Π½Π°Ρ‚Π΅Π»ΡŒ оставляСм Π±Π΅Π· измСнСния:

Π­Ρ‚ΠΎΡ‚ ΠΏΡ€ΠΈΠΌΠ΅Ρ€ ΠΌΠΎΠΆΠ½ΠΎ Π»Π΅Π³ΠΊΠΎ ΠΏΠΎΠ½ΡΡ‚ΡŒ, Ссли Π²ΡΠΏΠΎΠΌΠ½ΠΈΡ‚ΡŒ ΠΏΡ€ΠΎ ΠΏΠΈΡ†Ρ†Ρƒ, которая Ρ€Π°Π·Π΄Π΅Π»Π΅Π½Π° Π½Π° Ρ‚Ρ€ΠΈ части. Если ΠΎΡ‚ ΠΏΠΈΡ†Ρ†Ρ‹ ΠΎΡ‚Ρ€Π΅Π·Π°Ρ‚ΡŒ ΠΏΠΈΡ†Ρ†Ρ‹, Ρ‚ΠΎ получится ΠΏΠΈΡ†Ρ†Ρ‹:

ΠŸΡ€ΠΈΠΌΠ΅Ρ€ 3. Найти Π·Π½Π°Ρ‡Π΅Π½ΠΈΠ΅ выраТСния

Π­Ρ‚ΠΎΡ‚ ΠΏΡ€ΠΈΠΌΠ΅Ρ€ Ρ€Π΅ΡˆΠ°Π΅Ρ‚ΡΡ Ρ‚ΠΎΡ‡Π½ΠΎ Ρ‚Π°ΠΊΠΆΠ΅, ΠΊΠ°ΠΊ ΠΈ ΠΏΡ€Π΅Π΄Ρ‹Π΄ΡƒΡ‰ΠΈΠ΅. Из числитСля ΠΏΠ΅Ρ€Π²ΠΎΠΉ Π΄Ρ€ΠΎΠ±ΠΈ Π½ΡƒΠΆΠ½ΠΎ Π²Ρ‹Ρ‡Π΅ΡΡ‚ΡŒ числитСли ΠΎΡΡ‚Π°Π»ΡŒΠ½Ρ‹Ρ… Π΄Ρ€ΠΎΠ±Π΅ΠΉ:

Как Π²ΠΈΠ΄ΠΈΡ‚Π΅ Π² Π²Ρ‹Ρ‡ΠΈΡ‚Π°Π½ΠΈΠΈ Π΄Ρ€ΠΎΠ±Π΅ΠΉ с ΠΎΠ΄ΠΈΠ½Π°ΠΊΠΎΠ²Ρ‹ΠΌΠΈ знамСнатСлями Π½ΠΈΡ‡Π΅Π³ΠΎ слоТного Π½Π΅Ρ‚. Достаточно ΠΏΠΎΠ½ΠΈΠΌΠ°Ρ‚ΡŒ ΡΠ»Π΅Π΄ΡƒΡŽΡ‰ΠΈΠ΅ ΠΏΡ€Π°Π²ΠΈΠ»Π°:

  1. Π§Ρ‚ΠΎΠ±Ρ‹ Π²Ρ‹Ρ‡Π΅ΡΡ‚ΡŒ ΠΈΠ· ΠΎΠ΄Π½ΠΎΠΉ Π΄Ρ€ΠΎΠ±ΠΈ Π΄Ρ€ΡƒΠ³ΡƒΡŽ, Π½ΡƒΠΆΠ½ΠΎ ΠΈΠ· числитСля ΠΏΠ΅Ρ€Π²ΠΎΠΉ Π΄Ρ€ΠΎΠ±ΠΈ Π²Ρ‹Ρ‡Π΅ΡΡ‚ΡŒ Ρ‡ΠΈΡΠ»ΠΈΡ‚Π΅Π»ΡŒ Π²Ρ‚ΠΎΡ€ΠΎΠΉ Π΄Ρ€ΠΎΠ±ΠΈ, Π° Π·Π½Π°ΠΌΠ΅Π½Π°Ρ‚Π΅Π»ΡŒ ΠΎΡΡ‚Π°Π²ΠΈΡ‚ΡŒ Π±Π΅Π· измСнСния;
  2. Если Π² ΠΎΡ‚Π²Π΅Ρ‚Π΅ ΠΏΠΎΠ»ΡƒΡ‡ΠΈΠ»Π°ΡΡŒ Π½Π΅ΠΏΡ€Π°Π²ΠΈΠ»ΡŒΠ½Π°Ρ Π΄Ρ€ΠΎΠ±ΡŒ, Ρ‚ΠΎ Π½ΡƒΠΆΠ½ΠΎ Π²Ρ‹Π΄Π΅Π»ΠΈΡ‚ΡŒ Π² Π½Π΅ΠΉ Ρ†Π΅Π»ΡƒΡŽ Ρ‡Π°ΡΡ‚ΡŒ.

Π’Ρ‹Ρ‡ΠΈΡ‚Π°Π½ΠΈΠ΅ Π΄Ρ€ΠΎΠ±Π΅ΠΉ с Ρ€Π°Π·Π½Ρ‹ΠΌΠΈ знамСнатСлями

НапримСр, ΠΎΡ‚ Π΄Ρ€ΠΎΠ±ΠΈ ΠΌΠΎΠΆΠ½ΠΎ Π²Ρ‹Ρ‡Π΅ΡΡ‚ΡŒ Π΄Ρ€ΠΎΠ±ΡŒ , ΠΏΠΎΡΠΊΠΎΠ»ΡŒΠΊΡƒ Ρƒ этих Π΄Ρ€ΠΎΠ±Π΅ΠΉ ΠΎΠ΄ΠΈΠ½Π°ΠΊΠΎΠ²Ρ‹Π΅ Π·Π½Π°ΠΌΠ΅Π½Π°Ρ‚Π΅Π»ΠΈ. А Π²ΠΎΡ‚ ΠΎΡ‚ Π΄Ρ€ΠΎΠ±ΠΈ нСльзя Π²Ρ‹Ρ‡Π΅ΡΡ‚ΡŒ Π΄Ρ€ΠΎΠ±ΡŒ , ΠΏΠΎΡΠΊΠΎΠ»ΡŒΠΊΡƒ Ρƒ этих Π΄Ρ€ΠΎΠ±Π΅ΠΉ Ρ€Π°Π·Π½Ρ‹Π΅ Π·Π½Π°ΠΌΠ΅Π½Π°Ρ‚Π΅Π»ΠΈ. Π’ Ρ‚Π°ΠΊΠΈΡ… случаях Π΄Ρ€ΠΎΠ±ΠΈ Π½ΡƒΠΆΠ½ΠΎ ΠΏΡ€ΠΈΠ²ΠΎΠ΄ΠΈΡ‚ΡŒ ΠΊ ΠΎΠ΄ΠΈΠ½Π°ΠΊΠΎΠ²ΠΎΠΌΡƒ (ΠΎΠ±Ρ‰Π΅ΠΌΡƒ) Π·Π½Π°ΠΌΠ΅Π½Π°Ρ‚Π΅Π»ΡŽ.

ΠžΠ±Ρ‰ΠΈΠΉ Π·Π½Π°ΠΌΠ΅Π½Π°Ρ‚Π΅Π»ΡŒ находят ΠΏΠΎ Ρ‚ΠΎΠΌΡƒ ΠΆΠ΅ ΠΏΡ€ΠΈΠ½Ρ†ΠΈΠΏΡƒ, ΠΊΠΎΡ‚ΠΎΡ€Ρ‹ΠΌ ΠΌΡ‹ пользовались ΠΏΡ€ΠΈ слоТСнии Π΄Ρ€ΠΎΠ±Π΅ΠΉ с Ρ€Π°Π·Π½Ρ‹ΠΌΠΈ знамСнатСлями. Π’ ΠΏΠ΅Ρ€Π²ΡƒΡŽ ΠΎΡ‡Π΅Ρ€Π΅Π΄ΡŒ находят НОК Π·Π½Π°ΠΌΠ΅Π½Π°Ρ‚Π΅Π»Π΅ΠΉ ΠΎΠ±Π΅ΠΈΡ… Π΄Ρ€ΠΎΠ±Π΅ΠΉ. Π—Π°Ρ‚Π΅ΠΌ НОК дСлят Π½Π° Π·Π½Π°ΠΌΠ΅Π½Π°Ρ‚Π΅Π»ΡŒ ΠΏΠ΅Ρ€Π²ΠΎΠΉ Π΄Ρ€ΠΎΠ±ΠΈ ΠΈ ΠΏΠΎΠ»ΡƒΡ‡Π°ΡŽΡ‚ ΠΏΠ΅Ρ€Π²Ρ‹ΠΉ Π΄ΠΎΠΏΠΎΠ»Π½ΠΈΡ‚Π΅Π»ΡŒΠ½Ρ‹ΠΉ ΠΌΠ½ΠΎΠΆΠΈΡ‚Π΅Π»ΡŒ, ΠΊΠΎΡ‚ΠΎΡ€Ρ‹ΠΉ записываСтся Π½Π°Π΄ ΠΏΠ΅Ρ€Π²ΠΎΠΉ Π΄Ρ€ΠΎΠ±ΡŒΡŽ. Аналогично НОК дСлят Π½Π° Π·Π½Π°ΠΌΠ΅Π½Π°Ρ‚Π΅Π»ΡŒ Π²Ρ‚ΠΎΡ€ΠΎΠΉ Π΄Ρ€ΠΎΠ±ΠΈ ΠΈ ΠΏΠΎΠ»ΡƒΡ‡Π°ΡŽΡ‚ Π²Ρ‚ΠΎΡ€ΠΎΠΉ Π΄ΠΎΠΏΠΎΠ»Π½ΠΈΡ‚Π΅Π»ΡŒΠ½Ρ‹ΠΉ ΠΌΠ½ΠΎΠΆΠΈΡ‚Π΅Π»ΡŒ, ΠΊΠΎΡ‚ΠΎΡ€Ρ‹ΠΉ записываСтся Π½Π°Π΄ Π²Ρ‚ΠΎΡ€ΠΎΠΉ Π΄Ρ€ΠΎΠ±ΡŒΡŽ.

Π—Π°Ρ‚Π΅ΠΌ Π΄Ρ€ΠΎΠ±ΠΈ ΡƒΠΌΠ½ΠΎΠΆΠ°ΡŽΡ‚ΡΡ Π½Π° свои Π΄ΠΎΠΏΠΎΠ»Π½ΠΈΡ‚Π΅Π»ΡŒΠ½Ρ‹Π΅ ΠΌΠ½ΠΎΠΆΠΈΡ‚Π΅Π»ΠΈ. Π’ Ρ€Π΅Π·ΡƒΠ»ΡŒΡ‚Π°Ρ‚Π΅ этих ΠΎΠΏΠ΅Ρ€Π°Ρ†ΠΈΠΉ, Π΄Ρ€ΠΎΠ±ΠΈ Ρƒ ΠΊΠΎΡ‚ΠΎΡ€Ρ‹Ρ… Π±Ρ‹Π»ΠΈ Ρ€Π°Π·Π½Ρ‹Π΅ Π·Π½Π°ΠΌΠ΅Π½Π°Ρ‚Π΅Π»ΠΈ, ΠΎΠ±Ρ€Π°Ρ‰Π°ΡŽΡ‚ΡΡ Π² Π΄Ρ€ΠΎΠ±ΠΈ, Ρƒ ΠΊΠΎΡ‚ΠΎΡ€Ρ‹Ρ… ΠΎΠ΄ΠΈΠ½Π°ΠΊΠΎΠ²Ρ‹Π΅ Π·Π½Π°ΠΌΠ΅Π½Π°Ρ‚Π΅Π»ΠΈ. А ΠΊΠ°ΠΊ Π²Ρ‹Ρ‡ΠΈΡ‚Π°Ρ‚ΡŒ Ρ‚Π°ΠΊΠΈΠ΅ Π΄Ρ€ΠΎΠ±ΠΈ ΠΌΡ‹ ΡƒΠΆΠ΅ Π·Π½Π°Π΅ΠΌ.

ΠŸΡ€ΠΈΠΌΠ΅Ρ€ 1. Найти Π·Π½Π°Ρ‡Π΅Π½ΠΈΠ΅ выраТСния:

Π£ этих Π΄Ρ€ΠΎΠ±Π΅ΠΉ Ρ€Π°Π·Π½Ρ‹Π΅ Π·Π½Π°ΠΌΠ΅Π½Π°Ρ‚Π΅Π»ΠΈ, поэтому Π½ΡƒΠΆΠ½ΠΎ привСсти ΠΈΡ… ΠΊ ΠΎΠ΄ΠΈΠ½Π°ΠΊΠΎΠ²ΠΎΠΌΡƒ (ΠΎΠ±Ρ‰Π΅ΠΌΡƒ) Π·Π½Π°ΠΌΠ΅Π½Π°Ρ‚Π΅Π»ΡŽ.

Π‘Π½Π°Ρ‡Π°Π»Π° Π½Π°Ρ…ΠΎΠ΄ΠΈΠΌ НОК Π·Π½Π°ΠΌΠ΅Π½Π°Ρ‚Π΅Π»Π΅ΠΉ ΠΎΠ±Π΅ΠΈΡ… Π΄Ρ€ΠΎΠ±Π΅ΠΉ. Π—Π½Π°ΠΌΠ΅Π½Π°Ρ‚Π΅Π»ΡŒ ΠΏΠ΅Ρ€Π²ΠΎΠΉ Π΄Ρ€ΠΎΠ±ΠΈ это число 3, Π° Π·Π½Π°ΠΌΠ΅Π½Π°Ρ‚Π΅Π»ΡŒ Π²Ρ‚ΠΎΡ€ΠΎΠΉ Π΄Ρ€ΠΎΠ±ΠΈ β€” число 4. НаимСньшСС ΠΎΠ±Ρ‰Π΅Π΅ ΠΊΡ€Π°Ρ‚Π½ΠΎΠ΅ этих чисСл Ρ€Π°Π²Π½ΠΎ 12

НОК (3 и 4) = 12

Π’Π΅ΠΏΠ΅Ρ€ΡŒ возвращаСмся ΠΊ дробям ΠΈ

Найдём Π΄ΠΎΠΏΠΎΠ»Π½ΠΈΡ‚Π΅Π»ΡŒΠ½Ρ‹ΠΉ ΠΌΠ½ΠΎΠΆΠΈΡ‚Π΅Π»ΡŒ для ΠΏΠ΅Ρ€Π²ΠΎΠΉ Π΄Ρ€ΠΎΠ±ΠΈ. Для этого Ρ€Π°Π·Π΄Π΅Π»ΠΈΠΌ НОК Π½Π° Π·Π½Π°ΠΌΠ΅Π½Π°Ρ‚Π΅Π»ΡŒ ΠΏΠ΅Ρ€Π²ΠΎΠΉ Π΄Ρ€ΠΎΠ±ΠΈ. НОК это число 12, Π° Π·Π½Π°ΠΌΠ΅Π½Π°Ρ‚Π΅Π»ΡŒ ΠΏΠ΅Ρ€Π²ΠΎΠΉ Π΄Ρ€ΠΎΠ±ΠΈ β€” число 3. Π”Π΅Π»ΠΈΠΌ 12 Π½Π° 3, ΠΏΠΎΠ»ΡƒΡ‡Π°Π΅ΠΌ 4. ЗаписываСм Ρ‡Π΅Ρ‚Π²Ρ‘Ρ€ΠΊΡƒ Π½Π°Π΄ ΠΏΠ΅Ρ€Π²ΠΎΠΉ Π΄Ρ€ΠΎΠ±ΡŒΡŽ:

Аналогично поступаСм ΠΈ со Π²Ρ‚ΠΎΡ€ΠΎΠΉ Π΄Ρ€ΠΎΠ±ΡŒΡŽ. Π”Π΅Π»ΠΈΠΌ НОК Π½Π° Π·Π½Π°ΠΌΠ΅Π½Π°Ρ‚Π΅Π»ΡŒ Π²Ρ‚ΠΎΡ€ΠΎΠΉ Π΄Ρ€ΠΎΠ±ΠΈ. НОК это число 12, Π° Π·Π½Π°ΠΌΠ΅Π½Π°Ρ‚Π΅Π»ΡŒ Π²Ρ‚ΠΎΡ€ΠΎΠΉ Π΄Ρ€ΠΎΠ±ΠΈ β€” число 4. Π”Π΅Π»ΠΈΠΌ 12 Π½Π° 4, ΠΏΠΎΠ»ΡƒΡ‡Π°Π΅ΠΌ 3. ЗаписываСм Ρ‚Ρ€ΠΎΠΉΠΊΡƒ Π½Π°Π΄ Π²Ρ‚ΠΎΡ€ΠΎΠΉ Π΄Ρ€ΠΎΠ±ΡŒΡŽ:

Π’Π΅ΠΏΠ΅Ρ€ΡŒ Ρƒ нас всё Π³ΠΎΡ‚ΠΎΠ²ΠΎ для вычитания. ΠžΡΡ‚Π°Π»ΠΎΡΡŒ ΡƒΠΌΠ½ΠΎΠΆΠΈΡ‚ΡŒ Π΄Ρ€ΠΎΠ±ΠΈ Π½Π° свои Π΄ΠΎΠΏΠΎΠ»Π½ΠΈΡ‚Π΅Π»ΡŒΠ½Ρ‹Π΅ ΠΌΠ½ΠΎΠΆΠΈΡ‚Π΅Π»ΠΈ:

ΠœΡ‹ ΠΏΡ€ΠΈΡˆΠ»ΠΈ ΠΊ Ρ‚ΠΎΠΌΡƒ, Ρ‡Ρ‚ΠΎ Π΄Ρ€ΠΎΠ±ΠΈ Ρƒ ΠΊΠΎΡ‚ΠΎΡ€Ρ‹Ρ… Π±Ρ‹Π»ΠΈ Ρ€Π°Π·Π½Ρ‹Π΅ Π·Π½Π°ΠΌΠ΅Π½Π°Ρ‚Π΅Π»ΠΈ, ΠΏΡ€Π΅Π²Ρ€Π°Ρ‚ΠΈΠ»ΠΈΡΡŒ Π² Π΄Ρ€ΠΎΠ±ΠΈ Ρƒ ΠΊΠΎΡ‚ΠΎΡ€Ρ‹Ρ… ΠΎΠ΄ΠΈΠ½Π°ΠΊΠΎΠ²Ρ‹Π΅ Π·Π½Π°ΠΌΠ΅Π½Π°Ρ‚Π΅Π»ΠΈ. А ΠΊΠ°ΠΊ Π²Ρ‹Ρ‡ΠΈΡ‚Π°Ρ‚ΡŒ Ρ‚Π°ΠΊΠΈΠ΅ Π΄Ρ€ΠΎΠ±ΠΈ ΠΌΡ‹ ΡƒΠΆΠ΅ Π·Π½Π°Π΅ΠΌ. Π”Π°Π²Π°ΠΉΡ‚Π΅ Π΄ΠΎΡ€Π΅ΡˆΠ°Π΅ΠΌ этот ΠΏΡ€ΠΈΠΌΠ΅Ρ€ Π΄ΠΎ ΠΊΠΎΠ½Ρ†Π°:

ΠŸΠΎΠ»ΡƒΡ‡ΠΈΠ»ΠΈ ΠΎΡ‚Π²Π΅Ρ‚

ΠŸΠΎΠΏΡ€ΠΎΠ±ΡƒΠ΅ΠΌ ΠΈΠ·ΠΎΠ±Ρ€Π°Π·ΠΈΡ‚ΡŒ нашС Ρ€Π΅ΡˆΠ΅Π½ΠΈΠ΅ с ΠΏΠΎΠΌΠΎΡ‰ΡŒΡŽ рисунка. Если ΠΎΡ‚ ΠΏΠΈΡ†Ρ†Ρ‹ ΠΎΡ‚Ρ€Π΅Π·Π°Ρ‚ΡŒ ΠΏΠΈΡ†Ρ†Ρ‹, Ρ‚ΠΎ получится ΠΏΠΈΡ†Ρ†Ρ‹

Π­Ρ‚ΠΎ подробная вСрсия Ρ€Π΅ΡˆΠ΅Π½ΠΈΡ. ΠΠ°Ρ…ΠΎΠ΄ΡΡΡŒ Π² школС, Π½Π°ΠΌ ΠΏΡ€ΠΈΡˆΠ»ΠΎΡΡŒ Π±Ρ‹ Ρ€Π΅ΡˆΠΈΡ‚ΡŒ этот ΠΏΡ€ΠΈΠΌΠ΅Ρ€ ΠΏΠΎΠΊΠΎΡ€ΠΎΡ‡Π΅. ВыглядСло Π±Ρ‹ Ρ‚Π°ΠΊΠΎΠ΅ Ρ€Π΅ΡˆΠ΅Π½ΠΈΠ΅ ΡΠ»Π΅Π΄ΡƒΡŽΡ‰ΠΈΠΌ ΠΎΠ±Ρ€Π°Π·ΠΎΠΌ:

ΠŸΡ€ΠΈΠ²Π΅Π΄Π΅Π½ΠΈΠ΅ Π΄Ρ€ΠΎΠ±Π΅ΠΉ ΠΈ ΠΊ ΠΎΠ±Ρ‰Π΅ΠΌΡƒ Π·Π½Π°ΠΌΠ΅Π½Π°Ρ‚Π΅Π»ΡŽ Ρ‚Π°ΠΊΠΆΠ΅ ΠΌΠΎΠΆΠ΅Ρ‚ Π±Ρ‹Ρ‚ΡŒ ΠΈΠ·ΠΎΠ±Ρ€Π°ΠΆΠ΅Π½ΠΎ с ΠΏΠΎΠΌΠΎΡ‰ΡŒΡŽ рисунка. ΠŸΡ€ΠΈΠ²Π΅Π΄Ρ эти Π΄Ρ€ΠΎΠ±ΠΈ ΠΊ ΠΎΠ±Ρ‰Π΅ΠΌΡƒ Π·Π½Π°ΠΌΠ΅Π½Π°Ρ‚Π΅Π»ΡŽ, ΠΌΡ‹ ΠΏΠΎΠ»ΡƒΡ‡ΠΈΠ»ΠΈ Π΄Ρ€ΠΎΠ±ΠΈ ΠΈ . Π­Ρ‚ΠΈ Π΄Ρ€ΠΎΠ±ΠΈ Π±ΡƒΠ΄ΡƒΡ‚ ΠΈΠ·ΠΎΠ±Ρ€Π°ΠΆΠ°Ρ‚ΡŒΡΡ Ρ‚Π΅ΠΌΠΈ ΠΆΠ΅ кусочками ΠΏΠΈΡ†Ρ†, Π½ΠΎ Π² этот Ρ€Π°Π· ΠΎΠ½ΠΈ Π±ΡƒΠ΄ΡƒΡ‚ Ρ€Π°Π·Π΄Π΅Π»Π΅Π½Ρ‹ Π½Π° ΠΎΠ΄ΠΈΠ½Π°ΠΊΠΎΠ²Ρ‹Π΅ Π΄ΠΎΠ»ΠΈ (ΠΏΡ€ΠΈΠ²Π΅Π΄Π΅Π½Ρ‹ ΠΊ ΠΎΠ΄ΠΈΠ½Π°ΠΊΠΎΠ²ΠΎΠΌΡƒ Π·Π½Π°ΠΌΠ΅Π½Π°Ρ‚Π΅Π»ΡŽ):

ΠŸΠ΅Ρ€Π²Ρ‹ΠΉ рисунок ΠΈΠ·ΠΎΠ±Ρ€Π°ΠΆΠ°Π΅Ρ‚ Π΄Ρ€ΠΎΠ±ΡŒ (восСмь кусочков ΠΈΠ· Π΄Π²Π΅Π½Π°Π΄Ρ†Π°Ρ‚ΠΈ), Π° Π²Ρ‚ΠΎΡ€ΠΎΠΉ рисунок β€” Π΄Ρ€ΠΎΠ±ΡŒ (Ρ‚Ρ€ΠΈ кусочка ΠΈΠ· Π΄Π²Π΅Π½Π°Π΄Ρ†Π°Ρ‚ΠΈ). ΠžΡ‚Ρ€Π΅Π·Π°Π² ΠΎΡ‚ восьми кусочков Ρ‚Ρ€ΠΈ кусочка ΠΌΡ‹ ΠΏΠΎΠ»ΡƒΡ‡Π°Π΅ΠΌ ΠΏΡΡ‚ΡŒ кусочков ΠΈΠ· Π΄Π²Π΅Π½Π°Π΄Ρ†Π°Ρ‚ΠΈ. Π”Ρ€ΠΎΠ±ΡŒ ΠΈ описываСт эти ΠΏΡΡ‚ΡŒ кусочков.

ΠŸΡ€ΠΈΠΌΠ΅Ρ€ 2. Найти Π·Π½Π°Ρ‡Π΅Π½ΠΈΠ΅ выраТСния

Π£ этих Π΄Ρ€ΠΎΠ±Π΅ΠΉ Ρ€Π°Π·Π½Ρ‹Π΅ Π·Π½Π°ΠΌΠ΅Π½Π°Ρ‚Π΅Π»ΠΈ, поэтому сначала Π½ΡƒΠΆΠ½ΠΎ привСсти ΠΈΡ… ΠΊ ΠΎΠ΄ΠΈΠ½Π°ΠΊΠΎΠ²ΠΎΠΌΡƒ (ΠΎΠ±Ρ‰Π΅ΠΌΡƒ) Π·Π½Π°ΠΌΠ΅Π½Π°Ρ‚Π΅Π»ΡŽ.

Найдём НОК Π·Π½Π°ΠΌΠ΅Π½Π°Ρ‚Π΅Π»Π΅ΠΉ этих Π΄Ρ€ΠΎΠ±Π΅ΠΉ.

Π—Π½Π°ΠΌΠ΅Π½Π°Ρ‚Π΅Π»ΠΈ Π΄Ρ€ΠΎΠ±Π΅ΠΉ это числа 10, 3 ΠΈ 5. НаимСньшСС ΠΎΠ±Ρ‰Π΅Π΅ ΠΊΡ€Π°Ρ‚Π½ΠΎΠ΅ этих чисСл Ρ€Π°Π²Π½ΠΎ 30

НОК (10, 3, 5) = 30

Π’Π΅ΠΏΠ΅Ρ€ΡŒ Π½Π°Ρ…ΠΎΠ΄ΠΈΠΌ Π΄ΠΎΠΏΠΎΠ»Π½ΠΈΡ‚Π΅Π»ΡŒΠ½Ρ‹Π΅ ΠΌΠ½ΠΎΠΆΠΈΡ‚Π΅Π»ΠΈ для ΠΊΠ°ΠΆΠ΄ΠΎΠΉ Π΄Ρ€ΠΎΠ±ΠΈ. Для этого Ρ€Π°Π·Π΄Π΅Π»ΠΈΠΌ НОК Π½Π° Π·Π½Π°ΠΌΠ΅Π½Π°Ρ‚Π΅Π»ΡŒ ΠΊΠ°ΠΆΠ΄ΠΎΠΉ Π΄Ρ€ΠΎΠ±ΠΈ.

Найдём Π΄ΠΎΠΏΠΎΠ»Π½ΠΈΡ‚Π΅Π»ΡŒΠ½Ρ‹ΠΉ ΠΌΠ½ΠΎΠΆΠΈΡ‚Π΅Π»ΡŒ для ΠΏΠ΅Ρ€Π²ΠΎΠΉ Π΄Ρ€ΠΎΠ±ΠΈ. НОК это число 30, Π° Π·Π½Π°ΠΌΠ΅Π½Π°Ρ‚Π΅Π»ΡŒ ΠΏΠ΅Ρ€Π²ΠΎΠΉ Π΄Ρ€ΠΎΠ±ΠΈ β€” число 10. Π”Π΅Π»ΠΈΠΌ 30 Π½Π° 10, ΠΏΠΎΠ»ΡƒΡ‡Π°Π΅ΠΌ ΠΏΠ΅Ρ€Π²Ρ‹ΠΉ Π΄ΠΎΠΏΠΎΠ»Π½ΠΈΡ‚Π΅Π»ΡŒΠ½Ρ‹ΠΉ ΠΌΠ½ΠΎΠΆΠΈΡ‚Π΅Π»ΡŒ 3. ЗаписываСм Π΅Π³ΠΎ Π½Π°Π΄ ΠΏΠ΅Ρ€Π²ΠΎΠΉ Π΄Ρ€ΠΎΠ±ΡŒΡŽ:

Π’Π΅ΠΏΠ΅Ρ€ΡŒ Π½Π°Ρ…ΠΎΠ΄ΠΈΠΌ Π΄ΠΎΠΏΠΎΠ»Π½ΠΈΡ‚Π΅Π»ΡŒΠ½Ρ‹ΠΉ ΠΌΠ½ΠΎΠΆΠΈΡ‚Π΅Π»ΡŒ для Π²Ρ‚ΠΎΡ€ΠΎΠΉ Π΄Ρ€ΠΎΠ±ΠΈ. Π Π°Π·Π΄Π΅Π»ΠΈΠΌ НОК Π½Π° Π·Π½Π°ΠΌΠ΅Π½Π°Ρ‚Π΅Π»ΡŒ Π²Ρ‚ΠΎΡ€ΠΎΠΉ Π΄Ρ€ΠΎΠ±ΠΈ. НОК это число 30, Π° Π·Π½Π°ΠΌΠ΅Π½Π°Ρ‚Π΅Π»ΡŒ Π²Ρ‚ΠΎΡ€ΠΎΠΉ Π΄Ρ€ΠΎΠ±ΠΈ β€” число 3. Π”Π΅Π»ΠΈΠΌ 30 Π½Π° 3, ΠΏΠΎΠ»ΡƒΡ‡Π°Π΅ΠΌ Π²Ρ‚ΠΎΡ€ΠΎΠΉ Π΄ΠΎΠΏΠΎΠ»Π½ΠΈΡ‚Π΅Π»ΡŒΠ½Ρ‹ΠΉ ΠΌΠ½ΠΎΠΆΠΈΡ‚Π΅Π»ΡŒ 10. ЗаписываСм Π΅Π³ΠΎ Π½Π°Π΄ Π²Ρ‚ΠΎΡ€ΠΎΠΉ Π΄Ρ€ΠΎΠ±ΡŒΡŽ:

Π’Π΅ΠΏΠ΅Ρ€ΡŒ Π½Π°Ρ…ΠΎΠ΄ΠΈΠΌ Π΄ΠΎΠΏΠΎΠ»Π½ΠΈΡ‚Π΅Π»ΡŒΠ½Ρ‹ΠΉ ΠΌΠ½ΠΎΠΆΠΈΡ‚Π΅Π»ΡŒ для Ρ‚Ρ€Π΅Ρ‚ΡŒΠ΅ΠΉ Π΄Ρ€ΠΎΠ±ΠΈ. Π Π°Π·Π΄Π΅Π»ΠΈΠΌ НОК Π½Π° Π·Π½Π°ΠΌΠ΅Π½Π°Ρ‚Π΅Π»ΡŒ Ρ‚Ρ€Π΅Ρ‚ΡŒΠ΅ΠΉ Π΄Ρ€ΠΎΠ±ΠΈ. НОК это число 30, Π° Π·Π½Π°ΠΌΠ΅Π½Π°Ρ‚Π΅Π»ΡŒ Ρ‚Ρ€Π΅Ρ‚ΡŒΠ΅ΠΉ Π΄Ρ€ΠΎΠ±ΠΈ β€” число 5. Π”Π΅Π»ΠΈΠΌ 30 Π½Π° 5, ΠΏΠΎΠ»ΡƒΡ‡Π°Π΅ΠΌ Ρ‚Ρ€Π΅Ρ‚ΠΈΠΉ Π΄ΠΎΠΏΠΎΠ»Π½ΠΈΡ‚Π΅Π»ΡŒΠ½Ρ‹ΠΉ ΠΌΠ½ΠΎΠΆΠΈΡ‚Π΅Π»ΡŒ 6. ЗаписываСм Π΅Π³ΠΎ Π½Π°Π΄ Ρ‚Ρ€Π΅Ρ‚ΡŒΠ΅ΠΉ Π΄Ρ€ΠΎΠ±ΡŒΡŽ:

Π’Π΅ΠΏΠ΅Ρ€ΡŒ всё Π³ΠΎΡ‚ΠΎΠ²ΠΎ для вычитания. ΠžΡΡ‚Π°Π»ΠΎΡΡŒ ΡƒΠΌΠ½ΠΎΠΆΠΈΡ‚ΡŒ Π΄Ρ€ΠΎΠ±ΠΈ Π½Π° свои Π΄ΠΎΠΏΠΎΠ»Π½ΠΈΡ‚Π΅Π»ΡŒΠ½Ρ‹Π΅ ΠΌΠ½ΠΎΠΆΠΈΡ‚Π΅Π»ΠΈ:

ΠœΡ‹ ΠΏΡ€ΠΈΡˆΠ»ΠΈ ΠΊ Ρ‚ΠΎΠΌΡƒ, Ρ‡Ρ‚ΠΎ Π΄Ρ€ΠΎΠ±ΠΈ Ρƒ ΠΊΠΎΡ‚ΠΎΡ€Ρ‹Ρ… Π±Ρ‹Π»ΠΈ Ρ€Π°Π·Π½Ρ‹Π΅ Π·Π½Π°ΠΌΠ΅Π½Π°Ρ‚Π΅Π»ΠΈ, ΠΏΡ€Π΅Π²Ρ€Π°Ρ‚ΠΈΠ»ΠΈΡΡŒ Π² Π΄Ρ€ΠΎΠ±ΠΈ Ρƒ ΠΊΠΎΡ‚ΠΎΡ€Ρ‹Ρ… ΠΎΠ΄ΠΈΠ½Π°ΠΊΠΎΠ²Ρ‹Π΅ (ΠΎΠ±Ρ‰ΠΈΠ΅) Π·Π½Π°ΠΌΠ΅Π½Π°Ρ‚Π΅Π»ΠΈ. А ΠΊΠ°ΠΊ Π²Ρ‹Ρ‡ΠΈΡ‚Π°Ρ‚ΡŒ Ρ‚Π°ΠΊΠΈΠ΅ Π΄Ρ€ΠΎΠ±ΠΈ ΠΌΡ‹ ΡƒΠΆΠ΅ Π·Π½Π°Π΅ΠΌ. Π”Π°Π²Π°ΠΉΡ‚Π΅ Π΄ΠΎΡ€Π΅ΡˆΠ°Π΅ΠΌ этот ΠΏΡ€ΠΈΠΌΠ΅Ρ€.

ΠŸΡ€ΠΎΠ΄ΠΎΠ»ΠΆΠ΅Π½ΠΈΠ΅ ΠΏΡ€ΠΈΠΌΠ΅Ρ€Π° Π½Π΅ помСстится Π½Π° ΠΎΠ΄Π½ΠΎΠΉ строкС, поэтому пСрСносим ΠΏΡ€ΠΎΠ΄ΠΎΠ»ΠΆΠ΅Π½ΠΈΠ΅ Π½Π° ΡΠ»Π΅Π΄ΡƒΡŽΡ‰ΡƒΡŽ строку. НС Π·Π°Π±Ρ‹Π²Π°Π΅ΠΌ ΠΏΡ€ΠΎ Π·Π½Π°ΠΊ равСнства (=) Π½Π° Π½ΠΎΠ²ΠΎΠΉ строкС:

Π’ ΠΎΡ‚Π²Π΅Ρ‚Π΅ ΠΏΠΎΠ»ΡƒΡ‡ΠΈΠ»Π°ΡΡŒ ΠΏΡ€Π°Π²ΠΈΠ»ΡŒΠ½Π°Ρ Π΄Ρ€ΠΎΠ±ΡŒ, ΠΈ Π²Ρ€ΠΎΠ΄Π΅ Π±Ρ‹ нас всё устраиваСт, Π½ΠΎ ΠΎΠ½Π° слишком Π³Ρ€ΠΎΠΌΠΎΠ·Π΄ΠΊΠ° ΠΈ нСкрасива. Надо Π±Ρ‹ ΡΠ΄Π΅Π»Π°Ρ‚ΡŒ Π΅Ρ‘ ΠΏΡ€ΠΎΡ‰Π΅. А Ρ‡Ρ‚ΠΎ ΠΌΠΎΠΆΠ½ΠΎ ΡΠ΄Π΅Π»Π°Ρ‚ΡŒ? МоТно ΡΠΎΠΊΡ€Π°Ρ‚ΠΈΡ‚ΡŒ эту Π΄Ρ€ΠΎΠ±ΡŒ.

Π§Ρ‚ΠΎΠ±Ρ‹ ΡΠΎΠΊΡ€Π°Ρ‚ΠΈΡ‚ΡŒ Π΄Ρ€ΠΎΠ±ΡŒ , Π½ΡƒΠΆΠ½ΠΎ Ρ€Π°Π·Π΄Π΅Π»ΠΈΡ‚ΡŒ Π΅Ρ‘ Ρ‡ΠΈΡΠ»ΠΈΡ‚Π΅Π»ΡŒ ΠΈ Π·Π½Π°ΠΌΠ΅Π½Π°Ρ‚Π΅Π»ΡŒ Π½Π° (ΠΠžΠ”) чисСл 20 ΠΈ 30.

Π˜Ρ‚Π°ΠΊ, Π½Π°Ρ…ΠΎΠ΄ΠΈΠΌ ΠΠžΠ” чисСл 20 ΠΈ 30:

Π’Π΅ΠΏΠ΅Ρ€ΡŒ возвращаСмся ΠΊ Π½Π°ΡˆΠ΅ΠΌΡƒ ΠΏΡ€ΠΈΠΌΠ΅Ρ€Ρƒ ΠΈ Π΄Π΅Π»ΠΈΠΌ Ρ‡ΠΈΡΠ»ΠΈΡ‚Π΅Π»ΡŒ ΠΈ Π·Π½Π°ΠΌΠ΅Π½Π°Ρ‚Π΅Π»ΡŒ Π΄Ρ€ΠΎΠ±ΠΈ Π½Π° Π½Π°ΠΉΠ΄Π΅Π½Π½Ρ‹ΠΉ ΠΠžΠ”, Ρ‚ΠΎ Π΅ΡΡ‚ΡŒ Π½Π° 10

ΠŸΠΎΠ»ΡƒΡ‡ΠΈΠ»ΠΈ ΠΎΡ‚Π²Π΅Ρ‚

Π£ΠΌΠ½ΠΎΠΆΠ΅Π½ΠΈΠ΅ Π΄Ρ€ΠΎΠ±ΠΈ Π½Π° число

Π§Ρ‚ΠΎΠ±Ρ‹ ΡƒΠΌΠ½ΠΎΠΆΠΈΡ‚ΡŒ Π΄Ρ€ΠΎΠ±ΡŒ Π½Π° число, Π½ΡƒΠΆΠ½ΠΎ Ρ‡ΠΈΡΠ»ΠΈΡ‚Π΅Π»ΡŒ Π΄Π°Π½Π½ΠΎΠΉ Π΄Ρ€ΠΎΠ±ΠΈ ΡƒΠΌΠ½ΠΎΠΆΠΈΡ‚ΡŒ Π½Π° это число, Π° Π·Π½Π°ΠΌΠ΅Π½Π°Ρ‚Π΅Π»ΡŒ ΠΎΡΡ‚Π°Π²ΠΈΡ‚ΡŒ Π±Π΅Π· ΠΈΠ·ΠΌΠ΅Π½Π΅Π½ΠΈΠΉ.

ΠŸΡ€ΠΈΠΌΠ΅Ρ€ 1 . Π£ΠΌΠ½ΠΎΠΆΠΈΡ‚ΡŒ Π΄Ρ€ΠΎΠ±ΡŒ Π½Π° число 1 .

Π£ΠΌΠ½ΠΎΠΆΠΈΠΌ Ρ‡ΠΈΡΠ»ΠΈΡ‚Π΅Π»ΡŒ Π΄Ρ€ΠΎΠ±ΠΈ Π½Π° число 1

Π—Π°ΠΏΠΈΡΡŒ ΠΌΠΎΠΆΠ½ΠΎ ΠΏΠΎΠ½ΠΈΠΌΠ°Ρ‚ΡŒ, ΠΊΠ°ΠΊ Π²Π·ΡΡ‚ΡŒ ΠΏΠΎΠ»ΠΎΠ²ΠΈΠ½Ρƒ 1 Ρ€Π°Π·. К ΠΏΡ€ΠΈΠΌΠ΅Ρ€Ρƒ, Ссли ΠΏΠΈΡ†Ρ†Ρ‹ Π²Π·ΡΡ‚ΡŒ 1 Ρ€Π°Π·, Ρ‚ΠΎ получится ΠΏΠΈΡ†Ρ†Ρ‹

Из Π·Π°ΠΊΠΎΠ½ΠΎΠ² умноТСния ΠΌΡ‹ Π·Π½Π°Π΅ΠΌ, Ρ‡Ρ‚ΠΎ Ссли ΠΌΠ½ΠΎΠΆΠΈΠΌΠΎΠ΅ ΠΈ ΠΌΠ½ΠΎΠΆΠΈΡ‚Π΅Π»ΡŒ ΠΏΠΎΠΌΠ΅Π½ΡΡ‚ΡŒ мСстами, Ρ‚ΠΎ ΠΏΡ€ΠΎΠΈΠ·Π²Π΅Π΄Π΅Π½ΠΈΠ΅ Π½Π΅ измСнится. Если Π²Ρ‹Ρ€Π°ΠΆΠ΅Π½ΠΈΠ΅ , Π·Π°ΠΏΠΈΡΠ°Ρ‚ΡŒ ΠΊΠ°ΠΊ , Ρ‚ΠΎ ΠΏΡ€ΠΎΠΈΠ·Π²Π΅Π΄Π΅Π½ΠΈΠ΅ ΠΏΠΎ ΠΏΡ€Π΅ΠΆΠ½Π΅ΠΌΡƒ Π±ΡƒΠ΄Π΅Ρ‚ Ρ€Π°Π²Π½ΠΎ . ΠžΠΏΡΡ‚ΡŒ ΠΆΠ΅ срабатываСт ΠΏΡ€Π°Π²ΠΈΠ»ΠΎ пСрСмноТСния Ρ†Π΅Π»ΠΎΠ³ΠΎ числа ΠΈ Π΄Ρ€ΠΎΠ±ΠΈ:

Π­Ρ‚Ρƒ запись ΠΌΠΎΠΆΠ½ΠΎ ΠΏΠΎΠ½ΠΈΠΌΠ°Ρ‚ΡŒ, ΠΊΠ°ΠΊ взятиС ΠΏΠΎΠ»ΠΎΠ²ΠΈΠ½Ρ‹ ΠΎΡ‚ Π΅Π΄ΠΈΠ½ΠΈΡ†Ρ‹. К ΠΏΡ€ΠΈΠΌΠ΅Ρ€Ρƒ, Ссли имССтся 1 цСлая ΠΏΠΈΡ†Ρ†Π° ΠΈ ΠΌΡ‹ возьмСм ΠΎΡ‚ Π½Π΅Ρ‘ ΠΏΠΎΠ»ΠΎΠ²ΠΈΠ½Ρƒ, Ρ‚ΠΎ Ρƒ нас окаТСтся ΠΏΠΈΡ†Ρ†Ρ‹:

ΠŸΡ€ΠΈΠΌΠ΅Ρ€ 2 . Найти Π·Π½Π°Ρ‡Π΅Π½ΠΈΠ΅ выраТСния

Π£ΠΌΠ½ΠΎΠΆΠΈΠΌ Ρ‡ΠΈΡΠ»ΠΈΡ‚Π΅Π»ΡŒ Π΄Ρ€ΠΎΠ±ΠΈ Π½Π° 4

Π’ ΠΎΡ‚Π²Π΅Ρ‚Π΅ ΠΏΠΎΠ»ΡƒΡ‡ΠΈΠ»Π°ΡΡŒ Π½Π΅ΠΏΡ€Π°Π²ΠΈΠ»ΡŒΠ½Π°Ρ Π΄Ρ€ΠΎΠ±ΡŒ. Π’Ρ‹Π΄Π΅Π»ΠΈΠΌ Π² Π½Π΅ΠΉ Ρ†Π΅Π»ΡƒΡŽ Ρ‡Π°ΡΡ‚ΡŒ:

Π’Ρ‹Ρ€Π°ΠΆΠ΅Π½ΠΈΠ΅ ΠΌΠΎΠΆΠ½ΠΎ ΠΏΠΎΠ½ΠΈΠΌΠ°Ρ‚ΡŒ, ΠΊΠ°ΠΊ взятиС Π΄Π²ΡƒΡ… Ρ‡Π΅Ρ‚Π²Π΅Ρ€Ρ‚Π΅ΠΉ 4 Ρ€Π°Π·Π°. К ΠΏΡ€ΠΈΠΌΠ΅Ρ€Ρƒ, Ссли ΠΏΠΈΡ†Ρ†Ρ‹ Π²Π·ΡΡ‚ΡŒ 4 Ρ€Π°Π·Π°, Ρ‚ΠΎ получится Π΄Π²Π΅ Ρ†Π΅Π»Ρ‹Π΅ ΠΏΠΈΡ†Ρ†Ρ‹

А Ссли ΠΏΠΎΠΌΠ΅Π½ΡΡ‚ΡŒ ΠΌΠ½ΠΎΠΆΠΈΠΌΠΎΠ΅ ΠΈ ΠΌΠ½ΠΎΠΆΠΈΡ‚Π΅Π»ΡŒ мСстами, Ρ‚ΠΎ ΠΏΠΎΠ»ΡƒΡ‡ΠΈΠΌ Π²Ρ‹Ρ€Π°ΠΆΠ΅Π½ΠΈΠ΅ . Оно Ρ‚ΠΎΠΆΠ΅ Π±ΡƒΠ΄Π΅Ρ‚ Ρ€Π°Π²Π½ΠΎ 2. Π­Ρ‚ΠΎ Π²Ρ‹Ρ€Π°ΠΆΠ΅Π½ΠΈΠ΅ ΠΌΠΎΠΆΠ½ΠΎ ΠΏΠΎΠ½ΠΈΠΌΠ°Ρ‚ΡŒ, ΠΊΠ°ΠΊ взятиС Π΄Π²ΡƒΡ… ΠΏΠΈΡ†Ρ† ΠΎΡ‚ Ρ‡Π΅Ρ‚Ρ‹Ρ€Π΅Ρ… Ρ†Π΅Π»Ρ‹Ρ… ΠΏΠΈΡ†Ρ†:

Число, ΠΊΠΎΡ‚ΠΎΡ€ΠΎΠ΅ умноТаСтся Π½Π° Π΄Ρ€ΠΎΠ±ΡŒ, ΠΈ Π·Π½Π°ΠΌΠ΅Π½Π°Ρ‚Π΅Π»ΡŒ Π΄Ρ€ΠΎΠ±ΠΈ Ρ€Π°Π·Ρ€Π΅ΡˆΠ°Π΅Ρ‚ΡΡ , Ссли ΠΎΠ½ΠΈ ΠΈΠΌΠ΅ΡŽΡ‚ ΠΎΠ±Ρ‰ΠΈΠΉ Π΄Π΅Π»ΠΈΡ‚Π΅Π»ΡŒ, бóльший Π΅Π΄ΠΈΠ½ΠΈΡ†Ρ‹.

НапримСр, Π²Ρ‹Ρ€Π°ΠΆΠ΅Π½ΠΈΠ΅ ΠΌΠΎΠΆΠ½ΠΎ Π²Ρ‹Ρ‡ΠΈΡΠ»ΠΈΡ‚ΡŒ двумя способами.

ΠŸΠ΅Ρ€Π²Ρ‹ΠΉ способ . Π£ΠΌΠ½ΠΎΠΆΠΈΡ‚ΡŒ число 4 Π½Π° Ρ‡ΠΈΡΠ»ΠΈΡ‚Π΅Π»ΡŒ Π΄Ρ€ΠΎΠ±ΠΈ, Π° Π·Π½Π°ΠΌΠ΅Π½Π°Ρ‚Π΅Π»ΡŒ Π΄Ρ€ΠΎΠ±ΠΈ ΠΎΡΡ‚Π°Π²ΠΈΡ‚ΡŒ Π±Π΅Π· ΠΈΠ·ΠΌΠ΅Π½Π΅Π½ΠΈΠΉ:

Π’Ρ‚ΠΎΡ€ΠΎΠΉ способ . Π£ΠΌΠ½ΠΎΠΆΠ°Π΅ΠΌΡƒΡŽ Ρ‡Π΅Ρ‚Π²Ρ‘Ρ€ΠΊΡƒ ΠΈ Ρ‡Π΅Ρ‚Π²Ρ‘Ρ€ΠΊΡƒ, Π½Π°Ρ…ΠΎΠ΄ΡΡ‰ΡƒΡŽΡΡ Π² Π·Π½Π°ΠΌΠ΅Π½Π°Ρ‚Π΅Π»Π΅ Π΄Ρ€ΠΎΠ±ΠΈ , ΠΌΠΎΠΆΠ½ΠΎ ΡΠΎΠΊΡ€Π°Ρ‚ΠΈΡ‚ΡŒ. Π‘ΠΎΠΊΡ€Π°Ρ‚ΠΈΡ‚ΡŒ эти Ρ‡Π΅Ρ‚Π²Ρ‘Ρ€ΠΊΠΈ ΠΌΠΎΠΆΠ½ΠΎ Π½Π° 4 , ΠΏΠΎΡΠΊΠΎΠ»ΡŒΠΊΡƒ наибольший ΠΎΠ±Ρ‰ΠΈΠΉ Π΄Π΅Π»ΠΈΡ‚Π΅Π»ΡŒ для Π΄Π²ΡƒΡ… Ρ‡Π΅Ρ‚Π²Ρ‘Ρ€ΠΎΠΊ Π΅ΡΡ‚ΡŒ сама Ρ‡Π΅Ρ‚Π²Ρ‘Ρ€ΠΊΠ°:

ΠŸΠΎΠ»ΡƒΡ‡ΠΈΠ»ΡΡ Ρ‚ΠΎΡ‚ ΠΆΠ΅ Ρ€Π΅Π·ΡƒΠ»ΡŒΡ‚Π°Ρ‚ 3. ПослС сокращСния Ρ‡Π΅Ρ‚Π²Ρ‘Ρ€ΠΎΠΊ, Π½Π° ΠΈΡ… мСстС ΠΎΠ±Ρ€Π°Π·ΡƒΡŽΡ‚ΡΡ Π½ΠΎΠ²Ρ‹Π΅ числа: Π΄Π²Π΅ Π΅Π΄ΠΈΠ½ΠΈΡ†Ρ‹. Но ΠΏΠ΅Ρ€Π΅ΠΌΠ½ΠΎΠΆΠ΅Π½ΠΈΠ΅ Π΅Π΄ΠΈΠ½ΠΈΡ†Ρ‹ с Ρ‚Ρ€ΠΎΠΉΠΊΠΎΠΉ, ΠΈ Π΄Π°Π»Π΅Π΅ Π΄Π΅Π»Π΅Π½ΠΈΠ΅ Π½Π° Π΅Π΄ΠΈΠ½ΠΈΡ†Ρƒ Π½ΠΈΡ‡Π΅Π³ΠΎ Π½Π΅ мСняСт. ΠŸΠΎΡΡ‚ΠΎΠΌΡƒ Ρ€Π΅ΡˆΠ΅Π½ΠΈΠ΅ ΠΌΠΎΠΆΠ½ΠΎ Π·Π°ΠΏΠΈΡΠ°Ρ‚ΡŒ ΠΏΠΎΠΊΠΎΡ€ΠΎΡ‡Π΅:

Π‘ΠΎΠΊΡ€Π°Ρ‰Π΅Π½ΠΈΠ΅ ΠΌΠΎΠΆΠ΅Ρ‚ Π±Ρ‹Ρ‚ΡŒ Π²Ρ‹ΠΏΠΎΠ»Π½Π΅Π½ΠΎ Π΄Π°ΠΆΠ΅ Ρ‚ΠΎΠ³Π΄Π°, ΠΊΠΎΠ³Π΄Π° ΠΌΡ‹ Ρ€Π΅ΡˆΠΈΠ»ΠΈ Π²ΠΎΡΠΏΠΎΠ»ΡŒΠ·ΠΎΠ²Π°Ρ‚ΡŒΡΡ ΠΏΠ΅Ρ€Π²Ρ‹ΠΌ способом, Π½ΠΎ Π½Π° этапС пСрСмноТСния числа 4 ΠΈ числитСля 3 Ρ€Π΅ΡˆΠΈΠ»ΠΈ Π²ΠΎΡΠΏΠΎΠ»ΡŒΠ·ΠΎΠ²Π°Ρ‚ΡŒΡΡ сокращСниСм:

А Π²ΠΎΡ‚ ΠΊ ΠΏΡ€ΠΈΠΌΠ΅Ρ€Ρƒ Π²Ρ‹Ρ€Π°ΠΆΠ΅Π½ΠΈΠ΅ ΠΌΠΎΠΆΠ½ΠΎ Π²Ρ‹Ρ‡ΠΈΡΠ»ΠΈΡ‚ΡŒ Ρ‚ΠΎΠ»ΡŒΠΊΠΎ ΠΏΠ΅Ρ€Π²Ρ‹ΠΌ способом β€” ΡƒΠΌΠ½ΠΎΠΆΠΈΡ‚ΡŒ 7 Π½Π° Π·Π½Π°ΠΌΠ΅Π½Π°Ρ‚Π΅Π»ΡŒ Π΄Ρ€ΠΎΠ±ΠΈ , Π° Π·Π½Π°ΠΌΠ΅Π½Π°Ρ‚Π΅Π»ΡŒ ΠΎΡΡ‚Π°Π²ΠΈΡ‚ΡŒ Π±Π΅Π· ΠΈΠ·ΠΌΠ΅Π½Π΅Π½ΠΈΠΉ:

Бвязано это с Ρ‚Π΅ΠΌ, Ρ‡Ρ‚ΠΎ число 7 ΠΈ Π·Π½Π°ΠΌΠ΅Π½Π°Ρ‚Π΅Π»ΡŒ Π΄Ρ€ΠΎΠ±ΠΈ Π½Π΅ ΠΈΠΌΠ΅ΡŽΡ‚ ΠΎΠ±Ρ‰Π΅Π³ΠΎ дСлитСля, бóльшСго Π΅Π΄ΠΈΠ½ΠΈΡ†Ρ‹, ΠΈ соотвСтствСнно Π½Π΅ ΡΠΎΠΊΡ€Π°Ρ‰Π°ΡŽΡ‚ΡΡ.

НСкоторыС ΡƒΡ‡Π΅Π½ΠΈΠΊΠΈ ΠΏΠΎ ошибкС ΡΠΎΠΊΡ€Π°Ρ‰Π°ΡŽΡ‚ ΡƒΠΌΠ½ΠΎΠΆΠ°Π΅ΠΌΠΎΠ΅ число ΠΈ Ρ‡ΠΈΡΠ»ΠΈΡ‚Π΅Π»ΡŒ Π΄Ρ€ΠΎΠ±ΠΈ. Π”Π΅Π»Π°Ρ‚ΡŒ этого нСльзя. НапримСр, ΡΠ»Π΅Π΄ΡƒΡŽΡ‰Π°Ρ запись Π½Π΅ являСтся ΠΏΡ€Π°Π²ΠΈΠ»ΡŒΠ½ΠΎΠΉ:

Π‘ΠΎΠΊΡ€Π°Ρ‰Π΅Π½ΠΈΠ΅ Π΄Ρ€ΠΎΠ±ΠΈ ΠΏΠΎΠ΄Ρ€Π°Π·ΡƒΠΌΠ΅Π²Π°Π΅Ρ‚, Ρ‡Ρ‚ΠΎ ΠΈ Ρ‡ΠΈΡΠ»ΠΈΡ‚Π΅Π»ΡŒ ΠΈ Π·Π½Π°ΠΌΠ΅Π½Π°Ρ‚Π΅Π»ΡŒ Π±ΡƒΠ΄Π΅Ρ‚ Ρ€Π°Π·Π΄Π΅Π»Ρ‘Π½ Π½Π° ΠΎΠ΄Π½ΠΎ ΠΈ Ρ‚ΠΎΠΆΠ΅ число. Π’ ситуации с Π²Ρ‹Ρ€Π°ΠΆΠ΅Π½ΠΈΠ΅ΠΌ Π΄Π΅Π»Π΅Π½ΠΈΠ΅ Π²Ρ‹ΠΏΠΎΠ»Π½Π΅Π½ΠΎ Ρ‚ΠΎΠ»ΡŒΠΊΠΎ Π² числитСлС, ΠΏΠΎΡΠΊΠΎΠ»ΡŒΠΊΡƒ Π·Π°ΠΏΠΈΡΠ°Ρ‚ΡŒ это всё Ρ€Π°Π²Π½ΠΎ, Ρ‡Ρ‚ΠΎ Π·Π°ΠΏΠΈΡΠ°Ρ‚ΡŒ . Π’ΠΈΠ΄ΠΈΠΌ, Ρ‡Ρ‚ΠΎ Π΄Π΅Π»Π΅Π½ΠΈΠ΅ Π²Ρ‹ΠΏΠΎΠ»Π½Π΅Π½ΠΎ Ρ‚ΠΎΠ»ΡŒΠΊΠΎ Π² числитСлС, Π° Π² Π·Π½Π°ΠΌΠ΅Π½Π°Ρ‚Π΅Π»Π΅ Π½ΠΈΠΊΠ°ΠΊΠΎΠ³ΠΎ дСлСния Π½Π΅ происходит.

Π£ΠΌΠ½ΠΎΠΆΠ΅Π½ΠΈΠ΅ Π΄Ρ€ΠΎΠ±Π΅ΠΉ

Π§Ρ‚ΠΎΠ±Ρ‹ ΠΏΠ΅Ρ€Π΅ΠΌΠ½ΠΎΠΆΠΈΡ‚ΡŒ Π΄Ρ€ΠΎΠ±ΠΈ, Π½ΡƒΠΆΠ½ΠΎ ΠΏΠ΅Ρ€Π΅ΠΌΠ½ΠΎΠΆΠΈΡ‚ΡŒ ΠΈΡ… числитСли ΠΈ Π·Π½Π°ΠΌΠ΅Π½Π°Ρ‚Π΅Π»ΠΈ. Если Π² ΠΎΡ‚Π²Π΅Ρ‚Π΅ получится Π½Π΅ΠΏΡ€Π°Π²ΠΈΠ»ΡŒΠ½Π°Ρ Π΄Ρ€ΠΎΠ±ΡŒ, Π½ΡƒΠΆΠ½ΠΎ Π²Ρ‹Π΄Π΅Π»ΠΈΡ‚ΡŒ Π² Π½Π΅ΠΉ Ρ†Π΅Π»ΡƒΡŽ Ρ‡Π°ΡΡ‚ΡŒ.

ΠŸΡ€ΠΈΠΌΠ΅Ρ€ 1. Найти Π·Π½Π°Ρ‡Π΅Π½ΠΈΠ΅ выраТСния .

ΠŸΠΎΠ»ΡƒΡ‡ΠΈΠ»ΠΈ ΠΎΡ‚Π²Π΅Ρ‚ . Π–Π΅Π»Π°Ρ‚Π΅Π»ΡŒΠ½ΠΎ ΡΠΎΠΊΡ€Π°Ρ‚ΠΈΡ‚ΡŒ Π΄Π°Π½Π½ΡƒΡŽ Π΄Ρ€ΠΎΠ±ΡŒ. Π”Ρ€ΠΎΠ±ΡŒ ΠΌΠΎΠΆΠ½ΠΎ ΡΠΎΠΊΡ€Π°Ρ‚ΠΈΡ‚ΡŒ Π½Π° 2. Π’ΠΎΠ³Π΄Π° ΠΎΠΊΠΎΠ½Ρ‡Π°Ρ‚Π΅Π»ΡŒΠ½ΠΎΠ΅ Ρ€Π΅ΡˆΠ΅Π½ΠΈΠ΅ ΠΏΡ€ΠΈΠΌΠ΅Ρ‚ ΡΠ»Π΅Π΄ΡƒΡŽΡ‰ΠΈΠΉ Π²ΠΈΠ΄:

Π’Ρ‹Ρ€Π°ΠΆΠ΅Π½ΠΈΠ΅ ΠΌΠΎΠΆΠ½ΠΎ ΠΏΠΎΠ½ΠΈΠΌΠ°Ρ‚ΡŒ, ΠΊΠ°ΠΊ взятиС ΠΏΠΈΡ†Ρ†Ρ‹ ΠΎΡ‚ ΠΏΠΎΠ»ΠΎΠ²ΠΈΠ½Ρ‹ ΠΏΠΈΡ†Ρ†Ρ‹. Допустим, Ρƒ нас Π΅ΡΡ‚ΡŒ ΠΏΠΎΠ»ΠΎΠ²ΠΈΠ½Π° ΠΏΠΈΡ†Ρ†Ρ‹:

Как Π²Π·ΡΡ‚ΡŒ ΠΎΡ‚ этой ΠΏΠΎΠ»ΠΎΠ²ΠΈΠ½Ρ‹ Π΄Π²Π΅ Ρ‚Ρ€Π΅Ρ‚ΡŒΠΈΡ…? Π‘Π½Π°Ρ‡Π°Π»Π° Π½ΡƒΠΆΠ½ΠΎ ΠΏΠΎΠ΄Π΅Π»ΠΈΡ‚ΡŒ эту ΠΏΠΎΠ»ΠΎΠ²ΠΈΠ½Ρƒ Π½Π° Ρ‚Ρ€ΠΈ Ρ€Π°Π²Π½Ρ‹Π΅ части:

И Π²Π·ΡΡ‚ΡŒ ΠΎΡ‚ этих Ρ‚Ρ€Π΅Ρ… кусочков Π΄Π²Π°:

Π£ нас получится ΠΏΠΈΡ†Ρ†Ρ‹. ВспомнитС, ΠΊΠ°ΠΊ выглядит ΠΏΠΈΡ†Ρ†Π°, раздСлСнная Π½Π° Ρ‚Ρ€ΠΈ части:

Один кусок ΠΎΡ‚ этой ΠΏΠΈΡ†Ρ†Ρ‹ ΠΈ взятыС Π½Π°ΠΌΠΈ Π΄Π²Π° кусочка Π±ΡƒΠ΄ΡƒΡ‚ ΠΈΠΌΠ΅Ρ‚ΡŒ ΠΎΠ΄ΠΈΠ½Π°ΠΊΠΎΠ²Ρ‹Π΅ Ρ€Π°Π·ΠΌΠ΅Ρ€Ρ‹:

Π”Ρ€ΡƒΠ³ΠΈΠΌΠΈ словами, Ρ€Π΅Ρ‡ΡŒ ΠΈΠ΄Π΅Ρ‚ ΠΎΠ± ΠΎΠ΄Π½ΠΎΠΌ ΠΈ Ρ‚ΠΎΠΌ ΠΆΠ΅ Ρ€Π°Π·ΠΌΠ΅Ρ€Π΅ ΠΏΠΈΡ†Ρ†Ρ‹. ΠŸΠΎΡΡ‚ΠΎΠΌΡƒ Π·Π½Π°Ρ‡Π΅Π½ΠΈΠ΅ выраТСния Ρ€Π°Π²Π½ΠΎ

ΠŸΡ€ΠΈΠΌΠ΅Ρ€ 2 . Найти Π·Π½Π°Ρ‡Π΅Π½ΠΈΠ΅ выраТСния

Π£ΠΌΠ½ΠΎΠΆΠ°Π΅ΠΌ Ρ‡ΠΈΡΠ»ΠΈΡ‚Π΅Π»ΡŒ ΠΏΠ΅Ρ€Π²ΠΎΠΉ Π΄Ρ€ΠΎΠ±ΠΈ Π½Π° Ρ‡ΠΈΡΠ»ΠΈΡ‚Π΅Π»ΡŒ Π²Ρ‚ΠΎΡ€ΠΎΠΉ Π΄Ρ€ΠΎΠ±ΠΈ, Π° Π·Π½Π°ΠΌΠ΅Π½Π°Ρ‚Π΅Π»ΡŒ ΠΏΠ΅Ρ€Π²ΠΎΠΉ Π΄Ρ€ΠΎΠ±ΠΈ Π½Π° Π·Π½Π°ΠΌΠ΅Π½Π°Ρ‚Π΅Π»ΡŒ Π²Ρ‚ΠΎΡ€ΠΎΠΉ Π΄Ρ€ΠΎΠ±ΠΈ:

Π’ ΠΎΡ‚Π²Π΅Ρ‚Π΅ ΠΏΠΎΠ»ΡƒΡ‡ΠΈΠ»Π°ΡΡŒ Π½Π΅ΠΏΡ€Π°Π²ΠΈΠ»ΡŒΠ½Π°Ρ Π΄Ρ€ΠΎΠ±ΡŒ. Π’Ρ‹Π΄Π΅Π»ΠΈΠΌ Π² Π½Π΅ΠΉ Ρ†Π΅Π»ΡƒΡŽ Ρ‡Π°ΡΡ‚ΡŒ:

ΠŸΡ€ΠΈΠΌΠ΅Ρ€ 3. Найти Π·Π½Π°Ρ‡Π΅Π½ΠΈΠ΅ выраТСния

Π£ΠΌΠ½ΠΎΠΆΠ°Π΅ΠΌ Ρ‡ΠΈΡΠ»ΠΈΡ‚Π΅Π»ΡŒ ΠΏΠ΅Ρ€Π²ΠΎΠΉ Π΄Ρ€ΠΎΠ±ΠΈ Π½Π° Ρ‡ΠΈΡΠ»ΠΈΡ‚Π΅Π»ΡŒ Π²Ρ‚ΠΎΡ€ΠΎΠΉ Π΄Ρ€ΠΎΠ±ΠΈ, Π° Π·Π½Π°ΠΌΠ΅Π½Π°Ρ‚Π΅Π»ΡŒ ΠΏΠ΅Ρ€Π²ΠΎΠΉ Π΄Ρ€ΠΎΠ±ΠΈ Π½Π° Π·Π½Π°ΠΌΠ΅Π½Π°Ρ‚Π΅Π»ΡŒ Π²Ρ‚ΠΎΡ€ΠΎΠΉ Π΄Ρ€ΠΎΠ±ΠΈ:

Π’ ΠΎΡ‚Π²Π΅Ρ‚Π΅ ΠΏΠΎΠ»ΡƒΡ‡ΠΈΠ»Π°ΡΡŒ ΠΏΡ€Π°Π²ΠΈΠ»ΡŒΠ½Π°Ρ Π΄Ρ€ΠΎΠ±ΡŒ, Π½ΠΎ Π±ΡƒΠ΄Π΅Ρ‚ Ρ…ΠΎΡ€ΠΎΡˆΠΎ, Ссли Π΅Ρ‘ ΡΠΎΠΊΡ€Π°Ρ‚ΠΈΡ‚ΡŒ. Π§Ρ‚ΠΎΠ±Ρ‹ ΡΠΎΠΊΡ€Π°Ρ‚ΠΈΡ‚ΡŒ эту Π΄Ρ€ΠΎΠ±ΡŒ, Π½ΡƒΠΆΠ½ΠΎ Ρ‡ΠΈΡΠ»ΠΈΡ‚Π΅Π»ΡŒ ΠΈ Π·Π½Π°ΠΌΠ΅Π½Π°Ρ‚Π΅Π»ΡŒ Π΄Π°Π½Π½ΠΎΠΉ Π΄Ρ€ΠΎΠ±ΠΈ Ρ€Π°Π·Π΄Π΅Π»ΠΈΡ‚ΡŒ Π½Π° наибольший ΠΎΠ±Ρ‰ΠΈΠΉ Π΄Π΅Π»ΠΈΡ‚Π΅Π»ΡŒ (ΠΠžΠ”) чисСл 105 ΠΈ 450.

Π˜Ρ‚Π°ΠΊ, Π½Π°ΠΉΠ΄Ρ‘ΠΌ ΠΠžΠ” чисСл 105 ΠΈ 450:

Π’Π΅ΠΏΠ΅Ρ€ΡŒ Π΄Π΅Π»ΠΈΠΌ Ρ‡ΠΈΡΠ»ΠΈΡ‚Π΅Π»ΡŒ ΠΈ Π·Π½Π°ΠΌΠ΅Π½Π°Ρ‚Π΅Π»ΡŒ нашСго ΠΎΡ‚Π²Π΅Ρ‚Π° Π½Π° ΠΠžΠ”, ΠΊΠΎΡ‚ΠΎΡ€Ρ‹ΠΉ ΠΌΡ‹ сСйчас нашли, Ρ‚ΠΎ Π΅ΡΡ‚ΡŒ Π½Π° 15

ΠŸΡ€Π΅Π΄ΡΡ‚Π°Π²Π»Π΅Π½ΠΈΠ΅ Ρ†Π΅Π»ΠΎΠ³ΠΎ числа Π² Π²ΠΈΠ΄Π΅ Π΄Ρ€ΠΎΠ±ΠΈ

Π›ΡŽΠ±ΠΎΠ΅ Ρ†Π΅Π»ΠΎΠ΅ число ΠΌΠΎΠΆΠ½ΠΎ ΠΏΡ€Π΅Π΄ΡΡ‚Π°Π²ΠΈΡ‚ΡŒ Π² Π²ΠΈΠ΄Π΅ Π΄Ρ€ΠΎΠ±ΠΈ. НапримСр, число 5 ΠΌΠΎΠΆΠ½ΠΎ ΠΏΡ€Π΅Π΄ΡΡ‚Π°Π²ΠΈΡ‚ΡŒ ΠΊΠ°ΠΊ . ΠžΡ‚ этого пятёрка своСго значСния Π½Π΅ помСняСт, ΠΏΠΎΡΠΊΠΎΠ»ΡŒΠΊΡƒ Π²Ρ‹Ρ€Π°ΠΆΠ΅Π½ΠΈΠ΅ ΠΎΠ·Π½Π°Ρ‡Π°Π΅Ρ‚ «число ΠΏΡΡ‚ΡŒ Ρ€Π°Π·Π΄Π΅Π»ΠΈΡ‚ΡŒ Π½Π° Π΅Π΄ΠΈΠ½ΠΈΡ†ΡƒΒ», Π° это, ΠΊΠ°ΠΊ извСстно Ρ€Π°Π²Π½ΠΎ пятёркС:

ΠžΠ±Ρ€Π°Ρ‚Π½Ρ‹Π΅ числа

БСйчас ΠΌΡ‹ познакомимся с ΠΎΡ‡Π΅Π½ΡŒ интСрСсной Ρ‚Π΅ΠΌΠΎΠΉ Π² ΠΌΠ°Ρ‚Π΅ΠΌΠ°Ρ‚ΠΈΠΊΠ΅. Она называСтся Β«ΠΎΠ±Ρ€Π°Ρ‚Π½Ρ‹Π΅ числа».

ΠžΠΏΡ€Π΅Π΄Π΅Π»Π΅Π½ΠΈΠ΅. ΠžΠ±Ρ€Π°Ρ‚Π½Ρ‹ΠΌ ΠΊ числу a называСтся число, ΠΊΠΎΡ‚ΠΎΡ€ΠΎΠ΅ ΠΏΡ€ΠΈ ΡƒΠΌΠ½ΠΎΠΆΠ΅Π½ΠΈΠΈ Π½Π° a Π΄Π°Ρ‘Ρ‚ Π΅Π΄ΠΈΠ½ΠΈΡ†Ρƒ.

Π”Π°Π²Π°ΠΉΡ‚Π΅ подставим Π² это ΠΎΠΏΡ€Π΅Π΄Π΅Π»Π΅Π½ΠΈΠ΅ вмСсто ΠΏΠ΅Ρ€Π΅ΠΌΠ΅Π½Π½ΠΎΠΉ a число 5 ΠΈ ΠΏΠΎΠΏΡ€ΠΎΠ±ΡƒΠ΅ΠΌ ΠΏΡ€ΠΎΡ‡ΠΈΡ‚Π°Ρ‚ΡŒ ΠΎΠΏΡ€Π΅Π΄Π΅Π»Π΅Π½ΠΈΠ΅:

ΠžΠ±Ρ€Π°Ρ‚Π½Ρ‹ΠΌ ΠΊ числу 5 называСтся число, ΠΊΠΎΡ‚ΠΎΡ€ΠΎΠ΅ ΠΏΡ€ΠΈ ΡƒΠΌΠ½ΠΎΠΆΠ΅Π½ΠΈΠΈ Π½Π° 5 Π΄Π°Ρ‘Ρ‚ Π΅Π΄ΠΈΠ½ΠΈΡ†Ρƒ.

МоТно Π»ΠΈ Π½Π°ΠΉΡ‚ΠΈ Ρ‚Π°ΠΊΠΎΠ΅ число, ΠΊΠΎΡ‚ΠΎΡ€ΠΎΠ΅ ΠΏΡ€ΠΈ ΡƒΠΌΠ½ΠΎΠΆΠ΅Π½ΠΈΠΈ Π½Π° 5, Π΄Π°Ρ‘Ρ‚ Π΅Π΄ΠΈΠ½ΠΈΡ†Ρƒ? ΠžΠΊΠ°Π·Ρ‹Π²Π°Π΅Ρ‚ΡΡ ΠΌΠΎΠΆΠ½ΠΎ. ΠŸΡ€Π΅Π΄ΡΡ‚Π°Π²ΠΈΠΌ пятёрку Π² Π²ΠΈΠ΄Π΅ Π΄Ρ€ΠΎΠ±ΠΈ:

Π—Π°Ρ‚Π΅ΠΌ ΡƒΠΌΠ½ΠΎΠΆΠΈΡ‚ΡŒ эту Π΄Ρ€ΠΎΠ±ΡŒ Π½Π° саму сСбя, Ρ‚ΠΎΠ»ΡŒΠΊΠΎ помСняСм мСстами Ρ‡ΠΈΡΠ»ΠΈΡ‚Π΅Π»ΡŒ ΠΈ Π·Π½Π°ΠΌΠ΅Π½Π°Ρ‚Π΅Π»ΡŒ. Π”Ρ€ΡƒΠ³ΠΈΠΌΠΈ словами, ΡƒΠΌΠ½ΠΎΠΆΠΈΠΌ Π΄Ρ€ΠΎΠ±ΡŒ Π½Π° саму сСбя, Ρ‚ΠΎΠ»ΡŒΠΊΠΎ ΠΏΠ΅Ρ€Π΅Π²Ρ‘Ρ€Π½ΡƒΡ‚ΡƒΡŽ:

Π§Ρ‚ΠΎ получится Π² Ρ€Π΅Π·ΡƒΠ»ΡŒΡ‚Π°Ρ‚Π΅ этого? Если ΠΌΡ‹ ΠΏΡ€ΠΎΠ΄ΠΎΠ»ΠΆΠΈΠΌ Ρ€Π΅ΡˆΠ°Ρ‚ΡŒ этот ΠΏΡ€ΠΈΠΌΠ΅Ρ€, Ρ‚ΠΎ ΠΏΠΎΠ»ΡƒΡ‡ΠΈΠΌ Π΅Π΄ΠΈΠ½ΠΈΡ†Ρƒ:

Π—Π½Π°Ρ‡ΠΈΡ‚ ΠΎΠ±Ρ€Π°Ρ‚Π½Ρ‹ΠΌ ΠΊ числу 5, являСтся число , ΠΏΠΎΡΠΊΠΎΠ»ΡŒΠΊΡƒ ΠΏΡ€ΠΈ ΡƒΠΌΠ½ΠΎΠΆΠ΅Π½ΠΈΠΈ 5 Π½Π° получаСтся Π΅Π΄ΠΈΠ½ΠΈΡ†Π°.

ΠžΠ±Ρ€Π°Ρ‚Π½ΠΎΠ΅ число ΠΌΠΎΠΆΠ½ΠΎ Π½Π°ΠΉΡ‚ΠΈ Ρ‚Π°ΠΊΠΆΠ΅ для любого Π΄Ρ€ΡƒΠ³ΠΎΠ³ΠΎ Ρ†Π΅Π»ΠΎΠ³ΠΎ числа.

Найти ΠΎΠ±Ρ€Π°Ρ‚Π½ΠΎΠ΅ число ΠΌΠΎΠΆΠ½ΠΎ Ρ‚Π°ΠΊΠΆΠ΅ для любой Π΄Ρ€ΡƒΠ³ΠΎΠΉ Π΄Ρ€ΠΎΠ±ΠΈ. Для этого достаточно ΠΏΠ΅Ρ€Π΅Π²Π΅Ρ€Π½ΡƒΡ‚ΡŒ Π΅Ρ‘.

Π”Π΅Π»Π΅Π½ΠΈΠ΅ Π΄Ρ€ΠΎΠ±ΠΈ Π½Π° число

Допустим, Ρƒ нас имССтся ΠΏΠΎΠ»ΠΎΠ²ΠΈΠ½Π° ΠΏΠΈΡ†Ρ†Ρ‹:

Π Π°Π·Π΄Π΅Π»ΠΈΠΌ Π΅Ρ‘ ΠΏΠΎΡ€ΠΎΠ²Π½Ρƒ Π½Π° Π΄Π²ΠΎΠΈΡ…. Бколько ΠΏΠΈΡ†Ρ†Ρ‹ достанСтся ΠΊΠ°ΠΆΠ΄ΠΎΠΌΡƒ?

Π’ΠΈΠ΄Π½ΠΎ, Ρ‡Ρ‚ΠΎ послС раздСлСния ΠΏΠΎΠ»ΠΎΠ²ΠΈΠ½Ρ‹ ΠΏΠΈΡ†Ρ†Ρ‹ ΠΏΠΎΠ»ΡƒΡ‡ΠΈΠ»ΠΎΡΡŒ Π΄Π²Π° Ρ€Π°Π²Π½Ρ‹Ρ… кусочка, ΠΊΠ°ΠΆΠ΄Ρ‹ΠΉ ΠΈΠ· ΠΊΠΎΡ‚ΠΎΡ€Ρ‹Ρ… составляСт ΠΏΠΈΡ†Ρ†Ρ‹. Π—Π½Π°Ρ‡ΠΈΡ‚ ΠΊΠ°ΠΆΠ΄ΠΎΠΌΡƒ достанСтся ΠΏΠΎ ΠΏΠΈΡ†Ρ†Ρ‹.

Π£ΠΌΠ½ΠΎΠΆΠ΅Π½ΠΈΠ΅ ΠΈ Π΄Π΅Π»Π΅Π½ΠΈΠ΅ Π΄Ρ€ΠΎΠ±Π΅ΠΉ.

Π’Π½ΠΈΠΌΠ°Π½ΠΈΠ΅!
К этой Ρ‚Π΅ΠΌΠ΅ ΠΈΠΌΠ΅ΡŽΡ‚ΡΡ Π΄ΠΎΠΏΠΎΠ»Π½ΠΈΡ‚Π΅Π»ΡŒΠ½Ρ‹Π΅
ΠΌΠ°Ρ‚Π΅Ρ€ΠΈΠ°Π»Ρ‹ Π² Особом Ρ€Π°Π·Π΄Π΅Π»Π΅ 555.
Для Ρ‚Π΅Ρ…, ΠΊΡ‚ΠΎ сильно «Π½Π΅ ΠΎΡ‡Π΅Π½ΡŒ…»
И для Ρ‚Π΅Ρ…, ΠΊΡ‚ΠΎ «ΠΎΡ‡Π΅Π½ΡŒ Π΄Π°ΠΆΠ΅…»)

Π­Ρ‚Π° опСрация Π³ΠΎΡ€Π°Π·Π΄ΠΎ приятнСС слоТСния-вычитания ! ΠŸΠΎΡ‚ΠΎΠΌΡƒ Ρ‡Ρ‚ΠΎ ΠΏΡ€ΠΎΡ‰Π΅. Напоминаю: Ρ‡Ρ‚ΠΎΠ±Ρ‹ ΡƒΠΌΠ½ΠΎΠΆΠΈΡ‚ΡŒ Π΄Ρ€ΠΎΠ±ΡŒ Π½Π° Π΄Ρ€ΠΎΠ±ΡŒ, Π½ΡƒΠΆΠ½ΠΎ ΠΏΠ΅Ρ€Π΅ΠΌΠ½ΠΎΠΆΠΈΡ‚ΡŒ числитСли (это Π±ΡƒΠ΄Π΅Ρ‚ Ρ‡ΠΈΡΠ»ΠΈΡ‚Π΅Π»ΡŒ Ρ€Π΅Π·ΡƒΠ»ΡŒΡ‚Π°Ρ‚Π°) ΠΈ Π·Π½Π°ΠΌΠ΅Π½Π°Ρ‚Π΅Π»ΠΈ (это Π±ΡƒΠ΄Π΅Ρ‚ Π·Π½Π°ΠΌΠ΅Π½Π°Ρ‚Π΅Π»ΡŒ). Π’ΠΎ Π΅ΡΡ‚ΡŒ:

НапримСр:

Всё ΠΏΡ€Π΅Π΄Π΅Π»ΡŒΠ½ΠΎ просто . И, поТалуйста, Π½Π΅ ΠΈΡ‰ΠΈΡ‚Π΅ ΠΎΠ±Ρ‰ΠΈΠΉ Π·Π½Π°ΠΌΠ΅Π½Π°Ρ‚Π΅Π»ΡŒ! НС Π½Π°Π΄ΠΎ Π΅Π³ΠΎ Π·Π΄Π΅ΡΡŒβ€¦

Π§Ρ‚ΠΎΠ±Ρ‹ Ρ€Π°Π·Π΄Π΅Π»ΠΈΡ‚ΡŒ Π΄Ρ€ΠΎΠ±ΡŒ Π½Π° Π΄Ρ€ΠΎΠ±ΡŒ, Π½ΡƒΠΆΠ½ΠΎ ΠΏΠ΅Ρ€Π΅Π²Π΅Ρ€Π½ΡƒΡ‚ΡŒ Π²Ρ‚ΠΎΡ€ΡƒΡŽ (это Π²Π°ΠΆΠ½ΠΎ!) Π΄Ρ€ΠΎΠ±ΡŒ ΠΈ ΠΈΡ… ΠΏΠ΅Ρ€Π΅ΠΌΠ½ΠΎΠΆΠΈΡ‚ΡŒ, Ρ‚. Π΅.:

НапримСр:

Если попалось ΡƒΠΌΠ½ΠΎΠΆΠ΅Π½ΠΈΠ΅ ΠΈΠ»ΠΈ Π΄Π΅Π»Π΅Π½ΠΈΠ΅ с Ρ†Π΅Π»Ρ‹ΠΌΠΈ числами ΠΈ дробями — Π½ΠΈΡ‡Π΅Π³ΠΎ ΡΡ‚Ρ€Π°ΡˆΠ½ΠΎΠ³ΠΎ. Как ΠΈ ΠΏΡ€ΠΈ слоТСнии, Π΄Π΅Π»Π°Π΅ΠΌ ΠΈΠ· Ρ†Π΅Π»ΠΎΠ³ΠΎ числа Π΄Ρ€ΠΎΠ±ΡŒ с Π΅Π΄ΠΈΠ½ΠΈΡ†Π΅ΠΉ Π² Π·Π½Π°ΠΌΠ΅Π½Π°Ρ‚Π΅Π»Π΅ — ΠΈ Π²ΠΏΠ΅Ρ€Ρ‘Π΄! НапримСр:

Π’ ΡΡ‚Π°Ρ€ΡˆΠΈΡ… классах часто приходится ΠΈΠΌΠ΅Ρ‚ΡŒ Π΄Π΅Π»ΠΎ с трСхэтаТными (Π° Ρ‚ΠΎ ΠΈ чСтырСхэтаТными!) дробями. НапримСр:

Как эту Π΄Ρ€ΠΎΠ±ΡŒ привСсти ΠΊ ΠΏΡ€ΠΈΠ»ΠΈΡ‡Π½ΠΎΠΌΡƒ Π²ΠΈΠ΄Ρƒ? Π”Π° ΠΎΡ‡Π΅Π½ΡŒ просто! Π˜ΡΠΏΠΎΠ»ΡŒΠ·ΠΎΠ²Π°Ρ‚ΡŒ Π΄Π΅Π»Π΅Π½ΠΈΠ΅ Ρ‡Π΅Ρ€Π΅Π· Π΄Π²Π΅ Ρ‚ΠΎΡ‡ΠΊΠΈ:

Но Π½Π΅ Π·Π°Π±Ρ‹Π²Π°ΠΉΡ‚Π΅ ΠΎ порядкС дСлСния! Π’ ΠΎΡ‚Π»ΠΈΡ‡ΠΈΠ΅ ΠΎΡ‚ умноТСния, здСсь это ΠΎΡ‡Π΅Π½ΡŒ Π²Π°ΠΆΠ½ΠΎ! ΠšΠΎΠ½Π΅Ρ‡Π½ΠΎ, 4:2, ΠΈΠ»ΠΈ 2:4 ΠΌΡ‹ Π½Π΅ спутаСм. А Π²ΠΎΡ‚ Π² трёхэтаТной Π΄Ρ€ΠΎΠ±ΠΈ Π»Π΅Π³ΠΊΠΎ ΠΎΡˆΠΈΠ±ΠΈΡ‚ΡŒΡΡ. ΠžΠ±Ρ€Π°Ρ‚ΠΈΡ‚Π΅ Π²Π½ΠΈΠΌΠ°Π½ΠΈΠ΅, Π½Π°ΠΏΡ€ΠΈΠΌΠ΅Ρ€:

Π’ ΠΏΠ΅Ρ€Π²ΠΎΠΌ случаС (Π²Ρ‹Ρ€Π°ΠΆΠ΅Π½ΠΈΠ΅ слСва):

Π’ΠΎ Π²Ρ‚ΠΎΡ€ΠΎΠΌ (Π²Ρ‹Ρ€Π°ΠΆΠ΅Π½ΠΈΠ΅ справа):

ЧувствуСтС Ρ€Π°Π·Π½ΠΈΡ†Ρƒ? 4 ΠΈ 1/9!

А Ρ‡Π΅ΠΌ задаСтся порядок дСлСния? Или скобками, ΠΈΠ»ΠΈ (ΠΊΠ°ΠΊ здСсь) Π΄Π»ΠΈΠ½ΠΎΠΉ Π³ΠΎΡ€ΠΈΠ·ΠΎΠ½Ρ‚Π°Π»ΡŒΠ½Ρ‹Ρ… Ρ‡Π΅Ρ€Ρ‚ΠΎΡ‡Π΅ΠΊ. Π Π°Π·Π²ΠΈΠ²Π°ΠΉΡ‚Π΅ Π³Π»Π°Π·ΠΎΠΌΠ΅Ρ€. А Ссли Π½Π΅Ρ‚ Π½ΠΈ скобок, Π½ΠΈ Ρ‡Π΅Ρ€Ρ‚ΠΎΡ‡Π΅ΠΊ, Ρ‚ΠΈΠΏΠ°:

Ρ‚ΠΎ Π΄Π΅Π»ΠΈΠΌ-ΡƒΠΌΠ½ΠΎΠΆΠ°Π΅ΠΌ ΠΏΠΎ порядочку, слСва Π½Π°ΠΏΡ€Π°Π²ΠΎ !

И Π΅Ρ‰Π΅ ΠΎΡ‡Π΅Π½ΡŒ простой ΠΈ Π²Π°ΠΆΠ½Ρ‹ΠΉ ΠΏΡ€ΠΈΡ‘ΠΌ. Π’ дСйствиях со стСпСнями ΠΎΠ½ Π²Π°ΠΌ ΠΎΡ… ΠΊΠ°ΠΊ пригодится! ПодСлим Π΅Π΄ΠΈΠ½ΠΈΡ†Ρƒ Π½Π° Π»ΡŽΠ±ΡƒΡŽ Π΄Ρ€ΠΎΠ±ΡŒ, Π½Π°ΠΏΡ€ΠΈΠΌΠ΅Ρ€, Π½Π° 13/15:

Π”Ρ€ΠΎΠ±ΡŒ ΠΏΠ΅Ρ€Π΅Π²Π΅Ρ€Π½ΡƒΠ»Π°ΡΡŒ! И Ρ‚Π°ΠΊ Π±Ρ‹Π²Π°Π΅Ρ‚ всСгда. ΠŸΡ€ΠΈ Π΄Π΅Π»Π΅Π½ΠΈΠΈ 1 Π½Π° Π»ΡŽΠ±ΡƒΡŽ Π΄Ρ€ΠΎΠ±ΡŒ, Π² Ρ€Π΅Π·ΡƒΠ»ΡŒΡ‚Π°Ρ‚Π΅ ΠΏΠΎΠ»ΡƒΡ‡Π°Π΅ΠΌ Ρ‚Ρƒ ΠΆΠ΅ Π΄Ρ€ΠΎΠ±ΡŒ, Ρ‚ΠΎΠ»ΡŒΠΊΠΎ ΠΏΠ΅Ρ€Π΅Π²Π΅Ρ€Π½ΡƒΡ‚ΡƒΡŽ.

Π’ΠΎΡ‚ ΠΈ всС дСйствия с дробями. Π’Π΅Ρ‰ΡŒ достаточно простая, Π½ΠΎ ошибок Π΄Π°Ρ‘Ρ‚ Π±ΠΎΠ»Π΅Π΅, Ρ‡Π΅ΠΌ достаточно. ΠŸΡ€ΠΈΠΌΠΈΡ‚Π΅ ΠΊ свСдСнию практичСскиС совСты, ΠΈ ΠΈΡ… (ошибок) Π±ΡƒΠ΄Π΅Ρ‚ мСньшС!

ΠŸΡ€Π°ΠΊΡ‚ΠΈΡ‡Π΅ΡΠΊΠΈΠ΅ совСты:

1. Π‘Π°ΠΌΠΎΠ΅ Π³Π»Π°Π²Π½ΠΎΠ΅ ΠΏΡ€ΠΈ Ρ€Π°Π±ΠΎΡ‚Π΅ с Π΄Ρ€ΠΎΠ±Π½Ρ‹ΠΌΠΈ выраТСниями — Π°ΠΊΠΊΡƒΡ€Π°Ρ‚Π½ΠΎΡΡ‚ΡŒ ΠΈ Π²Π½ΠΈΠΌΠ°Ρ‚Π΅Π»ΡŒΠ½ΠΎΡΡ‚ΡŒ! Π­Ρ‚ΠΎ Π½Π΅ ΠΎΠ±Ρ‰ΠΈΠ΅ слова, Π½Π΅ Π±Π»Π°Π³ΠΈΠ΅ поТСлания! Π­Ρ‚ΠΎ суровая Π½Π΅ΠΎΠ±Ρ…ΠΎΠ΄ΠΈΠΌΠΎΡΡ‚ΡŒ! ВсС вычислСния Π½Π° Π•Π“Π­ Π΄Π΅Π»Π°ΠΉΡ‚Π΅ ΠΊΠ°ΠΊ ΠΏΠΎΠ»Π½ΠΎΡ†Π΅Π½Π½ΠΎΠ΅ Π·Π°Π΄Π°Π½ΠΈΠ΅, сосрСдоточСнно ΠΈ Ρ‡Ρ‘Ρ‚ΠΊΠΎ. Π›ΡƒΡ‡ΡˆΠ΅ Π½Π°ΠΏΠΈΡΠ°Ρ‚ΡŒ Π΄Π²Π΅ лишниС строчки Π² Ρ‡Π΅Ρ€Π½ΠΎΠ²ΠΈΠΊΠ΅, Ρ‡Π΅ΠΌ Π½Π°ΠΊΠΎΡΡΡ‡ΠΈΡ‚ΡŒ ΠΏΡ€ΠΈ расчётС Π² ΡƒΠΌΠ΅.

2. Π’ ΠΏΡ€ΠΈΠΌΠ΅Ρ€Π°Ρ… с Ρ€Π°Π·Π½Ρ‹ΠΌΠΈ Π²ΠΈΠ΄Π°ΠΌΠΈ Π΄Ρ€ΠΎΠ±Π΅ΠΉ — ΠΏΠ΅Ρ€Π΅Ρ…ΠΎΠ΄ΠΈΠΌ ΠΊ ΠΎΠ±Ρ‹ΠΊΠ½ΠΎΠ²Π΅Π½Π½Ρ‹ΠΌ дробям.

3. ВсС Π΄Ρ€ΠΎΠ±ΠΈ сокращаСм Π΄ΠΎ ΡƒΠΏΠΎΡ€Π°.

4. ΠœΠ½ΠΎΠ³ΠΎΡΡ‚Π°ΠΆΠ½Ρ‹Π΅ Π΄Ρ€ΠΎΠ±Π½Ρ‹Π΅ выраТСния сводим ΠΊ ΠΎΠ±Ρ‹ΠΊΠ½ΠΎΠ²Π΅Π½Π½Ρ‹ΠΌ, ΠΈΡΠΏΠΎΠ»ΡŒΠ·ΡƒΡ Π΄Π΅Π»Π΅Π½ΠΈΠ΅ Ρ‡Π΅Ρ€Π΅Π· Π΄Π²Π΅ Ρ‚ΠΎΡ‡ΠΊΠΈ (слСдим Π·Π° порядком дСлСния!).

5. Π•Π΄ΠΈΠ½ΠΈΡ†Ρƒ Π½Π° Π΄Ρ€ΠΎΠ±ΡŒ Π΄Π΅Π»ΠΈΠΌ Π² ΡƒΠΌΠ΅, просто пСрСворачивая Π΄Ρ€ΠΎΠ±ΡŒ.

Π’ΠΎΡ‚ Π²Π°ΠΌ задания, ΠΊΠΎΡ‚ΠΎΡ€Ρ‹Π΅ Π½ΡƒΠΆΠ½ΠΎ ΠΎΠ±ΡΠ·Π°Ρ‚Π΅Π»ΡŒΠ½ΠΎ ΠΏΡ€ΠΎΡ€Π΅ΡˆΠ°Ρ‚ΡŒ. ΠžΡ‚Π²Π΅Ρ‚Ρ‹ Π΄Π°Π½Ρ‹ послС всСх Π·Π°Π΄Π°Π½ΠΈΠΉ. Π˜ΡΠΏΠΎΠ»ΡŒΠ·ΡƒΠΉΡ‚Π΅ ΠΌΠ°Ρ‚Π΅Ρ€ΠΈΠ°Π»Ρ‹ этой Ρ‚Π΅ΠΌΡ‹ ΠΈ практичСскиС совСты. ΠŸΡ€ΠΈΠΊΠΈΠ½ΡŒΡ‚Π΅, сколько ΠΏΡ€ΠΈΠΌΠ΅Ρ€ΠΎΠ² Π²Ρ‹ смогли Ρ€Π΅ΡˆΠΈΡ‚ΡŒ ΠΏΡ€Π°Π²ΠΈΠ»ΡŒΠ½ΠΎ. Π‘ ΠΏΠ΅Ρ€Π²ΠΎΠ³ΠΎ Ρ€Π°Π·Π°! Π‘Π΅Π· ΠΊΠ°Π»ΡŒΠΊΡƒΠ»ΡΡ‚ΠΎΡ€Π°! И сдСлайтС Π²Π΅Ρ€Π½Ρ‹Π΅ Π²Ρ‹Π²ΠΎΠ΄Ρ‹…

ΠŸΠΎΠΌΠ½ΠΈΡ‚Π΅ – ΠΏΡ€Π°Π²ΠΈΠ»ΡŒΠ½Ρ‹ΠΉ ΠΎΡ‚Π²Π΅Ρ‚, ΠΏΠΎΠ»ΡƒΡ‡Π΅Π½Π½Ρ‹ΠΉ со Π²Ρ‚ΠΎΡ€ΠΎΠ³ΠΎ (Ρ‚Π΅ΠΌ Π±ΠΎΠ»Π΅Π΅ – Ρ‚Ρ€Π΅Ρ‚ΡŒΠ΅Π³ΠΎ) Ρ€Π°Π·Π° – Π½Π΅ считаСтся! Π’Π°ΠΊΠΎΠ²Π° суровая Тизнь.

Π˜Ρ‚Π°ΠΊ, Ρ€Π΅ΡˆΠ°Π΅ΠΌ Π² Ρ€Π΅ΠΆΠΈΠΌΠ΅ экзамСна ! Π­Ρ‚ΠΎ ΡƒΠΆΠ΅ ΠΏΠΎΠ΄Π³ΠΎΡ‚ΠΎΠ²ΠΊΠ° ΠΊ Π•Π“Π­, ΠΌΠ΅ΠΆΠ΄Ρƒ ΠΏΡ€ΠΎΡ‡ΠΈΠΌ. РСшаСм ΠΏΡ€ΠΈΠΌΠ΅Ρ€, провСряСм, Ρ€Π΅ΡˆΠ°Π΅ΠΌ ΡΠ»Π΅Π΄ΡƒΡŽΡ‰ΠΈΠΉ. РСшили всС — ΠΏΡ€ΠΎΠ²Π΅Ρ€ΠΈΠ»ΠΈ снова с ΠΏΠ΅Ρ€Π²ΠΎΠ³ΠΎ ΠΏΠΎ послСдний. И Ρ‚ΠΎΠ»ΡŒΠΊΠΎ ΠΏΠΎΡ‚ΠΎΠΌ смотрим ΠΎΡ‚Π²Π΅Ρ‚Ρ‹.

Π’Ρ‹Ρ‡ΠΈΡΠ»ΠΈΡ‚ΡŒ:

ΠŸΠΎΡ€Π΅ΡˆΠ°Π»ΠΈ?

Π˜Ρ‰Π΅ΠΌ ΠΎΡ‚Π²Π΅Ρ‚Ρ‹, ΠΊΠΎΡ‚ΠΎΡ€Ρ‹Π΅ ΡΠΎΠ²ΠΏΠ°Π΄Π°ΡŽΡ‚ с вашими. Π― ΡΠΏΠ΅Ρ†ΠΈΠ°Π»ΡŒΠ½ΠΎ ΠΈΡ… Π² бСспорядкС записал, подальшС ΠΎΡ‚ соблазна, Ρ‚Π°ΠΊ ΡΠΊΠ°Π·Π°Ρ‚ΡŒ… Π’ΠΎΡ‚ ΠΎΠ½ΠΈ, ΠΎΡ‚Π²Π΅Ρ‚Ρ‹, Ρ‡Π΅Ρ€Π΅Π· Ρ‚ΠΎΡ‡ΠΊΡƒ с запятой записаны.

0; 17/22; 3/4; 2/5; 1; 25.

А Ρ‚Π΅ΠΏΠ΅Ρ€ΡŒ Π΄Π΅Π»Π°Π΅ΠΌ Π²Ρ‹Π²ΠΎΠ΄Ρ‹. Если всё ΠΏΠΎΠ»ΡƒΡ‡ΠΈΠ»ΠΎΡΡŒ — Ρ€Π°Π΄ Π·Π° вас! Π­Π»Π΅ΠΌΠ΅Π½Ρ‚Π°Ρ€Π½Ρ‹Π΅ вычислСния с дробями — Π½Π΅ ваша ΠΏΡ€ΠΎΠ±Π»Π΅ΠΌΠ°! МоТно Π·Π°Π½ΡΡ‚ΡŒΡΡ Π±ΠΎΠ»Π΅Π΅ ΡΠ΅Ρ€ΡŒΡ‘Π·Π½Ρ‹ΠΌΠΈ Π²Π΅Ρ‰Π°ΠΌΠΈ. Если Π½Π΅Ρ‚…

Π—Π½Π°Ρ‡ΠΈΡ‚, Ρƒ вас ΠΎΠ΄Π½Π° ΠΈΠ· Π΄Π²ΡƒΡ… ΠΏΡ€ΠΎΠ±Π»Π΅ΠΌ. Или ΠΎΠ±Π΅ сразу.) НСхватка Π·Π½Π°Π½ΠΈΠΉ ΠΈ (ΠΈΠ»ΠΈ) Π½Π΅Π²Π½ΠΈΠΌΠ°Ρ‚Π΅Π»ΡŒΠ½ΠΎΡΡ‚ΡŒ. Но… Π­Ρ‚ΠΎ Ρ€Π΅ΡˆΠ°Π΅ΠΌΡ‹Π΅ ΠΏΡ€ΠΎΠ±Π»Π΅ΠΌΡ‹.

Если Π’Π°ΠΌ нравится этот сайт…

ΠšΡΡ‚Π°Ρ‚ΠΈ, Ρƒ мСня Π΅ΡΡ‚ΡŒ Π΅Ρ‰Ρ‘ ΠΏΠ°Ρ€ΠΎΡ‡ΠΊΠ° интСрСсных сайтов для Вас.)

МоТно ΠΏΠΎΡ‚Ρ€Π΅Π½ΠΈΡ€ΠΎΠ²Π°Ρ‚ΡŒΡΡ Π² Ρ€Π΅ΡˆΠ΅Π½ΠΈΠΈ ΠΏΡ€ΠΈΠΌΠ΅Ρ€ΠΎΠ² ΠΈ ΡƒΠ·Π½Π°Ρ‚ΡŒ свой ΡƒΡ€ΠΎΠ²Π΅Π½ΡŒ. ВСстированиС с ΠΌΠ³Π½ΠΎΠ²Π΅Π½Π½ΠΎΠΉ ΠΏΡ€ΠΎΠ²Π΅Ρ€ΠΊΠΎΠΉ. Учимся — с интСрСсом!)

ΠΌΠΎΠΆΠ½ΠΎ ΠΏΠΎΠ·Π½Π°ΠΊΠΎΠΌΠΈΡ‚ΡŒΡΡ с функциями ΠΈ ΠΏΡ€ΠΎΠΈΠ·Π²ΠΎΠ΄Π½Ρ‹ΠΌΠΈ.

ΠžΠ±Ρ‹ΠΊΠ½ΠΎΠ²Π΅Π½Π½Ρ‹Π΅ Π΄Ρ€ΠΎΠ±Π½Ρ‹Π΅ числа Π²ΠΏΠ΅Ρ€Π²Ρ‹Π΅ Π²ΡΡ‚Ρ€Π΅Ρ‡Π°ΡŽΡ‚ школьников Π² 5 классС ΠΈ ΡΠΎΠΏΡ€ΠΎΠ²ΠΎΠΆΠ΄Π°ΡŽΡ‚ ΠΈΡ… Π½Π° протяТСнии всСй ΠΆΠΈΠ·Π½ΠΈ, Ρ‚Π°ΠΊ ΠΊΠ°ΠΊ Π² Π±Ρ‹Ρ‚Ρƒ Π·Π°Ρ‡Π°ΡΡ‚ΡƒΡŽ трСбуСтся Ρ€Π°ΡΡΠΌΠ°Ρ‚Ρ€ΠΈΠ²Π°Ρ‚ΡŒ ΠΈΠ»ΠΈ ΠΈΡΠΏΠΎΠ»ΡŒΠ·ΠΎΠ²Π°Ρ‚ΡŒ ΠΊΠ°ΠΊΠΎΠΉ-Ρ‚ΠΎ ΠΎΠ±ΡŠΠ΅ΠΊΡ‚ Π½Π΅ Ρ†Π΅Π»ΠΈΠΊΠΎΠΌ, Π° ΠΎΡ‚Π΄Π΅Π»ΡŒΠ½Ρ‹ΠΌΠΈ кусками. Начало изучСния этой Ρ‚Π΅ΠΌΡ‹ — Π΄ΠΎΠ»ΠΈ. Π”ΠΎΠ»ΠΈ — это Ρ€Π°Π²Π½Ρ‹Π΅ части , Π½Π° ΠΊΠΎΡ‚ΠΎΡ€Ρ‹Π΅ Ρ€Π°Π·Π΄Π΅Π»Π΅Π½ Ρ‚ΠΎΡ‚ ΠΈΠ»ΠΈ ΠΈΠ½ΠΎΠΉ ΠΏΡ€Π΅Π΄ΠΌΠ΅Ρ‚. Π’Π΅Π΄ΡŒ Π½Π΅ всСгда получаСтся Π²Ρ‹Ρ€Π°Π·ΠΈΡ‚ΡŒ, допустим, Π΄Π»ΠΈΠ½Ρƒ ΠΈΠ»ΠΈ Ρ†Π΅Π½Ρƒ Ρ‚ΠΎΠ²Π°Ρ€Π° Ρ†Π΅Π»Ρ‹ΠΌ числом, слСдуСт ΠΏΡ€ΠΈΠ½ΡΡ‚ΡŒ Π²ΠΎ Π²Π½ΠΈΠΌΠ°Π½ΠΈΠ΅ части ΠΈΠ»ΠΈ Π΄ΠΎΠ»ΠΈ ΠΊΠ°ΠΊΠΎΠΉ-Π»ΠΈΠ±ΠΎ ΠΌΠ΅Ρ€Ρ‹. ΠžΠ±Ρ€Π°Π·ΠΎΠ²Π°Π½Π½ΠΎΠ΅ ΠΎΡ‚ Π³Π»Π°Π³ΠΎΠ»Π° Β«Π΄Ρ€ΠΎΠ±ΠΈΡ‚ΡŒΒ» — Ρ€Π°Π·Π΄Π΅Π»ΡΡ‚ΡŒ Π½Π° части, ΠΈ имСя арабскиС ΠΊΠΎΡ€Π½ΠΈ, Π² VIII Π²Π΅ΠΊΠ΅ Π²ΠΎΠ·Π½ΠΈΠΊΠ»ΠΎ само слово Β«Π΄Ρ€ΠΎΠ±ΡŒΒ» Π² русском языкС.

Π’ΠΊΠΎΠ½Ρ‚Π°ΠΊΡ‚Π΅

Π”Ρ€ΠΎΠ±Π½Ρ‹Π΅ выраТСния ΠΏΡ€ΠΎΠ΄ΠΎΠ»ΠΆΠΈΡ‚Π΅Π»ΡŒΠ½ΠΎΠ΅ врСмя считали самым слоТным Ρ€Π°Π·Π΄Π΅Π»ΠΎΠΌ ΠΌΠ°Ρ‚Π΅ΠΌΠ°Ρ‚ΠΈΠΊΠΈ. Π’ XVII Π²Π΅ΠΊΠ΅, ΠΏΡ€ΠΈ появлСнии ΠΏΠ΅Ρ€Π²ΠΎΡƒΡ‡Π΅Π±Π½ΠΈΠΊΠΎΠ² ΠΏΠΎ ΠΌΠ°Ρ‚Π΅ΠΌΠ°Ρ‚ΠΈΠΊΠ΅, ΠΈΡ… Π½Π°Π·Ρ‹Π²Π°Π»ΠΈ Β«Π»ΠΎΠΌΠ°Π½Ρ‹Π΅ числа», Ρ‡Ρ‚ΠΎ ΠΎΡ‡Π΅Π½ΡŒ слоТно ΠΎΡ‚ΠΎΠ±Ρ€Π°ΠΆΠ°Π»ΠΎΡΡŒ Π² ΠΏΠΎΠ½ΠΈΠΌΠ°Π½ΠΈΠΈ людСй.

Π‘ΠΎΠ²Ρ€Π΅ΠΌΠ΅Π½Π½ΠΎΠΌΡƒ Π²ΠΈΠ΄Ρƒ простых Π΄Ρ€ΠΎΠ±Π½Ρ‹Ρ… остатков, части ΠΊΠΎΡ‚ΠΎΡ€Ρ‹Ρ… Ρ€Π°Π·Π΄Π΅Π»Π΅Π½Ρ‹ ΠΈΠΌΠ΅Π½Π½ΠΎ Π³ΠΎΡ€ΠΈΠ·ΠΎΠ½Ρ‚Π°Π»ΡŒΠ½ΠΎΠΉ Ρ‡Π΅Ρ€Ρ‚ΠΎΠΉ, Π²ΠΏΠ΅Ρ€Π²Ρ‹Π΅ поспособствовал Π€ΠΈΠ±ΠΎΠ½Π°Ρ‡Ρ‡ΠΈ — Π›Π΅ΠΎΠ½Π°Ρ€Π΄ΠΎ Пизанский. Π•Π³ΠΎ Ρ‚Ρ€ΡƒΠ΄Ρ‹ Π΄Π°Ρ‚ΠΈΡ€ΠΎΠ²Π°Π½Ρ‹ Π² 1202 Π³ΠΎΠ΄Ρƒ. Но Ρ†Π΅Π»ΡŒ этой ΡΡ‚Π°Ρ‚ΡŒΠΈ — просто ΠΈ понятно ΠΎΠ±ΡŠΡΡΠ½ΠΈΡ‚ΡŒ Ρ‡ΠΈΡ‚Π°Ρ‚Π΅Π»ΡŽ, ΠΊΠ°ΠΊ происходит ΡƒΠΌΠ½ΠΎΠΆΠ΅Π½ΠΈΠ΅ ΡΠΌΠ΅ΡˆΠ°Π½Π½Ρ‹Ρ… Π΄Ρ€ΠΎΠ±Π΅ΠΉ с Ρ€Π°Π·Π½Ρ‹ΠΌΠΈ знамСнатСлями.

Π£ΠΌΠ½ΠΎΠΆΠ΅Π½ΠΈΠ΅ Π΄Ρ€ΠΎΠ±Π΅ΠΉ с Ρ€Π°Π·Π½Ρ‹ΠΌΠΈ знамСнатСлями

Π˜Π·Π½Π°Ρ‡Π°Π»ΡŒΠ½ΠΎ стоит ΠΎΠΏΡ€Π΅Π΄Π΅Π»ΠΈΡ‚ΡŒ разновидности Π΄Ρ€ΠΎΠ±Π΅ΠΉ :

  • ΠΏΡ€Π°Π²ΠΈΠ»ΡŒΠ½Ρ‹Π΅;
  • Π½Π΅ΠΏΡ€Π°Π²ΠΈΠ»ΡŒΠ½Ρ‹Π΅;
  • ΡΠΌΠ΅ΡˆΠ°Π½Π½Ρ‹Π΅.

Π”Π°Π»Π΅Π΅ Π½ΡƒΠΆΠ½ΠΎ Π²ΡΠΏΠΎΠΌΠ½ΠΈΡ‚ΡŒ, ΠΊΠ°ΠΊ происходит ΡƒΠΌΠ½ΠΎΠΆΠ΅Π½ΠΈΠ΅ Π΄Ρ€ΠΎΠ±Π½Ρ‹Ρ… чисСл с ΠΎΠ΄ΠΈΠ½Π°ΠΊΠΎΠ²Ρ‹ΠΌΠΈ знамСнатСлями. Π‘Π°ΠΌΠΎ ΠΏΡ€Π°Π²ΠΈΠ»ΠΎ этого процСсса нСслоТно ΡΡ„ΠΎΡ€ΠΌΡƒΠ»ΠΈΡ€ΠΎΠ²Π°Ρ‚ΡŒ ΡΠ°ΠΌΠΎΡΡ‚ΠΎΡΡ‚Π΅Π»ΡŒΠ½ΠΎ: Ρ€Π΅Π·ΡƒΠ»ΡŒΡ‚Π°Ρ‚ΠΎΠΌ умноТСния простых Π΄Ρ€ΠΎΠ±Π΅ΠΉ с ΠΎΠ΄ΠΈΠ½Π°ΠΊΠΎΠ²Ρ‹ΠΌΠΈ знамСнатСлями являСтся Π΄Ρ€ΠΎΠ±Π½ΠΎΠ΅ Π²Ρ‹Ρ€Π°ΠΆΠ΅Π½ΠΈΠ΅, Ρ‡ΠΈΡΠ»ΠΈΡ‚Π΅Π»ΡŒ ΠΊΠΎΡ‚ΠΎΡ€ΠΎΠΉ Π΅ΡΡ‚ΡŒ ΠΏΡ€ΠΎΠΈΠ·Π²Π΅Π΄Π΅Π½ΠΈΠ΅ числитСлСй, Π° Π·Π½Π°ΠΌΠ΅Π½Π°Ρ‚Π΅Π»ΡŒ — ΠΏΡ€ΠΎΠΈΠ·Π²Π΅Π΄Π΅Π½ΠΈΠ΅ Π·Π½Π°ΠΌΠ΅Π½Π°Ρ‚Π΅Π»Π΅ΠΉ Π΄Π°Π½Π½Ρ‹Ρ… Π΄Ρ€ΠΎΠ±Π΅ΠΉ. Π’ΠΎ Π΅ΡΡ‚ΡŒ, ΠΏΠΎ сути, Π½ΠΎΠ²Ρ‹ΠΉ Π·Π½Π°ΠΌΠ΅Π½Π°Ρ‚Π΅Π»ΡŒ Π΅ΡΡ‚ΡŒ ΠΊΠ²Π°Π΄Ρ€Π°Ρ‚ ΠΎΠ΄Π½ΠΎΠ³ΠΎ ΠΈΠ· ΡΡƒΡ‰Π΅ΡΡ‚Π²ΡƒΡŽΡ‰ΠΈΡ… ΠΈΠ·Π½Π°Ρ‡Π°Π»ΡŒΠ½ΠΎ.

ΠŸΡ€ΠΈ ΡƒΠΌΠ½ΠΎΠΆΠ΅Π½ΠΈΠΈ простых Π΄Ρ€ΠΎΠ±Π΅ΠΉ с Ρ€Π°Π·Π½Ρ‹ΠΌΠΈ знамСнатСлями для Π΄Π²ΡƒΡ… ΠΈ Π±ΠΎΠ»Π΅Π΅ ΠΌΠ½ΠΎΠΆΠΈΡ‚Π΅Π»Π΅ΠΉ ΠΏΡ€Π°Π²ΠΈΠ»ΠΎ Π½Π΅ мСняСтся:

a/ b * c/ d = a*c / b*d.

ЕдинствСнноС ΠΎΡ‚Π»ΠΈΡ‡ΠΈΠ΅ Π² Ρ‚ΠΎΠΌ, Ρ‡Ρ‚ΠΎ ΠΎΠ±Ρ€Π°Π·ΠΎΠ²Π°Π½Π½ΠΎΠ΅ число ΠΏΠΎΠ΄ Π΄Ρ€ΠΎΠ±Π½ΠΎΠΉ Ρ‡Π΅Ρ€Ρ‚ΠΎΠΉ Π±ΡƒΠ΄Π΅Ρ‚ ΠΏΡ€ΠΎΠΈΠ·Π²Π΅Π΄Π΅Π½ΠΈΠ΅ΠΌ Ρ€Π°Π·Π½Ρ‹Ρ… чисСл ΠΈ, СстСствСнно, ΠΊΠ²Π°Π΄Ρ€Π°Ρ‚ΠΎΠΌ ΠΎΠ΄Π½ΠΎΠ³ΠΎ числового выраТСния Π΅Π³ΠΎ Π½Π°Π·Π²Π°Ρ‚ΡŒ Π½Π΅Π²ΠΎΠ·ΠΌΠΎΠΆΠ½ΠΎ.

Π‘Ρ‚ΠΎΠΈΡ‚ Ρ€Π°ΡΡΠΌΠΎΡ‚Ρ€Π΅Ρ‚ΡŒ ΡƒΠΌΠ½ΠΎΠΆΠ΅Π½ΠΈΠ΅ Π΄Ρ€ΠΎΠ±Π΅ΠΉ с Ρ€Π°Π·Π½Ρ‹ΠΌΠΈ знамСнатСлями Π½Π° ΠΏΡ€ΠΈΠΌΠ΅Ρ€Π°Ρ…:

  • 8/ 9 * 6/ 7 = 8*6 / 9*7 = 48/ 63 = 16/2 1 ;
  • 4/ 6 * 3/ 7 = 2/ 3 * 3/7 2*3 / 3*7 = 6/ 21 .

Π’ ΠΏΡ€ΠΈΠΌΠ΅Ρ€Π°Ρ… ΠΏΡ€ΠΈΠΌΠ΅Π½ΡΡŽΡ‚ΡΡ способы сокращСния Π΄Ρ€ΠΎΠ±Π½Ρ‹Ρ… Π²Ρ‹Ρ€Π°ΠΆΠ΅Π½ΠΈΠΉ. МоТно ΡΠΎΠΊΡ€Π°Ρ‰Π°Ρ‚ΡŒ Ρ‚ΠΎΠ»ΡŒΠΊΠΎ числа числитСля с числами знамСнатСля, рядом стоящиС ΠΌΠ½ΠΎΠΆΠΈΡ‚Π΅Π»ΠΈ Π½Π°Π΄ Π΄Ρ€ΠΎΠ±Π½ΠΎΠΉ Ρ‡Π΅Ρ€Ρ‚ΠΎΠΉ ΠΈΠ»ΠΈ ΠΏΠΎΠ΄ Π½Π΅ΠΉ ΡΠΎΠΊΡ€Π°Ρ‰Π°Ρ‚ΡŒ нСльзя.

Наряду с простыми Π΄Ρ€ΠΎΠ±Π½Ρ‹ΠΌΠΈ числами, сущСствуСт понятиС ΡΠΌΠ΅ΡˆΠ°Π½Π½Ρ‹Ρ… Π΄Ρ€ΠΎΠ±Π΅ΠΉ. БмСшанноС число состоит ΠΈΠ· Ρ†Π΅Π»ΠΎΠ³ΠΎ числа ΠΈ Π΄Ρ€ΠΎΠ±Π½ΠΎΠΉ части, Ρ‚ΠΎ Π΅ΡΡ‚ΡŒ являСтся суммой этих чисСл:

1 4/ 11 =1 + 4/ 11.

Как происходит ΠΏΠ΅Ρ€Π΅ΠΌΠ½ΠΎΠΆΠ΅Π½ΠΈΠ΅

ΠŸΡ€Π΅Π΄Π»Π°Π³Π°Π΅Ρ‚ΡΡ нСсколько ΠΏΡ€ΠΈΠΌΠ΅Ρ€ΠΎΠ² для рассмотрСния.

2 1/ 2 * 7 3/ 5 = 2 + 1/ 2 * 7 + 3/ 5 = 2*7 + 2* 3/ 5 + 1/ 2 * 7 + 1/ 2 * 3/ 5 = 14 + 6/5 + 7/ 2 + 3/ 10 = 14 + 12/ 10 + 35/ 10 + 3/ 10 = 14 + 50/ 10 = 14 + 5=19.

Π’ ΠΏΡ€ΠΈΠΌΠ΅Ρ€Π΅ ΠΈΡΠΏΠΎΠ»ΡŒΠ·ΡƒΠ΅Ρ‚ΡΡ ΡƒΠΌΠ½ΠΎΠΆΠ΅Π½ΠΈΠ΅ числа Π½Π° ΠΎΠ±Ρ‹ΠΊΠ½ΠΎΠ²Π΅Π½Π½ΡƒΡŽ Π΄Ρ€ΠΎΠ±Π½ΡƒΡŽ Ρ‡Π°ΡΡ‚ΡŒ , Π·Π°ΠΏΠΈΡΠ°Ρ‚ΡŒ ΠΏΡ€Π°Π²ΠΈΠ»ΠΎ для этого дСйствия ΠΌΠΎΠΆΠ½ΠΎ Ρ„ΠΎΡ€ΠΌΡƒΠ»ΠΎΠΉ:

a * b/ c = a*b / c.

По сути, Ρ‚Π°ΠΊΠΎΠ΅ ΠΏΡ€ΠΎΠΈΠ·Π²Π΅Π΄Π΅Π½ΠΈΠ΅ Π΅ΡΡ‚ΡŒ сумма ΠΎΠ΄ΠΈΠ½Π°ΠΊΠΎΠ²Ρ‹Ρ… Π΄Ρ€ΠΎΠ±Π½Ρ‹Ρ… остатков, Π° количСство слагаСмых ΡƒΠΊΠ°Π·Ρ‹Π²Π°Π΅Ρ‚ это Π½Π°Ρ‚ΡƒΡ€Π°Π»ΡŒΠ½ΠΎΠ΅ число. Частный случай:

4 * 12/ 15 = 12/ 15 + 12/ 15 + 12/ 15 + 12/ 15 = 48/ 15 = 3 1/ 5.

БущСствуСт Π΅Ρ‰Π΅ ΠΎΠ΄ΠΈΠ½ Π²Π°Ρ€ΠΈΠ°Π½Ρ‚ Ρ€Π΅ΡˆΠ΅Π½ΠΈΡ умноТСния числа Π½Π° Π΄Ρ€ΠΎΠ±Π½Ρ‹ΠΉ остаток. Π‘Ρ‚ΠΎΠΈΡ‚ просто Ρ€Π°Π·Π΄Π΅Π»ΠΈΡ‚ΡŒ Π·Π½Π°ΠΌΠ΅Π½Π°Ρ‚Π΅Π»ΡŒ Π½Π° это число:

d * e/ f = e/ f: d.

Π­Ρ‚ΠΈΠΌ ΠΏΡ€ΠΈΠ΅ΠΌΠΎΠΌ ΠΏΠΎΠ»Π΅Π·Π½ΠΎ ΠΏΠΎΠ»ΡŒΠ·ΠΎΠ²Π°Ρ‚ΡŒΡΡ, ΠΊΠΎΠ³Π΄Π° Π·Π½Π°ΠΌΠ΅Π½Π°Ρ‚Π΅Π»ΡŒ дСлится Π½Π° Π½Π°Ρ‚ΡƒΡ€Π°Π»ΡŒΠ½ΠΎΠ΅ число Π±Π΅Π· остатка ΠΈΠ»ΠΈ, ΠΊΠ°ΠΊ говорится, Π½Π°Ρ†Π΅Π»ΠΎ.

ΠŸΠ΅Ρ€Π΅Π²Π΅ΡΡ‚ΠΈ ΡΠΌΠ΅ΡˆΠ°Π½Π½Ρ‹Π΅ числа Π² Π½Π΅ΠΏΡ€Π°Π²ΠΈΠ»ΡŒΠ½Ρ‹Π΅ Π΄Ρ€ΠΎΠ±ΠΈ ΠΈ ΠΏΠΎΠ»ΡƒΡ‡ΠΈΡ‚ΡŒ ΠΏΡ€ΠΎΠΈΠ·Π²Π΅Π΄Π΅Π½ΠΈΠ΅ Ρ€Π°Π½Π΅Π΅ описанным способом:

1 2/ 3 * 4 1/ 5 = 5/ 3 * 21/ 5 = 5*21 / 3*5 =7.

Π’ этом ΠΏΡ€ΠΈΠΌΠ΅Ρ€Π΅ участвуСт способ прСдставлСния смСшанной Π΄Ρ€ΠΎΠ±ΠΈ Π² Π½Π΅ΠΏΡ€Π°Π²ΠΈΠ»ΡŒΠ½ΡƒΡŽ, Π΅Π³ΠΎ Ρ‚Π°ΠΊΠΆΠ΅ ΠΌΠΎΠΆΠ½ΠΎ ΠΏΡ€Π΅Π΄ΡΡ‚Π°Π²ΠΈΡ‚ΡŒ Π² Π²ΠΈΠ΄Π΅ ΠΎΠ±Ρ‰Π΅ΠΉ Ρ„ΠΎΡ€ΠΌΡƒΠ»Ρ‹:

a b c = a * b + c / c, Π³Π΄Π΅ Π·Π½Π°ΠΌΠ΅Π½Π°Ρ‚Π΅Π»ΡŒ Π½ΠΎΠ²ΠΎΠΉ Π΄Ρ€ΠΎΠ±ΠΈ образуСтся ΠΏΡ€ΠΈ ΡƒΠΌΠ½ΠΎΠΆΠ΅Π½ΠΈΠΈ Ρ†Π΅Π»ΠΎΠΉ части со Π·Π½Π°ΠΌΠ΅Π½Π°Ρ‚Π΅Π»Π΅ΠΌ ΠΈ ΠΏΡ€ΠΈ слоТСнии Π΅Π³ΠΎ с числитСлСм исходного Π΄Ρ€ΠΎΠ±Π½ΠΎΠ³ΠΎ остатка, Π° Π·Π½Π°ΠΌΠ΅Π½Π°Ρ‚Π΅Π»ΡŒ остаСтся ΠΏΡ€Π΅ΠΆΠ½ΠΈΠΌ.

Π­Ρ‚ΠΎΡ‚ процСсс Ρ€Π°Π±ΠΎΡ‚Π°Π΅Ρ‚ ΠΈ Π² ΠΎΠ±Ρ€Π°Ρ‚Π½ΡƒΡŽ сторону. Для выдСлСния Ρ†Π΅Π»ΠΎΠΉ части ΠΈ Π΄Ρ€ΠΎΠ±Π½ΠΎΠ³ΠΎ остатка Π½ΡƒΠΆΠ½ΠΎ ΠΏΠΎΠ΄Π΅Π»ΠΈΡ‚ΡŒ Ρ‡ΠΈΡΠ»ΠΈΡ‚Π΅Π»ΡŒ Π½Π΅ΠΏΡ€Π°Π²ΠΈΠ»ΡŒΠ½ΠΎΠΉ Π΄Ρ€ΠΎΠ±ΠΈ Π½Π° Π΅Π΅ Π·Π½Π°ΠΌΠ΅Π½Π°Ρ‚Π΅Π»ΡŒ Β«ΡƒΠ³ΠΎΠ»ΠΊΠΎΠΌΒ».

Π£ΠΌΠ½ΠΎΠΆΠ΅Π½ΠΈΠ΅ Π½Π΅ΠΏΡ€Π°Π²ΠΈΠ»ΡŒΠ½Ρ‹Ρ… Π΄Ρ€ΠΎΠ±Π΅ΠΉ производят общСпринятым способом. Когда запись ΠΈΠ΄Π΅Ρ‚ ΠΏΠΎΠ΄ Π΅Π΄ΠΈΠ½ΠΎΠΉ Π΄Ρ€ΠΎΠ±Π½ΠΎΠΉ Ρ‡Π΅Ρ€Ρ‚ΠΎΠΉ, ΠΏΠΎ ΠΌΠ΅Ρ€Π΅ нСобходимости Π½ΡƒΠΆΠ½ΠΎ ΡΠ΄Π΅Π»Π°Ρ‚ΡŒ сокращСниС Π΄Ρ€ΠΎΠ±Π΅ΠΉ, Ρ‡Ρ‚ΠΎΠ±Ρ‹ ΡƒΠΌΠ΅Π½ΡŒΡˆΠΈΡ‚ΡŒ Ρ‚Π°ΠΊΠΈΠΌ ΠΌΠ΅Ρ‚ΠΎΠ΄ΠΎΠΌ числа ΠΈ ΠΏΡ€ΠΎΡ‰Π΅ ΠΏΠΎΡΡ‡ΠΈΡ‚Π°Ρ‚ΡŒ Ρ€Π΅Π·ΡƒΠ»ΡŒΡ‚Π°Ρ‚.

Π’ ΠΈΠ½Ρ‚Π΅Ρ€Π½Π΅Ρ‚Π΅ сущСствуСт мноТСство ΠΏΠΎΠΌΠΎΡ‰Π½ΠΈΠΊΠΎΠ², Ρ‡Ρ‚ΠΎΠ±Ρ‹ Ρ€Π΅ΡˆΠ°Ρ‚ΡŒ Π΄Π°ΠΆΠ΅ слоТныС матСматичСскиС Π·Π°Π΄Π°Ρ‡ΠΈ Π² Ρ€Π°Π·Π»ΠΈΡ‡Π½Ρ‹Ρ… вариациях ΠΏΡ€ΠΎΠ³Ρ€Π°ΠΌΠΌ. ДостаточноС количСство Ρ‚Π°ΠΊΠΈΡ… сСрвисов ΠΏΡ€Π΅Π΄Π»Π°Π³Π°ΡŽΡ‚ свою ΠΏΠΎΠΌΠΎΡ‰ΡŒ ΠΏΡ€ΠΈ счСтС умноТСния Π΄Ρ€ΠΎΠ±Π΅ΠΉ с Ρ€Π°Π·Π½Ρ‹ΠΌΠΈ числами Π² знамСнатСлях — Ρ‚Π°ΠΊ Π½Π°Π·Ρ‹Π²Π°Π΅ΠΌΡ‹Π΅ ΠΎΠ½Π»Π°ΠΉΠ½-ΠΊΠ°Π»ΡŒΠΊΡƒΠ»ΡΡ‚ΠΎΡ€Ρ‹ для расчСта Π΄Ρ€ΠΎΠ±Π΅ΠΉ. Они способны Π½Π΅ Ρ‚ΠΎΠ»ΡŒΠΊΠΎ ΡƒΠΌΠ½ΠΎΠΆΠΈΡ‚ΡŒ, Π½ΠΎ ΠΈ произвСсти всС ΠΎΡΡ‚Π°Π»ΡŒΠ½Ρ‹Π΅ ΠΏΡ€ΠΎΡΡ‚Π΅ΠΉΡˆΠΈΠ΅ арифмСтичСскиС ΠΎΠΏΠ΅Ρ€Π°Ρ†ΠΈΠΈ с ΠΎΠ±Ρ‹ΠΊΠ½ΠΎΠ²Π΅Π½Π½Ρ‹ΠΌΠΈ дробями ΠΈ ΡΠΌΠ΅ΡˆΠ°Π½Π½Ρ‹ΠΌΠΈ числами. Π Π°Π±ΠΎΡ‚Π°Ρ‚ΡŒ с Π½ΠΈΠΌ нСслоТно, Π½Π° страницС сайта Π·Π°ΠΏΠΎΠ»Π½ΡΡŽΡ‚ΡΡ ΡΠΎΠΎΡ‚Π²Π΅Ρ‚ΡΡ‚Π²ΡƒΡŽΡ‰ΠΈΠ΅ поля, выбираСтся Π·Π½Π°ΠΊ матСматичСского дСйствия ΠΈ наТимаСтся Β«Π²Ρ‹Ρ‡ΠΈΡΠ»ΠΈΡ‚ΡŒΒ». ΠŸΡ€ΠΎΠ³Ρ€Π°ΠΌΠΌΠ° считаСт автоматичСски.

Π’Π΅ΠΌΠ° арифмСтичСских дСйствий с Π΄Ρ€ΠΎΠ±Π½Ρ‹ΠΌΠΈ числами Π°ΠΊΡ‚ΡƒΠ°Π»ΡŒΠ½Π° Π½Π° всСм протяТСнии обучСния школьников срСднСго ΠΈ ΡΡ‚Π°Ρ€ΡˆΠ΅Π³ΠΎ Π·Π²Π΅Π½Π°. Π’ ΡΡ‚Π°Ρ€ΡˆΠΈΡ… классах Ρ€Π°ΡΡΠΌΠ°Ρ‚Ρ€ΠΈΠ²Π°ΡŽΡ‚ ΡƒΠΆΠ΅ Π½Π΅ ΠΏΡ€ΠΎΡΡ‚Π΅ΠΉΡˆΠΈΠ΅ Π²ΠΈΠ΄Ρ‹, Π° Ρ†Π΅Π»Ρ‹Π΅ Π΄Ρ€ΠΎΠ±Π½Ρ‹Π΅ выраТСния , Π½ΠΎ знания ΠΏΡ€Π°Π²ΠΈΠ» ΠΏΠΎ ΠΏΡ€Π΅ΠΎΠ±Ρ€Π°Π·ΠΎΠ²Π°Π½ΠΈΡŽ ΠΈ расчСтам, ΠΏΠΎΠ»ΡƒΡ‡Π΅Π½Π½Ρ‹Π΅ Ρ€Π°Π½Π΅Π΅, ΠΏΡ€ΠΈΠΌΠ΅Π½ΡΡŽΡ‚ΡΡ Π² ΠΏΠ΅Ρ€Π²ΠΎΠ·Π΄Π°Π½Π½ΠΎΠΌ Π²ΠΈΠ΄Π΅. Π₯ΠΎΡ€ΠΎΡˆΠΎ усвоСнныС Π±Π°Π·ΠΎΠ²Ρ‹Π΅ знания Π΄Π°ΡŽΡ‚ ΠΏΠΎΠ»Π½ΡƒΡŽ ΡƒΠ²Π΅Ρ€Π΅Π½Π½ΠΎΡΡ‚ΡŒ Π² ΡƒΠ΄Π°Ρ‡Π½ΠΎΠΌ Ρ€Π΅ΡˆΠ΅Π½ΠΈΠΈ Π½Π°ΠΈΠ±ΠΎΠ»Π΅Π΅ слоТных Π·Π°Π΄Π°Ρ‡.

Π’ Π·Π°ΠΊΠ»ΡŽΡ‡Π΅Π½ΠΈΠ΅ ΠΈΠΌΠ΅Π΅Ρ‚ смысл привСсти слова Π›ΡŒΠ²Π° НиколаСвича Волстого, ΠΊΠΎΡ‚ΠΎΡ€Ρ‹ΠΉ писал: Β«Π§Π΅Π»ΠΎΠ²Π΅ΠΊ Π΅ΡΡ‚ΡŒ Π΄Ρ€ΠΎΠ±ΡŒ. Π£Π²Π΅Π»ΠΈΡ‡ΠΈΡ‚ΡŒ своСго числитСля — свои достоинства, — Π½Π΅ Π²ΠΎ власти Ρ‡Π΅Π»ΠΎΠ²Π΅ΠΊΠ°, Π½ΠΎ всякий ΠΌΠΎΠΆΠ΅Ρ‚ ΡƒΠΌΠ΅Π½ΡŒΡˆΠΈΡ‚ΡŒ своСго знамСнатСля — своё ΠΌΠ½Π΅Π½ΠΈΠ΅ ΠΎ самом сСбС, ΠΈ этим ΡƒΠΌΠ΅Π½ΡŒΡˆΠ΅Π½ΠΈΠ΅ΠΌ ΠΏΡ€ΠΈΠ±Π»ΠΈΠ·ΠΈΡ‚ΡŒΡΡ ΠΊ своСму ΡΠΎΠ²Π΅Ρ€ΡˆΠ΅Π½ΡΡ‚Π²ΡƒΒ».

Π‘ дробями ΠΌΠΎΠΆΠ½ΠΎ Π²Ρ‹ΠΏΠΎΠ»Π½ΡΡ‚ΡŒ всС дСйствия, Π² Ρ‚ΠΎΠΌ числС ΠΈ Π΄Π΅Π»Π΅Π½ΠΈΠ΅. Данная ΡΡ‚Π°Ρ‚ΡŒΡ ΠΏΠΎΠΊΠ°Π·Ρ‹Π²Π°Π΅Ρ‚ Π΄Π΅Π»Π΅Π½ΠΈΠ΅ ΠΎΠ±Ρ‹ΠΊΠ½ΠΎΠ²Π΅Π½Π½Ρ‹Ρ… Π΄Ρ€ΠΎΠ±Π΅ΠΉ. Π‘ΡƒΠ΄ΡƒΡ‚ Π΄Π°Π½Ρ‹ опрСдСлСния, рассмотрСны ΠΏΡ€ΠΈΠΌΠ΅Ρ€Ρ‹. ΠŸΠΎΠ΄Ρ€ΠΎΠ±Π½ΠΎ остановимся Π½Π° Π΄Π΅Π»Π΅Π½ΠΈΠΈ Π΄Ρ€ΠΎΠ±Π΅ΠΉ Π½Π° Π½Π°Ρ‚ΡƒΡ€Π°Π»ΡŒΠ½Ρ‹Π΅ числа ΠΈ Π½Π°ΠΎΠ±ΠΎΡ€ΠΎΡ‚. Π‘ΡƒΠ΄Π΅Ρ‚ рассмотрСно Π΄Π΅Π»Π΅Π½ΠΈΠ΅ ΠΎΠ±Ρ‹ΠΊΠ½ΠΎΠ²Π΅Π½Π½ΠΎΠΉ Π΄Ρ€ΠΎΠ±ΠΈ Π½Π° смСшанноС число.

Π”Π΅Π»Π΅Π½ΠΈΠ΅ ΠΎΠ±Ρ‹ΠΊΠ½ΠΎΠ²Π΅Π½Π½Ρ‹Ρ… Π΄Ρ€ΠΎΠ±Π΅ΠΉ

ДСлСния являСтся ΠΎΠ±Ρ€Π°Ρ‚Π½Ρ‹ΠΌ ΡƒΠΌΠ½ΠΎΠΆΠ΅Π½ΠΈΡŽ. ΠŸΡ€ΠΈ Π΄Π΅Π»Π΅Π½ΠΈΠΈ нСизвСстный ΠΌΠ½ΠΎΠΆΠΈΡ‚Π΅Π»ΡŒ находится ΠΏΡ€ΠΈ извСстном ΠΏΡ€ΠΎΠΈΠ·Π²Π΅Π΄Π΅Π½ΠΈΠΈ ΠΈ Π΄Ρ€ΡƒΠ³ΠΎΠ³ΠΎ мноТитСля, Π³Π΄Π΅ ΠΈ сохраняСтся Π΅Π³ΠΎ Π΄Π°Π½Π½Ρ‹ΠΉ смысл с ΠΎΠ±Ρ‹ΠΊΠ½ΠΎΠ²Π΅Π½Π½Ρ‹ΠΌΠΈ дробями.

Если Π½Π΅ΠΎΠ±Ρ…ΠΎΠ΄ΠΈΠΌΠΎ произвСсти Π΄Π΅Π»Π΅Π½ΠΈΠ΅ ΠΎΠ±Ρ‹ΠΊΠ½ΠΎΠ²Π΅Π½Π½ΠΎΠΉ Π΄Ρ€ΠΎΠ±ΠΈ a b Π½Π° c d , Ρ‚ΠΎΠ³Π΄Π° для опрСдСлСния Ρ‚Π°ΠΊΠΎΠ³ΠΎ числа Π½ΡƒΠΆΠ½ΠΎ произвСсти ΡƒΠΌΠ½ΠΎΠΆΠ΅Π½ΠΈΠ΅ Π½Π° Π΄Π΅Π»ΠΈΡ‚Π΅Π»ΡŒ c d , это даст Π² ΠΈΡ‚ΠΎΠ³Π΅ Π΄Π΅Π»ΠΈΠΌΠΎΠ΅ a b . ΠŸΠΎΠ»ΡƒΡ‡ΠΈΠΌ число ΠΈ запишСм Π΅Π³ΠΎ a b Β· d c , Π³Π΄Π΅ d c являСтся ΠΎΠ±Ρ€Π°Ρ‚Π½Ρ‹ΠΌ c d числу. РавСнства ΠΌΠΎΠΆΠ½ΠΎ Π·Π°ΠΏΠΈΡΠ°Ρ‚ΡŒ ΠΏΡ€ΠΈ ΠΏΠΎΠΌΠΎΡ‰ΠΈ свойств умноТСния, Π° ΠΈΠΌΠ΅Π½Π½ΠΎ: a b Β· d c Β· c d = a b Β· d c Β· c d = a b Β· 1 = a b , Π³Π΄Π΅ Π²Ρ‹Ρ€Π°ΠΆΠ΅Π½ΠΈΠ΅ a b Β· d c являСтся частным ΠΎΡ‚ дСлСния a b Π½Π° c d .

ΠžΡ‚ΡΡŽΠ΄Π° ΠΏΠΎΠ»ΡƒΡ‡ΠΈΠΌ ΠΈ сформулируСм ΠΏΡ€Π°Π²ΠΈΠ»ΠΎ дСлСния ΠΎΠ±Ρ‹ΠΊΠ½ΠΎΠ²Π΅Π½Π½Ρ‹Ρ… Π΄Ρ€ΠΎΠ±Π΅ΠΉ:

ΠžΠΏΡ€Π΅Π΄Π΅Π»Π΅Π½ΠΈΠ΅ 1

Π§Ρ‚ΠΎΠ±Ρ‹ Ρ€Π°Π·Π΄Π΅Π»ΠΈΡ‚ΡŒ ΠΎΠ±Ρ‹ΠΊΠ½ΠΎΠ²Π΅Π½Π½ΡƒΡŽ Π΄Ρ€ΠΎΠ±ΡŒ a b Π½Π° c d , Π½Π΅ΠΎΠ±Ρ…ΠΎΠ΄ΠΈΠΌΠΎ Π΄Π΅Π»ΠΈΠΌΠΎΠ΅ ΡƒΠΌΠ½ΠΎΠΆΠΈΡ‚ΡŒ Π½Π° число, ΠΎΠ±Ρ€Π°Ρ‚Π½ΠΎΠ΅ Π΄Π΅Π»ΠΈΡ‚Π΅Π»ΡŽ.

Π—Π°ΠΏΠΈΡˆΠ΅ΠΌ ΠΏΡ€Π°Π²ΠΈΠ»ΠΎ Π² Π²ΠΈΠ΄Π΅ выраТСния: a b: c d = a b Β· d c

ΠŸΡ€Π°Π²ΠΈΠ»Π° дСлСния сводятся ΠΊ ΡƒΠΌΠ½ΠΎΠΆΠ΅Π½ΠΈΡŽ. Π§Ρ‚ΠΎΠ±Ρ‹ ΠΏΡ€ΠΈΠ΄Π΅Ρ€ΠΆΠΈΠ²Π°Ρ‚ΡŒΡΡ Π΅Π³ΠΎ, Π½ΡƒΠΆΠ½ΠΎ Ρ…ΠΎΡ€ΠΎΡˆΠΎ Ρ€Π°Π·Π±ΠΈΡ€Π°Ρ‚ΡŒΡΡ Π² Π²Ρ‹ΠΏΠΎΠ»Π½Π΅Π½ΠΈΠΈ умноТСния ΠΎΠ±Ρ‹ΠΊΠ½ΠΎΠ²Π΅Π½Π½Ρ‹Ρ… Π΄Ρ€ΠΎΠ±Π΅ΠΉ.

ΠŸΠ΅Ρ€Π΅ΠΉΠ΄Π΅ΠΌ ΠΊ Ρ€Π°ΡΡΠΌΠΎΡ‚Ρ€Π΅Π½ΠΈΡŽ дСлСния ΠΎΠ±Ρ‹ΠΊΠ½ΠΎΠ²Π΅Π½Π½Ρ‹Ρ… Π΄Ρ€ΠΎΠ±Π΅ΠΉ.

ΠŸΡ€ΠΈΠΌΠ΅Ρ€ 1

Π’Ρ‹ΠΏΠΎΠ»Π½ΠΈΡ‚ΡŒ Π΄Π΅Π»Π΅Π½ΠΈΠ΅ 9 7 Π½Π° 5 3 . Π Π΅Π·ΡƒΠ»ΡŒΡ‚Π°Ρ‚ Π·Π°ΠΏΠΈΡΠ°Ρ‚ΡŒ Π² Π²ΠΈΠ΄Π΅ Π΄Ρ€ΠΎΠ±ΠΈ.

РСшСниС

Число 5 3 – это обратная Π΄Ρ€ΠΎΠ±ΡŒ 3 5 . НСобходимо ΠΈΡΠΏΠΎΠ»ΡŒΠ·ΠΎΠ²Π°Ρ‚ΡŒ ΠΏΡ€Π°Π²ΠΈΠ»ΠΎ дСлСния ΠΎΠ±Ρ‹ΠΊΠ½ΠΎΠ²Π΅Π½Π½Ρ‹Ρ… Π΄Ρ€ΠΎΠ±Π΅ΠΉ. Π­Ρ‚ΠΎ Π²Ρ‹Ρ€Π°ΠΆΠ΅Π½ΠΈΠ΅ запишСм Ρ‚Π°ΠΊ: 9 7: 5 3 = 9 7 Β· 3 5 = 9 Β· 3 7 Β· 5 = 27 35 .

ΠžΡ‚Π²Π΅Ρ‚: 9 7: 5 3 = 27 35 .

ΠŸΡ€ΠΈ сокращСнии Π΄Ρ€ΠΎΠ±Π΅ΠΉ слСдуСт Π²Ρ‹Π΄Π΅Π»ΡΡ‚ΡŒ Ρ†Π΅Π»ΡƒΡŽ Ρ‡Π°ΡΡ‚ΡŒ, Ссли Ρ‡ΠΈΡΠ»ΠΈΡ‚Π΅Π»ΡŒ большС знамСнатСля.

ΠŸΡ€ΠΈΠΌΠ΅Ρ€ 2

Π Π°Π·Π΄Π΅Π»ΠΈΡ‚ΡŒ 8 15: 24 65 . ΠžΡ‚Π²Π΅Ρ‚ Π·Π°ΠΏΠΈΡΠ°Ρ‚ΡŒ Π² Π²ΠΈΠ΄Π΅ Π΄Ρ€ΠΎΠ±ΠΈ.

РСшСниС

Для Ρ€Π΅ΡˆΠ΅Π½ΠΈΡ Π½ΡƒΠΆΠ½ΠΎ ΠΏΠ΅Ρ€Π΅ΠΉΡ‚ΠΈ ΠΎΡ‚ дСлСния ΠΊ ΡƒΠΌΠ½ΠΎΠΆΠ΅Π½ΠΈΡŽ. Π—Π°ΠΏΠΈΡˆΠ΅ΠΌ это Π² Ρ‚Π°ΠΊΠΎΠΉ Ρ„ΠΎΡ€ΠΌΠ΅: 8 15: 24 65 = 2 Β· 2 Β· 2 Β· 5 Β· 13 3 Β· 5 Β· 2 Β· 2 Β· 2 Β· 3 = 13 3 Β· 3 = 13 9

НСобходимо произвСсти сокращСниС, Π° это выполняСтся ΡΠ»Π΅Π΄ΡƒΡŽΡ‰ΠΈΠΌ ΠΎΠ±Ρ€Π°Π·ΠΎΠΌ: 8 Β· 65 15 Β· 24 = 2 Β· 2 Β· 2 Β· 5 Β· 13 3 Β· 5 Β· 2 Β· 2 Β· 2 Β· 3 = 13 3 Β· 3 = 13 9

ВыдСляСм Ρ†Π΅Π»ΡƒΡŽ Ρ‡Π°ΡΡ‚ΡŒ ΠΈ ΠΏΠΎΠ»ΡƒΡ‡Π°Π΅ΠΌ 13 9 = 1 4 9 .

ΠžΡ‚Π²Π΅Ρ‚: 8 15: 24 65 = 1 4 9 .

Π”Π΅Π»Π΅Π½ΠΈΠ΅ Π½Π΅ΠΎΠ±Ρ‹ΠΊΠ½ΠΎΠ²Π΅Π½Π½ΠΎΠΉ Π΄Ρ€ΠΎΠ±ΠΈ Π½Π° Π½Π°Ρ‚ΡƒΡ€Π°Π»ΡŒΠ½ΠΎΠ΅ число

Π˜ΡΠΏΠΎΠ»ΡŒΠ·ΡƒΠ΅ΠΌ ΠΏΡ€Π°Π²ΠΈΠ»ΠΎ дСлСния Π΄Ρ€ΠΎΠ±ΠΈ Π½Π° Π½Π°Ρ‚ΡƒΡ€Π°Π»ΡŒΠ½ΠΎΠ΅ число:Ρ‡Ρ‚ΠΎΠ±Ρ‹ Ρ€Π°Π·Π΄Π΅Π»ΠΈΡ‚ΡŒ a b Π½Π° Π½Π°Ρ‚ΡƒΡ€Π°Π»ΡŒΠ½ΠΎΠ΅ число n , Π½Π΅ΠΎΠ±Ρ…ΠΎΠ΄ΠΈΠΌΠΎ ΡƒΠΌΠ½ΠΎΠΆΠΈΡ‚ΡŒ Ρ‚ΠΎΠ»ΡŒΠΊΠΎ Π·Π½Π°ΠΌΠ΅Π½Π°Ρ‚Π΅Π»ΡŒ Π½Π° n . ΠžΡ‚ΡΡŽΠ΄Π° ΠΏΠΎΠ»ΡƒΡ‡ΠΈΠΌ Π²Ρ‹Ρ€Π°ΠΆΠ΅Π½ΠΈΠ΅: a b: n = a b Β· n .

ΠŸΡ€Π°Π²ΠΈΠ»ΠΎ дСлСния являСтся слСдствиСм ΠΏΡ€Π°Π²ΠΈΠ»Π° умноТСния. ΠŸΠΎΡΡ‚ΠΎΠΌΡƒ прСдставлСниС Π½Π°Ρ‚ΡƒΡ€Π°Π»ΡŒΠ½ΠΎΠ³ΠΎ числа Π² Π²ΠΈΠ΄Π΅ Π΄Ρ€ΠΎΠ±ΠΈ даст равСнство Ρ‚Π°ΠΊΠΎΠ³ΠΎ Ρ‚ΠΈΠΏΠ°: a b: n = a b: n 1 = a b Β· 1 n = a b Β· n .

Рассмотрим Π΄Π°Π½Π½ΠΎΠ΅ Π΄Π΅Π»Π΅Π½ΠΈΠ΅ Π΄Ρ€ΠΎΠ±ΠΈ Π½Π° число.

ΠŸΡ€ΠΈΠΌΠ΅Ρ€ 3

ΠŸΡ€ΠΎΠΈΠ·Π²Π΅ΡΡ‚ΠΈ Π΄Π΅Π»Π΅Π½ΠΈΠ΅ Π΄Ρ€ΠΎΠ±ΠΈ 16 45 Π½Π° число 12 .

РСшСниС

ΠŸΡ€ΠΈΠΌΠ΅Π½ΠΈΠΌ ΠΏΡ€Π°Π²ΠΈΠ»ΠΎ дСлСния Π΄Ρ€ΠΎΠ±ΠΈ Π½Π° число. ΠŸΠΎΠ»ΡƒΡ‡ΠΈΠΌ Π²Ρ‹Ρ€Π°ΠΆΠ΅Π½ΠΈΠ΅ Π²ΠΈΠ΄Π° 16 45: 12 = 16 45 Β· 12 .

ΠŸΡ€ΠΎΠΈΠ·Π²Π΅Π΄Π΅ΠΌ сокращСниС Π΄Ρ€ΠΎΠ±ΠΈ. ΠŸΠΎΠ»ΡƒΡ‡ΠΈΠΌ 16 45 Β· 12 = 2 Β· 2 Β· 2 Β· 2 (3 Β· 3 Β· 5) Β· (2 Β· 2 Β· 3) = 2 Β· 2 3 Β· 3 Β· 3 Β· 5 = 4 135 .

ΠžΡ‚Π²Π΅Ρ‚: 16 45: 12 = 4 135 .

Π”Π΅Π»Π΅Π½ΠΈΠ΅ Π½Π°Ρ‚ΡƒΡ€Π°Π»ΡŒΠ½ΠΎΠ³ΠΎ числа Π½Π° ΠΎΠ±Ρ‹ΠΊΠ½ΠΎΠ²Π΅Π½Π½ΡƒΡŽ Π΄Ρ€ΠΎΠ±ΡŒ

ΠŸΡ€Π°Π²ΠΈΠ»ΠΎ дСлСния Π°Π½Π°Π»ΠΎΠ³ΠΈΡ‡Π½ΠΎ ΠΏΡ€Π°Π²ΠΈΠ»Ρƒ дСлСния Π½Π°Ρ‚ΡƒΡ€Π°Π»ΡŒΠ½ΠΎΠ³ΠΎ числа Π½Π° ΠΎΠ±Ρ‹ΠΊΠ½ΠΎΠ²Π΅Π½Π½ΡƒΡŽ Π΄Ρ€ΠΎΠ±ΡŒ: Ρ‡Ρ‚ΠΎΠ±Ρ‹ Ρ€Π°Π·Π΄Π΅Π»ΠΈΡ‚ΡŒ Π½Π°Ρ‚ΡƒΡ€Π°Π»ΡŒΠ½ΠΎΠ΅ число n Π½Π° ΠΎΠ±Ρ‹ΠΊΠ½ΠΎΠ²Π΅Π½Π½ΡƒΡŽ a b , Π½Π΅ΠΎΠ±Ρ…ΠΎΠ΄ΠΈΠΌΠΎ произвСсти ΡƒΠΌΠ½ΠΎΠΆΠ΅Π½ΠΈΠ΅ числа n Π½Π° ΠΎΠ±Ρ€Π°Ρ‚Π½ΠΎΠ΅ Π΄Ρ€ΠΎΠ±ΠΈ a b .

Π˜ΡΡ…ΠΎΠ΄Ρ ΠΈΠ· ΠΏΡ€Π°Π²ΠΈΠ»Π°, ΠΈΠΌΠ΅Π΅ΠΌ n: a b = n Β· b a , Π° благодаря ΠΏΡ€Π°Π²ΠΈΠ»Ρƒ умноТСния Π½Π°Ρ‚ΡƒΡ€Π°Π»ΡŒΠ½ΠΎΠ³ΠΎ числа Π½Π° ΠΎΠ±Ρ‹ΠΊΠ½ΠΎΠ²Π΅Π½Π½ΡƒΡŽ Π΄Ρ€ΠΎΠ±ΡŒ, ΠΏΠΎΠ»ΡƒΡ‡ΠΈΠΌ нашС Π²Ρ‹Ρ€Π°ΠΆΠ΅Π½ΠΈΠ΅ Π² Π²ΠΈΠ΄Π΅ n: a b = n Β· b a . НСобходимо Ρ€Π°ΡΡΠΌΠΎΡ‚Ρ€Π΅Ρ‚ΡŒ Π΄Π°Π½Π½ΠΎΠ΅ Π΄Π΅Π»Π΅Π½ΠΈΠ΅ Π½Π° ΠΏΡ€ΠΈΠΌΠ΅Ρ€Π΅.

ΠŸΡ€ΠΈΠΌΠ΅Ρ€ 4

Π”Π΅Π»ΠΈΡ‚ΡŒ 25 Π½Π° 15 28 .

РСшСниС

Нам Π½Π΅ΠΎΠ±Ρ…ΠΎΠ΄ΠΈΠΌΠΎ ΠΏΠ΅Ρ€Π΅Ρ…ΠΎΠ΄ΠΈΡ‚ΡŒ ΠΎΡ‚ дСлСния ΠΊ ΡƒΠΌΠ½ΠΎΠΆΠ΅Π½ΠΈΡŽ. Π—Π°ΠΏΠΈΡˆΠ΅ΠΌ Π² Π²ΠΈΠ΄Π΅ выраТСния 25: 15 28 = 25 Β· 28 15 = 25 Β· 28 15 . Π‘ΠΎΠΊΡ€Π°Ρ‚ΠΈΠΌ Π΄Ρ€ΠΎΠ±ΡŒ ΠΈ ΠΏΠΎΠ»ΡƒΡ‡ΠΈΠΌ Ρ€Π΅Π·ΡƒΠ»ΡŒΡ‚Π°Ρ‚ Π² Π²ΠΈΠ΄Π΅ Π΄Ρ€ΠΎΠ±ΠΈ 46 2 3 .

ΠžΡ‚Π²Π΅Ρ‚: 25: 15 28 = 46 2 3 .

Π”Π΅Π»Π΅Π½ΠΈΠ΅ ΠΎΠ±Ρ‹ΠΊΠ½ΠΎΠ²Π΅Π½Π½ΠΎΠΉ Π΄Ρ€ΠΎΠ±ΠΈ Π½Π° смСшанноС число

ΠŸΡ€ΠΈ Π΄Π΅Π»Π΅Π½ΠΈΠΈ ΠΎΠ±Ρ‹ΠΊΠ½ΠΎΠ²Π΅Π½Π½ΠΎΠΉ Π΄Ρ€ΠΎΠ±ΠΈ Π½Π° смСшанноС числолСгко ΠΌΠΎΠΆΠ½ΠΎ свСти ΠΊ дСлСнию ΠΎΠ±Ρ‹ΠΊΠ½ΠΎΠ²Π΅Π½Π½Ρ‹Ρ… Π΄Ρ€ΠΎΠ±Π΅ΠΉ. НуТно ΡΠΎΠ²Π΅Ρ€ΡˆΠΈΡ‚ΡŒ ΠΏΠ΅Ρ€Π΅Π²ΠΎΠ΄ смСшанного числа Π² Π½Π΅ΠΏΡ€Π°Π²ΠΈΠ»ΡŒΠ½ΡƒΡŽ Π΄Ρ€ΠΎΠ±ΡŒ.

ΠŸΡ€ΠΈΠΌΠ΅Ρ€ 5

Π Π°Π·Π΄Π΅Π»ΠΈΡ‚ΡŒ Π΄Ρ€ΠΎΠ±ΡŒ 35 16 Π½Π° 3 1 8 .

РСшСниС

Π’Π°ΠΊ ΠΊΠ°ΠΊ 3 1 8 — смСшанноС число, прСдставим Π΅Π³ΠΎ Π² Π²ΠΈΠ΄Π΅ Π½Π΅ΠΏΡ€Π°Π²ΠΈΠ»ΡŒΠ½ΠΎΠΉ Π΄Ρ€ΠΎΠ±ΠΈ. Π’ΠΎΠ³Π΄Π° ΠΏΠΎΠ»ΡƒΡ‡ΠΈΠΌ 3 1 8 = 3 Β· 8 + 1 8 = 25 8 . Π’Π΅ΠΏΠ΅Ρ€ΡŒ ΠΏΡ€ΠΎΠΈΠ·Π²Π΅Π΄Π΅ΠΌ Π΄Π΅Π»Π΅Π½ΠΈΠ΅ Π΄Ρ€ΠΎΠ±Π΅ΠΉ. ΠŸΠΎΠ»ΡƒΡ‡ΠΈΠΌ 35 16: 3 1 8 = 35 16: 25 8 = 35 16 Β· 8 25 = 35 Β· 8 16 Β· 25 = 5 Β· 7 Β· 2 Β· 2 Β· 2 2 Β· 2 Β· 2 Β· 2 Β· (5 Β· 5) = 7 10

ΠžΡ‚Π²Π΅Ρ‚: 35 16: 3 1 8 = 7 10 .

Π”Π΅Π»Π΅Π½ΠΈΠ΅ смСшанного числа производится Ρ‚Π°ΠΊΠΈΠΌ ΠΆΠ΅ ΠΎΠ±Ρ€Π°Π·ΠΎΠΌ, ΠΊΠ°ΠΊ ΠΈ ΠΎΠ±Ρ‹ΠΊΠ½ΠΎΠ²Π΅Π½Π½Ρ‹Ρ….

Если Π²Ρ‹ Π·Π°ΠΌΠ΅Ρ‚ΠΈΠ»ΠΈ ΠΎΡˆΠΈΠ±ΠΊΡƒ Π² тСкстС, поТалуйста, Π²Ρ‹Π΄Π΅Π»ΠΈΡ‚Π΅ Π΅Ρ‘ ΠΈ Π½Π°ΠΆΠΌΠΈΡ‚Π΅ Ctrl+Enter

Π”Ρ€ΠΎΠ±ΡŒ – это ΠΎΠ΄Π½Π° ΠΈΠ»ΠΈ Π±ΠΎΠ»Π΅Π΅ Π΄ΠΎΠ»Π΅ΠΉ Ρ†Π΅Π»ΠΎΠ³ΠΎ, Π·Π° ΠΊΠΎΡ‚ΠΎΡ€ΠΎΠ΅ ΠΎΠ±Ρ‹Ρ‡Π½ΠΎ принимаСтся Π΅Π΄ΠΈΠ½ΠΈΡ†Π° (1). Как ΠΈ с Π½Π°Ρ‚ΡƒΡ€Π°Π»ΡŒΠ½Ρ‹ΠΌΠΈ числами, с дробями ΠΌΠΎΠΆΠ½ΠΎ Π²Ρ‹ΠΏΠΎΠ»Π½ΡΡ‚ΡŒ всС основныС арифмСтичСскиС дСйствия (слоТСниС, Π²Ρ‹Ρ‡ΠΈΡ‚Π°Π½ΠΈΠ΅, Π΄Π΅Π»Π΅Π½ΠΈΠ΅, умноТСния), для этого Π½ΡƒΠΆΠ½ΠΎ Π·Π½Π°Ρ‚ΡŒ особСнности Ρ€Π°Π±ΠΎΡ‚Ρ‹ с дробями ΠΈ Ρ€Π°Π·Π»ΠΈΡ‡Π°Ρ‚ΡŒ ΠΈΡ… Π²ΠΈΠ΄Ρ‹. БущСствуСт нСсколько Π²ΠΈΠ΄ΠΎΠ² Π΄Ρ€ΠΎΠ±Π΅ΠΉ: дСсятичныС ΠΈ ΠΎΠ±Ρ‹ΠΊΠ½ΠΎΠ²Π΅Π½Π½Ρ‹Π΅, ΠΈΠ»ΠΈ простыС. Бвоя спСцифика Π΅ΡΡ‚ΡŒ Ρƒ ΠΊΠ°ΠΆΠ΄ΠΎΠ³ΠΎ Π²ΠΈΠ΄Π° Π΄Ρ€ΠΎΠ±Π΅ΠΉ, Π½ΠΎ, ΠΎΠ±ΡΡ‚ΠΎΡΡ‚Π΅Π»ΡŒΠ½ΠΎ Ρ€Π°Π·ΠΎΠ±Ρ€Π°Π²ΡˆΠΈΡΡŒ ΠΎΠ΄ΠΈΠ½ Ρ€Π°Π·, ΠΊΠ°ΠΊ с Π½ΠΈΠΌΠΈ ΠΎΠ±Ρ€Π°Ρ‰Π°Ρ‚ΡŒΡΡ, Π²Ρ‹ смоТСтС Ρ€Π΅ΡˆΠ°Ρ‚ΡŒ Π»ΡŽΠ±Ρ‹Π΅ ΠΏΡ€ΠΈΠΌΠ΅Ρ€Ρ‹ с дробями, ΠΏΠΎΡΠΊΠΎΠ»ΡŒΠΊΡƒ Π±ΡƒΠ΄Π΅Ρ‚Π΅ Π·Π½Π°Ρ‚ΡŒ основныС ΠΏΡ€ΠΈΠ½Ρ†ΠΈΠΏΡ‹ выполнСния арифмСтичСских вычислСний с дробями. Рассмотрим Π½Π° ΠΏΡ€ΠΈΠΌΠ΅Ρ€Π°Ρ… ΠΊΠ°ΠΊ Ρ€Π°Π·Π΄Π΅Π»ΠΈΡ‚ΡŒ Π΄Ρ€ΠΎΠ±ΡŒ Π½Π° Ρ†Π΅Π»ΠΎΠ΅ число, ΠΈΡΠΏΠΎΠ»ΡŒΠ·ΡƒΡ Ρ€Π°Π·Π½Ρ‹Π΅ Π²ΠΈΠ΄Ρ‹ Π΄Ρ€ΠΎΠ±Π΅ΠΉ.

Как Ρ€Π°Π·Π΄Π΅Π»ΠΈΡ‚ΡŒ ΠΏΡ€ΠΎΡΡ‚ΡƒΡŽ Π΄Ρ€ΠΎΠ±ΡŒ Π½Π° Π½Π°Ρ‚ΡƒΡ€Π°Π»ΡŒΠ½ΠΎΠ΅ число?
ΠžΠ±Ρ‹ΠΊΠ½ΠΎΠ²Π΅Π½Π½Ρ‹ΠΌΠΈ ΠΈΠ»ΠΈ простыми Π½Π°Π·Ρ‹Π²Π°ΡŽΡ‚ Π΄Ρ€ΠΎΠ±ΠΈ, Π·Π°ΠΏΠΈΡΡ‹Π²Π°ΡŽΡ‰ΠΈΠ΅ΡΡ Π² Π²ΠΈΠ΄Π΅ Ρ‚Π°ΠΊΠΎΠ³ΠΎ ΠΎΡ‚Π½ΠΎΡˆΠ΅Π½ΠΈΡ чисСл, ΠΏΡ€ΠΈ ΠΊΠΎΡ‚ΠΎΡ€ΠΎΠΌ Π²Π²Π΅Ρ€Ρ…Ρƒ Π΄Ρ€ΠΎΠ±ΠΈ указываСтся Π΄Π΅Π»ΠΈΠΌΠΎΠ΅ (Ρ‡ΠΈΡΠ»ΠΈΡ‚Π΅Π»ΡŒ), Π° Π²Π½ΠΈΠ·Ρƒ – Π΄Π΅Π»ΠΈΡ‚Π΅Π»ΡŒ (Π·Π½Π°ΠΌΠ΅Π½Π°Ρ‚Π΅Π»ΡŒ) Π΄Ρ€ΠΎΠ±ΠΈ. Как Ρ€Π°Π·Π΄Π΅Π»ΠΈΡ‚ΡŒ Ρ‚Π°ΠΊΡƒΡŽ Π΄Ρ€ΠΎΠ±ΡŒ Π½Π° Ρ†Π΅Π»ΠΎΠ΅ число? Рассмотрим Π½Π° ΠΏΡ€ΠΈΠΌΠ΅Ρ€Π΅! Допустим, Π½Π°ΠΌ Π½ΡƒΠΆΠ½ΠΎ Ρ€Π°Π·Π΄Π΅Π»ΠΈΡ‚ΡŒ 8/12 Π½Π° 2.


Для этого ΠΌΡ‹ Π΄ΠΎΠ»ΠΆΠ½Ρ‹ Π²Ρ‹ΠΏΠΎΠ»Π½ΠΈΡ‚ΡŒ ряд дСйствий:
Π’Π°ΠΊΠΈΠΌ ΠΎΠ±Ρ€Π°Π·ΠΎΠΌ, Ссли ΠΏΠ΅Ρ€Π΅Π΄ Π½Π°ΠΌΠΈ стоит Π·Π°Π΄Π°Ρ‡Π° Ρ€Π°Π·Π΄Π΅Π»ΠΈΡ‚ΡŒ Π΄Ρ€ΠΎΠ±ΡŒ Π½Π° Ρ†Π΅Π»ΠΎΠ΅ число, схСма Ρ€Π΅ΡˆΠ΅Π½ΠΈΡ Π±ΡƒΠ΄Π΅Ρ‚ Π²Ρ‹Π³Π»ΡΠ΄Π΅Ρ‚ΡŒ ΠΏΡ€ΠΈΠΌΠ΅Ρ€Π½ΠΎ Ρ‚Π°ΠΊ:


ΠŸΠΎΠ΄ΠΎΠ±Π½Ρ‹ΠΌ ΠΎΠ±Ρ€Π°Π·ΠΎΠΌ ΠΌΠΎΠΆΠ½ΠΎ Ρ€Π°Π·Π΄Π΅Π»ΠΈΡ‚ΡŒ Π»ΡŽΠ±ΡƒΡŽ ΠΎΠ±Ρ‹ΠΊΠ½ΠΎΠ²Π΅Π½Π½ΡƒΡŽ (ΠΏΡ€ΠΎΡΡ‚ΡƒΡŽ) Π΄Ρ€ΠΎΠ±ΡŒ Π½Π° Ρ†Π΅Π»ΠΎΠ΅ число.

Как Ρ€Π°Π·Π΄Π΅Π»ΠΈΡ‚ΡŒ Π΄Π΅ΡΡΡ‚ΠΈΡ‡Π½ΡƒΡŽ Π΄Ρ€ΠΎΠ±ΡŒ Π½Π° Ρ†Π΅Π»ΠΎΠ΅ число?
ДСсятичная Π΄Ρ€ΠΎΠ±ΡŒ — это такая Π΄Ρ€ΠΎΠ±ΡŒ, которая получаСтся вслСдствиС дСлСния Π΅Π΄ΠΈΠ½ΠΈΡ†Ρ‹ Π½Π° Π΄Π΅ΡΡΡ‚ΡŒ, тысячу ΠΈ Ρ‚Π°ΠΊ Π΄Π°Π»Π΅Π΅ частСй. АрифмСтичСскиС дСйствия с дСсятичными дробями Π²Ρ‹ΠΏΠΎΠ»Π½ΡΡŽΡ‚ΡΡ довольно просто.

Рассмотрим Π½Π° ΠΏΡ€ΠΈΠΌΠ΅Ρ€Π΅ ΠΊΠ°ΠΊ Ρ€Π°Π·Π΄Π΅Π»ΠΈΡ‚ΡŒ Π΄Ρ€ΠΎΠ±ΡŒ Π½Π° Ρ†Π΅Π»ΠΎΠ΅ число. Допустим, Π½Π°ΠΌ Π½ΡƒΠΆΠ½ΠΎ ΠΏΠΎΠ΄Π΅Π»ΠΈΡ‚ΡŒ Π΄Π΅ΡΡΡ‚ΠΈΡ‡Π½ΡƒΡŽ Π΄Ρ€ΠΎΠ±ΡŒ 0,925 Π½Π° Π½Π°Ρ‚ΡƒΡ€Π°Π»ΡŒΠ½ΠΎΠ΅ число 5.


Подводя ΠΈΡ‚ΠΎΠ³ΠΈ, остановимся Π½Π° Π΄Π²ΡƒΡ… основных ΠΌΠΎΠΌΠ΅Π½Ρ‚Π°Ρ…, ΠΊΠΎΡ‚ΠΎΡ€Ρ‹Π΅ Π²Π°ΠΆΠ½Ρ‹ ΠΏΡ€ΠΈ Π²Ρ‹ΠΏΠΎΠ»Π½Π΅Π½ΠΈΠΈ ΠΎΠΏΠ΅Ρ€Π°Ρ†ΠΈΠΈ дСлСния дСсятичных Π΄Ρ€ΠΎΠ±Π΅ΠΉ Π½Π° Ρ†Π΅Π»ΠΎΠ΅ число:
  • для раздСлСния дСсятичной Π΄Ρ€ΠΎΠ±ΠΈ Π½Π° Π½Π°Ρ‚ΡƒΡ€Π°Π»ΡŒΠ½ΠΎΠ΅ число ΠΏΡ€ΠΈΠΌΠ΅Π½ΡΡŽΡ‚ Π΄Π΅Π»Π΅Π½ΠΈΠ΅ Π² столбик;
  • запятая ставится Π² частном Ρ‚ΠΎΠ³Π΄Π°, ΠΊΠΎΠ³Π΄Π° Π·Π°ΠΊΠΎΠ½Ρ‡Π΅Π½ΠΎ Π΄Π΅Π»Π΅Π½ΠΈΠ΅ Ρ†Π΅Π»ΠΎΠΉ части Π΄Π΅Π»ΠΈΠΌΠΎΠ³ΠΎ.
ΠŸΡ€ΠΈΠΌΠ΅Π½ΡΡ эти простыС ΠΏΡ€Π°Π²ΠΈΠ»Π°, всСгда ΠΌΠΎΠΆΠ½ΠΎ Π±Π΅Π· особого Ρ‚Ρ€ΡƒΠ΄Π° Ρ€Π°Π·Π΄Π΅Π»ΠΈΡ‚ΡŒ Π»ΡŽΠ±ΡƒΡŽ Π΄Π΅ΡΡΡ‚ΠΈΡ‡Π½ΡƒΡŽ ΠΈΠ»ΠΈ ΠΏΡ€ΠΎΡΡ‚ΡƒΡŽ Π΄Ρ€ΠΎΠ±ΠΈ Π½Π° Ρ†Π΅Π»ΠΎΠ΅ число.

Как ΡΠΊΠ»Π°Π΄Ρ‹Π²Π°Ρ‚ΡŒ, Π²Ρ‹Ρ‡ΠΈΡ‚Π°Ρ‚ΡŒ, ΡƒΠΌΠ½ΠΎΠΆΠ°Ρ‚ΡŒ ΠΈ Π΄Π΅Π»ΠΈΡ‚ΡŒ Π΄Ρ€ΠΎΠ±ΠΈ: с Π²ΠΈΠ΄Π΅ΠΎ ΠΈ ΠΏΡ€ΠΈΠΌΠ΅Ρ€Π°ΠΌΠΈ

Π‘ΠΎΠ»ΡŒΡˆΠΈΠ½ΡΡ‚Π²ΠΎ ΡƒΡ‡ΠΈΡ‚Π΅Π»Π΅ΠΉ Π½Π°ΡƒΡ‡Π°Ρ‚ вас, ΠΊΠ°ΠΊ ΡΠΊΠ»Π°Π΄Ρ‹Π²Π°Ρ‚ΡŒ ΠΈ Π²Ρ‹Ρ‡ΠΈΡ‚Π°Ρ‚ΡŒ Π΄Ρ€ΠΎΠ±ΠΈ, ΠΏΡ€Π΅ΠΆΠ΄Π΅ Ρ‡Π΅ΠΌ ΠΎΠ½ΠΈ Π½Π°ΡƒΡ‡Π°Ρ‚ вас, ΠΊΠ°ΠΊ ΠΈΡ… ΡƒΠΌΠ½ΠΎΠΆΠ°Ρ‚ΡŒ. Однако ΠΏΡ€ΠΎΡ‰Π΅ всСго Π²Ρ‹ΡƒΡ‡ΠΈΡ‚ΡŒ Π΅Π³ΠΎ Π² ΠΎΠ±Ρ€Π°Ρ‚Π½ΠΎΠΌ порядкС. ΠŸΠΎΡΠΌΠΎΡ‚Ρ€ΠΈΠΌ, ΠΊΠ°ΠΊ это Ρ€Π°Π±ΠΎΡ‚Π°Π΅Ρ‚.

Π’Ρ‹ Ρ‚Π°ΠΊΠΆΠ΅ ΠΌΠΎΠΆΠ΅Ρ‚Π΅ ΠΏΠΎΠΏΡ€ΠΎΠ±ΠΎΠ²Π°Ρ‚ΡŒ наш ΠšΠ°Π»ΡŒΠΊΡƒΠ»ΡΡ‚ΠΎΡ€ Π΄Ρ€ΠΎΠ±Π΅ΠΉ

Как ΡƒΠΌΠ½ΠΎΠΆΠ°Ρ‚ΡŒ Π΄Ρ€ΠΎΠ±ΠΈ

Вопрос:- Π£ΠΌΠ½ΠΎΠΆΠΈΡ‚ΡŒ 3 0 ⁄ 92 ΠΈ 5 ⁄ 4

ΠžΡ‚Π²Π΅Ρ‚:

Π’ Ρ‡Π΅ΠΌ ΠΏΠΎΠ΄Π²ΠΎΡ…: НичСго. ΠŸΡ€ΠΎΡΡ‚ΠΎ Π΄ΠΎΠ±Π°Π²ΡŒΡ‚Π΅ Π²Π΅Ρ€Ρ…Π½Π΅Π΅ число ΠΈ Π½ΠΈΠΆΠ½Π΅Π΅ число, ΠΈ всС Π³ΠΎΡ‚ΠΎΠ²ΠΎ.

Вопрос: Π£ΠΌΠ½ΠΎΠΆΠ΅Π½ΠΈΠ΅ 2 ⁄ 6 ΠΈ 8 ⁄ 2 ΠΈ 4 ⁄ 5

ΠžΡ‚Π²Π΅Ρ‚: Упростит эти Ρ†ΠΈΡ„Ρ€Ρ‹, ΠΊΠΎΡ‚ΠΎΡ€Ρ‹Π΅ ΠΌΠΎΠ³ΡƒΡ‚ Π±Ρ‹Ρ‚ΡŒ ΡƒΠΏΡ€ΠΎΡ‰Π΅Π½Ρ‹ (ΠΈΠ»ΠΈ Π²Ρ‹ ΠΌΠΎΠΆΠ΅Ρ‚Π΅ ΡΠ΄Π΅Π»Π°Ρ‚ΡŒ это. послС умноТСния. Π­Ρ‚ΠΎ зависит ΠΎΡ‚ вас, ΠΊΠΎΠ³Π΄Π° Π²Ρ‹ Ρ…ΠΎΡ‚ΠΈΡ‚Π΅ ΡƒΠΏΡ€ΠΎΡΡ‚ΠΈΡ‚ΡŒ. ΠŸΠΎΡΠΊΠΎΠ»ΡŒΠΊΡƒ это ΡƒΠΌΠ½ΠΎΠΆΠ΅Π½ΠΈΠ΅, поэтому Π½Π΅ ΠΈΠΌΠ΅Π΅Ρ‚ значСния, ΠΊΠΎΠ³Π΄Π° Π²Ρ‹ это Π΄Π΅Π»Π°Π΅Ρ‚Π΅.)

2 ⁄ 6 ПослС упрощСния = 1 ⁄ 3

8 ⁄ 2 ПослС упрощСния = 4 ⁄ 1

. 3 X 4 ⁄ 1 X 4 ⁄ 5 = 16 ⁄ 15

ΠœΡ‹ снова ΠΏΠ΅Ρ€Π΅ΠΌΠ½ΠΎΠΆΠΈΠ»ΠΈ Π²Π΅Ρ€Ρ…Π½ΠΈΠ΅ числа ΠΈ числа снизу.

Как Π΄Π΅Π»ΠΈΡ‚ΡŒ Π΄Ρ€ΠΎΠ±ΠΈ

ΠžΠΏΡΡ‚ΡŒ ΠΆΠ΅, это просто. Если Π²Ρ‹ Π·Π½Π°Π΅Ρ‚Π΅, ΠΊΠ°ΠΊ ΡƒΠΌΠ½ΠΎΠΆΠ°Ρ‚ΡŒ, Π²Ρ‹ ΡƒΠΆΠ΅ Π·Π½Π°Π΅Ρ‚Π΅, ΠΊΠ°ΠΊ Π΄Π΅Π»ΠΈΡ‚ΡŒ. ΠŸΠΎΡΠΌΠΎΡ‚Ρ€ΠΈΠΌ, ΠΊΠ°ΠΊ.

Вопрос: Π Π΅ΡˆΠΈΡ‚Π΅ это: 3/8 Γ· 4/7

РСшСниС: 

Для понимания: ВсС, Ρ‡Ρ‚ΠΎ Π²Π°ΠΌ Π½ΡƒΠΆΠ½ΠΎ ΡΠ΄Π΅Π»Π°Ρ‚ΡŒ, это ΠΏΠΎΠ²Π΅Ρ€Π½ΡƒΡ‚ΡŒ Π·Π½Π°Ρ‡Π΅Π½ΠΈΠ΅ Π²Ρ‚ΠΎΡ€ΠΎΠΉ Π΄Ρ€ΠΎΠ±ΠΈ. НапримСр, 4/7 Π±ΡƒΠ΄Π΅Ρ‚ 7/4, Π·Π°Ρ‚Π΅ΠΌ ΡƒΠΌΠ½ΠΎΠΆΡŒΡ‚Π΅ Π΄Ρ€ΠΎΠ±ΠΈ, ΠΈ Π²Ρ‹ ΠΏΠΎΠ»ΡƒΡ‡ΠΈΡ‚Π΅ ΠΎΡ‚Π²Π΅Ρ‚.

Ρ‚Π°ΠΊ, 3/8 x 7/4

=> 21/32

просто ΠΈΡ… Π²Π΅Π·Π΄Π΅ Π³Π΄Π΅ Π½Π°Π΄ΠΎ.

Π”Ρ€ΡƒΠ³ΠΎΠΉ вопрос: Π Π΅ΡˆΠΈΡ‚Π΅ это 10/4 Γ· 8/5

РСшСниС:

10/4 Γ· 8/5

=> 10/4 x 5/8

=> 50/32

=> 25 /16

Π‘Π»ΠΎΠΆΠ΅Π½ΠΈΠ΅ ΠΈ Π²Ρ‹Ρ‡ΠΈΡ‚Π°Π½ΠΈΠ΅ Π΄Ρ€ΠΎΠ±Π΅ΠΉ

Как ΡΠΊΠ»Π°Π΄Ρ‹Π²Π°Ρ‚ΡŒ Π΄Ρ€ΠΎΠ±ΠΈ, Ссли Π½ΠΈΠΆΠ½ΠΈΠ΅ числа ΡΠΎΠ²ΠΏΠ°Π΄Π°ΡŽΡ‚? Или ΠΊΠ°ΠΊ ΡΠΊΠ»Π°Π΄Ρ‹Π²Π°Ρ‚ΡŒ Π΄Ρ€ΠΎΠ±ΠΈ с ΠΎΠ΄ΠΈΠ½Π°ΠΊΠΎΠ²Ρ‹ΠΌΠΈ знамСнатСлями ΠΈΠ»ΠΈ ΠΎΠ΄ΠΈΠ½Π°ΠΊΠΎΠ²Ρ‹ΠΌΠΈ знамСнатСлями?

Один ΠΈ Ρ‚ΠΎΡ‚ ΠΆΠ΅ вопрос ΠΎΡ‚Π²Π΅Ρ‚ΠΈΠ» 2 способами. Π’Π΅ ΠΆΠ΅ самыС Π½ΠΈΠΆΠ½ΠΈΠ΅ числа извСстны ΠΊΠ°ΠΊ ΠΈΠ΄Π΅Π½Ρ‚ΠΈΡ‡Π½Ρ‹Π΅ ΠΈΠ»ΠΈ ΠΏΠΎΡ…ΠΎΠΆΠΈΠ΅ Π·Π½Π°ΠΌΠ΅Π½Π°Ρ‚Π΅Π»ΠΈ.

Π’ΠžΠŸΠ ΠžΠ‘: Π”ΠΎΠ±Π°Π²ΠΈΡ‚ΡŒ 1 ⁄ 2 ΠΈ 3 ⁄ 2

ΠžΡ‚Π²Π΅Ρ‚: 2

. ΠŸΠΎΡΡΠ½ΠΈΡ‚Π΅: Π’Π°ΠΊΠΈΠΌ ΠΎΠ±Ρ€Π°Π·ΠΎΠΌ, ΠΎΡ‚Π²Π΅Ρ‚ — 4 or 29006 2 2 2 . Π²Ρ‹ ΠΌΠΎΠΆΠ΅Ρ‚Π΅ Π΅Ρ‰Π΅ большС ΡƒΠΏΡ€ΠΎΡΡ‚ΠΈΡ‚ΡŒ, ΠΈ это Π±ΡƒΠ΄Π΅Ρ‚ 2 ⁄ 1 , ΠΈ ΠΌΡ‹ напишСм это ΠΊΠ°ΠΊ 2, ΠΏΠΎΡ‚ΠΎΠΌΡƒ Ρ‡Ρ‚ΠΎ 1 Π²Π½ΠΈΠ·Ρƒ Π½Π΅ ΠΈΠΌΠ΅Π΅Ρ‚ смысла.

Вопрос: Π”ΠΎΠ±Π°Π²ΠΈΡ‚ΡŒ 3 ⁄ 2 ΠΈ 7 ⁄ 2 ΠΈ 4 ⁄ 2

ΠžΡ‚Π²Π΅Ρ‚:

Подсказка: МоТно ΡƒΠΏΡ€ΠΎΡΡ‚ΠΈΡ‚ΡŒ 4 ⁄ 2 , Π½ΠΎ ΠΌΡ‹ Π½Π΅ Π±ΡƒΠ΄Π΅ΠΌ этого Π΄Π΅Π»Π°Ρ‚ΡŒ Π΄ΠΎ слоТСния. Если ΠΌΡ‹ это сдСлаСм, ΠΌΡ‹ потСряСм ΠΎΠ±Ρ‰Π΅Π΅ Π½ΠΈΠΆΠ½Π΅Π΅ Π·Π½Π°Ρ‡Π΅Π½ΠΈΠ΅ ΠΈΠ· нашСй Ρ‚Ρ€Π΅Ρ‚ΡŒΠ΅ΠΉ Π΄Ρ€ΠΎΠ±ΠΈ, Ρ€Π°Π²Π½ΠΎΠ΅ 2. ΠŸΠΎΡΡ‚ΠΎΠΌΡƒ ΠΌΡ‹ собираСмся ΡƒΠΏΡ€ΠΎΡΡ‚ΠΈΡ‚ΡŒ ΠΏΠΎΠ·ΠΆΠ΅, послС Π·Π°Π²Π΅Ρ€ΡˆΠ΅Π½ΠΈΡ процСсса слоТСния.

SO, 3 ⁄ 2 + 7 ⁄ 2 + 4 ⁄ 2

=> 14 ⁄ 2

=> 14 ~ 2

020

(ΠœΡ‹ просто Π΄ΠΎΠ±Π°Π²ΠΈΠ»ΠΈ Π²Π΅Ρ€Ρ…Π½ΠΈΠ΅ числа ΠΈ взяли 2 Π² качСствС ΠΎΠ±Ρ‰Π΅Π³ΠΎ числа для Π½ΠΈΠΆΠ½ΠΈΡ…, ΠΏΠΎΡ‚ΠΎΠΌΡƒ Ρ‡Ρ‚ΠΎ всС ΠΎΠ½ΠΈ ΠΈΠΌΠ΅ΡŽΡ‚ ΠΎΠ΄Π½ΠΎ ΠΈ Ρ‚ΠΎ ΠΆΠ΅ число Π²Π½ΠΈΠ·Ρƒ)

=> УпроститС 14 ⁄ 2 , ΠΈ Π²Ρ‹ ΠΏΠΎΠ»ΡƒΡ‡ΠΈΡ‚Π΅ 7 ⁄ 1 Ρ‚ΠΎ Π±ΡƒΠ΄Π΅Ρ‚ записано ΠΊΠ°ΠΊ 7

Π˜Ρ‚Π°ΠΊ, 7 это ΠΎΡ‚Π²Π΅Ρ‚ Π½Π° вопрос.

Π’Ρ‹Ρ‡ΠΈΡ‚Π°Π½ΠΈΠ΅ Π΄Ρ€ΠΎΠ±ΠΈ ΠΏΡ€ΠΈ ΠΎΠ΄ΠΈΠ½Π°ΠΊΠΎΠ²ΠΎΠΌ Π½ΠΈΠΆΠ½Π΅ΠΌ Π·Π½Π°Ρ‡Π΅Π½ΠΈΠΈ (Π½Π°ΠΏΡ€ΠΈΠΌΠ΅Ρ€, Π·Π½Π°ΠΌΠ΅Π½Π°Ρ‚Π΅Π»ΠΈ ΠΈΠ»ΠΈ ΠΈΠ΄Π΅Π½Ρ‚ΠΈΡ‡Π½Ρ‹ΠΉ Π·Π½Π°ΠΌΠ΅Π½Π°Ρ‚Π΅Π»ΡŒ)

Π’Ρ‹Ρ‡ΠΈΡ‚Π°Π½ΠΈΠ΅ Π΄Ρ€ΠΎΠ±ΠΈ Ρ€Π°Π±ΠΎΡ‚Π°Π΅Ρ‚ Ρ‚Π°ΠΊ ΠΆΠ΅, ΠΊΠ°ΠΊ ΠΌΡ‹ Π³ΠΎΠ²ΠΎΡ€ΠΈΠ»ΠΈ ΠΎ Π΄ΠΎΠ±Π°Π²Π»Π΅Π½ΠΈΠΈ вычитания.

  • Когда ΠΎΠ±Π΅ Π΄Ρ€ΠΎΠ±ΠΈ ΠΈΠΌΠ΅ΡŽΡ‚ ΠΎΠ΄ΠΈΠ½Π°ΠΊΠΎΠ²ΠΎΠ΅ Π½ΠΈΠΆΠ½Π΅Π΅ Π·Π½Π°Ρ‡Π΅Π½ΠΈΠ΅, ΠΌΡ‹ ΠΏΡ€ΠΈΠ½ΠΈΠΌΠ°Π΅ΠΌ Π΅Π³ΠΎ ΠΊΠ°ΠΊ ΠΎΠ±Ρ‰Π΅Π΅ Π·Π½Π°Ρ‡Π΅Π½ΠΈΠ΅ ΠΈ Π²Ρ‹Ρ‡ΠΈΡ‚Π°Π΅ΠΌ Π²Π΅Ρ€Ρ…Π½ΠΈΠ΅ числа.

Question: Subtract 1 ⁄ 3 from 4 ⁄ 3 Β  (OrΒ  get the value of 4 ⁄ 3 – 1 ⁄ 3 )

Answer:

Π£Ρ‚ΠΎΡ‡Π½Π΅Π½ΠΈΠ΅: Как ΠΌΡ‹ сказали, ΠΊΠΎΠ³Π΄Π° Π½ΠΈΠΆΠ½Π΅Π΅ Π·Π½Π°Ρ‡Π΅Π½ΠΈΠ΅ ΠΎΠ΄ΠΈΠ½Π°ΠΊΠΎΠ²ΠΎ для ΠΎΠ±ΠΎΠΈΡ… чисСл, ΠΌΡ‹ ΠΏΡ€ΠΈΠ½ΠΈΠΌΠ°Π΅ΠΌ Π΅Π³ΠΎ Π·Π° ΠΎΠ±Ρ‰Π΅Π΅. Π’Π°ΠΊΠΈΠΌ ΠΎΠ±Ρ€Π°Π·ΠΎΠΌ, ΠΌΡ‹ взяли 3 ΠΊΠ°ΠΊ ΠΎΠ±Ρ‰Π΅Π΅ для Π½ΠΈΠΆΠ½Π΅Π³ΠΎ значСния. ΠœΡ‹ Π²Ρ‹Ρ‡Π»ΠΈ 4 – 1, Ρ‡Ρ‚ΠΎΠ±Ρ‹ ΠΏΠΎΠ»ΡƒΡ‡ΠΈΡ‚ΡŒ максимальноС Π·Π½Π°Ρ‡Π΅Π½ΠΈΠ΅. Π˜Ρ‚Π°ΠΊ, ΠΌΡ‹ ΠΏΠΎΠ»ΡƒΡ‡ΠΈΠ»ΠΈ 3 ⁄ 3 . Π—Π°Ρ‚Π΅ΠΌ ΠΌΡ‹ упростили 3 ⁄ 3 ΠΈ ΠΏΠΎΠ»ΡƒΡ‡ΠΈΠ»ΠΈ 1 ΠΊΠ°ΠΊ ΠΎΠΊΠΎΠ½Ρ‡Π°Ρ‚Π΅Π»ΡŒΠ½ΠΎΠ΅ Π·Π½Π°Ρ‡Π΅Π½ΠΈΠ΅, ΠΊΠΎΡ‚ΠΎΡ€ΠΎΠ΅ являСтся ΠΎΡ‚Π²Π΅Ρ‚ΠΎΠΌ.

Π‘ΡƒΠ΄ΡŒ Ρ‚ΠΎ 2 значСния, 3 ΠΈΠ»ΠΈ 4, ΠΌΡ‹ Π±ΡƒΠ΄Π΅ΠΌ ΠΏΡ€ΠΈΠΌΠ΅Π½ΡΡ‚ΡŒ ΠΎΠ΄ΠΈΠ½ ΠΈ Ρ‚ΠΎΡ‚ ΠΆΠ΅ ΠΌΠ΅Ρ‚ΠΎΠ΄, ΠΊΠΎΠ³Π΄Π° Π½ΠΈΠΆΠ½Π΅Π΅ Π·Π½Π°Ρ‡Π΅Π½ΠΈΠ΅ ΠΎΠ΄ΠΈΠ½Π°ΠΊΠΎΠ²ΠΎ.

Вопрос: ΠŸΠΎΠ»ΡƒΡ‡ΠΈΡ‚Π΅ Π·Π½Π°Ρ‡Π΅Π½ΠΈΠ΅ для 2 ⁄ 3 1 ⁄ 3 6 ⁄ 3

. Π²Ρ‹Ρ‡ΠΈΡ‚Π°Π½ΠΈΠ΅, ΠΈΠ½Π°Ρ‡Π΅ измСнится Π½ΠΈΠΆΠ½Π΅Π΅ Π·Π½Π°Ρ‡Π΅Π½ΠΈΠ΅ Ρ‚Ρ€Π΅Ρ‚ΡŒΠ΅ΠΉ Π΄Ρ€ΠΎΠ±ΠΈ.

Π’ΠΎΠ·ΡŒΠΌΠ΅ΠΌ 3 Π² качСствС Π½ΠΈΠΆΠ½Π΅Π³ΠΎ значСния ΠΈ Π²Ρ‹Ρ‡Ρ‚Π΅ΠΌ 2 – 1 – 6.

ΠŸΠΎΠ»ΡƒΡ‡Π°Π΅ΠΌ =>Β  -5 ⁄ 3

Π’ ΠΎΡ‚Π»ΠΈΡ‡ΠΈΠ΅ ΠΎΡ‚ Π΄Ρ€ΠΎΠ±ΠΈ

7 Π²Ρ‹Ρ‡Π΅ΡΡ‚ΡŒ Π½Π΅ΠΈΠ΄Π΅Π½Ρ‚ΠΈΡ‡Π½ΡƒΡŽ ΠΈΠ»ΠΈ Π½Π΅ΠΈΠ΄Π΅Π½Ρ‚ΠΈΡ‡Π½ΡƒΡŽ Π΄Ρ€ΠΎΠ±ΡŒ (ΠΊΠΎΠ³Π΄Π° Π½ΠΈΠΆΠ½ΠΈΠ΅ значСния Π½Π΅ ΡΠΎΠ²ΠΏΠ°Π΄Π°ΡŽΡ‚ для ΠΎΠ±ΠΎΠΈΡ… чисСл)

Π”Π°Π²Π°ΠΉΡ‚Π΅ Ρ€Π΅ΡˆΠΈΠΌ нСсколько ΠΏΡ€ΠΈΠΌΠ΅Ρ€ΠΎΠ², Ρ‡Ρ‚ΠΎΠ±Ρ‹ ΡƒΠ²ΠΈΠ΄Π΅Ρ‚ΡŒ это Π² дСйствии.

Вопрос: ΠŸΠΎΠ»ΡƒΡ‡ΠΈΡ‚ΡŒ Π·Π½Π°Ρ‡Π΅Π½ΠΈΠ΅ 2 ⁄ 7 + 1 ⁄ 3

ΠžΡ‚Π²Π΅Ρ‚: ΠŸΠΎΡΠΊΠΎΠ»ΡŒΠΊΡƒ Π½ΠΈΠΆΠ½Π΅Π΅ Π·Π½Π°Ρ‡Π΅Π½ΠΈΠ΅ для ΠΎΠ±ΠΎΠΈΡ… Π½Π΅ ΠΎΠ΄Π½ΠΎ ΠΈ Ρ‚ΠΎ ΠΆΠ΅, Π½Π°ΠΌ Π½ΡƒΠΆΠ½ΠΎ ΠΏΡ€Π΅ΠΎΠ±Ρ€Π°Π·ΠΎΠ²Π°Ρ‚ΡŒ ΠΈΡ… Π² ΠΏΠΎΠ΄ΠΎΠ±Π½Ρ‹Π΅ Π΄Ρ€ΠΎΠ±ΠΈ. Но ΠΊΠ°ΠΊ ΠΌΡ‹ это Π΄Π΅Π»Π°Π΅ΠΌ? Π­Ρ‚ΠΎ ΠΎΡ‡Π΅Π½ΡŒ Π»Π΅Π³ΠΊΠΎ.

Как ΠΏΡ€Π΅ΠΎΠ±Ρ€Π°Π·ΠΎΠ²Π°Ρ‚ΡŒ Π½Π΅ΠΏΠΎΡ…ΠΎΠΆΡƒΡŽ Π΄Ρ€ΠΎΠ±ΡŒ Π² ΠΏΠΎΡ…ΠΎΠΆΡƒΡŽ Π΄Ρ€ΠΎΠ±ΡŒ:

Π’ΠΎΠ·ΡŒΠΌΠΈΡ‚Π΅ Π½ΠΈΠΆΠ½Π΅Π΅ Π·Π½Π°Ρ‡Π΅Π½ΠΈΠ΅ ΠΏΠ΅Ρ€Π²ΠΎΠΉ Π΄Ρ€ΠΎΠ±ΠΈ, Π° Π·Π°Ρ‚Π΅ΠΌ ΡƒΠΌΠ½ΠΎΠΆΡŒΡ‚Π΅ Π΅Π³ΠΎ Π½Π° Π²Π΅Ρ€Ρ…Π½Π΅Π΅ ΠΈ Π½ΠΈΠΆΠ½Π΅Π΅ Π·Π½Π°Ρ‡Π΅Π½ΠΈΠ΅ Π²Ρ‚ΠΎΡ€ΠΎΠΉ Π΄Ρ€ΠΎΠ±ΠΈ. А Ρ‚Π°ΠΊΠΆΠ΅ Π²ΠΎΠ·ΡŒΠΌΠΈΡ‚Π΅ Π½ΠΈΠΆΠ½Π΅Π΅ Π·Π½Π°Ρ‡Π΅Π½ΠΈΠ΅ Π²Ρ‚ΠΎΡ€ΠΎΠΉ Π΄Ρ€ΠΎΠ±ΠΈ ΠΈ ΡƒΠΌΠ½ΠΎΠΆΡŒΡ‚Π΅ Π΅Π³ΠΎ Π½Π° Π²Π΅Ρ€Ρ…Π½Π΅Π΅ ΠΈ Π½ΠΈΠΆΠ½Π΅Π΅ Π·Π½Π°Ρ‡Π΅Π½ΠΈΠ΅ ΠΏΠ΅Ρ€Π²ΠΎΠΉ Π΄Ρ€ΠΎΠ±ΠΈ.

So our question was to get the value of 2 ⁄ 7 + 1 ⁄ 3

so we are going to multiply 3 ⁄ 3 with 2 ⁄ 7 and 7 ⁄ 7 with 1 ⁄ 3

=> 3 ⁄ 3 x 2 ⁄ 7 + 1 ⁄ 3 Ρ… 7 ⁄ 7

=> 6 ⁄ 21 + 7 ⁄ 21

БСйчас Ρƒ нас нравятся Ρ„Ρ€Π°ΠΊΡ†ΠΈΠΈ. ΠœΡ‹ Π³ΠΎΡ‚ΠΎΠ²Ρ‹ Π΄ΠΎΠ±Π°Π²ΠΈΡ‚ΡŒ ΠΈΡ…, ΠΏΠΎΡΠΊΠΎΠ»ΡŒΠΊΡƒ ΠΌΡ‹ Π΄ΠΎΠ±Π°Π²ΠΈΠ»ΠΈ ΠΏΠΎΡ…ΠΎΠΆΠΈΠ΅ Π΄Ρ€ΠΎΠ±ΠΈ Π² Ρ€Π°Π·Π΄Π΅Π»Π΅ ΠΏΠΎΡ…ΠΎΠΆΠΈΡ… Π΄Ρ€ΠΎΠ±Π΅ΠΉ Π²Ρ‹ΡˆΠ΅.

Π’ΠΎΠ·ΡŒΠΌΠ΅ΠΌ 21 Π² качСствС ΠΎΠ±Ρ‰Π΅Π³ΠΎ значСния для Π½ΠΈΠΆΠ½Π΅Π³ΠΎ, ΠΏΠΎΡ‚ΠΎΠΌΡƒ Ρ‡Ρ‚ΠΎ ΠΎΠ±Π΅ Π΄Ρ€ΠΎΠ±ΠΈ ΠΈΠΌΠ΅ΡŽΡ‚ 21 Π² Π½ΠΈΠΆΠ½Π΅ΠΉ части ΠΈ добавят Π²Π΅Ρ€Ρ…Π½ΠΈΠ΅ значСния. 0006 13 ⁄ 21

Π”Ρ€ΡƒΠ³ΠΎΠΉ вопрос: ΠŸΠΎΠ»ΡƒΡ‡ΠΈΡ‚Π΅ Π·Π½Π°Ρ‡Π΅Π½ΠΈΠ΅ 2 ⁄ 7 1 ⁄ 3

. .

2 ⁄ 7 –  1 ⁄ 3

=> 3 ⁄ 3 x 2 ⁄ 7 – 1 ⁄ 3 x 7 ⁄ 7

=> 6 ⁄ 21 7 ⁄ 21

=> -1 ~ 21

9000 2
. ΠΈΠ»ΠΈ Π²Ρ‹Ρ‡Π΅ΡΡ‚ΡŒ?

Как ΡΠΊΠ»Π°Π΄Ρ‹Π²Π°Ρ‚ΡŒ ΠΈ Π²Ρ‹Ρ‡ΠΈΡ‚Π°Ρ‚ΡŒ Π΄Ρ€ΠΎΠ±ΠΈ, ΠΊΠΎΡ‚ΠΎΡ€Ρ‹Π΅ ΠΈΠΌΠ΅ΡŽΡ‚ Π±ΠΎΠ»Π΅Π΅ 2 Π·Π½Π°Ρ‡Π΅Π½ΠΈΠΉ

Π­Ρ‚ΠΎΡ‚ ΠΌΠ΅Ρ‚ΠΎΠ΄ Ρ‚Π°ΠΊΠΆΠ΅ ΠΌΠΎΠΆΠ½ΠΎ ΠΈΡΠΏΠΎΠ»ΡŒΠ·ΠΎΠ²Π°Ρ‚ΡŒ для слоТСния ΠΈ вычитания Π΄Ρ€ΠΎΠ±Π΅ΠΉ, ΠΊΠΎΡ‚ΠΎΡ€Ρ‹Π΅ ΠΈΠΌΠ΅ΡŽΡ‚ Ρ‚ΠΎΠ»ΡŒΠΊΠΎ 2 значСния, Π½ΠΎ Π²Ρ‹ΡˆΠ΅ΡƒΠΏΠΎΠΌΡΠ½ΡƒΡ‚Ρ‹ΠΉ ΠΌΠ΅Ρ‚ΠΎΠ΄ Π±Ρ‹Π» самым простым, поэтому ΠΌΡ‹ Π½Π΅ Π½Π΅ обсуТдайтС это Ρ‚Π°ΠΌ.

Как Π΄ΠΎΠ±Π°Π²ΠΈΡ‚ΡŒ ΠΈΡ…: 3 ⁄ 2 + 7 ⁄ 4 + 4 ⁄ 3

ΠŸΠ΅Ρ€Π²Ρ‹ΠΉ, Π½Π°ΠΌ Π½ΡƒΠΆΠ½ΠΎ ΠΏΠΎΠ»ΡƒΡ‡ΠΈΡ‚ΡŒ LCD (ΠΏΠΎ мСньшСй ΠΌΠ΅Ρ€Π΅, DENININTATR)

. Как ΠΏΠΎΠ»ΡƒΡ‡ΠΈΡ‚ΡŒ наимСньший ΠΎΠ±Ρ‰ΠΈΠΉ Π·Π½Π°ΠΌΠ΅Π½Π°Ρ‚Π΅Π»ΡŒ (LCD)?

Π£ΠΌΠ½ΠΎΠΆΡŒΡ‚Π΅ Π½ΠΈΠΆΠ½ΠΈΠ΅ числа Π½Π° 2, 3, 4 ΠΈ Ρ‚Π°ΠΊ далСС… ΠΏΠΎΠΊΠ° Π½Π΅ Π½Π°ΠΉΠ΄Π΅Ρ‚Π΅ наимСньший ΠΎΠ±Ρ‰ΠΈΠΉ Π·Π½Π°ΠΌΠ΅Π½Π°Ρ‚Π΅Π»ΡŒ для всСх 3.

2 4 3
2 Ρ… 4 8 6
3 Ρ… 6 12 9
4 Ρ… 8 16 12
5 Ρ… 10 20 15
6 Ρ… 12 24 18

Наш наимСньший ΠΎΠ±Ρ‰ΠΈΠΉ Π·Π½Π°ΠΌΠ΅Π½Π°Ρ‚Π΅Π»ΡŒ Ρ€Π°Π²Π΅Π½ 12, ΠΏΠΎΡ‚ΠΎΠΌΡƒ Ρ‡Ρ‚ΠΎ 12 β€” СдинствСнноС число, доступноС Π²ΠΎ всСх Ρ‚Ρ€Π΅Ρ… столбцах, ΠΊΠΎΡ‚ΠΎΡ€Ρ‹Π΅ ΠΌΡ‹ Π²Ρ‹Π΄Π΅Π»ΠΈΠ»ΠΈ Π·Π΅Π»Π΅Π½Ρ‹ΠΌ Ρ†Π²Π΅Ρ‚ΠΎΠΌ.

Наш вопрос Π±Ρ‹Π» 3 ⁄ 2 + 7 ⁄ 4 + 4 ⁄ 3 , Π³Π΄Π΅

  • ΠŸΠ΅Ρ€Π²ΠΎΠ΅ Π΄Π½ΠΎ составляСт 2, Ρ‡Ρ‚ΠΎ ΠΌΡ‹ ΡƒΠΌΠ½ΠΎΠΆΠ΅Π½Ρ‹ Π½Π° 6, Ρ‡Ρ‚ΠΎΠ±Ρ‹ ΠΏΠΎΠ»ΡƒΡ‡ΠΈΡ‚ΡŒ наимСньшСС ΠΎΠ±Ρ‰Π΅Π΅. посмотритС Π½Π° Ρ‚Π°Π±Π»ΠΈΡ†Ρƒ Π²Ρ‹ΡˆΠ΅), поэтому ΠΌΡ‹ собираСмся ΡƒΠΌΠ½ΠΎΠΆΠΈΡ‚ΡŒ 3 ⁄ 2 Π½Π° 6 ⁄ 6 . ΠŸΠΎΠ»ΡƒΡ‡Π°Π΅Ρ‚ΡΡ: 3 ⁄ 2 x 6 ⁄ 6
  • Π’Ρ‚ΠΎΡ€ΠΎΠ΅ Π½ΠΈΠΆΠ½Π΅Π΅ Π·Π½Π°Ρ‡Π΅Π½ΠΈΠ΅ β€” 4, ΠΊΠΎΡ‚ΠΎΡ€ΠΎΠ΅ ΠΌΡ‹ ΡƒΠΌΠ½ΠΎΠΆΠΈΠ»ΠΈ Π½Π° 3, Ρ‡Ρ‚ΠΎΠ±Ρ‹ ΠΏΠΎΠ»ΡƒΡ‡ΠΈΡ‚ΡŒ Π–Πš-дисплСй (см. Ρ‚Π°Π±Π»ΠΈΡ†Ρƒ Π²Ρ‹ΡˆΠ΅), поэтому ΠΌΡ‹ собираСмся ΡƒΠΌΠ½ΠΎΠΆΠΈΡ‚ΡŒ 7 ⁄ 4 Π½Π° 3 ⁄ 3 . Π—Π½Π°Ρ‡ΠΈΡ‚ Π±ΡƒΠ΄Π΅Ρ‚: 7 ⁄ 4 Ρ… 3 ⁄ 3
  • Π’Ρ€Π΅Ρ‚ΡŒΠ΅ Π½ΠΈΠΆΠ½Π΅Π΅ Π·Π½Π°Ρ‡Π΅Π½ΠΈΠ΅ Ρ€Π°Π²Π½ΠΎ 3, ΠΊΠΎΡ‚ΠΎΡ€ΠΎΠ΅ ΠΌΡ‹ ΡƒΠΌΠ½ΠΎΠΆΠΈΠ»ΠΈ Π½Π° 4, Ρ‡Ρ‚ΠΎΠ±Ρ‹ ΠΏΠΎΠ»ΡƒΡ‡ΠΈΡ‚ΡŒ Π–Πš-дисплСй (см. Ρ‚Π°Π±Π»ΠΈΡ†Ρƒ Π²Ρ‹ΡˆΠ΅), поэтому ΠΌΡ‹ собираСмся ΡƒΠΌΠ½ΠΎΠΆΠΈΡ‚ΡŒ 4 ⁄ 3 с 4 ⁄ 4 . Π—Π½Π°Ρ‡ΠΈΡ‚ Π±ΡƒΠ΄Π΅Ρ‚: 4 ⁄ 3 Ρ… 4 ⁄ 4

S0, 3 ⁄ 2 + 7 ⁄ 4 + 4 ⁄ 3

=> 3 ⁄ 2Β  x 6 ⁄ 6Β  + 7 ⁄ 4 x 3 ⁄ 3Β  + 4 ⁄ 3 x 4 ⁄ 4

ΠŸΠΎΠΌΠ½ΠΈΡ‚Π΅, Ρ‡Ρ‚ΠΎ Π² ΠΌΠ°Ρ‚Π΅ΠΌΠ°Ρ‚ΠΈΠΊΠ΅ ΠΌΡ‹ всСгда сначала ΡƒΠΌΠ½ΠΎΠΆΠ°Π΅ΠΌ ΠΏΠ΅Ρ€Π΅Π΄ слоТСниСм ΠΈΠ»ΠΈ Π²Ρ‹Ρ‡ΠΈΡ‚Π°Π½ΠΈΠ΅ΠΌ. Π­Ρ‚ΠΎ матСматичСскоС ΠΏΡ€Π°Π²ΠΈΠ»ΠΎ.

ПослС упрощСния. Π’Π°ΠΊΠΈΠΌ ΠΎΠ±Ρ€Π°Π·ΠΎΠΌ, ΠΌΡ‹ возьмСм 12 ΠΊΠ°ΠΊ ΠΎΠ±Ρ‹Ρ‡Π½Ρ‹Π΅ для Π½ΠΈΠΆΠ½ΠΈΡ… ΠΈ Π΄ΠΎΠ±Π°Π²ΠΈΠΌ Π²Π΅Ρ€Ρ…Π½ΠΈΠ΅ числа.

Π˜Ρ‚Π°ΠΊ, 18 ⁄ 12 + 21 ⁄ 12 + 16 ⁄ 12

= 55 ⁄ 12

Π’Π°ΠΊΠΈΠΌ ΠΎΠ±Ρ€Π°Π·ΠΎΠΌ, ΠΎΡ‚Π²Π΅Ρ‚ 55 ⁄ 12

Π”Ρ€ΡƒΠ³ΠΎΠΉ Вопрос: ΠŸΠΎΠ΄Ρ‚Π΅Ρ€Π΅ΠΆ 5

. Π”Ρ€ΡƒΠ³ΠΎΠΉ Вопрос: ΠΏΠΎΠ΄Ρ‚Π΅Ρ€Π΅ΠΆΠ½ΠΈΡ‚ 5

. 2 – 7 ⁄ 9
6 2 9
2 Ρ… 12 4 18
3 Ρ… 18 6 27
4 Ρ… 24 8 36
5 Ρ… 30 10 45
6 Ρ… 36 12 54
7 Ρ… 42 14 63
8 Ρ… 48 16 72
9 Ρ… 54 18 81

Π’Π°ΠΊΠΈΠΌ ΠΎΠ±Ρ€Π°Π·ΠΎΠΌ, наимСньший ΠΎΠ±Ρ‰ΠΈΠΉ Π·Π½Π°ΠΌΠ΅Π½Π°Ρ‚Π΅Π»ΡŒ для Π½ΠΈΠΆΠ½ΠΈΡ… Π·Π½Π°Ρ‡Π΅Π½ΠΈΠΉ (6, 2 ΠΈ 9) Ρ€Π°Π²Π΅Π½ 18.

Π’Π΅ΠΏΠ΅Ρ€ΡŒ ΠΌΡ‹ собираСмся ΡƒΠΌΠ½ΠΎΠΆΠΈΡ‚ΡŒ ΠΊΠ°ΠΆΠ΄ΡƒΡŽ Π΄Ρ€ΠΎΠ±ΡŒ Ρ‚Π°ΠΊ ΠΆΠ΅, ΠΊΠ°ΠΊ ΠΌΡ‹ Π΄Π΅Π»Π°Π»ΠΈ это Π² ΠΏΡ€Π΅Π΄Ρ‹Π΄ΡƒΡ‰Π΅ΠΌ вопросС, глядя Π½Π° Ρ‚Π°Π±Π»ΠΈΡ†Ρƒ.

  • 5 ⁄ 6 Π±ΡƒΠ΄Π΅Ρ‚ ΡƒΠΌΠ½ΠΎΠΆΠ΅Π½ΠΎ Π½Π° 3 ⁄ 3
  • 1 ⁄ 2 Π±ΡƒΠ΄Π΅Ρ‚ ΡƒΠΌΠ½ΠΎΠΆΠ΅Π½ΠΎ Π½Π° 9 ⁄9
  • 7 ⁄ 9 Π±ΡƒΠ΄Π΅Ρ‚ ΡƒΠΌΠ½ΠΎΠΆΠ΅Π½ΠΎ Π½Π° 2 ⁄ 2

S0, 5 ⁄ 6 – 1 ⁄ 2 – 7 ⁄ 9

=> 5 ⁄ 6Β  x 3 ⁄ 3Β  – 1 ⁄ 2 x 9 ⁄ 7 ⁄ 9 x 2 ⁄ 2

=> 15 ~ 18 – 9 ⁄ 18 – 14 ⁄ 18

Π’Π΅ΠΏΠ΅Ρ€ΡŒ Ρƒ нас ΠΎΠ΄ΠΈΠ½Π°ΠΊΠΎΠ²Ρ‹Π΅ Π΄Ρ€ΠΎΠ±ΠΈ. Π’Π°ΠΊΠΈΠΌ ΠΎΠ±Ρ€Π°Π·ΠΎΠΌ, ΠΌΡ‹ возьмСм 18 Π² качСствС ΠΎΠ±Ρ‰Π΅Π³ΠΎ значСния для Π½ΠΈΠΆΠ½ΠΈΡ… ΠΈ Π²Ρ‹Ρ‡Ρ‚Π΅ΠΌ Π²Π΅Ρ€Ρ…Π½ΠΈΠ΅ числа.

=> -8 ⁄ 18

=> -4 ⁄ 9

Как насчСт слоТСния ΠΈΠ»ΠΈ вычитания Π±ΠΎΠ»ΡŒΡˆΠΈΡ… чисСл, ΠΊΠΎΡ‚ΠΎΡ€Ρ‹Π΅ содСрТат Π΄Ρ€ΠΎΠ±ΠΈ?

Вопрос: — ΠŸΠΎΠ»ΡƒΡ‡ΠΈΡ‚Π΅ Π·Π½Π°Ρ‡Π΅Π½ΠΈΠ΅ для 35 ⁄ 105 + 70 ⁄ 80 + 42 ⁄ 90

Π‘ΠΎΠ²Π΅Ρ‚:- Когда Π½ΠΈΠΆΠ½ΠΈΠ΅ значСния Π΄Ρ€ΠΎΠ±Π΅ΠΉ Π½Π΅ ΡΠΎΠ²ΠΏΠ°Π΄Π°ΡŽΡ‚. Π’Ρ‹ всСгда Π΄ΠΎΠ»ΠΆΠ½Ρ‹ сначала ΡƒΠΏΡ€ΠΎΡΡ‚ΠΈΡ‚ΡŒ ΠΈΡ…, ΠΏΡ€Π΅ΠΆΠ΄Π΅ Ρ‡Π΅ΠΌ Π²Ρ‹Ρ‡ΠΈΡΠ»ΡΡ‚ΡŒ. Π­Ρ‚ΠΎ сэкономит вашС врСмя.

Let’s Simplify: 35 ⁄ 105 + 70 ⁄ 80 + 42 ⁄ 90

and you will get: 1 ⁄ 5 + 7 ⁄ 8 + 7 ⁄ 15

Π’Π΅ΠΏΠ΅Ρ€ΡŒ Π½Π°ΠΌ Π½ΡƒΠΆΠ½ΠΎ Π½Π°ΠΉΡ‚ΠΈ наимСньший ΠΎΠ±Ρ‰ΠΈΠΉ Π·Π½Π°ΠΌΠ΅Π½Π°Ρ‚Π΅Π»ΡŒ для чисСл 5, 8 ΠΈ 15

5 8 15
2 Ρ… 10 16 30
3 Ρ… 15 24 45
4 Ρ… 20 32 60
5 Ρ… 25 40 75
6 Ρ… 30 48 90
7 Ρ… 35 56 105
8 Ρ… 40 64 120
9 Ρ… 45 72 135
10 Ρ… 50 80 150
11 Ρ… 55 88 165
12 Ρ… 60 96 180
13 Ρ… 65 104 195
14 Ρ… 70 112 210
15 Ρ… 75 120 225
16 Ρ… 80 128 240
17 Ρ… 85 136 255
18 Ρ… 90 144 270
19 Ρ… 95 152 285
20 Ρ… 100 160 300
21 Ρ… 105 168 315
22 Ρ… 110 176 330
23 Ρ… 115 184 345
24 Ρ… 120 192 360

Π’Π°ΠΊΠΈΠΌ ΠΎΠ±Ρ€Π°Π·ΠΎΠΌ, ΠΌΡ‹ ΠΏΠΎΠ»ΡƒΡ‡ΠΈΠ»ΠΈ 120 Π² качСствС Π½Π°ΠΈΠΌΠ΅Π½Π΅Π΅ распространСнного знамСнатСля (Π–Πš -дисплСй) для 5, 8 ΠΈ 15

  • 1 ⁄ 5 Π±ΡƒΠ΄ΡƒΡ‚ ΡƒΠΌΠ½ΠΎΠΆΠ΅Π½Ρ‹ с 24 ⁄ 24 20202024024024024024024024024012
  • 7 ⁄ 8 Π±ΡƒΠ΄Π΅Ρ‚ ΡƒΠΌΠ½ΠΎΠΆΠ΅Π½ΠΎ Π½Π° 15 ⁄ 15
  • 7 ⁄ 15 Π±ΡƒΠ΄Π΅Ρ‚ ΡƒΠΌΠ½ΠΎΠΆΠ΅Π½ΠΎ Π½Π° 8 ⁄ 8

So, it will be written like this: 1 ⁄ 5Β  x 24 ⁄ 24 + 7 ⁄ 8Β  x 15 ⁄ 15Β  + 7 ⁄ 15 Ρ… 8 ⁄ 8

=> 24 ⁄ 120 + 105 ⁄ 120 + 5 6 900019 120

Π’Π΅ΠΏΠ΅Ρ€ΡŒ Ρƒ нас Π΅ΡΡ‚ΡŒ ΠΏΠΎΡ…ΠΎΠΆΠΈΠ΅ Π΄Ρ€ΠΎΠ±ΠΈ. ΠœΡ‹ возьмСм 120 Π² качСствС ΠΎΠ±Ρ‰Π΅Π³ΠΎ значСния для Π½ΠΈΠΆΠ½Π΅ΠΉ части ΠΈ Π΄ΠΎΠ±Π°Π²ΠΈΠΌ Π²Π΅Ρ€Ρ…Π½ΠΈΠ΅ значСния.

=> 185 ⁄ 120

=> ПослС упрощСния, Π²Ρ‹ ΠΏΠΎΠ»ΡƒΡ‡ΠΈΡ‚Π΅: 37 ⁄ 24

, Ρ‚Π°ΠΊ Ρ‡Ρ‚ΠΎ наш ΠΎΡ‚Π²Π΅Ρ‚ 37 ⁄ 24

9 37 ⁄ 24

. НС ΡΡ‚Π΅ΡΠ½ΡΠΉΡ‚Π΅ΡΡŒ ΠΊΠΎΠΌΠΌΠ΅Π½Ρ‚ΠΈΡ€ΠΎΠ²Π°Ρ‚ΡŒ.

ΠžΠΏΠ΅Ρ€Π°Ρ†ΠΈΠΈ Π½Π°Π΄ дробями | ΠœΠ°Ρ‚Π΅ΠΌΠ°Ρ‚ΠΈΠΊΠ° для Π³ΡƒΠΌΠ°Π½ΠΈΡ‚Π°Ρ€Π½Ρ‹Ρ… Π½Π°ΡƒΠΊ Corequisite

Π Π΅Π·ΡƒΠ»ΡŒΡ‚Π°Ρ‚Ρ‹ обучСния

  • Π‘Π»ΠΎΠΆΠ΅Π½ΠΈΠ΅ ΠΈΠ»ΠΈ Π²Ρ‹Ρ‡ΠΈΡ‚Π°Π½ΠΈΠ΅ Π΄Ρ€ΠΎΠ±Π΅ΠΉ.
  • УпроститС Π΄Ρ€ΠΎΠ±ΠΈ.
  • Π£ΠΌΠ½ΠΎΠΆΠ΅Π½ΠΈΠ΅ Π΄Ρ€ΠΎΠ±Π΅ΠΉ.
  • Π Π°Π·Π΄Π΅Π»ΠΈΡ‚ΡŒ Π΄Ρ€ΠΎΠ±ΠΈ.

Π‘Ρ‚ΡƒΠ΄Π΅Π½Ρ‚Ρ‹-ΠΌΠ°Ρ‚Π΅ΠΌΠ°Ρ‚ΠΈΠΊΠΈ ΠΈ Ρ€Π°Π±ΠΎΡ‚Π°ΡŽΡ‰ΠΈΠ΅ взрослыС часто ΠΎΠ±Π½Π°Ρ€ΡƒΠΆΠΈΠ²Π°ΡŽΡ‚, Ρ‡Ρ‚ΠΎ ΠΈΡ… знания ΠΎ Ρ‚ΠΎΠΌ, ΠΊΠ°ΠΊ ΡΠΊΠ»Π°Π΄Ρ‹Π²Π°Ρ‚ΡŒ, Π²Ρ‹Ρ‡ΠΈΡ‚Π°Ρ‚ΡŒ, ΡƒΠΌΠ½ΠΎΠΆΠ°Ρ‚ΡŒ ΠΈ Π΄Π΅Π»ΠΈΡ‚ΡŒ Π΄Ρ€ΠΎΠ±ΠΈ, Π·Π°Ρ€ΠΆΠ°Π²Π΅Π»ΠΈ ΠΎΡ‚ нСиспользования. ΠœΡ‹ склонны ΠΏΠΎΠ»Π°Π³Π°Ρ‚ΡŒΡΡ Π½Π° ΠΊΠ°Π»ΡŒΠΊΡƒΠ»ΡΡ‚ΠΎΡ€Ρ‹, ΠΊΠΎΡ‚ΠΎΡ€Ρ‹Π΅ Π΄Π΅Π»Π°ΡŽΡ‚ Π·Π° нас Π±ΠΎΠ»ΡŒΡˆΡƒΡŽ Ρ‡Π°ΡΡ‚ΡŒ Ρ€Π°Π±ΠΎΡ‚Ρ‹ с дробями. Π’Π΅ΠΌ Π½Π΅ ΠΌΠ΅Π½Π΅Π΅, студСнчСская Π°Π»Π³Π΅Π±Ρ€Π° создаСт Π½Π΅ΠΊΠΎΡ‚ΠΎΡ€Ρ‹Π΅ Π²Π°ΠΆΠ½Ρ‹Π΅ ΠΌΠ΅Ρ‚ΠΎΠ΄Ρ‹ Ρ€Π°Π±ΠΎΡ‚Ρ‹ с выраТСниями ΠΈ уравнСниями, основанныС Π½Π° опСрациях Π½Π°Π΄ дробями. ΠŸΠΎΡΡ‚ΠΎΠΌΡƒ Π²Π°ΠΆΠ½ΠΎ Π·Π°Π½ΠΎΠ²ΠΎ ΠΎΡΠ²ΠΎΠΈΡ‚ΡŒ эти Π½Π°Π²Ρ‹ΠΊΠΈ. Π­Ρ‚ΠΎΡ‚ Ρ€Π°Π·Π΄Π΅Π» Π½Π°ΠΏΠΎΠΌΠ½ΠΈΡ‚ Π²Π°ΠΌ, ΠΊΠ°ΠΊ Π²Ρ‹ΠΏΠΎΠ»Π½ΡΡ‚ΡŒ дСйствия Π½Π°Π΄ дробями. По ΠΌΠ΅Ρ€Π΅ прохоТдСния ΠΎΡΡ‚Π°Π²ΡˆΠ΅ΠΉΡΡ части курса Π²Ρ‹ ΠΌΠΎΠΆΠ΅Ρ‚Π΅ Π²ΠΎΠ·Π²Ρ€Π°Ρ‰Π°Ρ‚ΡŒΡΡ ΠΊ этому Ρ€Π°Π·Π΄Π΅Π»Ρƒ ΠΏΠΎ ΠΌΠ΅Ρ€Π΅ нСобходимости для быстрого напоминания ΠΎΠ± опСрациях с дробями.

ΠŸΡ€Π΅ΠΆΠ΄Π΅ Ρ‡Π΅ΠΌ ΠΌΡ‹ Π½Π°Ρ‡Π½Π΅ΠΌ, Π΄Π°Π²Π°ΠΉΡ‚Π΅ опрСдСлимся с Ρ‚Π΅Ρ€ΠΌΠΈΠ½ΠΎΠ»ΠΎΠ³ΠΈΠ΅ΠΉ.

  • ΠΏΡ€ΠΎΠΈΠ·Π²Π΅Π΄Π΅Π½ΠΈΠ΅: Ρ€Π΅Π·ΡƒΠ»ΡŒΡ‚Π°Ρ‚ умноТСния
  • Ρ„Π°ΠΊΡ‚ΠΎΡ€: Ρ‡Ρ‚ΠΎ-Ρ‚ΠΎ умноТаСтся — для Β [латСкс]3 \cdot 2 = 6[/латСкс] , ΠΈ [латСкс]3[/латСкс] ΠΈ [латСкс]2[/латСкс] ΡΠ²Π»ΡΡŽΡ‚ΡΡ мноТитСлями [латСкс]6[ /латСкс]
  • Ρ‡ΠΈΡΠ»ΠΈΡ‚Π΅Π»ΡŒ: вСрхняя Ρ‡Π°ΡΡ‚ΡŒ Π΄Ρ€ΠΎΠ±ΠΈ – Ρ‡ΠΈΡΠ»ΠΈΡ‚Π΅Π»ΡŒ Π΄Ρ€ΠΎΠ±ΠΈΒ [латСкс]\ΠΊΡ€ΡƒΠΏΠ½Ρ‹ΠΉ\Ρ„Ρ€Π°ΠΊ{2}{3}[/латСкс] Ρ€Π°Π²Π΅Π½ [латСкс]2[/латСкс]
  • Π·Π½Π°ΠΌΠ΅Π½Π°Ρ‚Π΅Π»ΡŒ: ниТняя Ρ‡Π°ΡΡ‚ΡŒ Π΄Ρ€ΠΎΠ±ΠΈ β€” Π·Π½Π°ΠΌΠ΅Π½Π°Ρ‚Π΅Π»ΡŒ Π΄Ρ€ΠΎΠ±ΠΈΒ [латСкс]\большой\фракция{2}{3}[/латСкс] Ρ€Π°Π²Π΅Π½ [латСкс]3[/латСкс]

ΠŸΡ€ΠΈΠΌΠ΅Ρ‡Π°Π½ΠΈΠ΅ ΠΎΠ± инструкциях

ΠžΠΏΡ€Π΅Π΄Π΅Π»Π΅Π½Π½Ρ‹Π΅ слова ΠΈΡΠΏΠΎΠ»ΡŒΠ·ΡƒΡŽΡ‚ΡΡ Π² ΡƒΡ‡Π΅Π±Π½ΠΈΠΊΠ°Ρ… ΠΏΠΎ ΠΌΠ°Ρ‚Π΅ΠΌΠ°Ρ‚ΠΈΠΊΠ΅ ΠΈ учитСлями, Ρ‡Ρ‚ΠΎΠ±Ρ‹ ΠΏΡ€Π΅Π΄ΠΎΡΡ‚Π°Π²ΠΈΡ‚ΡŒ учащимся инструкции ΠΎ Ρ‚ΠΎΠΌ, Ρ‡Ρ‚ΠΎ Π΄Π΅Π»Π°Ρ‚ΡŒ с Π΄Π°Π½Π½ΠΎΠΉ Π·Π°Π΄Π°Ρ‡Π΅ΠΉ. НапримСр, Π²Ρ‹ ΠΌΠΎΠΆΠ΅Ρ‚Π΅ ΡƒΠ²ΠΈΠ΄Π΅Ρ‚ΡŒ Ρ‚Π°ΠΊΠΈΠ΅ инструкции, ΠΊΠ°ΠΊ Π½Π°ΠΉΡ‚ΠΈ ΠΈΠ»ΠΈ ΡƒΠΏΡ€ΠΎΡΡ‚ΠΈΡ‚ΡŒ. Β Π’Π°ΠΆΠ½ΠΎ ΠΏΠΎΠ½ΠΈΠΌΠ°Ρ‚ΡŒ, Ρ‡Ρ‚ΠΎ ΠΎΠ·Π½Π°Ρ‡Π°ΡŽΡ‚ эти слова, Ρ‡Ρ‚ΠΎΠ±Ρ‹ ΡƒΡΠΏΠ΅ΡˆΠ½ΠΎ Ρ€Π΅ΡˆΠ°Ρ‚ΡŒ Π·Π°Π΄Π°Ρ‡ΠΈ этого курса. Π’ΠΎΡ‚ ΠΊΡ€Π°Ρ‚ΠΊΠΈΠΉ список Π½Π΅ΠΊΠΎΡ‚ΠΎΡ€Ρ‹Ρ… инструкций ΠΏΠΎ Ρ€Π΅ΡˆΠ΅Π½ΠΈΡŽ ΠΏΡ€ΠΎΠ±Π»Π΅ΠΌ вмСстС с ΠΈΡ… описаниями, ΠΏΠΎΡΠΊΠΎΠ»ΡŒΠΊΡƒ ΠΎΠ½ΠΈ Π±ΡƒΠ΄ΡƒΡ‚ ΠΈΡΠΏΠΎΠ»ΡŒΠ·ΠΎΠ²Π°Ρ‚ΡŒΡΡ Π² этом ΠΌΠΎΠ΄ΡƒΠ»Π΅.

Π˜Π½ΡΡ‚Ρ€ΡƒΠΊΡ†ΠΈΡ Π˜Π½Ρ‚Π΅Ρ€ΠΏΡ€Π΅Ρ‚Π°Ρ†ΠΈΡ
Найти Π’Ρ‹ΠΏΠΎΠ»Π½ΠΈΡ‚ΡŒ ΡƒΠΊΠ°Π·Π°Π½Π½Ρ‹Π΅ матСматичСскиС ΠΎΠΏΠ΅Ρ€Π°Ρ†ΠΈΠΈ, ΠΊΠΎΡ‚ΠΎΡ€Ρ‹Π΅ ΠΌΠΎΠ³ΡƒΡ‚ Π²ΠΊΠ»ΡŽΡ‡Π°Ρ‚ΡŒ Π² сСбя слоТСниС, Π²Ρ‹Ρ‡ΠΈΡ‚Π°Π½ΠΈΠ΅, ΡƒΠΌΠ½ΠΎΠΆΠ΅Π½ΠΈΠ΅, Π΄Π΅Π»Π΅Π½ΠΈΠ΅ (ΠΏΠΎΠ·ΠΆΠ΅ использованиС слова Π½Π°ΠΉΡ‚ΠΈ Π±ΡƒΠ΄Π΅Ρ‚ Ρ€Π°ΡΡˆΠΈΡ€Π΅Π½ΠΎ Π΄ΠΎ Ρ€Π΅ΡˆΠ΅Π½ΠΈΡ ΡƒΡ€Π°Π²Π½Π΅Π½ΠΈΠΉ, ΠΊΠ°ΠΊ Π² Π½Π°ΠΉΡ‚ΠΈ Π·Π½Π°Ρ‡Π΅Π½ΠΈΠ΅ ΠΏΠ΅Ρ€Π΅ΠΌΠ΅Π½Π½ΠΎΠΉ).
Β Π£ΠΏΡ€ΠΎΡΡ‚ΠΈΡ‚ΡŒ 1) Π’Ρ‹ΠΏΠΎΠ»Π½ΡΡ‚ΡŒ ΡƒΠΊΠ°Π·Π°Π½Π½Ρ‹Π΅ матСматичСскиС дСйствия, Π²ΠΊΠ»ΡŽΡ‡Π°Ρ слоТСниС, Π²Ρ‹Ρ‡ΠΈΡ‚Π°Π½ΠΈΠ΅, ΡƒΠΌΠ½ΠΎΠΆΠ΅Π½ΠΈΠ΅, Π΄Π΅Π»Π΅Π½ΠΈΠ΅

2) Π—Π°ΠΏΠΈΡˆΠΈΡ‚Π΅ ΠΌΠ°Ρ‚Π΅ΠΌΠ°Ρ‚ΠΈΡ‡Π΅ΡΠΊΡƒΡŽ Ρ„ΠΎΡ€ΠΌΡƒΠ»ΠΈΡ€ΠΎΠ²ΠΊΡƒ Π² Π½Π°ΠΈΠΌΠ΅Π½ΡŒΡˆΠΈΡ… выраТСниях, Ρ‡Ρ‚ΠΎΠ±Ρ‹ Π½Π΅ Π±Ρ‹Π»ΠΎ Π΄Ρ€ΡƒΠ³ΠΈΡ… матСматичСских ΠΎΠΏΠ΅Ρ€Π°Ρ†ΠΈΠΉ, ΠΊΠΎΡ‚ΠΎΡ€Ρ‹Π΅ ΠΌΠΎΠΆΠ½ΠΎ Π±Ρ‹Π»ΠΎ Π±Ρ‹ Π²Ρ‹ΠΏΠΎΠ»Π½ΠΈΡ‚ΡŒ β€” часто встрСчаСтся Π² Π·Π°Π΄Π°Ρ‡Π°Ρ…, связанных с дробями ΠΈ порядком ΠΎΠΏΠ΅Ρ€Π°Ρ†ΠΈΠΉ

ΠžΡ†Π΅Π½ΠΊΠ° 1) Π’Ρ‹ΠΏΠΎΠ»Π½ΡΡ‚ΡŒ ΡƒΠΊΠ°Π·Π°Π½Π½Ρ‹Π΅ матСматичСскиС дСйствия, Π²ΠΊΠ»ΡŽΡ‡Π°Ρ слоТСниС, Π²Ρ‹Ρ‡ΠΈΡ‚Π°Π½ΠΈΠ΅, ΡƒΠΌΠ½ΠΎΠΆΠ΅Π½ΠΈΠ΅, Π΄Π΅Π»Π΅Π½ΠΈΠ΅

2) ΠŸΠΎΠ΄ΡΡ‚Π°Π²ΠΈΡ‚ΡŒ Π·Π°Π΄Π°Π½Π½ΠΎΠ΅ Π·Π½Π°Ρ‡Π΅Π½ΠΈΠ΅ ΠΏΠ΅Ρ€Π΅ΠΌΠ΅Π½Π½ΠΎΠΉ Π² Π²Ρ‹Ρ€Π°ΠΆΠ΅Π½ΠΈΠ΅ ΠΈ Π·Π°Ρ‚Π΅ΠΌ Π²Ρ‹ΠΏΠΎΠ»Π½ΠΈΡ‚ΡŒ ΡƒΠΊΠ°Π·Π°Π½Π½Ρ‹Π΅ матСматичСскиС ΠΎΠΏΠ΅Ρ€Π°Ρ†ΠΈΠΈ

Π£ΠΌΠ΅Π½ΡŒΡˆΠΈΡ‚ΡŒ Π—Π°ΠΏΠΈΡˆΠΈΡ‚Π΅ матСматичСскоС Π²Ρ‹Ρ€Π°ΠΆΠ΅Π½ΠΈΠ΅ Π² наимСньшСм ΠΈΠ»ΠΈ минимальном Π²Ρ‹Ρ€Π°ΠΆΠ΅Π½ΠΈΠΈ, Ρ‡Ρ‚ΠΎΠ±Ρ‹ Π½Π΅ Π±Ρ‹Π»ΠΎ Π΄Ρ€ΡƒΠ³ΠΈΡ… матСматичСских ΠΎΠΏΠ΅Ρ€Π°Ρ†ΠΈΠΉ, ΠΊΠΎΡ‚ΠΎΡ€Ρ‹Π΅ ΠΌΠΎΠΆΠ½ΠΎ Π±Ρ‹Π»ΠΎ Π±Ρ‹ Π²Ρ‹ΠΏΠΎΠ»Π½ΠΈΡ‚ΡŒ β€” часто встрСчаСтся Π² Π·Π°Π΄Π°Ρ‡Π°Ρ…, связанных с дробями ΠΈΠ»ΠΈ Π΄Π΅Π»Π΅Π½ΠΈΠ΅ΠΌ

Π‘Π»ΠΎΠΆΠ΅Π½ΠΈΠ΅ Π΄Ρ€ΠΎΠ±Π΅ΠΉ

Когда Π²Π°ΠΌ Π½ΡƒΠΆΠ½ΠΎ ΡΠΊΠ»Π°Π΄Ρ‹Π²Π°Ρ‚ΡŒ ΠΈΠ»ΠΈ Π²Ρ‹Ρ‡ΠΈΡ‚Π°Ρ‚ΡŒ Π΄Ρ€ΠΎΠ±ΠΈ, сначала Π½ΡƒΠΆΠ½ΠΎ ΡƒΠ±Π΅Π΄ΠΈΡ‚ΡŒΡΡ, Ρ‡Ρ‚ΠΎ Π΄Ρ€ΠΎΠ±ΠΈ ΠΈΠΌΠ΅ΡŽΡ‚ ΠΎΠ΄ΠΈΠ½Π°ΠΊΠΎΠ²Ρ‹ΠΉ Π·Π½Π°ΠΌΠ΅Π½Π°Ρ‚Π΅Π»ΡŒ. Π—Π½Π°ΠΌΠ΅Π½Π°Ρ‚Π΅Π»ΡŒ Π³ΠΎΠ²ΠΎΡ€ΠΈΡ‚ Π²Π°ΠΌ, Π½Π° сколько частСй Ρ€Π°Π·Π±ΠΈΡ‚ΠΎ Ρ†Π΅Π»ΠΎΠ΅, Π° Ρ‡ΠΈΡΠ»ΠΈΡ‚Π΅Π»ΡŒ Π³ΠΎΠ²ΠΎΡ€ΠΈΡ‚ Π²Π°ΠΌ, сколько ΠΈΠ· этих частСй Π²Ρ‹ ΠΈΡΠΏΠΎΠ»ΡŒΠ·ΡƒΠ΅Ρ‚Π΅.

ΠšΠΎΠ½Ρ†Π΅ΠΏΡ†ΠΈΡ «части Ρ†Π΅Π»ΠΎΠ³ΠΎΒ» ΠΌΠΎΠΆΠ΅Ρ‚ Π±Ρ‹Ρ‚ΡŒ смодСлирована с ΠΏΠΎΠΌΠΎΡ‰ΡŒΡŽ ΠΏΠΈΡ†Ρ†Ρ‹ ΠΈ кусочков ΠΏΠΈΡ†Ρ†Ρ‹. НапримСр, ΠΏΡ€Π΅Π΄ΡΡ‚Π°Π²ΡŒΡ‚Π΅, Ρ‡Ρ‚ΠΎ ΠΏΠΈΡ†Ρ†Π° Ρ€Π°Π·Ρ€Π΅Π·Π°Π½Π° Π½Π° [латСкс]4[/латСкс] куска, ΠΈ ΠΊΡ‚ΠΎ-Ρ‚ΠΎ Π±Π΅Ρ€Π΅Ρ‚ [латСкс]1[/латСкс] кусок. Π’Π΅ΠΏΠ΅Ρ€ΡŒ [latex]\Large\frac{1}{4}[/latex]Β ΠΈΠ· ΠΏΠΈΡ†Ρ†Ρ‹ исчСзаСт, Π° [latex]\Large\frac{3}{4}[/latex] остаСтся. ΠžΠ±Ρ€Π°Ρ‚ΠΈΡ‚Π΅ Π²Π½ΠΈΠΌΠ°Π½ΠΈΠ΅, Ρ‡Ρ‚ΠΎ ΠΎΠ±Π΅ эти Π΄Ρ€ΠΎΠ±ΠΈ ΠΈΠΌΠ΅ΡŽΡ‚ Π·Π½Π°ΠΌΠ΅Π½Π°Ρ‚Π΅Π»ΡŒ [латСкс]4[/латСкс], ΠΊΠΎΡ‚ΠΎΡ€Ρ‹ΠΉ относится ΠΊ количСству Π»ΠΎΠΌΡ‚ΠΈΠΊΠΎΠ², Π½Π° ΠΊΠΎΡ‚ΠΎΡ€Ρ‹Π΅ Π±Ρ‹Π»Π° Ρ€Π°Π·Ρ€Π΅Π·Π°Π½Π° вся ΠΏΠΈΡ†Ρ†Π°. Π§Ρ‚ΠΎ, Ссли Ρƒ вас Π΅ΡΡ‚ΡŒ Π΅Ρ‰Π΅ ΠΎΠ΄Π½Π° ΠΏΠΈΡ†Ρ†Π°, разрСзанная Π½Π° [латСкс]8[/латСкс] Ρ€Π°Π²Π½Ρ‹Ρ… частСй, ΠΈ [латСкс]3[/латСкс] ΠΈΠ· этих частСй исчСзли, оставив [латСкс]\большой\Ρ„Ρ€Π°ΠΊ{5}{8} [/латСкс]?

Как ΠΎΠΏΠΈΡΠ°Ρ‚ΡŒ ΠΎΠ±Ρ‰Π΅Π΅ количСство ΠΎΡΡ‚Π°Π²ΡˆΠ΅ΠΉΡΡ ΠΏΠΈΡ†Ρ†Ρ‹ ΠΎΠ΄Π½ΠΈΠΌ числом, Π° Π½Π΅ двумя Ρ€Π°Π·Π½Ρ‹ΠΌΠΈ дробями? Π’Π°ΠΌ Π½ΡƒΠΆΠ΅Π½ ΠΎΠ±Ρ‰ΠΈΠΉ Π·Π½Π°ΠΌΠ΅Π½Π°Ρ‚Π΅Π»ΡŒ, тСхничСски Π½Π°Π·Ρ‹Π²Π°Π΅ΠΌΡ‹ΠΉ наимСньшим ΠΎΠ±Ρ‰ΠΈΠΌ ΠΊΡ€Π°Ρ‚Π½Ρ‹ΠΌ . ΠŸΠΎΠΌΠ½ΠΈΡ‚Π΅, что Ссли число ΠΊΡ€Π°Ρ‚Π½ΠΎ Π΄Ρ€ΡƒΠ³ΠΎΠΌΡƒ, Π²Ρ‹ ΠΌΠΎΠΆΠ΅Ρ‚Π΅ Ρ€Π°Π·Π΄Π΅Π»ΠΈΡ‚ΡŒ ΠΈΡ… ΠΈ Π½Π΅ ΠΏΠΎΠ»ΡƒΡ‡ΠΈΡ‚ΡŒ остатка.

Один ΠΈΠ· способов Π½Π°ΠΉΡ‚ΠΈ наимСньшСС ΠΎΠ±Ρ‰Π΅Π΅ ΠΊΡ€Π°Ρ‚Π½ΠΎΠ΅ Π΄Π²ΡƒΡ… ΠΈΠ»ΠΈ Π±ΠΎΠ»Π΅Π΅ чисСл – сначала ΡƒΠΌΠ½ΠΎΠΆΠΈΡ‚ΡŒ ΠΊΠ°ΠΆΠ΄ΠΎΠ΅ ΠΈΠ· Π½ΠΈΡ… Π½Π°Β [латСкс]1, 2, 3, 4[/латСкс] ΠΈΒ Ρ‚.Β Π΄. НапримСр, Π½Π°ΠΉΡ‚ΠΈ наимСньшСС ΠΎΠ±Ρ‰Π΅Π΅ ΠΊΡ€Π°Ρ‚Π½ΠΎΠ΅ [латСкс] 2[/латСкс] ΠΈ [латСкс]5[/латСкс].

Π‘Π½Π°Ρ‡Π°Π»Π° пСрСчислитС всС ΠΊΡ€Π°Ρ‚Π½Ρ‹Π΅ [латСкс]2[/латСкс]: Π—Π°Ρ‚Π΅ΠΌ пСрСчислитС всС числа, ΠΊΡ€Π°Ρ‚Π½Ρ‹Π΅ 5:
[латСкс]2\cdot 1 = 2[/латСкс] [латСкс]5\cdot 1 = 5[/латСкс]
[латСкс]2\cdot 2 = 4[/латСкс] [латСкс]5\cdot 2 = 10[/латСкс]
[латСкс]2\cdot 3 = 6[/латСкс] [латСкс]5\cdot 3 = 15[/латСкс]
[латСкс]2\cdot 4 = 8[/латСкс] [латСкс]5\cdot 4 = 20[/латСкс]
[латСкс]2\cdot 5 = 10[/латСкс] [латСкс]5\cdot 5 = 25[/латСкс]

НаимСньшСС ΠΊΡ€Π°Ρ‚Π½ΠΎΠ΅, ΠΊΠΎΡ‚ΠΎΡ€ΠΎΠ΅ Ρƒ Π½ΠΈΡ… Π΅ΡΡ‚ΡŒ, Π±ΡƒΠ΄Π΅Ρ‚ ΠΎΠ±Ρ‰ΠΈΠΌ Π·Π½Π°ΠΌΠ΅Π½Π°Ρ‚Π΅Π»Π΅ΠΌ, ΠΈΡΠΏΠΎΠ»ΡŒΠ·ΡƒΠ΅ΠΌΡ‹ΠΌ для прСобразования ΠΊΠ°ΠΆΠ΄ΠΎΠΉ Π΄Ρ€ΠΎΠ±ΠΈ Π² эквивалСнтныС Π΄Ρ€ΠΎΠ±ΠΈ. Π‘ΠΌ. ΠΏΡ€ΠΈΠΌΠ΅Ρ€ Π½ΠΈΠΆΠ΅ для дСмонстрации нашСй Π·Π°Π΄Π°Ρ‡ΠΈ ΠΎ ΠΏΠΈΡ†Ρ†Π΅.

ΠŸΡ€ΠΈΠΌΠ΅Ρ€

Π’ ΠΎΠ΄Π½ΠΎΠΉ ΠΏΠΈΡ†Ρ†Π΅, Ρ€Π°Π·Ρ€Π΅Π·Π°Π½Π½ΠΎΠΉ Π½Π° Ρ‡Π΅Ρ‚Ρ‹Ρ€Π΅ Π»ΠΎΠΌΡ‚ΠΈΠΊΠ°, ΠΎΠ΄ΠΈΠ½ отсутствуСт. Другая ΠΏΠΈΡ†Ρ†Π° Ρ‚ΠΎΠ³ΠΎ ΠΆΠ΅ Ρ€Π°Π·ΠΌΠ΅Ρ€Π° Π±Ρ‹Π»Π° Ρ€Π°Π·Ρ€Π΅Π·Π°Π½Π° Π½Π° восСмь частСй, Ρ‚Ρ€ΠΈ ΠΈΠ· ΠΊΠΎΡ‚ΠΎΡ€Ρ‹Ρ… Π±Ρ‹Π»ΠΈ ΡƒΠ΄Π°Π»Π΅Π½Ρ‹. ΠžΠΏΠΈΡˆΠΈΡ‚Π΅ ΠΎΠ±Ρ‰Π΅Π΅ количСство ΠΏΠΈΡ†Ρ†Ρ‹, ΠΎΡΡ‚Π°Π²ΡˆΠ΅ΠΉΡΡ Π² Π΄Π²ΡƒΡ… ΠΏΠΈΡ†Ρ†Π°Ρ…, ΠΈΡΠΏΠΎΠ»ΡŒΠ·ΡƒΡ ΠΎΠ±Ρ‰ΠΈΠ΅ Ρ‚Π΅Ρ€ΠΌΠΈΠ½Ρ‹.

ΠŸΠΎΠΊΠ°Π·Π°Ρ‚ΡŒ Ρ€Π΅ΡˆΠ΅Π½ΠΈΠ΅

Π§Ρ‚ΠΎΠ±Ρ‹ ΡΠ»ΠΎΠΆΠΈΡ‚ΡŒ Π΄Ρ€ΠΎΠ±ΠΈ с Ρ€Π°Π·Π½Ρ‹ΠΌΠΈ знамСнатСлями, сначала ΠΏΠ΅Ρ€Π΅ΠΏΠΈΡˆΠΈΡ‚Π΅ ΠΈΡ… с ΠΎΠ΄ΠΈΠ½Π°ΠΊΠΎΠ²Ρ‹ΠΌΠΈ знамСнатСлями. Π—Π°Ρ‚Π΅ΠΌ Π΄ΠΎΠ±Π°Π²ΡŒΡ‚Π΅ ΠΈΠ»ΠΈ Π²Ρ‹Ρ‡Ρ‚ΠΈΡ‚Π΅ числитСли Π½Π°Π΄ ΠΎΠ±Ρ‰ΠΈΠΌ Π·Π½Π°ΠΌΠ΅Π½Π°Ρ‚Π΅Π»Π΅ΠΌ.

Π‘Π»ΠΎΠΆΠ΅Π½ΠΈΠ΅ Π΄Ρ€ΠΎΠ±Π΅ΠΉ с Ρ€Π°Π·Π½Ρ‹ΠΌΠΈ знамСнатСлями

  1. НайдитС ΠΎΠ±Ρ‰ΠΈΠΉ Π·Π½Π°ΠΌΠ΅Π½Π°Ρ‚Π΅Π»ΡŒ.
  2. ΠŸΠ΅Ρ€Π΅ΠΏΠΈΡˆΠΈΡ‚Π΅ ΠΊΠ°ΠΆΠ΄ΡƒΡŽ Π΄Ρ€ΠΎΠ±ΡŒ Π² Π²ΠΈΠ΄Π΅ эквивалСнтной Π΄Ρ€ΠΎΠ±ΠΈ, ΠΈΡΠΏΠΎΠ»ΡŒΠ·ΡƒΡ ΠΎΠ±Ρ‰ΠΈΠΉ Π·Π½Π°ΠΌΠ΅Π½Π°Ρ‚Π΅Π»ΡŒ.
  3. Π’Π΅ΠΏΠ΅Ρ€ΡŒ, ΠΊΠΎΠ³Π΄Π° Π΄Ρ€ΠΎΠ±ΠΈ ΠΈΠΌΠ΅ΡŽΡ‚ ΠΎΠ±Ρ‰ΠΈΠΉ Π·Π½Π°ΠΌΠ΅Π½Π°Ρ‚Π΅Π»ΡŒ, ΠΌΠΎΠΆΠ½ΠΎ ΡΠ»ΠΎΠΆΠΈΡ‚ΡŒ числитСли.
  4. УпроститС, ΡƒΠ±Ρ€Π°Π² всС ΠΎΠ±Ρ‰ΠΈΠ΅ ΠΌΠ½ΠΎΠΆΠΈΡ‚Π΅Π»ΠΈ Π² числитСлС ΠΈ Π·Π½Π°ΠΌΠ΅Π½Π°Ρ‚Π΅Π»Π΅.

Π£ΠΏΡ€ΠΎΡΡ‚ΠΈΡ‚ΡŒ Π΄Ρ€ΠΎΠ±ΡŒ

Π’ ΠΌΠ°Ρ‚Π΅ΠΌΠ°Ρ‚ΠΈΠΊΠ΅ принято ΠΏΡ€Π΅Π΄ΡΡ‚Π°Π²Π»ΡΡ‚ΡŒ Π΄Ρ€ΠΎΠ±ΠΈ Π² наимСньшСм Π²Ρ‹Ρ€Π°ΠΆΠ΅Π½ΠΈΠΈ. ΠœΡ‹ Π½Π°Π·Ρ‹Π²Π°Π΅ΠΌ эту ΠΏΡ€Π°ΠΊΡ‚ΠΈΠΊΡƒ ΡƒΠΏΡ€ΠΎΡ‰Π΅Π½ΠΈΠ΅ΠΌ ΠΈΠ»ΠΈ сокращСниСм Π΄Ρ€ΠΎΠ±ΠΈ, ΠΈ Π΅Π΅ ΠΌΠΎΠΆΠ½ΠΎ ΠΎΡΡƒΡ‰Π΅ΡΡ‚Π²ΠΈΡ‚ΡŒ ΠΏΡƒΡ‚Π΅ΠΌ сокращСния (раздСлСния) ΠΎΠ±Ρ‰ΠΈΡ… ΠΌΠ½ΠΎΠΆΠΈΡ‚Π΅Π»Π΅ΠΉ Π² числитСлС ΠΈ Π·Π½Π°ΠΌΠ΅Π½Π°Ρ‚Π΅Π»Π΅ Π΄Ρ€ΠΎΠ±ΠΈ. ΠœΡ‹ ΠΌΠΎΠΆΠ΅ΠΌ ΡΠ΄Π΅Π»Π°Ρ‚ΡŒ это, ΠΏΠΎΡ‚ΠΎΠΌΡƒ Ρ‡Ρ‚ΠΎ Π΄Ρ€ΠΎΠ±ΡŒ прСдставляСт собой Π΄Π΅Π»Π΅Π½ΠΈΠ΅, ΠΈ для любого числа [latex]a[/latex], [latex]\dfrac{a}{a}=1[/latex].

НапримСр, Ρ‡Ρ‚ΠΎΠ±Ρ‹ ΡƒΠΏΡ€ΠΎΡΡ‚ΠΈΡ‚ΡŒ [латСкс]\dfrac{6}{9}[/latex] Π²Ρ‹ ΠΌΠΎΠΆΠ΅Ρ‚Π΅ ΠΏΠ΅Ρ€Π΅ΠΏΠΈΡΠ°Ρ‚ΡŒ [latex]6[/latex] ΠΈ [latex]9[/latex]Β , ΠΈΡΠΏΠΎΠ»ΡŒΠ·ΡƒΡ наимСньшиС Π²ΠΎΠ·ΠΌΠΎΠΆΠ½Ρ‹Π΅ ΠΌΠ½ΠΎΠΆΠΈΡ‚Π΅Π»ΠΈ ΡΠ»Π΅Π΄ΡƒΡŽΡ‰ΠΈΠΌ ΠΎΠ±Ρ€Π°Π·ΠΎΠΌ:

[latex]\dfrac{6}{9}=\dfrac{ 2\cdot3}{3\cdot3}[/latex]

ΠŸΠΎΡΠΊΠΎΠ»ΡŒΠΊΡƒ [latex]3[/latex] Π΅ΡΡ‚ΡŒ ΠΈ Π² числитСлС, ΠΈ Π² Π·Π½Π°ΠΌΠ΅Π½Π°Ρ‚Π΅Π»Π΅, Π° Π΄Ρ€ΠΎΠ±ΠΈ ΠΌΠΎΠΆΠ½ΠΎ ΡΡ‡ΠΈΡ‚Π°Ρ‚ΡŒ Π΄Π΅Π»Π΅Π½ΠΈΠ΅ΠΌ, ΠΌΡ‹ ΠΌΠΎΠΆΠ΅ΠΌ Ρ€Π°Π·Π΄Π΅Π»ΠΈΡ‚ΡŒ [latex]3[ /latex] Π²Π²Π΅Ρ€Ρ…Ρƒ ΠΈ [latex]3[/latex] Π²Π½ΠΈΠ·Ρƒ, Ρ‡Ρ‚ΠΎΠ±Ρ‹ ΡΠΎΠΊΡ€Π°Ρ‚ΠΈΡ‚ΡŒ Π΄ΠΎ [latex]1[/latex].

[латСкс]\dfrac{6}{9}=\dfrac{2\cdot\cancel{3}}{3\cdot\cancel{3}}=\dfrac{2\cdot1}{3}=\dfrac {2}{3}[/латСкс]

Π’ ΡΠ»Π΅Π΄ΡƒΡŽΡ‰Π΅ΠΌ ΠΏΡ€ΠΈΠΌΠ΅Ρ€Π΅ ΠΏΠΎΠΊΠ°Π·Π°Π½ΠΎ, ΠΊΠ°ΠΊ ΡΠ»ΠΎΠΆΠΈΡ‚ΡŒ Π΄Π²Π΅ Π΄Ρ€ΠΎΠ±ΠΈ с Ρ€Π°Π·Π½Ρ‹ΠΌΠΈ знамСнатСлями, Π° Π·Π°Ρ‚Π΅ΠΌ ΡƒΠΏΡ€ΠΎΡΡ‚ΠΈΡ‚ΡŒ ΠΎΡ‚Π²Π΅Ρ‚.

ΠŸΡ€ΠΈΠΌΠ΅Ρ€

Π”ΠΎΠ±Π°Π²ΠΈΡ‚ΡŒ [латСкс]\Π‘ΠΎΠ»ΡŒΡˆΠΎΠΉ\фракция{2}{3}+\Π‘ΠΎΠ»ΡŒΡˆΠΎΠΉ\фракция{1}{5}[/латСкс]. УпроститС ΠΎΡ‚Π²Π΅Ρ‚.

ΠŸΠΎΠΊΠ°Π·Π°Ρ‚ΡŒ Ρ€Π΅ΡˆΠ΅Π½ΠΈΠ΅

Π’Ρ‹ ΠΌΠΎΠΆΠ΅Ρ‚Π΅ Π½Π°ΠΉΡ‚ΠΈ ΠΎΠ±Ρ‰ΠΈΠΉ Π·Π½Π°ΠΌΠ΅Π½Π°Ρ‚Π΅Π»ΡŒ, найдя ΠΎΠ±Ρ‰ΠΈΠ΅ ΠΊΡ€Π°Ρ‚Π½Ρ‹Π΅ Π·Π½Π°ΠΌΠ΅Π½Π°Ρ‚Π΅Π»Π΅ΠΉ. НаимСньшСС ΠΎΠ±Ρ‰Π΅Π΅ ΠΊΡ€Π°Ρ‚Π½ΠΎΠ΅ являСтся самым простым Π² использовании.

ΠŸΡ€ΠΈΠΌΠ΅Ρ€

Π”ΠΎΠ±Π°Π²ΠΈΡ‚ΡŒΒ [latex]\Large\frac{3}{7}+\Large\frac{2}{21}[/latex]. УпроститС ΠΎΡ‚Π²Π΅Ρ‚.

ΠŸΠΎΠΊΠ°Π·Π°Ρ‚ΡŒ Ρ€Π΅ΡˆΠ΅Π½ΠΈΠ΅

Π’ ΡΠ»Π΅Π΄ΡƒΡŽΡ‰Π΅ΠΌ Π²ΠΈΠ΄Π΅ΠΎ Π²Ρ‹ ΡƒΠ²ΠΈΠ΄ΠΈΡ‚Π΅ ΠΏΡ€ΠΈΠΌΠ΅Ρ€ слоТСния Π΄Π²ΡƒΡ… Π΄Ρ€ΠΎΠ±Π΅ΠΉ с Ρ€Π°Π·Π½Ρ‹ΠΌΠΈ знамСнатСлями.

Π’Ρ‹ Ρ‚Π°ΠΊΠΆΠ΅ ΠΌΠΎΠΆΠ΅Ρ‚Π΅ ΡΠ»ΠΎΠΆΠΈΡ‚ΡŒ Π±ΠΎΠ»Π΅Π΅ Π΄Π²ΡƒΡ… Π΄Ρ€ΠΎΠ±Π΅ΠΉ, Ссли сначала Π½Π°ΠΉΠ΄Π΅Ρ‚Π΅ для Π½ΠΈΡ… ΠΎΠ±Ρ‰ΠΈΠΉ Π·Π½Π°ΠΌΠ΅Π½Π°Ρ‚Π΅Π»ΡŒ. ΠŸΡ€ΠΈΠΌΠ΅Ρ€ суммы Ρ‚Ρ€Π΅Ρ… Π΄Ρ€ΠΎΠ±Π΅ΠΉ ΠΏΠΎΠΊΠ°Π·Π°Π½ Π½ΠΈΠΆΠ΅. Π’ этом ΠΏΡ€ΠΈΠΌΠ΅Ρ€Π΅ Π²Ρ‹ Π±ΡƒΠ΄Π΅Ρ‚Π΅ ΠΈΡΠΏΠΎΠ»ΡŒΠ·ΠΎΠ²Π°Ρ‚ΡŒ ΠΌΠ΅Ρ‚ΠΎΠ΄ простой Ρ„Π°ΠΊΡ‚ΠΎΡ€ΠΈΠ·Π°Ρ†ΠΈΠΈ, Ρ‡Ρ‚ΠΎΠ±Ρ‹ Π½Π°ΠΉΡ‚ΠΈ LCM.

ΠŸΠΎΠ΄ΡƒΠΌΠ°ΠΉ ΠΎΠ± этом

Π”ΠΎΠ±Π°Π²ΠΈΡ‚ΡŒ [латСкс]\Π‘ΠΎΠ»ΡŒΡˆΠΎΠΉ\фракция{3}{4}+\Π‘ΠΎΠ»ΡŒΡˆΠΎΠΉ\фракция{1}{6}+\Π‘ΠΎΠ»ΡŒΡˆΠΎΠΉ\фракция{5}{8}[/латСкс]. УпроститС ΠΎΡ‚Π²Π΅Ρ‚ ΠΈ Π·Π°ΠΏΠΈΡˆΠΈΡ‚Π΅ Π² Π²ΠΈΠ΄Π΅ смСшанного числа.

Π§Π΅ΠΌ этот ΠΏΡ€ΠΈΠΌΠ΅Ρ€ отличаСтся ΠΎΡ‚ ΠΏΡ€Π΅Π΄Ρ‹Π΄ΡƒΡ‰ΠΈΡ…? Π˜ΡΠΏΠΎΠ»ΡŒΠ·ΡƒΠΉΡ‚Π΅ ΠΏΠΎΠ»Π΅ Π½ΠΈΠΆΠ΅, Ρ‡Ρ‚ΠΎΠ±Ρ‹ Π·Π°ΠΏΠΈΡΠ°Ρ‚ΡŒ нСсколько мыслСй ΠΎ Ρ‚ΠΎΠΌ, ΠΊΠ°ΠΊ Π±Ρ‹ Π²Ρ‹ слоТили вмСстС Ρ‚Ρ€ΠΈ Π΄Ρ€ΠΎΠ±ΠΈ с Ρ€Π°Π·Π½Ρ‹ΠΌΠΈ знамСнатСлями.

ΠŸΠΎΠΊΠ°Π·Π°Ρ‚ΡŒ Ρ€Π΅ΡˆΠ΅Π½ΠΈΠ΅

Π’Ρ‹Ρ‡ΠΈΡ‚Π°Π½ΠΈΠ΅ Π΄Ρ€ΠΎΠ±Π΅ΠΉ

Π’Ρ‹Ρ‡ΠΈΡ‚Π°Π½ΠΈΠ΅ Π΄Ρ€ΠΎΠ±Π΅ΠΉ выполняСтся Ρ‚Π°ΠΊ ΠΆΠ΅, ΠΊΠ°ΠΊ ΠΈ слоТСниС. Π‘Π½Π°Ρ‡Π°Π»Π° ΠΎΠΏΡ€Π΅Π΄Π΅Π»ΠΈΡ‚Π΅, ΠΎΠ΄ΠΈΠ½Π°ΠΊΠΎΠ²Ρ‹ Π»ΠΈ Π·Π½Π°ΠΌΠ΅Π½Π°Ρ‚Π΅Π»ΠΈ. Если Π½Π΅Ρ‚, ΠΏΠ΅Ρ€Π΅ΠΏΠΈΡˆΠΈΡ‚Π΅ ΠΊΠ°ΠΆΠ΄ΡƒΡŽ Π΄Ρ€ΠΎΠ±ΡŒ ΠΊΠ°ΠΊ ΡΠΊΠ²ΠΈΠ²Π°Π»Π΅Π½Ρ‚Π½ΡƒΡŽ Π΄Ρ€ΠΎΠ±ΡŒ с ΠΎΠ΄ΠΈΠ½Π°ΠΊΠΎΠ²Ρ‹ΠΌ Π·Π½Π°ΠΌΠ΅Π½Π°Ρ‚Π΅Π»Π΅ΠΌ. НиТС ΠΏΡ€ΠΈΠ²Π΅Π΄Π΅Π½Ρ‹ ΠΏΡ€ΠΈΠΌΠ΅Ρ€Ρ‹ вычитания Π΄Ρ€ΠΎΠ±Π΅ΠΉ, Π·Π½Π°ΠΌΠ΅Π½Π°Ρ‚Π΅Π»ΠΈ ΠΊΠΎΡ‚ΠΎΡ€Ρ‹Ρ… Π½Π΅ ΡΠΎΠ²ΠΏΠ°Π΄Π°ΡŽΡ‚.

Π’ ΠΏΡ€ΠΈΠ²Π΅Π΄Π΅Π½Π½ΠΎΠΌ Π½ΠΈΠΆΠ΅ ΠΏΡ€ΠΈΠΌΠ΅Ρ€Π΅ ΠΏΠΎΠΊΠ°Π·Π°Π½ΠΎ, ΠΊΠ°ΠΊ с ΠΏΠΎΠΌΠΎΡ‰ΡŒΡŽ ΠΊΡ€Π°Ρ‚Π½Ρ‹Ρ… Π½Π°ΠΉΡ‚ΠΈ наимСньшСС ΠΎΠ±Ρ‰Π΅Π΅ ΠΊΡ€Π°Ρ‚Π½ΠΎΠ΅, ΠΊΠΎΡ‚ΠΎΡ€ΠΎΠ΅ Π±ΡƒΠ΄Π΅Ρ‚ ΡΠ²Π»ΡΡ‚ΡŒΡΡ наимСньшим ΠΎΠ±Ρ‰ΠΈΠΌ Π·Π½Π°ΠΌΠ΅Π½Π°Ρ‚Π΅Π»Π΅ΠΌ.

ΠŸΡ€ΠΈΠΌΠ΅Ρ€

Π’Ρ‹Ρ‡Π΅ΡΡ‚ΡŒ [латСкс]\ΠΊΡ€ΡƒΠΏΠ½Ρ‹ΠΉ\Ρ„Ρ€Π°ΠΊ{5}{6}-\ΠΊΡ€ΡƒΠΏΠ½Ρ‹ΠΉ\Ρ„Ρ€Π°ΠΊ{1}{4}[/латСкс]. УпроститС ΠΎΡ‚Π²Π΅Ρ‚.

ΠŸΠΎΠΊΠ°Π·Π°Ρ‚ΡŒ Ρ€Π΅ΡˆΠ΅Π½ΠΈΠ΅

Π’ ΡΠ»Π΅Π΄ΡƒΡŽΡ‰Π΅ΠΌ Π²ΠΈΠ΄Π΅ΠΎ Π²Ρ‹ ΡƒΠ²ΠΈΠ΄ΠΈΡ‚Π΅ ΠΏΡ€ΠΈΠΌΠ΅Ρ€ вычитания Π΄Ρ€ΠΎΠ±Π΅ΠΉ с Ρ€Π°Π·Π½Ρ‹ΠΌΠΈ знамСнатСлями.

Π£ΠΌΠ½ΠΎΠΆΠ΅Π½ΠΈΠ΅ Π΄Ρ€ΠΎΠ±Π΅ΠΉ

Π’ΠΎΡ‡Π½ΠΎ Ρ‚Π°ΠΊ ΠΆΠ΅, ΠΊΠ°ΠΊ слоТСниС, Π²Ρ‹Ρ‡ΠΈΡ‚Π°Π½ΠΈΠ΅, ΡƒΠΌΠ½ΠΎΠΆΠ΅Π½ΠΈΠ΅ ΠΈ Π΄Π΅Π»Π΅Π½ΠΈΠ΅ ΠΏΡ€ΠΈ Ρ€Π°Π±ΠΎΡ‚Π΅ с Ρ†Π΅Π»Ρ‹ΠΌΠΈ числами, Π²Ρ‹ Ρ‚Π°ΠΊΠΆΠ΅ ΠΈΡΠΏΠΎΠ»ΡŒΠ·ΡƒΠ΅Ρ‚Π΅ эти ΠΎΠΏΠ΅Ρ€Π°Ρ†ΠΈΠΈ ΠΏΡ€ΠΈ Ρ€Π°Π±ΠΎΡ‚Π΅ с дробями. Π•ΡΡ‚ΡŒ ΠΌΠ½ΠΎΠ³ΠΎ случаСв, ΠΊΠΎΠ³Π΄Π° Π½Π΅ΠΎΠ±Ρ…ΠΎΠ΄ΠΈΠΌΠΎ ΡƒΠΌΠ½ΠΎΠΆΠΈΡ‚ΡŒ Π΄Ρ€ΠΎΠ±ΠΈ. МодСль ΠΌΠΎΠΆΠ΅Ρ‚ ΠΏΠΎΠΌΠΎΡ‡ΡŒ Π²Π°ΠΌ ΠΏΠΎΠ½ΡΡ‚ΡŒ ΡƒΠΌΠ½ΠΎΠΆΠ΅Π½ΠΈΠ΅ Π΄Ρ€ΠΎΠ±Π΅ΠΉ.

Когда Π²Ρ‹ ΡƒΠΌΠ½ΠΎΠΆΠ°Π΅Ρ‚Π΅ Π΄Ρ€ΠΎΠ±ΡŒ Π½Π° Π΄Ρ€ΠΎΠ±ΡŒ, Π²Ρ‹ ΠΏΠΎΠ»ΡƒΡ‡Π°Π΅Ρ‚Π΅ «долю Π΄Ρ€ΠΎΠ±ΠΈΒ». ΠŸΡ€Π΅Π΄ΠΏΠΎΠ»ΠΎΠΆΠΈΠΌ, Ρƒ вас Π΅ΡΡ‚ΡŒ [latex]\Large\frac{3}{4}[/latex]Β ΠΊΠΎΠ½Ρ„Π΅Ρ‚Ρ‹, ΠΈ Π²Ρ‹ Ρ…ΠΎΡ‚ΠΈΡ‚Π΅ Π½Π°ΠΉΡ‚ΠΈ [latex]\Large\frac{1}{2}[/latex]Β ΠΈΠ· [latex ]\Large\frac{3}{4}[/latex]:

Π Π°Π·Π΄Π΅Π»ΠΈΠ² ΠΊΠ°ΠΆΠ΄ΡƒΡŽ Ρ‡Π΅Ρ‚Π²Π΅Ρ€Ρ‚ΡŒ ΠΏΠΎΠΏΠΎΠ»Π°ΠΌ, Π²Ρ‹ ΠΌΠΎΠΆΠ΅Ρ‚Π΅ Ρ€Π°Π·Π΄Π΅Π»ΠΈΡ‚ΡŒ ΡˆΠΎΠΊΠΎΠ»Π°Π΄Π½Ρ‹ΠΉ Π±Π°Ρ‚ΠΎΠ½Ρ‡ΠΈΠΊ Π½Π° Π²ΠΎΡΡŒΠΌΡ‹Π΅ части.

Π—Π°Ρ‚Π΅ΠΌ Π²Ρ‹Π±Π΅Ρ€ΠΈΡ‚Π΅ ΠΏΠΎΠ»ΠΎΠ²ΠΈΠ½Ρƒ ΠΈΠ· Π½ΠΈΡ…, Ρ‡Ρ‚ΠΎΠ±Ρ‹ ΠΏΠΎΠ»ΡƒΡ‡ΠΈΡ‚ΡŒ [latex]\Large\frac{3}{8}[/latex].

Π’ ΠΎΠ±ΠΎΠΈΡ… Π²Ρ‹ΡˆΠ΅ΠΏΠ΅Ρ€Π΅Ρ‡ΠΈΡΠ»Π΅Π½Π½Ρ‹Ρ… случаях, Ρ‡Ρ‚ΠΎΠ±Ρ‹ Π½Π°ΠΉΡ‚ΠΈ ΠΎΡ‚Π²Π΅Ρ‚, Π²Ρ‹ ΠΌΠΎΠΆΠ΅Ρ‚Π΅ ΠΏΠ΅Ρ€Π΅ΠΌΠ½ΠΎΠΆΠΈΡ‚ΡŒ числитСли вмСстС ΠΈ Π·Π½Π°ΠΌΠ΅Π½Π°Ρ‚Π΅Π»ΠΈ вмСстС.

Π£ΠΌΠ½ΠΎΠΆΠ΅Π½ΠΈΠ΅ Π΄Π²ΡƒΡ… Π΄Ρ€ΠΎΠ±Π΅ΠΉ

[latex]\Large\frac{a}{b}\cdot\Large\frac{c}{d}=\Large\frac{a\cdot c}{b\cdot d}= \Large\frac{\text{ΠΏΡ€ΠΎΠΈΠ·Π²Π΅Π΄Π΅Π½ΠΈΠ΅ числитСлСй}}{\text{ΠΏΡ€ΠΎΠΈΠ·Π²Π΅Π΄Π΅Π½ΠΈΠ΅ Π·Π½Π°ΠΌΠ΅Π½Π°Ρ‚Π΅Π»Π΅ΠΉ}}[/latex]

Π£ΠΌΠ½ΠΎΠΆΠ΅Π½ΠΈΠ΅ Π±ΠΎΠ»Π΅Π΅ Π΄Π²ΡƒΡ… Π΄Ρ€ΠΎΠ±Π΅ΠΉ

[латСкс]\Large\frac{a}{b} \cdot\Large\frac{c}{d}\cdot\Large\frac{e}{f}=\Large\frac{a\cdot c\cdot e}{b\cdot d\cdot f}[/latex ]

ΠŸΡ€ΠΈΠΌΠ΅Ρ€

Π£ΠΌΠ½ΠΎΠΆΠΈΡ‚ΡŒ [latex]\Large\frac{2}{3}\cdot\Large\frac{4}{5}[/latex]

ΠŸΠΎΠΊΠ°Π·Π°Ρ‚ΡŒ Ρ€Π΅ΡˆΠ΅Π½ΠΈΠ΅

ΠŸΠΎΠ²Ρ‚ΠΎΡ€ΠΈΠΌ: Ссли Ρƒ Π΄Ρ€ΠΎΠ±ΠΈ Π΅ΡΡ‚ΡŒΒ ΠΎΠ±Ρ‰ΠΈΠ΅ Π΄Π΅Π»ΠΈΡ‚Π΅Π»ΠΈ Π² числитСлС ΠΈ Π·Π½Π°ΠΌΠ΅Π½Π°Ρ‚Π΅Π»Π΅, ΠΌΡ‹ ΠΌΠΎΠΆΠ΅ΠΌ привСсти Π΄Ρ€ΠΎΠ±ΡŒ ΠΊ ΡƒΠΏΡ€ΠΎΡ‰Π΅Π½Π½ΠΎΠΉ Ρ„ΠΎΡ€ΠΌΠ΅, ΡƒΠ΄Π°Π»ΠΈΠ² ΠΎΠ±Ρ‰ΠΈΠ΅ Π΄Π΅Π»ΠΈΡ‚Π΅Π»ΠΈ.

НапримСр,

  • Учитывая [латСкс]\большой\Ρ„Ρ€Π°ΠΊ{8}{15}[/латСкс], коэффициСнты [латСкс]8[/латСкс] Ρ€Π°Π²Π½Ρ‹: [латСкс]1, 2, 4, 8 [/latex] ΠΈ коэффициСнты [latex]15[/latex]: [latex]1, 3, 5, 15[/latex]. [latex]\Large\frac{8}{15}[/latex] ΡƒΠΏΡ€ΠΎΡ‰Π΅Π½, ΠΏΠΎΡ‚ΠΎΠΌΡƒ Ρ‡Ρ‚ΠΎ Π½Π΅Ρ‚ ΠΎΠ±Ρ‰ΠΈΡ… ΠΌΠ½ΠΎΠΆΠΈΡ‚Π΅Π»Π΅ΠΉ для [latex]8[/latex] ΠΈ [latex]15[/latex].
  • Учитывая [латСкс]\большой\Ρ„Ρ€Π°ΠΊ{10}{15}[/латСкс], ΠΌΠ½ΠΎΠΆΠΈΡ‚Π΅Π»ΠΈ [латСкс]10[/латСкс] ΡΠ»Π΅Π΄ΡƒΡŽΡ‰ΠΈΠ΅: [латСкс]1, 2, 5, 10[/латСкс] ΠΈ ΠΌΠ½ΠΎΠΆΠΈΡ‚Π΅Π»ΠΈ ΠΈΠ· [латСкс]15[/латСкс]: [латСкс]1, 3, 5, 15[/латСкс]. [latex]\Large\frac{10}{15}[/latex] Π½Π΅ упрощаСтся, ΠΏΠΎΡΠΊΠΎΠ»ΡŒΠΊΡƒ [latex]5[/latex] являСтся ΠΎΠ±Ρ‰ΠΈΠΌ Π΄Π΅Π»ΠΈΡ‚Π΅Π»Π΅ΠΌ [latex]10[/latex]Β ΠΈ [latex]15[/latex]. ].

Π’Ρ‹ ΠΌΠΎΠΆΠ΅Ρ‚Π΅ сначала ΡƒΠΏΡ€ΠΎΡΡ‚ΠΈΡ‚ΡŒ, ΠΏΡ€Π΅ΠΆΠ΄Π΅ Ρ‡Π΅ΠΌ ΡƒΠΌΠ½ΠΎΠΆΠ°Ρ‚ΡŒ Π΄Π²Π΅ Π΄Ρ€ΠΎΠ±ΠΈ, Ρ‡Ρ‚ΠΎΠ±Ρ‹ ΠΎΠ±Π»Π΅Π³Ρ‡ΠΈΡ‚ΡŒ сСбС Ρ€Π°Π±ΠΎΡ‚Ρƒ. Π­Ρ‚ΠΎ позволяСт Π²Π°ΠΌ Ρ€Π°Π±ΠΎΡ‚Π°Ρ‚ΡŒ с мСньшими числами ΠΏΡ€ΠΈ ΡƒΠΌΠ½ΠΎΠΆΠ΅Π½ΠΈΠΈ.

Π’ ΡΠ»Π΅Π΄ΡƒΡŽΡ‰Π΅ΠΌ Π²ΠΈΠ΄Π΅ΠΎ Π²Ρ‹ ΡƒΠ²ΠΈΠ΄ΠΈΡ‚Π΅ ΠΏΡ€ΠΈΠΌΠ΅Ρ€ Ρ‚ΠΎΠ³ΠΎ, ΠΊΠ°ΠΊ ΡƒΠΌΠ½ΠΎΠΆΠΈΡ‚ΡŒ Π΄Π²Π΅ Π΄Ρ€ΠΎΠ±ΠΈ, Π° Π·Π°Ρ‚Π΅ΠΌ ΡƒΠΏΡ€ΠΎΡΡ‚ΠΈΡ‚ΡŒ ΠΎΡ‚Π²Π΅Ρ‚.

ΠŸΠΎΠ΄ΡƒΠΌΠ°ΠΉ ΠΎΠ± этом

Π£ΠΌΠ½ΠΎΠΆΠΈΡ‚ΡŒ [латСкс]\Π‘ΠΎΠ»ΡŒΡˆΠΎΠΉ\фракция{2}{3}\cdot\Π‘ΠΎΠ»ΡŒΡˆΠ°Ρ\фракция{1}{4}\cdot\Π‘ΠΎΠ»ΡŒΡˆΠ°Ρ\фракция{3}{5}[/латСкс ]. УпроститС ΠΎΡ‚Π²Π΅Ρ‚.

Π§Π΅ΠΌ этот ΠΏΡ€ΠΈΠΌΠ΅Ρ€ отличаСтся ΠΎΡ‚ ΠΏΡ€Π΅Π΄Ρ‹Π΄ΡƒΡ‰ΠΈΡ…? Π˜ΡΠΏΠΎΠ»ΡŒΠ·ΡƒΠΉΡ‚Π΅ ΠΏΠΎΠ»Π΅ Π½ΠΈΠΆΠ΅, Ρ‡Ρ‚ΠΎΠ±Ρ‹ Π·Π°ΠΏΠΈΡΠ°Ρ‚ΡŒ нСсколько мыслСй ΠΎ Ρ‚ΠΎΠΌ, ΠΊΠ°ΠΊ Π±Ρ‹ Π²Ρ‹ ΡƒΠΌΠ½ΠΎΠΆΠΈΠ»ΠΈ Ρ‚Ρ€ΠΈ Π΄Ρ€ΠΎΠ±ΠΈ.

ΠŸΠΎΠΊΠ°Π·Π°Ρ‚ΡŒ Ρ€Π΅ΡˆΠ΅Π½ΠΈΠ΅

Π Π°Π·Π΄Π΅Π»ΠΈΡ‚ΡŒ Π΄Ρ€ΠΎΠ±ΠΈ

Π‘Ρ‹Π²Π°ΡŽΡ‚ случаи, ΠΊΠΎΠ³Π΄Π° Π²Π°ΠΌ Π½ΡƒΠΆΠ½ΠΎ ΠΈΡΠΏΠΎΠ»ΡŒΠ·ΠΎΠ²Π°Ρ‚ΡŒ Π΄Π΅Π»Π΅Π½ΠΈΠ΅ для Ρ€Π΅ΡˆΠ΅Π½ΠΈΡ ΠΏΡ€ΠΎΠ±Π»Π΅ΠΌΡ‹. НапримСр, Ссли для покраски ΠΎΠ΄Π½ΠΎΠ³ΠΎ слоя краски Π½Π° стСнах ΠΊΠΎΠΌΠ½Π°Ρ‚Ρ‹ трСбуСтся [латСкс]3[/латСкс] Π»ΠΈΡ‚Ρ€Π° краски, ΠΈ Ρƒ вас Π΅ΡΡ‚ΡŒ Π²Π΅Π΄Ρ€ΠΎ, содСрТащСС [латСкс]6[/латСкс] ΠΊΠ²Π°Ρ€Ρ‚ краски, сколько слоСв краски краской ΠΌΠΎΠΆΠ½ΠΎ ΠΊΡ€Π°ΡΠΈΡ‚ΡŒ стСны? Π’Ρ‹ Π΄Π΅Π»ΠΈΡ‚Π΅ [латСкс]6[/латСкс] Π½Π° [латСкс]3[/латСкс] для ΠΎΡ‚Π²Π΅Ρ‚Π° [латСкс]2[/латСкс] слоСв. Π’Π°ΠΊΠΆΠ΅ Π±ΡƒΠ΄ΡƒΡ‚ случаи, ΠΊΠΎΠ³Π΄Π° Π²Π°ΠΌ Π½ΡƒΠΆΠ½ΠΎ Ρ€Π°Π·Π΄Π΅Π»ΠΈΡ‚ΡŒ Π½Π° Π΄Ρ€ΠΎΠ±ΡŒ. ΠŸΡ€Π΅Π΄ΠΏΠΎΠ»ΠΎΠΆΠΈΠΌ, Ρ‡Ρ‚ΠΎ для покраски ΡˆΠΊΠ°Ρ„Π° Π² ΠΎΠ΄ΠΈΠ½ слой трСбуСтся всСго [латСкс]\большой\Ρ„Ρ€Π°ΠΊ{1}{2}[/латСкс] Π»ΠΈΡ‚Ρ€ краски. Бколько слоСв ΠΌΠΎΠΆΠ½ΠΎ нанСсти 6 Π»ΠΈΡ‚Ρ€Π°ΠΌΠΈ краски? Π§Ρ‚ΠΎΠ±Ρ‹ Π½Π°ΠΉΡ‚ΠΈ ΠΎΡ‚Π²Π΅Ρ‚, Π²Π°ΠΌ Π½ΡƒΠΆΠ½ΠΎ Ρ€Π°Π·Π΄Π΅Π»ΠΈΡ‚ΡŒ [латСкс]2[/латСкс]Β Π½Π° Π΄Ρ€ΠΎΠ±ΡŒ [латСкс]\большой\фракция{1}{2}[/латСкс].

ΠŸΡ€Π΅ΠΆΠ΄Π΅ Ρ‡Π΅ΠΌ ΠΌΡ‹ Π½Π°Ρ‡Π½Π΅ΠΌ Π΄Π΅Π»ΠΈΡ‚ΡŒ Π΄Ρ€ΠΎΠ±ΠΈ, Π΄Π°Π²Π°ΠΉΡ‚Π΅ рассмотрим Π½Π΅ΠΊΠΎΡ‚ΠΎΡ€Ρ‹Π΅ Π²Π°ΠΆΠ½Ρ‹Π΅ Ρ‚Π΅Ρ€ΠΌΠΈΠ½Ρ‹.

  • ΠΎΠ±Ρ€Π°Ρ‚Π½ΠΎΠ΅: Π΄Π²Π΅ Π΄Ρ€ΠΎΠ±ΠΈ ΡΠ²Π»ΡΡŽΡ‚ΡΡ ΠΎΠ±Ρ€Π°Ρ‚Π½Ρ‹ΠΌΠΈ, Ссли ΠΈΡ… ΠΏΡ€ΠΎΠΈΠ·Π²Π΅Π΄Π΅Π½ΠΈΠ΅ Ρ€Π°Π²Π½ΠΎ [латСкс]1[/латСкс]Β (Π½Π΅ Π²ΠΎΠ»Π½ΡƒΠΉΡ‚Π΅ΡΡŒ, ΠΌΡ‹ ΠΏΠΎΠΊΠ°ΠΆΠ΅ΠΌ Π²Π°ΠΌ ΠΏΡ€ΠΈΠΌΠ΅Ρ€Ρ‹ Ρ‚ΠΎΠ³ΠΎ, Ρ‡Ρ‚ΠΎ это Π·Π½Π°Ρ‡ΠΈΡ‚.)
  • частноС: Ρ€Π΅Π·ΡƒΠ»ΡŒΡ‚Π°Ρ‚ дСлСния

Для дСлСния Π΄Ρ€ΠΎΠ±Π΅ΠΉ Π½Π΅ΠΎΠ±Ρ…ΠΎΠ΄ΠΈΠΌΠΎ ΠΈΡΠΏΠΎΠ»ΡŒΠ·ΠΎΠ²Π°Ρ‚ΡŒ ΠΎΠ±Ρ€Π°Ρ‚Π½ΠΎΠ΅ число ΠΈΠ»ΠΈ Π΄Ρ€ΠΎΠ±ΡŒ. Если Π²Ρ‹ ΡƒΠΌΠ½ΠΎΠΆΠ°Π΅Ρ‚Π΅ Π΄Π²Π° числа вмСстС ΠΈ ΠΏΠΎΠ»ΡƒΡ‡Π°Π΅Ρ‚Π΅ Π² Ρ€Π΅Π·ΡƒΠ»ΡŒΡ‚Π°Ρ‚Π΅ [латСкс]1[/латСкс],Β Ρ‚ΠΎΠ³Π΄Π° эти Π΄Π²Π° числа ΡΠ²Π»ΡΡŽΡ‚ΡΡ ΠΎΠ±Ρ€Π°Ρ‚Π½Ρ‹ΠΌΠΈ. Π’ΠΎΡ‚ нСсколько ΠΏΡ€ΠΈΠΌΠ΅Ρ€ΠΎΠ² Π²Π·Π°ΠΈΠΌΠ½ΠΎΠ³ΠΎ ΠΎΠ±ΠΌΠ΅Π½Π°:

Π˜ΡΡ…ΠΎΠ΄Π½Ρ‹ΠΉ Π½ΠΎΠΌΠ΅Ρ€ Π’Π·Π°ΠΈΠΌΠ½ΠΎΠ΅ ΠŸΡ€ΠΎΠ΄ΡƒΠΊΡ‚
[латСкс]\большой\Ρ„Ρ€Π°ΠΊ{3}{4}[/латСкс] [латСкс]\большой\Ρ„Ρ€Π°ΠΊ{4}{3}[/латСкс] [латСкс]\Large\frac{3}{4}\cdot\Large\frac{4}{3}=\Large\frac{3\cdot 4}{4\cdot 3}=\Large\frac{12 {12}=1[/латСкс]
[латСкс]\большой\Ρ„Ρ€Π°ΠΊ{1}{2}[/латСкс] [латСкс]\большой\Ρ„Ρ€Π°ΠΊ{2}{1}[/латСкс] [латСкс]\Large\frac{1}{2}\cdot\Large\frac{2}{1}=\Large\frac{1\cdot2}{2\cdot1}=\Large\frac{2}{ 2}=1[/латСкс]
[латСкс] 3=\Large\frac{3}{1}[/латСкс] [латСкс]\большой\Ρ„Ρ€Π°ΠΊ{1}{3}[/латСкс] [латСкс]\Large\frac{3}{1}\cdot\Large\frac{1}{3}=\Large\frac{3\cdot 1}{1\cdot 3}=\Large\frac{3 {3}=1[/латСкс]
[латСкс]2\Π‘ΠΎΠ»ΡŒΡˆΠΎΠΉ\фракция{1}{3}=\Π‘ΠΎΠ»ΡŒΡˆΠΎΠΉ\фракция{7}{3}[/латСкс] [латСкс]\большой\Ρ„Ρ€Π°ΠΊ{3}{7}[/латСкс] [латСкс]\Large\frac{7}{3}\cdot\Large\frac{3}{7}=\Large\frac{7\cdot3}{3\cdot7}=\Large\frac{21}{ 21}=\Π½ΠΎΡ€ΠΌΠ°Π»ΡŒΠ½Ρ‹ΠΉ Ρ€Π°Π·ΠΌΠ΅Ρ€ 1[/латСкс]

Иногда ΠΌΡ‹ Π½Π°Π·Ρ‹Π²Π°Π΅ΠΌΒ ΠΎΠ±Ρ€Π°Ρ‚Π½ΠΎΠ΅Β «ΠΎΡ‚Ρ€Π°ΠΆΠ΅Π½ΠΈΠ΅ΠΌ» Π΄Ρ€ΡƒΠ³ΠΎΠ³ΠΎ числа: ΠΏΠ΅Ρ€Π΅Π²Π΅Ρ€Π½ΠΈΡ‚Π΅ [латСкс]\Π‘ΠΎΠ»ΡŒΡˆΠΎΠΉ\Ρ„Ρ€Π°ΠΊ{2}{5}[/латСкс], Ρ‡Ρ‚ΠΎΠ±Ρ‹ ΠΏΠΎΠ»ΡƒΡ‡ΠΈΡ‚ΡŒ ΠΎΠ±Ρ€Π°Ρ‚Π½ΠΎΠ΅ [латСкс]\Π‘ΠΎΠ»ΡŒΡˆΠΎΠΉ\Ρ„Ρ€Π°ΠΊ{5}{ 2}[/латСкс].

Π”Π΅Π»Π΅Π½ΠΈΠ΅ Π½Π° ноль

Π’Ρ‹ Π·Π½Π°Π΅Ρ‚Π΅, Ρ‡Ρ‚ΠΎ Π·Π½Π°Ρ‡ΠΈΡ‚ Π΄Π΅Π»ΠΈΡ‚ΡŒ Π½Π° [латСкс]2[/латСкс] ΠΈΠ»ΠΈ Π΄Π΅Π»ΠΈΡ‚ΡŒ Π½Π° [латСкс]10[/латСкс], Π½ΠΎ Ρ‡Ρ‚ΠΎ Π·Π½Π°Ρ‡ΠΈΡ‚ Π΄Π΅Π»ΠΈΡ‚ΡŒ количСство Π½Π° [латСкс]0[ /латСкс]? Π­Ρ‚ΠΎ Π²ΠΎΠΎΠ±Ρ‰Π΅ Π²ΠΎΠ·ΠΌΠΎΠΆΠ½ΠΎ? ΠœΠΎΠΆΠ΅Ρ‚Π΅ Π»ΠΈ Π²Ρ‹ Ρ€Π°Π·Π΄Π΅Π»ΠΈΡ‚ΡŒ [латСкс]0[/латСкс] Π½Π° число? Рассмотрим Π΄Ρ€ΠΎΠ±ΡŒ

[латСкс]\большой\Ρ„Ρ€Π°ΠΊ{0}{8}[/латСкс]

ΠœΡ‹ ΠΌΠΎΠΆΠ΅ΠΌ ΠΏΡ€ΠΎΡ‡ΠΈΡ‚Π°Ρ‚ΡŒ это ΠΊΠ°ΠΊ «ноль Ρ€Π°Π·Π΄Π΅Π»ΠΈΡ‚ΡŒ Π½Π° восСмь». ΠŸΠΎΡΠΊΠΎΠ»ΡŒΠΊΡƒ ΡƒΠΌΠ½ΠΎΠΆΠ΅Π½ΠΈΠ΅ ΠΎΠ±Ρ€Π°Ρ‚Π½ΠΎ дСлСнию, ΠΌΡ‹ ΠΌΠΎΠ³Π»ΠΈ Π±Ρ‹ ΠΏΠ΅Ρ€Π΅ΠΏΠΈΡΠ°Ρ‚ΡŒ это ΠΊΠ°ΠΊ Π·Π°Π΄Π°Ρ‡Ρƒ Π½Π° ΡƒΠΌΠ½ΠΎΠΆΠ΅Π½ΠΈΠ΅.

[латСкс]\тСкст{?}\cdot{8}=0[/латСкс].

ΠœΡ‹ ΠΌΠΎΠΆΠ΅ΠΌ ΡΠ΄Π΅Π»Π°Ρ‚ΡŒ Π²Ρ‹Π²ΠΎΠ΄, Ρ‡Ρ‚ΠΎ нСизвСстноС Π΄ΠΎΠ»ΠΆΠ½ΠΎ Π±Ρ‹Ρ‚ΡŒ [латСкс]0[/латСкс], Ρ‚Π°ΠΊ ΠΊΠ°ΠΊ это СдинствСнноС число, ΠΊΠΎΡ‚ΠΎΡ€ΠΎΠ΅ даст Ρ€Π΅Π·ΡƒΠ»ΡŒΡ‚Π°Ρ‚ [латСкс]0[/латСкс] ΠΏΡ€ΠΈ ΡƒΠΌΠ½ΠΎΠΆΠ΅Π½ΠΈΠΈ Π½Π° [латСкс]8[/латСкс]. ].

Π’Π΅ΠΏΠ΅Ρ€ΡŒ рассмотрим ΠΎΠ±Ρ€Π°Ρ‚Π½ΡƒΡŽ Π²Π΅Π»ΠΈΡ‡ΠΈΠ½Ρƒ [латСкс]\большой\Ρ„Ρ€Π°ΠΊ{0}{8}[/латСкс], которая Π±ΡƒΠ΄Π΅Ρ‚ [латСкс]\большой\Ρ„Ρ€Π°ΠΊ{8}{0}[/латСкс]. Если ΠΌΡ‹ ΠΏΠ΅Ρ€Π΅ΠΏΠΈΡˆΠ΅ΠΌ это ΠΊΠ°ΠΊ Π·Π°Π΄Π°Ρ‡Ρƒ Π½Π° ΡƒΠΌΠ½ΠΎΠΆΠ΅Π½ΠΈΠ΅, Ρƒ нас Π±ΡƒΠ΄Π΅Ρ‚

[латСкс]\тСкст{?}\cdot{0}=8[/латСкс].

Π­Ρ‚ΠΎ Π½Π΅ ΠΈΠΌΠ΅Π΅Ρ‚ Π½ΠΈΠΊΠ°ΠΊΠΎΠ³ΠΎ смысла. НС сущСствуСт чисСл, ΠΊΠΎΡ‚ΠΎΡ€Ρ‹Π΅ ΠΌΠΎΠΆΠ½ΠΎ ΡƒΠΌΠ½ΠΎΠΆΠΈΡ‚ΡŒ Π½Π° ноль, Ρ‡Ρ‚ΠΎΠ±Ρ‹ ΠΏΠΎΠ»ΡƒΡ‡ΠΈΡ‚ΡŒ Ρ€Π΅Π·ΡƒΠ»ΡŒΡ‚Π°Ρ‚ 8. ΠžΠ±Ρ€Π°Ρ‚Π½Π°Ρ Π²Π΅Π»ΠΈΡ‡ΠΈΠ½Π°Β [латСкс]\большой\Ρ„Ρ€Π°ΠΊ{8}{0}[/латСкс] Π½Π΅ ΠΎΠΏΡ€Π΅Π΄Π΅Π»Π΅Π½Π°, ΠΈ фактичСски любоС Π΄Π΅Π»Π΅Π½ΠΈΠ΅ Π½Π° ноль Π½Π΅ ΠΎΠΏΡ€Π΅Π΄Π΅Π»Π΅Π½ΠΎ. .

Π’Π½ΠΈΠΌΠ°Π½ΠΈΠ΅! Π”Π΅Π»Π΅Π½ΠΈΠ΅ Π½Π° ноль Π½Π΅ ΠΎΠΏΡ€Π΅Π΄Π΅Π»Π΅Π½ΠΎ, ΠΊΠ°ΠΊ ΠΈ обратная Π²Π΅Π»ΠΈΡ‡ΠΈΠ½Π° любой Π΄Ρ€ΠΎΠ±ΠΈ с Π½ΡƒΠ»Π΅ΠΌ Π² числитСлС. Для любого Π΄Π΅ΠΉΡΡ‚Π²ΠΈΡ‚Π΅Π»ΡŒΠ½ΠΎΠ³ΠΎ числа a [латСкс]\большой\Ρ„Ρ€Π°ΠΊ{Π°}{0}[/латСкс] Π½Π΅ ΠΎΠΏΡ€Π΅Π΄Π΅Π»Π΅Π½. ΠšΡ€ΠΎΠΌΠ΅ Ρ‚ΠΎΠ³ΠΎ, обратная Π²Π΅Π»ΠΈΡ‡ΠΈΠ½Π° [latex]\Large\frac{0}{a}[/latex] всСгда Π±ΡƒΠ΄Π΅Ρ‚ Π½Π΅ΠΎΠΏΡ€Π΅Π΄Π΅Π»Π΅Π½Π½ΠΎΠΉ.

Π”Π΅Π»Π΅Π½ΠΈΠ΅ Π΄Ρ€ΠΎΠ±ΠΈ Π½Π° Ρ†Π΅Π»ΠΎΠ΅ число

ΠŸΡ€ΠΈ Π΄Π΅Π»Π΅Π½ΠΈΠΈ Π½Π° Ρ†Π΅Π»ΠΎΠ΅ число Π²Ρ‹ ΡƒΠΌΠ½ΠΎΠΆΠ°Π΅Ρ‚Π΅ Π΅Π³ΠΎ Π½Π° ΠΎΠ±Ρ€Π°Ρ‚Π½ΠΎΠ΅. Π’ ΠΏΡ€ΠΈΠΌΠ΅Ρ€Π΅ покраски, Π³Π΄Π΅ Π²Π°ΠΌ Π½ΡƒΠΆΠ½ΠΎ [латСкс]3[/латСкс] ΠΊΠ²Π°Ρ€Ρ‚Ρ‹ краски для слоя ΠΈ Ρƒ вас Π΅ΡΡ‚ΡŒ [латСкс]6[/латСкс] ΠΊΠ²Π°Ρ€Ρ‚Ρ‹ краски, Π²Ρ‹ ΠΌΠΎΠΆΠ΅Ρ‚Π΅ Π½Π°ΠΉΡ‚ΠΈ ΠΎΠ±Ρ‰Π΅Π΅ количСство слоСв, ΠΊΠΎΡ‚ΠΎΡ€Ρ‹Π΅ ΠΌΠΎΠΆΠ½ΠΎ ΠΏΠΎΠΊΡ€Π°ΡΠΈΡ‚ΡŒ, Ρ€Π°Π·Π΄Π΅Π»ΠΈΠ² [ латСкс]6[/латСкс] ΠΎΡ‚ [латСкс]3[/латСкс], [латСкс]6\div3=2[/латСкс]. Π’Ρ‹ Ρ‚Π°ΠΊΠΆΠ΅ ΠΌΠΎΠΆΠ΅Ρ‚Π΅ ΡƒΠΌΠ½ΠΎΠΆΠΈΡ‚ΡŒ [латСкс]6[/латСкс] Π½Π° ΠΎΠ±Ρ€Π°Ρ‚Π½ΡƒΡŽ Π²Π΅Π»ΠΈΡ‡ΠΈΠ½Ρƒ [латСкс]3[/латСкс], которая Ρ€Π°Π²Π½Π° [латСкс]\большой\Ρ„Ρ€Π°ΠΊ{1}{3}[/латСкс], поэтому Π·Π°Π΄Π°Ρ‡Π° умноТСния становится

[latex]\Large\frac{6}{1}\cdot\Large\frac{1}{3}=\Large\frac{6}{3}=\normalsize2[/latex]

Π”Π΅Π»Π΅Π½ΠΈΠ΅ Ρ€Π°Π²Π½ΠΎ ΡƒΠΌΠ½ΠΎΠΆΠ΅Π½ΠΈΡŽ ΠΏΠΎ ΠΎΠ±Ρ€Π°Ρ‚Π½ΠΎΠΌΡƒ

Для всСх Π΄Π΅Π»Π΅Π½ΠΈΠΉ Π²Ρ‹ ΠΌΠΎΠΆΠ΅Ρ‚Π΅ ΠΏΡ€Π΅Π²Ρ€Π°Ρ‚ΠΈΡ‚ΡŒ ΠΎΠΏΠ΅Ρ€Π°Ρ†ΠΈΡŽΒ Π² ΡƒΠΌΠ½ΠΎΠΆΠ΅Π½ΠΈΠ΅ с ΠΏΠΎΠΌΠΎΡ‰ΡŒΡŽ ΠΎΠ±Ρ€Π°Ρ‚Π½ΠΎΠ³ΠΎ. Π”Π΅Π»Π΅Π½ΠΈΠ΅ Ρ€Π°Π²Π½ΠΎΡΠΈΠ»ΡŒΠ½ΠΎ ΡƒΠΌΠ½ΠΎΠΆΠ΅Π½ΠΈΡŽ Π½Π° ΠΎΠ±Ρ€Π°Ρ‚Π½ΠΎΠ΅.

Π’Π° ΠΆΠ΅ идСя Π±ΡƒΠ΄Π΅Ρ‚ Ρ€Π°Π±ΠΎΡ‚Π°Ρ‚ΡŒ, ΠΊΠΎΠ³Π΄Π° Π΄Π΅Π»ΠΈΡ‚Π΅Π»ΡŒ (Π²Π΅Ρ‰ΡŒ, ΠΊΠΎΡ‚ΠΎΡ€ΡƒΡŽ дСлят) являСтся Π΄Ρ€ΠΎΠ±ΡŒΡŽ. Если Ρƒ вас Π΅ΡΡ‚ΡŒ [latex]\Large\frac{3}{4}[/latex] шоколадного Π±Π°Ρ‚ΠΎΠ½Ρ‡ΠΈΠΊΠ° ΠΈ Π²Π°ΠΌ Π½ΡƒΠΆΠ½ΠΎ Ρ€Π°Π·Π΄Π΅Π»ΠΈΡ‚ΡŒ Π΅Π³ΠΎ ΠΌΠ΅ΠΆΠ΄Ρƒ [latex]5[/latex] людьми, ΠΊΠ°ΠΆΠ΄Ρ‹ΠΉ Ρ‡Π΅Π»ΠΎΠ²Π΅ΠΊ ΠΏΠΎΠ»ΡƒΡ‡Π°Π΅Ρ‚ [latex]\Large\frac {1}{5}[/latex] доступных ΠΊΠΎΠ½Ρ„Π΅Ρ‚:

[латСкс]\Large\frac{1}{5}\normalsize\text{ of }\Large\frac{3}{4}=\Large\frac{1}{5}\cdot\Large\frac{ 3}{4}=\Large\frac{3}{20}[/latex]

ΠšΠ°ΠΆΠ΄Ρ‹ΠΉ Ρ‡Π΅Π»ΠΎΠ²Π΅ΠΊ ΠΏΠΎΠ»ΡƒΡ‡Π°Π΅Ρ‚ [latex]\Large\frac{3}{20}[/latex]Β Ρ†Π΅Π»ΠΎΠ³ΠΎ Π±Π°Ρ‚ΠΎΠ½Ρ‡ΠΈΠΊΠ°.

Если Ρƒ вас Π΅ΡΡ‚ΡŒ Ρ€Π΅Ρ†Π΅ΠΏΡ‚, ΠΊΠΎΡ‚ΠΎΡ€Ρ‹ΠΉ Π½ΡƒΠΆΠ½ΠΎ Ρ€Π°Π·Π΄Π΅Π»ΠΈΡ‚ΡŒ ΠΏΠΎΠΏΠΎΠ»Π°ΠΌ, Π²Ρ‹ ΠΌΠΎΠΆΠ΅Ρ‚Π΅ Ρ€Π°Π·Π΄Π΅Π»ΠΈΡ‚ΡŒ ΠΊΠ°ΠΆΠ΄Ρ‹ΠΉ ΠΈΠ½Π³Ρ€Π΅Π΄ΠΈΠ΅Π½Ρ‚ Π½Π° [латСкс]2[/латСкс] ΠΈΠ»ΠΈ ΡƒΠΌΠ½ΠΎΠΆΠΈΡ‚ΡŒ ΠΊΠ°ΠΆΠ΄Ρ‹ΠΉ ΠΈΠ½Π³Ρ€Π΅Π΄ΠΈΠ΅Π½Ρ‚ Π½Π° [латСкс]\большой\Ρ„Ρ€Π°ΠΊ{1}{2} [/latex], Ρ‡Ρ‚ΠΎΠ±Ρ‹ Π½Π°ΠΉΡ‚ΠΈ Π½ΠΎΠ²ΡƒΡŽ сумму.

НапримСр, Π΄Π΅Π»Π΅Π½ΠΈΠ΅ Π½Π° [латСкс]6[/латСкс] Ρ€Π°Π²Π½ΠΎΡΠΈΠ»ΡŒΠ½ΠΎ ΡƒΠΌΠ½ΠΎΠΆΠ΅Π½ΠΈΡŽ Π½Π° ΠΎΠ±Ρ€Π°Ρ‚Π½ΡƒΡŽ Π²Π΅Π»ΠΈΡ‡ΠΈΠ½Ρƒ [латСкс]6[/латСкс], которая Ρ€Π°Π²Π½Π° [латСкс]\большой\Ρ„Ρ€Π°ΠΊ{1}{6}[/ латСкс]. ΠŸΠΎΡΠΌΠΎΡ‚Ρ€ΠΈΡ‚Π΅ Π½Π° схСму Π΄Π²ΡƒΡ… ΠΏΠΈΡ†Ρ† Π½ΠΈΠΆΠ΅. Как ΠΌΠΎΠΆΠ½ΠΎ справСдливо Ρ€Π°Π·Π΄Π΅Π»ΠΈΡ‚ΡŒ Ρ‚ΠΎ, Ρ‡Ρ‚ΠΎ ΠΎΡΡ‚Π°Π»ΠΎΡΡŒ (ΠΎΠ±Π»Π°ΡΡ‚ΡŒ, Π·Π°ΡˆΡ‚Ρ€ΠΈΡ…ΠΎΠ²Π°Π½Π½Π°Ρ красным), ΠΌΠ΅ΠΆΠ΄Ρƒ [латСкс]6[/латСкс] людьми?

ΠšΠ°ΠΆΠ΄Ρ‹ΠΉ Ρ‡Π΅Π»ΠΎΠ²Π΅ΠΊ ΠΏΠΎΠ»ΡƒΡ‡Π°Π΅Ρ‚ ΠΎΠ΄ΠΈΠ½ кусок, поэтому ΠΊΠ°ΠΆΠ΄Ρ‹ΠΉ Ρ‡Π΅Π»ΠΎΠ²Π΅ΠΊ ΠΏΠΎΠ»ΡƒΡ‡Π°Π΅Ρ‚ [latex]\Large\frac{1}{4}[/latex] ΠΏΠΈΡ†Ρ†Ρ‹.

Π”Π΅Π»Π΅Π½ΠΈΠ΅ Π΄Ρ€ΠΎΠ±ΠΈ Π½Π° Ρ†Π΅Π»ΠΎΠ΅ β€” это Ρ‚ΠΎ ΠΆΠ΅ самоС, Ρ‡Ρ‚ΠΎ ΠΈ ΡƒΠΌΠ½ΠΎΠΆΠ΅Π½ΠΈΠ΅ Π½Π° ΠΎΠ±Ρ€Π°Ρ‚Π½ΡƒΡŽ, поэтому Π²Ρ‹ всСгда ΠΌΠΎΠΆΠ΅Ρ‚Π΅ ΠΈΡΠΏΠΎΠ»ΡŒΠ·ΠΎΠ²Π°Ρ‚ΡŒ ΡƒΠΌΠ½ΠΎΠΆΠ΅Π½ΠΈΠ΅ Π΄Ρ€ΠΎΠ±Π΅ΠΉ для Ρ€Π΅ΡˆΠ΅Π½ΠΈΡ Π·Π°Π΄Π°Ρ‡ Π½Π° Π΄Π΅Π»Π΅Π½ΠΈΠ΅.

ΠŸΡ€ΠΈΠΌΠ΅Ρ€

Найти [latex]\Large\frac{2}{3}\div \normalsize 4[/latex]

ΠŸΠΎΠΊΠ°Π·Π°Ρ‚ΡŒ Ρ€Π΅ΡˆΠ΅Π½ΠΈΠ΅

ΠŸΡ€ΠΈΠΌΠ΅Ρ€

Π Π°Π·Π΄Π΅Π»ΠΈΡ‚ΡŒ. [latex] 9\div\Large\frac{1}{2}[/latex]

ΠŸΠΎΠΊΠ°Π·Π°Ρ‚ΡŒ Ρ€Π΅ΡˆΠ΅Π½ΠΈΠ΅

Π Π°Π·Π΄Π΅Π»ΠΈΡ‚ΡŒ Π΄Ρ€ΠΎΠ±ΡŒ Π½Π° Π΄Ρ€ΠΎΠ±ΡŒ

Иногда Π²Π°ΠΌ Π½ΡƒΠΆΠ½ΠΎ Ρ€Π΅ΡˆΠΈΡ‚ΡŒ Π·Π°Π΄Π°Ρ‡Ρƒ, Ρ‚Ρ€Π΅Π±ΡƒΡŽΡ‰ΡƒΡŽ дСлСния Π½Π° Π΄Ρ€ΠΎΠ±ΡŒ. ΠŸΡ€Π΅Π΄ΠΏΠΎΠ»ΠΎΠΆΠΈΠΌ, Ρƒ вас Π΅ΡΡ‚ΡŒ ΠΏΠΈΡ†Ρ†Π°, которая ΡƒΠΆΠ΅ Π½Π°Ρ€Π΅Π·Π°Π½Π° Π½Π° [латСкс]4[/латСкс] Π»ΠΎΠΌΡ‚ΠΈΠΊΠ°. Бколько сущСствуСт Ρ„Ρ€Π°Π³ΠΌΠ΅Π½Ρ‚ΠΎΠ² [latex]\Large\frac{1}{2}[/latex]?

Π•ΡΡ‚ΡŒ [латСкс]8[/латСкс] срСзов. Π’Ρ‹ ΠΌΠΎΠΆΠ΅Ρ‚Π΅ Π²ΠΈΠ΄Π΅Ρ‚ΡŒ, Ρ‡Ρ‚ΠΎ Π΄Π΅Π»Π΅Π½ΠΈΠ΅ [латСкс]4[/латСкс] Π½Π° [латСкс]\большой\Ρ„Ρ€Π°ΠΊ{1}{2}[/латСкс] Π΄Π°Π΅Ρ‚ Ρ‚ΠΎΡ‚ ΠΆΠ΅ Ρ€Π΅Π·ΡƒΠ»ΡŒΡ‚Π°Ρ‚, Ρ‡Ρ‚ΠΎ ΠΈ ΡƒΠΌΠ½ΠΎΠΆΠ΅Π½ΠΈΠ΅ [латСкс]4[/латСкс] Π½Π° [латСкс]2. [/латСкс].

Π§Ρ‚ΠΎ ΠΏΡ€ΠΎΠΈΠ·ΠΎΠΉΠ΄Π΅Ρ‚, Ссли Π²Π°ΠΌ потрСбуСтся Ρ€Π°Π·Π΄Π΅Π»ΠΈΡ‚ΡŒ ΠΊΠ°ΠΆΠ΄Ρ‹ΠΉ срСз Π½Π° Ρ‚Ρ€ΠΈ части?

Π£ вас Π±ΡƒΠ΄Π΅Ρ‚ [латСкс]12[/латСкс] срСзов, Ρ‡Ρ‚ΠΎ Ρ€Π°Π²Π½ΠΎΡΠΈΠ»ΡŒΠ½ΠΎ ΡƒΠΌΠ½ΠΎΠΆΠ΅Π½ΠΈΡŽ [латСкс]4[/латСкс] Π½Π° [латСкс]3[/латСкс].

Π”Π΅Π»Π΅Π½ΠΈΠ΅ дробями

  1. НайдитС ΠΎΠ±Ρ€Π°Ρ‚Π½ΡƒΡŽ Π²Π΅Π»ΠΈΡ‡ΠΈΠ½Ρƒ числа, ΡΠ»Π΅Π΄ΡƒΡŽΡ‰Π΅Π³ΠΎ Π·Π° символом дСлСния.
  2. Π£ΠΌΠ½ΠΎΠΆΡŒΡ‚Π΅ ΠΏΠ΅Ρ€Π²ΠΎΠ΅ число (Ρ‚ΠΎ, Ρ‡Ρ‚ΠΎ ΠΏΠ΅Ρ€Π΅Π΄ Π·Π½Π°ΠΊΠΎΠΌ дСлСния) Π½Π° Π²Π΅Π»ΠΈΡ‡ΠΈΠ½Ρƒ, ΠΎΠ±Ρ€Π°Ρ‚Π½ΡƒΡŽ Π²Ρ‚ΠΎΡ€ΠΎΠΌΡƒ числу (послС Π·Π½Π°ΠΊΠ° дСлСния).

Π‘Π°ΠΌΡ‹ΠΉ простой способ Π·Π°ΠΏΠΎΠΌΠ½ΠΈΡ‚ΡŒ, ΠΊΠ°ΠΊ Π΄Π΅Π»ΠΈΡ‚ΡŒ Π΄Ρ€ΠΎΠ±ΠΈ, β€” это Ρ„Ρ€Π°Π·Π° Β«ΡΠΎΡ…Ρ€Π°Π½ΠΈΡ‚ΡŒ, ΠΈΠ·ΠΌΠ΅Π½ΠΈΡ‚ΡŒ, ΠΏΠ΅Ρ€Π΅Π²Π΅Ρ€Π½ΡƒΡ‚ΡŒΒ». Π­Ρ‚ΠΎ Π·Π½Π°Ρ‡ΠΈΡ‚ Π΄ΠΎ БОΠ₯РАНИВЬ ΠΏΠ΅Ρ€Π²ΠΎΠ΅ число, Π—ΠΠœΠ•ΠΠ˜Π’Π¬ Π·Π½Π°ΠΊ дСлСния Π½Π° ΡƒΠΌΠ½ΠΎΠΆΠ΅Π½ΠΈΠ΅, Π° Π·Π°Ρ‚Π΅ΠΌ ΠŸΠ•Π Π•Π’Π•Π ΠΠ£Π’Π¬ (ΠΈΡΠΏΠΎΠ»ΡŒΠ·ΠΎΠ²Π°Ρ‚ΡŒ ΠΎΠ±Ρ€Π°Ρ‚Π½ΠΎΠ΅) Π²Ρ‚ΠΎΡ€ΠΎΠ³ΠΎ числа.

ΠŸΡ€ΠΈΠΌΠ΅Ρ€

Π Π°Π·Π΄Π΅Π»ΠΈΡ‚ΡŒ [latex]\Large\frac{2}{3}\div\Large\frac{1}{6}[/latex]

ΠŸΠΎΠΊΠ°Π·Π°Ρ‚ΡŒ Ρ€Π΅ΡˆΠ΅Π½ΠΈΠ΅

ΠŸΡ€ΠΈΠΌΠ΅Ρ€

Π Π°Π·Π΄Π΅Π»ΠΈΡ‚ΡŒ [latex]\Large\frac{3}{5}\div\Large\frac{2}{3}[/latex]

ΠŸΠΎΠΊΠ°Π·Π°Ρ‚ΡŒ Ρ€Π΅ΡˆΠ΅Π½ΠΈΠ΅

ΠŸΡ€ΠΈ Ρ€Π΅ΡˆΠ΅Π½ΠΈΠΈ Π·Π°Π΄Π°Ρ‡ΠΈ Π½Π° Π΄Π΅Π»Π΅Π½ΠΈΠ΅ ΠΏΡƒΡ‚Π΅ΠΌ умноТСния Π½Π° ΠΎΠ±Ρ€Π°Ρ‚Π½ΠΎΠ΅ число Π½Π΅ Π·Π°Π±ΡƒΠ΄ΡŒΡ‚Π΅ Π·Π°ΠΏΠΈΡΠ°Ρ‚ΡŒ всС Ρ†Π΅Π»Ρ‹Π΅ числа ΠΈ ΡΠΌΠ΅ΡˆΠ°Π½Π½Ρ‹Π΅ числа Π² Π²ΠΈΠ΄Π΅ Π½Π΅ΠΏΡ€Π°Π²ΠΈΠ»ΡŒΠ½Ρ‹Ρ… Π΄Ρ€ΠΎΠ±Π΅ΠΉ. ΠžΠΊΠΎΠ½Ρ‡Π°Ρ‚Π΅Π»ΡŒΠ½Ρ‹ΠΉ ΠΎΡ‚Π²Π΅Ρ‚ слСдуСт ΡƒΠΏΡ€ΠΎΡΡ‚ΠΈΡ‚ΡŒ ΠΈ Π·Π°ΠΏΠΈΡΠ°Ρ‚ΡŒ Π² Π²ΠΈΠ΄Π΅ смСшанного числа.

Π’ ΡΠ»Π΅Π΄ΡƒΡŽΡ‰Π΅ΠΌ Π²ΠΈΠ΄Π΅ΠΎ Π²Ρ‹ ΡƒΠ²ΠΈΠ΄ΠΈΡ‚Π΅ ΠΏΡ€ΠΈΠΌΠ΅Ρ€ Ρ‚ΠΎΠ³ΠΎ, ΠΊΠ°ΠΊ Ρ€Π°Π·Π΄Π΅Π»ΠΈΡ‚ΡŒ Ρ†Π΅Π»ΠΎΠ΅ число Π½Π° Π΄Ρ€ΠΎΠ±ΡŒ, Π° Ρ‚Π°ΠΊΠΆΠ΅ ΠΏΡ€ΠΈΠΌΠ΅Ρ€ Ρ‚ΠΎΠ³ΠΎ, ΠΊΠ°ΠΊ Ρ€Π°Π·Π΄Π΅Π»ΠΈΡ‚ΡŒ ΠΎΠ΄Π½Ρƒ Π΄Ρ€ΠΎΠ±ΡŒ Π½Π° Π΄Ρ€ΡƒΠ³ΡƒΡŽ Π΄Ρ€ΠΎΠ±ΡŒ.

Π”Ρ€ΠΎΠ±ΠΈ ΠΈ ΠΊΠ°ΠΊ ΠΈΡ… ΡΠΊΠ»Π°Π΄Ρ‹Π²Π°Ρ‚ΡŒ, Π²Ρ‹Ρ‡ΠΈΡ‚Π°Ρ‚ΡŒ, ΡƒΠΌΠ½ΠΎΠΆΠ°Ρ‚ΡŒ ΠΈ Π΄Π΅Π»ΠΈΡ‚ΡŒ

1.2 — Π”Ρ€ΠΎΠ±ΠΈ ΠΈ ΠΊΠ°ΠΊ ΠΈΡ… ΡΠΊΠ»Π°Π΄Ρ‹Π²Π°Ρ‚ΡŒ, Π²Ρ‹Ρ‡ΠΈΡ‚Π°Ρ‚ΡŒ, ΡƒΠΌΠ½ΠΎΠΆΠ°Ρ‚ΡŒ ΠΈ Π΄Π΅Π»ΠΈΡ‚ΡŒ

1.2 — Π”Ρ€ΠΎΠ±ΠΈ ΠΈ ΠΊΠ°ΠΊ ΠΈΡ… ΡΠΊΠ»Π°Π΄Ρ‹Π²Π°Ρ‚ΡŒ, Π²Ρ‹Ρ‡ΠΈΡ‚Π°Ρ‚ΡŒ, ΡƒΠΌΠ½ΠΎΠΆΠ°Ρ‚ΡŒ ΠΈ Π΄Π΅Π»ΠΈΡ‚ΡŒ

ΠžΠ±ΠΎΠ·Π½Π°Ρ‡Π΅Π½ΠΈΠ΅ Π΄Ρ€ΠΎΠ±ΠΈ

Π”Ρ€ΠΎΠ±ΠΈ (ΠΈΠ»ΠΈ ΠΎΠ±Ρ‹ΠΊΠ½ΠΎΠ²Π΅Π½Π½Ρ‹Ρ… Π΄Ρ€ΠΎΠ±ΠΈ ) ΠΈΡΠΏΠΎΠ»ΡŒΠ·ΡƒΡŽΡ‚ΡΡ для описания части Ρ†Π΅Π»ΠΎΠ³ΠΎ ΠΎΠ±ΡŠΠ΅ΠΊΡ‚Π°. БущСствуСт нСсколько ΠΎΠ±ΠΎΠ·Π½Π°Ρ‡Π΅Π½ΠΈΠΉ Π΄Ρ€ΠΎΠ±Π΅ΠΉ:
a называСтся числитСлСм , Π° b называСтся Π·Π½Π°ΠΌΠ΅Π½Π°Ρ‚Π΅Π»Π΅ΠΌ . ΠžΠ±ΠΎΠ·Π½Π°Ρ‡Π΅Π½ΠΈΠ΅ ΠΎΠ·Π½Π°Ρ‡Π°Π΅Ρ‚, Ρ‡Ρ‚ΠΎ ΠΌΡ‹ Ρ€Π°Π·Π±ΠΈΠ²Π°Π΅ΠΌ ΠΎΠ±ΡŠΠ΅ΠΊΡ‚ Π½Π° b Ρ€Π°Π²Π½Ρ‹Ρ… частСй ΠΈ Ρƒ нас Π΅ΡΡ‚ΡŒ ΠΈ Ρ‚Π°ΠΊΠΈΡ… ΡˆΡ‚ΡƒΠΊ. Π§Π°ΡΡ‚ΡŒ ΠΈΠ»ΠΈ Ρ‡Π°ΡΡ‚ΡŒ ΠΎΠ±ΡŠΠ΅ΠΊΡ‚Π°, ΠΊΠΎΡ‚ΠΎΡ€Ρ‹ΠΉ Ρƒ нас Π΅ΡΡ‚ΡŒ это Π° / Π± . НапримСр, Ссли ΠΌΡ‹ Ρ€Π°Π·ΠΎΠ±ΡŒΠ΅ΠΌ ΠΏΠΈΡ€ΠΎΠ³ Π½Π° 4 Ρ€Π°Π²Π½Ρ‹Ρ… куска ΠΈ Π±Π΅Ρ€Π΅ΠΌ 1 кусок Ρ‚ΠΎ Ρƒ нас получаСтся 1/4 Ρ‡Π°ΡΡ‚ΡŒ ΠΏΠΈΡ€ΠΎΠ³Π°:



Π­ΠΊΠ²ΠΈΠ²Π°Π»Π΅Π½Ρ‚Π½Ρ‹Π΅ Π΄Ρ€ΠΎΠ±ΠΈ

ΠžΠ±Ρ€Π°Ρ‚ΠΈΡ‚Π΅ Π²Π½ΠΈΠΌΠ°Π½ΠΈΠ΅, Ρ‡Ρ‚ΠΎ ΠΌΡ‹ ΠΏΠΎΠ»ΡƒΡ‡Π°Π΅ΠΌ ΠΎΠ΄ΠΈΠ½Π°ΠΊΠΎΠ²ΠΎΠ΅ количСство ΠΏΠΈΡ€ΠΎΠ³Π° ΠΊΠ°ΠΊ Π² ΠΏΡ€Π΅Π΄Ρ‹Π΄ΡƒΡ‰Π΅ΠΌ ΠΏΡ€ΠΈΠΌΠ΅Ρ€Π΅, Ссли Ρ€Π°Π·Π΄Π΅Π»ΠΈΡ‚ΡŒ ΠΏΠΈΡ€ΠΎΠ³ Π½Π° 8 Ρ€Π°Π²Π½Ρ‹Ρ… частСй ΠΈ ΠΏΠΎΠ»ΡƒΡ‡ΠΈΡ‚ΡŒ 2 ΠΈΠ· Π½ΠΈΡ…:
Π’Π°ΠΊΠΈΠ΅ Π΄Ρ€ΠΎΠ±ΠΈ, ΠΊΠ°ΠΊ 1/4 ΠΈ 2/8, ΠΈΠΌΠ΅ΡŽΡ‰ΠΈΠ΅ ΠΎΠ΄ΠΈΠ½Π°ΠΊΠΎΠ²ΠΎΠ΅ Π·Π½Π°Ρ‡Π΅Π½ΠΈΠ΅, Π½Π°Π·Ρ‹Π²Π°ΡŽΡ‚ΡΡ эквивалСнтныС Π΄Ρ€ΠΎΠ±ΠΈ . Π­Ρ‚ΠΎΡ‚ ΠΏΡ€ΠΈΠΌΠ΅Ρ€ ΠΏΡ€Π΅Π΄Π»Π°Π³Π°Π΅Ρ‚ ΡΠ»Π΅Π΄ΡƒΡŽΡ‰ΠΈΠΉ ΠΌΠ΅Ρ‚ΠΎΠ΄ для ΠΏΡ€ΠΎΠ²Π΅Ρ€ΠΊΠ° Π½Π° равСнство Π΄Π²ΡƒΡ… Π΄Ρ€ΠΎΠ±Π΅ΠΉ.

Π”Π²Π΅ Π΄Ρ€ΠΎΠ±ΠΈ Ρ€Π°Π²Π½Ρ‹ , Ссли ΡƒΠΌΠ½ΠΎΠΆΠΈΡ‚ΡŒ Ρ‡ΠΈΡΠ»ΠΈΡ‚Π΅Π»ΡŒ ΠΈ Π·Π½Π°ΠΌΠ΅Π½Π°Ρ‚Π΅Π»ΡŒ ΠΎΠ΄Π½ΠΎΠΉ Π΄Ρ€ΠΎΠ±ΠΈ Π½Π° Ρ‚ΠΎ ΠΆΠ΅ Ρ†Π΅Π»ΠΎΠ΅ число Π΄Π°Π΅Ρ‚ другая Π΄Ρ€ΠΎΠ±ΡŒ.

НапримСр, 4/5 ΠΈ 24/30 эквивалСнтны, ΠΏΠΎΡ‚ΠΎΠΌΡƒ Ρ‡Ρ‚ΠΎ ΠΌΡ‹ ΠΌΠΎΠΆΠ΅ΠΌ Π½Π°Ρ‡Π°Ρ‚ΡŒ с 4/5. ΠΈ ΡƒΠΌΠ½ΠΎΠΆΡŒΡ‚Π΅ Ρ‡ΠΈΡΠ»ΠΈΡ‚Π΅Π»ΡŒ ΠΈ Π·Π½Π°ΠΌΠ΅Π½Π°Ρ‚Π΅Π»ΡŒ Π½Π° 6, Ρ‡Ρ‚ΠΎΠ±Ρ‹ ΠΏΠΎΠ»ΡƒΡ‡ΠΈΡ‚ΡŒ 24/30:
Π”Π²ΠΈΠΆΠ΅Π½ΠΈΠ΅ Π² ΠΎΠ±Ρ€Π°Ρ‚Π½ΠΎΠΌ Π½Π°ΠΏΡ€Π°Π²Π»Π΅Π½ΠΈΠΈ (ΠΎΡ‚ 24/30 Π΄ΠΎ 4/5) ΠΏΡ€Π΅Π΄ΠΏΠΎΠ»Π°Π³Π°Π΅Ρ‚ ΡΠ»Π΅Π΄ΡƒΡŽΡ‰ΠΈΠΉ ΠΌΠ΅Ρ‚ΠΎΠ΄. для сокращСния Π΄Ρ€ΠΎΠ±ΠΈ Π΄ΠΎ ΠΌΠ»Π°Π΄ΡˆΠΈΡ… Ρ‡Π»Π΅Π½ΠΎΠ² ΠΈΠ»ΠΈ Π΅Π³ΠΎ ΠΏΡ€ΠΎΡΡ‚Π΅ΠΉΡˆΠ°Ρ эквивалСнтная Π΄Ρ€ΠΎΠ±ΡŒ :


Π§Ρ‚ΠΎΠ±Ρ‹ привСсти Π΄Ρ€ΠΎΠ±ΡŒ ΠΊ Π½Π°ΠΈΠΌΠ΅Π½ΡŒΡˆΠ΅ΠΌΡƒ Ρ‡Π»Π΅Π½Ρƒ ΠΈΠ»ΠΈ ΠΊ Π΅Π΅ ΠΏΡ€ΠΎΡΡ‚Π΅ΠΉΡˆΠ΅ΠΌΡƒ эквивалСнту Π΄Ρ€ΠΎΠ±ΠΈ , ΠΏΠΎΠ»Π½ΠΎΡΡ‚ΡŒΡŽ Ρ€Π°Π·Π»ΠΎΠΆΠΈΡ‚ΡŒ ΠΊΠ°ΠΊ Ρ‡ΠΈΡΠ»ΠΈΡ‚Π΅Π»ΡŒ, Ρ‚Π°ΠΊ ΠΈ Π·Π½Π°ΠΌΠ΅Π½Π°Ρ‚Π΅Π»ΡŒ (Ρ‚. Π΅. Π½Π° простыС числа). Π—Π°Ρ‚Π΅ΠΌ сократитС ΠΊΠ°ΠΆΠ΄Ρ‹ΠΉ ΠΌΠ½ΠΎΠΆΠΈΡ‚Π΅Π»ΡŒ, ΠΊΠΎΡ‚ΠΎΡ€Ρ‹ΠΉ встрСчаСтся ΠΊΠ°ΠΊ Π² числитСлС, Ρ‚Π°ΠΊ ΠΈ Π² числитСлС. ΠΈ Π·Π½Π°ΠΌΠ΅Π½Π°Ρ‚Π΅Π»ΡŒ. ΠžΡΡ‚Π°Π΅Ρ‚ΡΡ ΠΏΡ€ΠΎΡΡ‚Π΅ΠΉΡˆΠ°Ρ эквивалСнтная Π΄Ρ€ΠΎΠ±ΡŒ .

НапримСр Π²ΠΎΡ‚ ΠΊΠ°ΠΊ ΡƒΠΌΠ΅Π½ΡŒΡˆΠΈΡ‚ΡŒ Π΄Ρ€ΠΎΠ±ΡŒ 24/42 это самоС простоС эквивалСнтная Π΄Ρ€ΠΎΠ±ΡŒ, Π° ΠΈΠΌΠ΅Π½Π½ΠΎ 4/7:


УпраТнСния Ρ‚Ρ€Π΅Π½Π΅Ρ€Π° ΠΏΠΎ Π°Π»Π³Π΅Π±Ρ€Π΅



ΠΠ΅ΠΏΡ€Π°Π²ΠΈΠ»ΡŒΠ½Ρ‹Π΅ Π΄Ρ€ΠΎΠ±ΠΈ, ΡΠΌΠ΅ΡˆΠ°Π½Π½Ρ‹Π΅ Π΄Ρ€ΠΎΠ±ΠΈ ΠΈ Π΄Π»ΠΈΠ½Π½ΠΎΠ΅ Π΄Π΅Π»Π΅Π½ΠΈΠ΅

Π”Ρ€ΠΎΠ±ΡŒ, Ρƒ ΠΊΠΎΡ‚ΠΎΡ€ΠΎΠΉ Ρ‡ΠΈΡΠ»ΠΈΡ‚Π΅Π»ΡŒ мСньшС знамСнатСля, называСтся ΠΏΡ€Π°Π²ΠΈΠ»ΡŒΠ½ΠΎΠΉ Π΄Ρ€ΠΎΠ±ΡŒΡŽ ΠΈ Π΄Ρ€ΠΎΠ±ΡŒ, Ρƒ ΠΊΠΎΡ‚ΠΎΡ€ΠΎΠΉ Ρ‡ΠΈΡΠ»ΠΈΡ‚Π΅Π»ΡŒ большС знамСнатСля, называСтся Π½Π΅ΠΏΡ€Π°Π²ΠΈΠ»ΡŒΠ½ΠΎΠΉ Π΄Ρ€ΠΎΠ±ΡŒΡŽ . ΠŸΡ€ΠΈΠΌΠ΅Ρ€ Π½Π΅ΠΏΡ€Π°Π²ΠΈΠ»ΡŒΠ½ΠΎΠΉ Π΄Ρ€ΠΎΠ±ΠΈ 7/4. Π˜ΡΠΏΠΎΠ»ΡŒΠ·ΡƒΡ ΠΏΡ€ΠΈΠΌΠ΅Ρ€ ΠΏΠΈΡ€ΠΎΠ³Π°, это ΠΎΠ·Π½Π°Ρ‡Π°Π΅Ρ‚, Ρ‡Ρ‚ΠΎ Π²Ρ‹ сломали ΠΌΠ½ΠΎΠ³ΠΎ ΠΏΠΈΡ€ΠΎΠ³ΠΎΠ², ΠΊΠ°ΠΆΠ΄Ρ‹ΠΉ ΠΈΠ· ΠΊΠΎΡ‚ΠΎΡ€Ρ‹Ρ… Ρ€Π°Π·Π΄Π΅Π»Π΅Π½ Π½Π° 4 Ρ€Π°Π²Π½Ρ‹Π΅ части, ΠΈ Ρƒ вас Π΅ΡΡ‚ΡŒ 7 Ρ‚Π°ΠΊΠΈΡ… частСй:
ΠΠ΅ΠΏΡ€Π°Π²ΠΈΠ»ΡŒΠ½Ρ‹Π΅ Π΄Ρ€ΠΎΠ±ΠΈ ΠΈΠ½ΠΎΠ³Π΄Π° Π·Π°ΠΏΠΈΡΡ‹Π²Π°ΡŽΡ‚ Π² Π²ΠΈΠ΄Π΅ смСшанной Π΄Ρ€ΠΎΠ±ΠΈ , которая прСдставляСт собой сумму Ρ†Π΅Π»ΠΎΠ΅ число ΠΈ ΠΏΡ€Π°Π²ΠΈΠ»ΡŒΠ½Π°Ρ Π΄Ρ€ΠΎΠ±ΡŒ, Π½ΠΎ Π±Π΅Π· Π·Π½Π°ΠΊΠ° +. НапримСр, 7/4 Π² ΠΎΠ±ΠΎΠ·Π½Π°Ρ‡Π΅Π½ΠΈΠΈ смСшанной Π΄Ρ€ΠΎΠ±ΠΈ. выглядит Ρ‚Π°ΠΊ:
ΠžΠ±ΠΎΠ·Π½Π°Ρ‡Π΅Π½ΠΈΠ΅ смСшанной Π΄Ρ€ΠΎΠ±ΠΈ Π½Π΅ ΠΈΡΠΏΠΎΠ»ΡŒΠ·ΡƒΠ΅Ρ‚ΡΡ Π² этой ΠΊΠ½ΠΈΠ³Π΅ Algebra Help e ΠΈΠ»ΠΈ Π² ΠΏΡ€ΠΎΠ³Ρ€Π°ΠΌΠΌΠ΅ Algebra Coach, ΠΏΠΎΡ‚ΠΎΠΌΡƒ Ρ‡Ρ‚ΠΎ Π΅Π³ΠΎ слишком Π»Π΅Π³ΠΊΠΎ ΡΠΏΡƒΡ‚Π°Ρ‚ΡŒ с ΠΏΡ€ΠΎΠΈΠ·Π²Π΅Π΄Π΅Π½ΠΈΠ΅ΠΌ Ρ†Π΅Π»ΠΎΠ³ΠΎ числа Π½Π° Π΄Ρ€ΠΎΠ±ΡŒ. ВмСсто Ρ‚ΠΎΠ³ΠΎ, Ρ‡Ρ‚ΠΎΠ±Ρ‹ ΠΏΠΈΡΠ°Ρ‚ΡŒ ΠΌΡ‹ Π±ΡƒΠ΄Π΅ΠΌ Π΄Π΅Ρ€ΠΆΠ°Ρ‚ΡŒ Π·Π½Π°ΠΊ + ΠΈ ΠΏΠΈΡΠ°Ρ‚ΡŒ.

Π”Π»ΠΈΠ½Π½ΠΎΠ΅ Π΄Π΅Π»Π΅Π½ΠΈΠ΅ β€” это ΠΌΠ΅Ρ‚ΠΎΠ΄ прСобразования Π½Π΅ΠΏΡ€Π°Π²ΠΈΠ»ΡŒΠ½ΠΎΠΉ Π΄Ρ€ΠΎΠ±ΠΈ Π² ΡΠΌΠ΅ΡˆΠ°Π½Π½ΡƒΡŽ. ΠŸΡ€ΠΎΠΈΠ»Π»ΡŽΡΡ‚Ρ€ΠΈΡ€ΡƒΠ΅ΠΌ ΠΌΠ΅Ρ‚ΠΎΠ΄ Π½Π° Π΄Ρ€ΠΎΠ±ΠΈ . Π’Ρ‹ΠΏΠΎΠ»Π½ΠΈΡ‚Π΅ ΡΠ»Π΅Π΄ΡƒΡŽΡ‰ΠΈΠ΅ шаги:
  • УстановитС Ρ„ΠΎΡ€ΠΌΠ°Ρ‚ Π΄Π»ΠΈΠ½Π½ΠΎΠ³ΠΎ дСлСния, Π° ΠΈΠΌΠ΅Π½Π½ΠΎ .
  • Π’Π°ΠΊ ΠΊΠ°ΠΊ 5 Π² 9 ΠΈΠ΄Π΅Ρ‚ 1 Ρ€Π°Π·, Π½Π°ΠΏΠΈΡˆΠΈΡ‚Π΅ Β«1Β» Π½Π°Π΄ 9, Π½Π°ΠΏΠΈΡˆΠΈΡ‚Π΅ 1 Γ— 5 ΠΈΠ»ΠΈ Β«5Β» Π½ΠΈΠΆΠ΅ 9 ΠΈ Π²Ρ‹Ρ‡Ρ‚ΠΈΡ‚Π΅ 5 ΠΈΠ· 9, Ρ‡Ρ‚ΠΎΠ±Ρ‹ ΠΏΠΎΠ»ΡƒΡ‡ΠΈΡ‚ΡŒ Ρ€Π°Π·Π½ΠΈΡ†Ρƒ Π² 4, Π½Π°ΠΏΡ€ΠΈΠΌΠ΅Ρ€:
  • Π—Π°Ρ‚Π΅ΠΌ опуститС 2 ΡΠ»Π΅Π΄ΡƒΡŽΡ‰ΠΈΠΌ ΠΎΠ±Ρ€Π°Π·ΠΎΠΌ:
    Π’ΠΎΡ‚ Ρ‡Ρ‚ΠΎ ΠΌΡ‹ Π½Π° самом Π΄Π΅Π»Π΅ сдСлали: Β«1Β» ΠΈ Β«5Β» находятся Π² дСсятки мСсто, поэтому ΠΎΠ½ΠΈ Π½Π° самом Π΄Π΅Π»Π΅ ΠΏΡ€Π΅Π΄ΡΡ‚Π°Π²Π»ΡΡŽΡ‚ числа 10 ΠΈ 50, ΠΊΠ°ΠΊ ΠΏΠΎΠΊΠ°Π·Π°Π½ΠΎ здСсь:
    ΠŸΠΎΡΡ‚ΠΎΠΌΡƒ ΠΌΡ‹ фактичСски ΠΏΠΎΠΊΠ°Π·Π°Π»ΠΈ, Ρ‡Ρ‚ΠΎ .
  • Π’Π΅ΠΏΠ΅Ρ€ΡŒ ΠΏΠΎΠ²Ρ‚ΠΎΡ€ΠΈΡ‚Π΅ вСсь процСсс с остатком 42. ΠŸΠΎΡΠΊΠΎΠ»ΡŒΠΊΡƒ 5 Π² 42 ΠΈΠ΄Π΅Ρ‚ 8 Ρ€Π°Π·, Π½Π°ΠΏΠΈΡˆΠΈΡ‚Π΅ Β«8Β» Π½Π°Π΄ 2, Π½Π°ΠΏΠΈΡˆΠΈΡ‚Π΅ 8 Γ— 5 ΠΈΠ»ΠΈ Β«40Β» Π½ΠΈΠΆΠ΅ 42, ΠΈ Π²Ρ‹Ρ‡Ρ‚ΠΈΡ‚Π΅ 40 ΠΈΠ· 42, Ρ‡Ρ‚ΠΎΠ±Ρ‹ ΠΏΠΎΠ»ΡƒΡ‡ΠΈΡ‚ΡŒ остаток 2, Π½Π°ΠΏΡ€ΠΈΠΌΠ΅Ρ€:
  • Π­Ρ‚ΠΎ ΠΏΠΎΠΊΠ°Π·Ρ‹Π²Π°Π΅Ρ‚, Ρ‡Ρ‚ΠΎ . ΠŸΠΎΡΠΊΠΎΠ»ΡŒΠΊΡƒ остаток 2 мСньшС дСлитСля 5, это наш ΠΎΠΊΠΎΠ½Ρ‡Π°Ρ‚Π΅Π»ΡŒΠ½Ρ‹ΠΉ Π²Π°Ρ€ΠΈΠ°Π½Ρ‚. Ρ€Π΅Π·ΡƒΠ»ΡŒΡ‚Π°Ρ‚ смСшанной Ρ„Ρ€Π°ΠΊΡ†ΠΈΠΈ.



НСкоторыС ΡΠΏΠ΅Ρ†ΠΈΠ°Π»ΡŒΠ½Ρ‹Π΅ Π΄Ρ€ΠΎΠ±ΠΈ

Π•ΡΡ‚ΡŒ нСсколько ΡΠΏΠ΅Ρ†ΠΈΠ°Π»ΡŒΠ½Ρ‹Ρ… Π΄Ρ€ΠΎΠ±Π΅ΠΉ, ΠΊΠΎΡ‚ΠΎΡ€Ρ‹Π΅ Π²Π°ΠΆΠ½ΠΎ Ρ€Π°ΡΠΏΠΎΠ·Π½Π°Ρ‚ΡŒ:
  • . Π›ΡŽΠ±ΠΎΠ΅ число n ΠΌΠΎΠΆΠ½ΠΎ ΠΏΡ€Π΅Π²Ρ€Π°Ρ‚ΠΈΡ‚ΡŒ Π² Π΄Ρ€ΠΎΠ±ΡŒ, написав Π΅Π΅ Π½Π°Π΄ Π·Π½Π°ΠΌΠ΅Π½Π°Ρ‚Π΅Π»Π΅ΠΌ 1.
  • . ВсС, Ρ‡Ρ‚ΠΎ дСлится само Π½Π° сСбя, Ρ€Π°Π²Π½ΠΎ 1. ΠœΡ‹ Π½Π°Π·Ρ‹Π²Π°Π΅ΠΌ это UFOO (a u seful f orm o f o ne). ΠŸΠΎΠ΄Ρ€ΠΎΠ±Π½Π΅Π΅ ΠΎΠ± ΠΠ›Πž ΠΏΠΎΠ·ΠΆΠ΅.
  • Если Ρ‡ΠΈΡΠ»ΠΈΡ‚Π΅Π»ΡŒ Π΄Ρ€ΠΎΠ±ΠΈ ΠΊΡ€Π°Ρ‚Π΅Π½ Π·Π½Π°ΠΌΠ΅Π½Π°Ρ‚Π΅Π»ΡŽ Ρ‚ΠΎ Π΄Ρ€ΠΎΠ±ΡŒ Ρ€Π°Π²Π½Π° Ρ†Π΅Π»ΠΎΠΌΡƒ числу. ΠŸΡ€ΠΈΠΌΠ΅Ρ€ΠΎΠΌ являСтся .
  • Β Π½Π΅ ΠΎΠΏΡ€Π΅Π΄Π΅Π»Π΅Π½ΠΎ для любого числитСля n . Π”Π΅Π»Π΅Π½ΠΈΠ΅ Π½Π° ноль Π² ΠΌΠ°Ρ‚Π΅ΠΌΠ°Ρ‚ΠΈΠΊΠ΅ Π·Π°ΠΏΡ€Π΅Ρ‰Π΅Π½ΠΎ.
  • . НулСвой Ρ‡ΠΈΡΠ»ΠΈΡ‚Π΅Π»ΡŒ Π½Π΅ ΠΏΡ€ΠΎΠ±Π»Π΅ΠΌΠ°. Π­Ρ‚Π° Π΄Ρ€ΠΎΠ±ΡŒ Ρ€Π°Π²Π½Π° 0,
  • .



Π‘Π»ΠΎΠΆΠ΅Π½ΠΈΠ΅ ΠΈΠ»ΠΈ Π²Ρ‹Ρ‡ΠΈΡ‚Π°Π½ΠΈΠ΅ Π΄Ρ€ΠΎΠ±Π΅ΠΉ

На этом рисункС ΠΏΠΎΠΊΠ°Π·Π°Π½ΠΎ, Ρ‡Ρ‚ΠΎ 2/8 ΠΏΠΈΡ€ΠΎΠ³Π° плюс 3/8 ΠΏΠΈΡ€ΠΎΠ³Π° Ρ€Π°Π²Π½ΡΡŽΡ‚ΡΡ 5/8 ΠΏΠΈΡ€ΠΎΠ³Π°:
Π”Ρ€ΠΎΠ±ΠΈ, ΠΈΠΌΠ΅ΡŽΡ‰ΠΈΠ΅ ΠΎΠ΄ΠΈΠ½Π°ΠΊΠΎΠ²Ρ‹Π΅ Π·Π½Π°ΠΌΠ΅Π½Π°Ρ‚Π΅Π»ΠΈ, Π½Π°Π·Ρ‹Π²Π°ΡŽΡ‚ΡΡ , ΠΊΠ°ΠΊ ΠΈ Π΄Ρ€ΠΎΠ±ΠΈ . Если ΠΏΠΎΠ΄ΡƒΠΌΠ°Ρ‚ΡŒ ΠΎΠ± этом ΠΏΡ€ΠΈΠΌΠ΅Ρ€Π΅, Ρ‚ΠΎ ΡΠ»Π΅Π΄ΡƒΡŽΡ‰Π°Ρ ΠΏΡ€ΠΎΡ†Π΅Π΄ΡƒΡ€Π° добавлСния ΠΈΠ»ΠΈ Π²Ρ‹Ρ‡ΠΈΡ‚Π°Π½ΠΈΠ΅ ΠΏΠΎΠ΄ΠΎΠ±Π½Ρ‹Ρ… Π΄Ρ€ΠΎΠ±Π΅ΠΉ ΠΎΡ‡Π΅Π²ΠΈΠ΄Π½ΠΎ:

Π§Ρ‚ΠΎΠ±Ρ‹ ΡΠ»ΠΎΠΆΠΈΡ‚ΡŒ ΠΈΠ»ΠΈ Π²Ρ‹Ρ‡Π΅ΡΡ‚ΡŒ Π΄Π²Π΅ ΠΎΠ΄ΠΈΠ½Π°ΠΊΠΎΠ²Ρ‹Π΅ Π΄Ρ€ΠΎΠ±ΠΈ (Π΄Ρ€ΠΎΠ±ΠΈ, ΠΈΠΌΠ΅ΡŽΡ‰ΠΈΠ΅ ΠΎΠ±Ρ‰ΠΈΠΉ Π·Π½Π°ΠΌΠ΅Π½Π°Ρ‚Π΅Π»ΡŒ) , просто слоТитС ΠΈΠ»ΠΈ Π²Ρ‹Ρ‡Ρ‚ΠΈΡ‚Π΅ числитСли ΠΈ ΠΏΡ€ΠΈΠ²Π΅Π΄ΠΈΡ‚Π΅ Ρ€Π΅Π·ΡƒΠ»ΡŒΡ‚Π°Ρ‚ ΠΊ ΠΎΠ±Ρ‰Π΅ΠΌΡƒ Π·Π½Π°ΠΌΠ΅Π½Π°Ρ‚Π΅Π»ΡŽ, ΠΊΠ°ΠΊ это:

А Ρ‡Ρ‚ΠΎ, Ссли Π΄Ρ€ΠΎΠ±ΠΈ Π½Π΅ ΠΈΠΌΠ΅ΡŽΡ‚ ΠΎΠ±Ρ‰Π΅Π³ΠΎ знамСнатСля? ΠžΡ‚Π²Π΅Ρ‚ Π·Π°ΠΊΠ»ΡŽΡ‡Π°Π΅Ρ‚ΡΡ Π² Ρ‚ΠΎΠΌ, Ρ‡Ρ‚ΠΎ ΠΎΠ½ΠΈ Π΄ΠΎΠ»ΠΆΠ½Ρ‹ Π±Ρ‹Ρ‚ΡŒ Π·Π°Ρ‚Π΅ΠΌ ΠΏΡ€Π΅ΠΎΠ±Ρ€Π°Π·ΠΎΠ²Π°Π½Ρ‹ ΠΊ эквивалСнтным дробям, ΠΊΠΎΡ‚ΠΎΡ€Ρ‹Π΅ Π΄Π΅Π»Π°ΡŽΡ‚ ΠΈΠΌΠ΅ΡŽΡ‚ ΠΎΠ±Ρ‰ΠΈΠΉ Π·Π½Π°ΠΌΠ΅Π½Π°Ρ‚Π΅Π»ΡŒ. ΠŸΡ€ΠΎΡ†Π΅Π΄ΡƒΡ€Π° ΠΏΡ€ΠΎΠΈΠ»Π»ΡŽΡΡ‚Ρ€ΠΈΡ€ΠΎΠ²Π°Π½Π° Π½Π° этом ΠΏΡ€ΠΈΠΌΠ΅Ρ€Π΅:

Π¨Π°Π³ΠΈ:
  1. НайдитС наимСньшСС ΠΎΠ±Ρ‰Π΅Π΅ ΠΊΡ€Π°Ρ‚Π½ΠΎΠ΅ Π΄Π²ΡƒΡ… Π·Π½Π°ΠΌΠ΅Π½Π°Ρ‚Π΅Π»ΠΈ 24 ΠΈ 30. ΠŸΡ€ΠΈΠΌΠ΅Π½ΠΈΡ‚Π΅Π»ΡŒΠ½ΠΎ ΠΊ дробям это число называСтся наимСньшим ΠΎΠ±Ρ‰ΠΈΠΌ Π·Π½Π°ΠΌΠ΅Π½Π°Ρ‚Π΅Π»ΡŒ (Π–Πš-дисплСй). Π’ этом ΠΏΡ€ΠΈΠΌΠ΅Ρ€Π΅ Π–Πš-дисплСй Ρ€Π°Π²Π΅Π½ 120.
  2. ΠŸΡ€Π΅ΠΎΠ±Ρ€Π°Π·ΡƒΠΉΡ‚Π΅ ΠΊΠ°ΠΆΠ΄ΡƒΡŽ Π΄Ρ€ΠΎΠ±ΡŒ Π² ΡΠΊΠ²ΠΈΠ²Π°Π»Π΅Π½Ρ‚Π½ΡƒΡŽ Π΄Ρ€ΠΎΠ±ΡŒ, ΠΈΠΌΠ΅ΡŽΡ‰ΡƒΡŽ Π–Πš 120 Π² Π·Π½Π°ΠΌΠ΅Π½Π°Ρ‚Π΅Π»Π΅. Для этого Π² Π΄Π°Π½Π½ΠΎΠΌ ΠΏΡ€ΠΈΠΌΠ΅Ρ€Π΅ ΡƒΠΌΠ½ΠΎΠΆΡŒΡ‚Π΅ Ρ‡ΠΈΡΠ»ΠΈΡ‚Π΅Π»ΡŒ ΠΈ Π·Π½Π°ΠΌΠ΅Π½Π°Ρ‚Π΅Π»ΡŒ ΠΏΠ΅Ρ€Π²ΠΎΠΉ Π΄Ρ€ΠΎΠ±ΠΈ Π½Π° 5, Π° Ρ‡ΠΈΡΠ»ΠΈΡ‚Π΅Π»ΡŒ ΠΈ Π·Π½Π°ΠΌΠ΅Π½Π°Ρ‚Π΅Π»ΡŒ Π²Ρ‚ΠΎΡ€ΠΎΠΉ Π΄Ρ€ΠΎΠ±ΠΈ Π½Π° 4 (ΠΏΠΎΠΊΠ°Π·Π°Π½Ρ‹ красным).
  3. Π‘Π»ΠΎΠΆΠΈΡ‚Π΅ числитСли ΠΈ помСститС Π½Π°Π΄ ΠΎΠ±Ρ‰ΠΈΠΌ Π·Π½Π°ΠΌΠ΅Π½Π°Ρ‚Π΅Π»Π΅ΠΌ.
Иногда Π΅ΡΡ‚ΡŒ Π΅Ρ‰Π΅ ΠΎΠ΄ΠΈΠ½ шаг. Π Π΅Π·ΡƒΠ»ΡŒΡ‚Π°Ρ‚ всСгда Π΄ΠΎΠ»ΠΆΠ΅Π½ Π±Ρ‹Ρ‚ΡŒ Π²Ρ‹Ρ€Π°ΠΆΠ΅Π½ ΠΊΠ°ΠΊ ΠΏΡ€ΠΎΡΡ‚Π΅ΠΉΡˆΠ°Ρ эквивалСнтная Π΄Ρ€ΠΎΠ±ΡŒ, Π½Π°ΠΏΡ€ΠΈΠΌΠ΅Ρ€:

Π’ΠΎΡ‚ Π΅Ρ‰Π΅ нСсколько ΠΏΡ€ΠΈΠΌΠ΅Ρ€ΠΎΠ²:

УпраТнСния Ρ‚Ρ€Π΅Π½Π΅Ρ€Π° ΠΏΠΎ Π°Π»Π³Π΅Π±Ρ€Π΅




Π£ΠΌΠ½ΠΎΠΆΠ΅Π½ΠΈΠ΅ Π΄Ρ€ΠΎΠ±Π΅ΠΉ


Π£ΠΌΠ½ΠΎΠΆΠ΅Π½ΠΈΠ΅ Π΄Ρ€ΠΎΠ±Π΅ΠΉ Π΄Π°Π΅Ρ‚ Π½ΠΎΠ²ΡƒΡŽ Π΄Ρ€ΠΎΠ±ΡŒ. Π£ΠΌΠ½ΠΎΠΆΡŒΡ‚Π΅ числитСли, Ρ‡Ρ‚ΠΎΠ±Ρ‹ ΠΏΠΎΠ»ΡƒΡ‡ΠΈΡ‚ΡŒ Π½ΠΎΠ²Ρ‹ΠΉ Ρ‡ΠΈΡΠ»ΠΈΡ‚Π΅Π»ΡŒ ΠΈ ΡƒΠΌΠ½ΠΎΠΆΠΈΡ‚ΡŒ Π·Π½Π°ΠΌΠ΅Π½Π°Ρ‚Π΅Π»ΠΈ, Ρ‡Ρ‚ΠΎΠ±Ρ‹ ΠΏΠΎΠ»ΡƒΡ‡ΠΈΡ‚ΡŒ Π½ΠΎΠ²Ρ‹ΠΉ Π·Π½Π°ΠΌΠ΅Π½Π°Ρ‚Π΅Π»ΡŒ, ΠΊΠ°ΠΊ это:
Π—Π°Ρ‚Π΅ΠΌ упроститС, сократив Π½ΠΎΠ²ΡƒΡŽ Π΄Ρ€ΠΎΠ±ΡŒ Π΄ΠΎ Π½Π°ΠΈΠΌΠ΅Π½ΡŒΡˆΠΈΡ… Ρ‡Π»Π΅Π½ΠΎΠ².

Π§Ρ‚ΠΎΠ±Ρ‹ ΡƒΠΌΠ½ΠΎΠΆΠΈΡ‚ΡŒ Π΄Ρ€ΠΎΠ±ΡŒ Π½Π° Ρ†Π΅Π»ΠΎΠ΅ число, просто ΡƒΠΌΠ½ΠΎΠΆΡŒΡ‚Π΅ Ρ‡ΠΈΡΠ»ΠΈΡ‚Π΅Π»ΡŒ Π΄Ρ€ΠΎΠ±ΠΈ Π½Π° Ρ†Π΅Π»ΠΎΠ΅ число, Ρ‡Ρ‚ΠΎΠ±Ρ‹ ΠΏΠΎΠ»ΡƒΡ‡ΠΈΡ‚ΡŒ Π½ΠΎΠ²Ρ‹ΠΉ Ρ‡ΠΈΡΠ»ΠΈΡ‚Π΅Π»ΡŒ, Π½Π°ΠΏΡ€ΠΈΠΌΠ΅Ρ€:

Π—Π°Ρ‚Π΅ΠΌ упроститС, сократив Π½ΠΎΠ²ΡƒΡŽ Π΄Ρ€ΠΎΠ±ΡŒ Π΄ΠΎ Π½Π°ΠΈΠΌΠ΅Π½ΡŒΡˆΠΈΡ… Ρ‡Π»Π΅Π½ΠΎΠ².

Π’ΠΎΡ‚ ΠΏΡ€ΠΈΠΌΠ΅Ρ€ Ρ‚ΠΎΠ³ΠΎ, ΠΏΠΎΡ‡Π΅ΠΌΡƒ Ρ€Π°Π±ΠΎΡ‚Π°Π΅Ρ‚ пСрвая ΠΏΡ€ΠΎΡ†Π΅Π΄ΡƒΡ€Π°. ΠŸΡ€Π΅Π΄ΠΏΠΎΠ»ΠΎΠΆΠΈΠΌ, Ρ‡Ρ‚ΠΎ Π΅ΡΡ‚ΡŒ ΠΏΠΎΠ»ΠΎΠ²ΠΈΠ½Π° ΠΏΠΈΡ€ΠΎΠ³Π° (Ρ‚. Π΄Ρ€ΠΎΠ±ΡŒ 1/2), ΠΊΠ°ΠΊ ΠΏΠΎΠΊΠ°Π·Π°Π½ΠΎ слСва. Π’Π΅ΠΏΠ΅Ρ€ΡŒ ΠΏΡ€Π΅Π΄ΠΏΠΎΠ»ΠΎΠΆΠΈΠΌ, Ρ‡Ρ‚ΠΎ Π²Ρ‹ взяли 2/3 ΠΈΠ· этой ΠΏΠΎΠ»ΠΎΠ²ΠΈΠ½Ρ‹ ΠΏΠΈΡ€ΠΎΠ³Π°. (Π‘Π»ΠΎΠ²ΠΎ Β«ΠΈΠ·Β» пСрСводится ΠΊΠ°ΠΊ матСматичСская опСрация Β«ΡƒΠΌΠ½ΠΎΠΆΠΈΡ‚ΡŒΒ».) Π­Ρ‚ΠΎ ΠΎΠ·Π½Π°Ρ‡Π°Π΅Ρ‚, Ρ‡Ρ‚ΠΎ Π²Ρ‹ Ρ€Π°Π·Ρ€Π΅Π·Π°Π΅Ρ‚Π΅ ΠΏΠΎΠ»ΠΎΠ²ΠΈΠ½Ρƒ ΠΏΠΈΡ€ΠΎΠ³Π° Π½Π° 3 Ρ€Π°Π²Π½Ρ‹Π΅ части ΠΈ Π±Π΅Ρ€Π΅Ρ‚Π΅ 2 ΠΈΠ· Π½ΠΈΡ…. Π’ Ρ€Π΅Π·ΡƒΠ»ΡŒΡ‚Π°Ρ‚Π΅ получаСтся 2/6 части ΠΏΠΈΡ€ΠΎΠ³Π°.


Π’ΠΎΡ‚ ΠΏΡ€ΠΈΠΌΠ΅Ρ€ Ρ‚ΠΎΠ³ΠΎ, ΠΏΠΎΡ‡Π΅ΠΌΡƒ вторая ΠΏΡ€ΠΎΡ†Π΅Π΄ΡƒΡ€Π° Ρ€Π°Π±ΠΎΡ‚Π°Π΅Ρ‚. ΠŸΡ€Π΅Π΄ΠΏΠΎΠ»ΠΎΠΆΠΈΠΌ, Π²Ρ‹ съСли 1/4 Ρ‡Π°ΡΡ‚ΡŒ ΠΏΠΈΡ€ΠΎΠ³Π°. ΠΈ Ρ‡Ρ‚ΠΎ Ρ‚Π²ΠΎΠΉ Π΄Ρ€ΡƒΠ³ съСл Π² 3 Ρ€Π°Π·Π° большС ΠΏΠΈΡ€ΠΎΠ³Π°, Ρ‡Π΅ΠΌ Ρ‚Ρ‹. Π­Ρ‚ΠΎ ΠΎΠ·Π½Π°Ρ‡Π°Π΅Ρ‚, Ρ‡Ρ‚ΠΎ ваш Π΄Ρ€ΡƒΠ³ съСл 3/4 ΠΏΠΈΡ€ΠΎΠ³Π°.

Π’ΠΎΡ‚ Π΅Ρ‰Π΅ нСсколько ΠΏΡ€ΠΈΠΌΠ΅Ρ€ΠΎΠ² умноТСния:

УпраТнСния Ρ‚Ρ€Π΅Π½Π΅Ρ€Π° ΠΏΠΎ Π°Π»Π³Π΅Π±Ρ€Π΅



ΠžΠ±Ρ€Π°Ρ‚Π½Ρ‹Π΅ ΠΈ дСлящиС Π΄Ρ€ΠΎΠ±ΠΈ

ΠžΠ±Ρ€Π°Ρ‚Π½Ρ‹Π΅ числа ΠΈΠ³Ρ€Π°ΡŽΡ‚ Π²Π°ΠΆΠ½ΡƒΡŽ Ρ€ΠΎΠ»ΡŒ ΠΏΡ€ΠΈ Π΄Π΅Π»Π΅Π½ΠΈΠΈ Π΄Ρ€ΠΎΠ±Π΅ΠΉ. Π”Π²Π° числа ΠΈΠ»ΠΈ Π΄Ρ€ΠΎΠ±ΠΈ Π½Π°Π·Ρ‹Π²Π°ΡŽΡ‚ΡΡ ΠΎΠ±Ρ€Π°Ρ‚Π½Ρ‹ΠΌΠΈ Π΄Ρ€ΡƒΠ³ Π΄Ρ€ΡƒΠ³Ρƒ. Ссли ΠΈΡ… ΠΏΡ€ΠΎΠΈΠ·Π²Π΅Π΄Π΅Π½ΠΈΠ΅ Ρ€Π°Π²Π½ΠΎ 1. НапримСр:
4/5 ΠΈ 5/4 ΠΎΠ±Ρ€Π°Ρ‚Π½Ρ‹, ΠΏΠΎΡ‚ΠΎΠΌΡƒ Ρ‡Ρ‚ΠΎ

8 ΠΈ 1/8 ΠΎΠ±Ρ€Π°Ρ‚Π½Ρ‹, ΠΏΠΎΡ‚ΠΎΠΌΡƒ Ρ‡Ρ‚ΠΎ


Π”Π΅Π»Π΅Π½ΠΈΠ΅ Π΄Ρ€ΠΎΠ±Π΅ΠΉ: ΠŸΡ€ΠΎΡ†Π΅Π΄ΡƒΡ€Π° Π·Π°ΠΊΠ»ΡŽΡ‡Π°Π΅Ρ‚ΡΡ Π² Π·Π°ΠΌΠ΅Π½Π΅ дСлСния Π½Π° Π΄Ρ€ΠΎΠ±ΡŒ Π½Π° ΡƒΠΌΠ½ΠΎΠΆΠ΅Π½ΠΈΠ΅ ΠΎΠ±Ρ€Π°Ρ‚Π½Ρ‹ΠΌ Π·Π½Π°Ρ‡Π΅Π½ΠΈΠ΅ΠΌ этой Π΄Ρ€ΠΎΠ±ΠΈ , Π½Π°ΠΏΡ€ΠΈΠΌΠ΅Ρ€:

ΠžΠ±Ρ€Π°Ρ‚ΠΈΡ‚Π΅ Π²Π½ΠΈΠΌΠ°Π½ΠΈΠ΅, Ρ‡Ρ‚ΠΎ Π²Ρ‹ Π±Π΅Ρ€Π΅Ρ‚Π΅ обратная Π΄Ρ€ΠΎΠ±ΡŒ Π½Π° Π΄Π½Π΅!

Π’ΠΎΡ‚ ΠΏΠΎΡ‡Π΅ΠΌΡƒ эта ΠΏΡ€ΠΎΡ†Π΅Π΄ΡƒΡ€Π° Ρ€Π°Π±ΠΎΡ‚Π°Π΅Ρ‚:
Π‘ΡƒΡ‚ΡŒ Π² Ρ‚ΠΎΠΌ, Ρ‡Ρ‚ΠΎ вмСсто Ρ‚ΠΎΠ³ΠΎ, Ρ‡Ρ‚ΠΎΠ±Ρ‹ Π²ΠΈΠ΄Π΅Ρ‚ΡŒ Π΄Ρ€ΠΎΠ±ΡŒ, Π΄Π΅Π»Π΅Π½Π½ΡƒΡŽ Π½Π° Π΄Ρ€ΠΎΠ±ΡŒ, ΠΈΡ‰ΠΈΡ‚Π΅ ΠΎΠ΄Π½Ρƒ Π΄Ρ€ΠΎΠ±ΡŒ, Ρ‡ΠΈΡΠ»ΠΈΡ‚Π΅Π»ΡŒ ΠΈ Π·Π½Π°ΠΌΠ΅Π½Π°Ρ‚Π΅Π»ΡŒ ΠΊΠΎΡ‚ΠΎΡ€ΠΎΠΉ ΡΠ²Π»ΡΡŽΡ‚ΡΡ дробями. На ΠΏΠ΅Ρ€Π²ΠΎΠΌ шагС ΠΌΡ‹ ΡƒΠΌΠ½ΠΎΠΆΠΈΠ»ΠΈ эту Π΄Ρ€ΠΎΠ±ΡŒ Π½Π° UFOO Ρ‡ΠΈΡΠ»ΠΈΡ‚Π΅Π»ΡŒ ΠΈ Π·Π½Π°ΠΌΠ΅Π½Π°Ρ‚Π΅Π»ΡŒ ΠΊΠΎΡ‚ΠΎΡ€ΠΎΠ³ΠΎ ΡΠ²Π»ΡΡŽΡ‚ΡΡ дробями. ΠΠ›Πž Π±Ρ‹Π» Π²Ρ‹Π±Ρ€Π°Π½ Ρ‚Π°ΠΊ, Ρ‡Ρ‚ΠΎΠ±Ρ‹ Π΄Ρ€ΠΎΠ±ΠΈ Π² Π·Π½Π°ΠΌΠ΅Π½Π°Ρ‚Π΅Π»Π΅ ΡΠΎΠΊΡ€Π°Ρ‰Π°Π»ΠΈΡΡŒ ΠΈ Π΄Π°Π²Π°Π»ΠΈ 1. ПослС Π΄Ρ€ΡƒΠ³ΠΎΠ³ΠΎ ΡƒΠΏΡ€ΠΎΡ‰Π΅Π½ΠΈΠ΅, ΠΎΡΡ‚Π°Π²ΠΈΠ²ΡˆΠ΅Π΅ Ρ‚ΠΎΠ»ΡŒΠΊΠΎ ΠΎΠΊΠΎΠ½Ρ‡Π°Ρ‚Π΅Π»ΡŒΠ½ΠΎΠ΅ ΡƒΠΌΠ½ΠΎΠΆΠ΅Π½ΠΈΠ΅ Π΄Ρ€ΠΎΠ±Π΅ΠΉ.

Π”ΠΎΠ±Π°Π²ΠΈΡ‚ΡŒ ΠΊΠΎΠΌΠΌΠ΅Π½Ρ‚Π°Ρ€ΠΈΠΉ

Π’Π°Ρˆ адрСс email Π½Π΅ Π±ΡƒΠ΄Π΅Ρ‚ ΠΎΠΏΡƒΠ±Π»ΠΈΠΊΠΎΠ²Π°Π½. ΠžΠ±ΡΠ·Π°Ρ‚Π΅Π»ΡŒΠ½Ρ‹Π΅ поля ΠΏΠΎΠΌΠ΅Ρ‡Π΅Π½Ρ‹ *

Β© 2015 - 2019 ΠœΡƒΠ½ΠΈΡ†ΠΈΠΏΠ°Π»ΡŒΠ½ΠΎΠ΅ ΠΊΠ°Π·Ρ‘Π½Π½ΠΎΠ΅ ΠΎΠ±Ρ‰Π΅ΠΎΠ±Ρ€Π°Π·ΠΎΠ²Π°Ρ‚Π΅Π»ΡŒΠ½ΠΎΠ΅ ΡƒΡ‡Ρ€Π΅ΠΆΠ΄Π΅Π½ΠΈΠ΅ «Валовская срСдняя школа»

ΠšΠ°Ρ€Ρ‚Π° сайта